Você está na página 1de 236

Constitutional Law 2 Bill of Rights Armando Santiago Jr

Section Number Section title Extend Coverage


1 Due process / Equal
Protection of laws clause
2 Search and Seizures Arrest
3 Right to privacy of RPC
correspondence
4 Freedom of Expression
5 Freedom of religion
6 Right to liberty of abode and
travel
7 Right to information
8 Right to association
9 Right to Expropriate Power of Eminent domain
10 Contract Clause / Non- Oblicon
Impairment laws clause
11
12
13
14
15

Page 1 of 236
Constitutional Law 2 Bill of Rights Armando Santiago Jr

CASE CONTENT FORMAT


CASE TITLE GR NO. CASE HINT

CASE TITLE FORMAT


PONENTE:

TOPIC:

TRIGGER OF THE FACTS:

TRIGGER OF THE ISSUE:

TRIGGER OF THE RULING:

VERDICT:

END POINT:

CITED DOCTRINE:

SEQUEL OF THE CASE:

IMPORTANT POINT/s:

FACTS
ISSUE
RULING

Page 2 of 236
Constitutional Law 2 Bill of Rights Armando Santiago Jr

RATIO

CLASS ANNOTATION
SYLLABI
CONCURRING
DISSENTING
SEPARATE OPINION
OBITER DICTUM

Page 3 of 236
Constitutional Law 2 Bill of Rights Armando Santiago Jr

CASE CONTENT FORMAT ............................................................................................................................................................................................2


CASE TITLE FORMAT....................................................................................................................................................................................................2
BILL OF RIGHTS .................................................................................................................................................................................................................... 10
SIGNIFICANCE OF THE BILL OF RIGHTS ...........................................................................................................................................................................10
CLASSIFICATION OF RIGHTS .............................................................................................................................................................................................. 10

THE INHERENT POWERS OF THE STATE


SIMILARITIES, DISTINCTIONS, AND LIMITATION ............................................................................................................................................................... 10
FUNDAMENTAL POWERS OF THE STATE (POLICE POWER, EMINENT DOMAIN, TAXATION) ...................................................................................... 12

POLICE POWER
REQUISITES FOR VALID EXERCISE POLICE POWER .....................................................................................................................................................12
LAWFUL SUBJECT ................................................................................................................................................................................................................ 12
LAWFUL MEANS .................................................................................................................................................................................................................... 12
TO WHAT AREAS OF LIEE POLICE POWER HAS PERVASIVE REACH? .......................................................................................................................... 12

cases
ORTIGAS & CO. V. CA ................................................................................................................................................................................................14
PHILIPPINE PRESS INSTITUTE, INC., VS. COMMISSION ON ELECTIONS ............................................................................................................ 15
VICENTE DE LA CRUZ VS EDGARDO PARAS ..........................................................................................................................................................16
LUCENA GRAND CENTRAL TERMINAL, INC., VS. JAC LINER, INC., ......................................................................................................................17
ERMITA-MALATE HOTEL & MOTEL OPERATORS ASSOC., INC VS MAYOR OF MANILA ......................................................................................18
FACTS ..........................................................................................................................................................................................................................18

POWER OF TAXATION
DEFINITION AND SCOPE ..................................................................................................................................................................................................... 20
PURPOSE ..............................................................................................................................................................................................................................20
SCOPE AND LIMITATION ..................................................................................................................................................................................................... 20

POWER OF EMINENT DOMAIN


REQUISITES ..........................................................................................................................................................................................................................24
IT IS THE RIGHT OF THE GOVERNMENT TO TAKE PRIVATE PROPERTY WITH JUST COMPENSATION..................................................................... 24
SCOPE AND LIMITATIONS ................................................................................................................................................................................................... 24
NECESSITY ...........................................................................................................................................................................................................................25
PRIVATE PROPERTY ...........................................................................................................................................................................................................25
TAKING .................................................................................................................................................................................................................................. 25
REQUISITES FOR A VALID TAKING .....................................................................................................................................................................................25
DUE PROCESS ..................................................................................................................................................................................................................... 25
TAKING VIA EMINENT DOMAIN VS. TAKING UNDER SOCIAL JUSTICE CLAUSE .................................................................................................... 25
EXPANSIVE CONCEPT OF PUBLIC USE ........................................................................................................................................................................26
JUST COMPENSATION ........................................................................................................................................................................................................26
DETERMINATION ................................................................................................................................................................................................................. 26
NON-PAYMENT OF JUST ......................................................................................................................................................................................................27
EFFECT OF DELAY ...............................................................................................................................................................................................................27
ABANDONMENT OF INTENDED USE AND RIGHT OF REPURCHASE ............................................................................................................................. 27
MISCELLANEOUS APPLICATION ........................................................................................................................................................................................27

CASES
REPUBLIC OF THE PHILIPPINES VS. VDA. DE CASTELLVI ....................................................................................................................................28
DIDIPIO EARTH SAVERS MULTIPURPOSE ASSOCIATION ET AL VS DENR SEC ELISEA GOZUN ET AL ............................................................29
MANOSCA VS. COURT OF APPEALS .......................................................................................................................................................................31
REPUBLIC VS. PHILIPPINE LONG DISTANCE TELEPHONE CO. ............................................................................................................................33
FILSTREAM INTERNATIONAL INC. ............................................................................................................................................................................35
PEOPLE VS. FAJARDO ...............................................................................................................................................................................................37
REPUBLIC VS. CA .......................................................................................................................................................................................................38

DUE PROCESS AND EQUAL PROTECTION OF LAWS CLAUSE


REQUISITES FOR VALID EXERCISE OF INHERENT POWERS ......................................................................................................................................... 42
GIVE THE TWO ASPECTS OF DUE PROCESS AND DISCUSS EACH ...............................................................................................................................42
DO LIFE AND PROPERTY ENJOY IDENTICAL PROTECTION FROM THE CONSTITUTION? ..........................................................................................42
DOES EQUAL PROTECTION OF LAW PROHIBIT CLASSIFICATION? ...............................................................................................................................42
GIVE THE MEANING/RELATIVITY OF DUE PROCESS ACCORDING TO THE CASE OF US V. LING SU FAN ................................................................ 42
GIVE THE ESSENTIALS OF PROCEDURAL DUE PROCESS IN JUDICIAL PROCEEDINGS ACCORDING TO THE CASE OF BANCO ESPANOL V.
PALANCA....................................................................................................................................................................................................................................... 43
GIVE THE CARDINAL PRIMARY REQUIREMENTS OF DUE PROCESS IN ADMINISTRATIVE PROCEEDINGS .............................................................43
GIVE THE MINIMUM STANDARS WHICH MUST BE MET BY THE SCHOOLS TO SATISFY THE DEMANDS OF PROCEDURAL DUE PROCESS .......43
CONSTITUTIONAL AND STATUTORY DUE PROCESS .......................................................................................................................................................43
EXPLAIN VOID FOR VAGUENESS DOCTRINE ...................................................................................................................................................................43
CONCEPT OF EQUAL PROTECTION .................................................................................................................................................................................. 44
STANDARDS OF JUDICIAL REVIEW ON EQUAL PROTECTION ........................................................................................................................................ 44
RATIONAL BASIS TEST .........................................................................................................................................................................................................44
STRICT SCRUTINY TEST ......................................................................................................................................................................................................44
INTERMEDIATE SCRUTINY TEST .......................................................................................................................................................................................44

CASES
ALFONSO C. BINCE, JR VS. COMMISSION ON ELECTIONS, PROVINCIAL BOARD OF CANVASSERS OF PANGASINAN, MUNICIPAL
BOARDS OF CANVASSERS OF TAYUG AND SAN MANUEL, PANGASINAN, AND EMILIANO MICU .....................................................................45

Page 4 of 236
Constitutional Law 2 Bill of Rights Armando Santiago Jr

TABUENA V. SANDIGANBAYAN ..................................................................................................................................................................................46


SON UN GIOK VS MATUSA ........................................................................................................................................................................................47
PEOPLE VS. JUDGE VERA .........................................................................................................................................................................................48
PEDRO VS PROVINCIAL BOARD OF RIZAL .............................................................................................................................................................50
ROLITO GO VS CA ...................................................................................................................................................................................................... 51
SUNTAY V. PEOPLE..................................................................................................................................................................................................... 52
PEOPLE VS. CAYAT..................................................................................................................................................................................................... 53
ISAE V QUISIMBING ....................................................................................................................................................................................................55
HELD ............................................................................................................................................................................................................................56
ORMOC SUGAR COMPANY INC. VS ORMOC CITY ET AL .......................................................................................................................................57
JAMES MIRASOL VS DEPARTMENT OF PUBLIC WORKS AND HIGHWAYS ..........................................................................................................58

Search and seizures


SECTION 2. DIGESTED PROVISION ....................................................................................................................................................................................60
REQUISITES OF A VALID WARRANT: .................................................................................................................................................................................. 60

Warrantless Searches
IS EVERY WARRANTLESS SEARCH AN ILLEGAL SEARCH? ............................................................................................................................................ 61
EXAMINATION OF APPLICATION (RULE 126, SECTION 4 OF THE RULES OF COURT) .................................................................................................62
PARTICULARITY OF PRESCRIPTION .................................................................................................................................................................................. 62
PROPERTIES SUBJECT TO SEIZURE (RULE 126, SECTION 2 OF THE RULES OF COURT) ......................................................................................... 62
ADMISSIBILITY OF ILLEGALLY SEIZED EVIDENCE (EXCLUSIONARY RULE) ART. III, SECTION 3 PARAGRAPH 2 .....................................................63
DO THE ORDINARY RIGHTS AGAINST UNREASONABLE SEARCHES AND SEIZURES APPLY TO SEARCHES CONDUCTED AT THE AIRPORT
PURSUANT TO ROUTINE AIRPORT SECURITY PROCEDURES? ............................................................................................................................................ 63

Warrantless Arrest
RULE 113 ARREST ................................................................................................................................................................................................................ 64
BOND (PYANSADOR) ............................................................................................................................................................................................................65
WHO MAY ISSUE WARRANT OF ARREST...........................................................................................................................................................................66
ADMINISTRATIVE ARRESTS ............................................................................................................................................................................................... 66
SUBJECT OF THE OFFENSE;............................................................................................................................................................................................... 66

CASES
PEOPLE VS. ANDRE MARTI .......................................................................................................................................................................................67
STONEHILL VS. DIOKNO ............................................................................................................................................................................................68
PEOPLE VS. SARAP....................................................................................................................................................................................................69
ALIB VS. JUDGE LABAYEN .........................................................................................................................................................................................71
PADILLA VS CA ............................................................................................................................................................................................................74

Privacy of communication and correspondence


SECTION 3. DIGESTED PROVISION ....................................................................................................................................................................................78
WHAT IS THE WRIT OF HABEAS DATA? ............................................................................................................................................................................ 78
WHO MAY FILE A PETITION FOR THE ISSUANCE OF A WRIT OF HABEAS DATA?......................................................................................................... 78
WHERE CAN THE PETITION BE FILED? ............................................................................................................................................................................. 79

Right to privacy
DISCOVERY AND REVELATION OF SECRETS (CHAPTER THREE) .................................................................................................................................80

Freedom of expression
EXPLAIN PRIOR RESTRAINT AND SUBSEQUENT PUNISHMENT ....................................................................................................................................81
4 ASPECTS OF FREEDOM OF PRESS ................................................................................................................................................................................ 81
TESTS: EXPLAIN DANGEROUS TENDENCY RULE, CLEAR AND PRESENT DANGER, AND THE BALANCING OF INTEREST TEST ........................81
DANGEROUS TENDENCY RULE .........................................................................................................................................................................................81
CLEAR AND PRESENT DANGER RULE............................................................................................................................................................................... 81
BALANCING OF INTEREST TEST ........................................................................................................................................................................................81
FACIAL CHALLENGE AND THE OVER BREADTH DOCTRINE ........................................................................................................................................... 82
COMMERCIAL SPEECH ........................................................................................................................................................................................................82
PRIVATE VS. GOVERNMENT SPEECH................................................................................................................................................................................ 82

CASES
PRIMICIAS VS. FUGOSO ...........................................................................................................................................................................................82
MIRIAM COLLEGE FOUNDATION INC. VS. COURT OF APPEALS ..........................................................................................................................84
FRANCISCO CHAVEZ V. RAUL M. GONZALES .........................................................................................................................................................85
BAYAN, ET AL., VS. EDUARDO ERMITA ....................................................................................................................................................................88
BURGOS, SR. VS. CHIEF OF STAFF, AFP ................................................................................................................................................................. 89

Freedom of Religion (SEPARATION OF CHURCH AND STATE)


SUPPORTED PROVISIONS .................................................................................................................................................................................................. 92
FREE EXERCISE CLAUSE.................................................................................................................................................................................................... 92
BENEVOLENT NEUTRALITY ................................................................................................................................................................................................ 93
BALANCING OF INTERESTS ............................................................................................................................................................................................93
STRICT NEUTRALITY ............................................................................................................................................................................................................93

CASES
TARUC VS. DELA CRUZ ..............................................................................................................................................................................................93
AGLIPAY VS. RUIZ .......................................................................................................................................................................................................94

Page 5 of 236
Constitutional Law 2 Bill of Rights Armando Santiago Jr

GERMAN VS. BARANGAN ..........................................................................................................................................................................................95


EBRALINAG ET AL VS. DIV. SUPT. OF SCHOOL OF CEBU .....................................................................................................................................97
(OTHER DIGEST VERSION) EBRALINAG ET AL VS. DIV. SUPT. OF SCHOOL OF CEBU.......................................................................................99
GERONA VS. SEC OF EDUCATION .........................................................................................................................................................................102

Liberty of Abode and Freedom of Movement


THE LIMITATIONS ON THE RIGHT TO TRAVEL ..................................................................................................................................................................105
RIGHTS GUARANTEED UNDER SECTION 6 .......................................................................................................................................................................105
RIGHT CURTAILMENT/LIMITATIONS OF RIGHT ................................................................................................................................................................. 105
FREEDOM OF MOVEMENT: LIBERTY OF ABODE AND OF TRAVEL .................................................................................................................................105
THE LIBERTY GUARANTEED BY THIS PROVISION INCLUDES ........................................................................................................................................ 105
ALIENS AND RIGHT TO ENTRY............................................................................................................................................................................................106

CASES
VILLAVICENCIO V LUKBAN ......................................................................................................................................................................................107
MARCOS,PETITIONERVS.MANGLAPUS,RESPONDENT (PART 2) ....................................................................................................................110
MARCOS,PETITIONERVS.MANGLAPUS (PART 2) ..............................................................................................................................................112
DEFENSOR-SANTIAGO VS. VASQUEZ, 217 SCRA 633 (1993)...............................................................................................................................116

Right of Association
RELATED PROVISIONS ........................................................................................................................................................................................................118
LABOR UNIONISM ................................................................................................................................................................................................................ 118
COMMUNIST AND SIMILAR ORGANIZATIONS ...................................................................................................................................................................118
INTEGRATED BAR OF THE PHILIPPINES ..........................................................................................................................................................................119

CASES
PEOPLE VS. FERRER ...............................................................................................................................................................................................120
BANGALISAN VS. HON. COURT OF APPEALS .......................................................................................................................................................124
UNITED PEPSI-COLA SUPERVISORY UNION (UPSU) VS HON. BIENVENIDO E. LAGUESMA ...........................................................................127

Contract Clause / Non-Impairment clause


REQUISITES OF VALID IMPAIRMENT.................................................................................................................................................................................. 130
WHEN DOES, A LAW IMPAIR THE OBLIGATION OF CONTRACTS? ..................................................................................................................................130
WHEN NON-IMPAIRMENT CLAUSE PREVAILS ...................................................................................................................................................................130
APPLICATION OF THE CONTRACT CLAUSE .....................................................................................................................................................................130
CONTEMPORARY APPLICATION OF THE CONTRACT CLAUSE ......................................................................................................................................131
LIMITATIONS .........................................................................................................................................................................................................................132
NATURE OF PROTECTION ................................................................................................................................................................................................... 132
CONTRACTS AFFECTED ......................................................................................................................................................................................................132
LIMITATIONS ..........................................................................................................................................................................................................................132
EFFECT OF EMERGENCY LEGISLATION ON CONTRACTS.............................................................................................................................................. 132
CURRENCY LEGISLATION AND CONTRACTS ....................................................................................................................................................................132
IMPAIRMENT ..........................................................................................................................................................................................................................133

CASES
ORTIGAS & COMPANY VS COURT OF APPEALS ...................................................................................................................................................135
MARIVELES VS. CA................................................................................................................................................................................................... 137
CLEMONS VS. NOLTING ...........................................................................................................................................................................................139
PHILRECA VS DILG ................................................................................................................................................................................................... 140
PHILRECA CASE WAS EXPLAINED BY JUDGE REYES .....................................................................................................................................................140

Access/Right to information
SUBIDO V. OZAETA ...............................................................................................................................................................................................................143
SCOPE .................................................................................................................................................................................................................................. 144
LIMITATIONS .........................................................................................................................................................................................................................144
RIGHT TO INFORMATION RELATIVE TO GOVERNMENT CONTRACT NEGOTIATIONS ................................................................................................145
DIPLOMATIC NEGOTIATIONS .............................................................................................................................................................................................145
COURT HEARINGS ...............................................................................................................................................................................................................145

CASES
CHAVEZ VS. PUBLIC ESTATES AUTHORITY ..........................................................................................................................................................146

Page 6 of 236
Constitutional Law 2 Bill of Rights Armando Santiago Jr

CHAVES CASE WAS EXPLAINED BY JUDGE REYES ........................................................................................................................................................146

Legal Assistance and Free Access to Courts


RA 9406 (PUBLIC ATTORNEYS OFFICE LAW - PAO) .......................................................................................................................................................... 152
QUALIFIED FOR PAO SERVICE (LIMITATION) ....................................................................................................................................................................152
MERIT TEST ...........................................................................................................................................................................................................................152

Assistance of Counsel
ELEMENTS OF THE RIGHT TO COUNSEL .........................................................................................................................................................................154

RIGHT TO BAIL
EXCEPTION TO THE RIGHT TO BAIL .................................................................................................................................................................................. 155
FORMS OF BAIL .................................................................................................................................................................................................................... 155
PRESUMPTION OF INNOCENCE .........................................................................................................................................................................................155
RIGHT TO BE HEARD............................................................................................................................................................................................................156
ASSISTANCE OF COUNSEL ................................................................................................................................................................................................. 156
TRIAL IN ABSENTIA ...............................................................................................................................................................................................................156
REQUISITE OF TRIAL IN ABSENTIA ....................................................................................................................................................................................156
RIGHT TO CONFRONTATION ............................................................................................................................................................................................... 157
COMPULSORY PROCESS .................................................................................................................................................................................................... 157
PROHIBITED PUNISHMENT ................................................................................................................................................................................................. 157

CASES
PEOPLE VS. ORDOO ................................................................................................................................................................................................158
DOCENA-CASPE VS BUGTAS .................................................................................................................................................................................. 158
COMENDADOR VS. DE VILLA .................................................................................................................................................................................. 159
PEOPLE VS. RIVERA ................................................................................................................................................................................................161
HONG KONG V. OLALIA ............................................................................................................................................................................................163
GOVERNMENT OF HONG KONG VS. OLALIA (OTHER VERSION) .......................................................................................................................166
GOVERNMENT OF THE UNITED STATES OF AMERICA HON. GUILLERMO G. PURGANAN, MORALES, AND PRESIDING JUDGE, REGIONAL
TRIAL COURT OF MANILA AND MARK B. JIMENEZ A.K.A. MARIO BATACAN CRESPO ......................................................................................167

RIGHT OF THE ACCUSED


Right to be Heard
Right to be Informed
RIGHT TO SPEEDY, IMPARTIAL AND PUBLIC TRIAL .........................................................................................................................................................172
WAIVER OF PRESENCE .......................................................................................................................................................................................................172
IDENTIFICATION (TIME WHEN THEY CANNOT WAIVED THEIR RIGHT TO BE PRESENT) ............................................................................................172
TRIAL MUST BE PUBLIC .......................................................................................................................................................................................................173
IMPARTIAL TRIAL: ................................................................................................................................................................................................................ 173

Right of Confrontation
THIS IS THE BASIS OF THE RIGHT TO CROSS-EXAMINATION. ......................................................................................................................................174

Trials In Absentia
WHEN CAN TRIAL IN ABSENTIA BE DONE .........................................................................................................................................................................174
WHEN PRESENCE OF THE ACCUSED IS A DUTY ............................................................................................................................................................. 174

Writ of Habeas Corpus


HABEAS CORPUS ................................................................................................................................................................................................................ 175
A PRIME SPECIFICATION OF AN APPLICATION FOR A WRIT OF HABEAS CORPUS IS RESTRAINT OF LIBERTY. .................................................... 175
WRIT SUSPENDED VS. PRIVILEGE OF THE WRIT WAS SUSPENDED ............................................................................................................................ 176
WHAT IS A WRIT OF HABEAS CORPUS? ............................................................................................................................................................................ 177
WHAT IS THE SCOPE OF THE WRIT OF HABEAS CORPUS? ........................................................................................................................................... 177
WHO MAY ISSUE THE WRIT OF HABEAS CORPUS?.........................................................................................................................................................177
TO WHOM IS THE WRIT DIRECTED? .................................................................................................................................................................................. 178
WHEN SHALL THE SUBJECT PERSON BE RELEASED? ...................................................................................................................................................178

CASES
SUMMARY ..................................................................................................................................................................................................................178
MALIWAT VS. COURT OF APPEALS AND REPUBLIC OF THE PHILIPPINES........................................................................................................179
GIMENEZ AND MERCADO VS. HON. NAZARENO AND DE LA VEGA, JR .............................................................................................................181
PEOPLE VS. BUENVIAJE ..........................................................................................................................................................................................183
CORPUZ VS. PEOPLE ...............................................................................................................................................................................................185
PEOPLE VS MONTERON ..........................................................................................................................................................................................188
ESTRADA VS SANDIGANBAYAN ..............................................................................................................................................................................191
PEOPLE VS. PURISIMA ............................................................................................................................................................................................197
BARCELON VS.BAKER, JR., AND THOMPSON .......................................................................................................................................................199
IN THE ISSUANCE OF THE WRIT OF HABEAS CORPUS FOR DR. PARONG ET. AL. VS. ENRILE ET. AL. .........................................................201
LIM VS. CA .................................................................................................................................................................................................................204
IN THE MATTER OF THE PETITION FOR HABEAS CORPUS OF LANSANG VS.GARCIA....................................................................................207

Page 7 of 236
Constitutional Law 2 Bill of Rights Armando Santiago Jr

Right against self-incrimination


JUSTIFICATION OF GUARANTEE: [US VS. NAVARRO] ......................................................................................................................................................219
PURPOSE ..............................................................................................................................................................................................................................219
WHO MAY DETERMINE: INCRIMINATING ...........................................................................................................................................................................219
SCOPE ................................................................................................................................................................................................................................... 219
NOT AN ABSOLUTE RIGHT .................................................................................................................................................................................................. 220
WHEN TO RAISE THE PRIVILEGE ......................................................................................................................................................................................220
WHEN AVAILABLE TO INVOKE THE PRIVILEGE AGAINST SELF-INCRIMINATION.......................................................................................................... 220
PROHIBITION DOES NOT PRECLUDE A BODY CHECKDUP; MECHANICAL ACTS ........................................................................................................ 220
HANDWRITING ..................................................................................................................................................................................................................... 220
WAIVER OF RIGHT ...............................................................................................................................................................................................................221
DOCUMENTS AND RECORDS .............................................................................................................................................................................................221

CASES
CASE CONTENT FORMAT ........................................................................................................................................................................................221
PEOPLE VS. GALLARDE ...........................................................................................................................................................................................221
REGALA VS.SANDIGANBAYAN ................................................................................................................................................................................224
SEPARATE OPINION ................................................................................................................................................................................................227
DISSENTING OPINION ..............................................................................................................................................................................................227
BELTRAN VS. SAMSON ............................................................................................................................................................................................228
ROSETE VS JULIANO LIM ........................................................................................................................................................................................231

Page 8 of 236
Constitutional Law 2 Bill of Rights Armando Santiago Jr

Page 9 of 236
Constitutional Law 2 Bill of Rights Armando Santiago Jr

BILL OF RIGHTS
set of prescriptions setting forth the fundamental civil and political rights of the individual, and imposing
limitations on the powers of the government as a means of securing the enjoyment of those rights.

SIGNIFICANCE OF THE BILL OF RIGHTS


Government is powerful. When unlimited, it becomes tyrannical. The Bill of Rights is a guarantee that there are
certain areas of a person's life, liberty, and property which governmental power may not touch.

Bill of Rights are generally self-implementing.

CLASSIFICATION OF RIGHTS
Political Rights granted by law to members of community in relation to their direct or indirect participation in the
establishment or administration of the government;

Civil Rights rights which municipal law will enforce at the instance of private individuals for the purpose of
securing them the enjoyment of their means of happiness;

Social and Economic Rights; and, Human Rights.

THE INHERENT POWERS OF THE


STATE
1. Police Power
2. Power of Eminent Domain
3. Power of Taxation

They are inherent powers because they belong to the very essence of government and without them no
government can exist.

SIMILARITIES, DISTINCTIONS, AND LIMITATION


Similarities:

they are inherent in the State


they are necessary and indispensable
they are methods by which the State interferes with private rights
they presuppose an equivalent compensation
they are exercised primarily by the legislature

Differences:
Police Power Eminent Domain Taxation

As to regulation regulates both liberty and regulates property rights regulates property rights
property only only
As to who may exercise only the government government and some only the government
private entities
As to the property taken destroyed because it is -wholesome -wholesome
noxious or intended for -taken for a public use or -taken for a public use or
noxious purpose purpose purpose
Page 10 of 236
Constitutional Law 2 Bill of Rights Armando Santiago Jr

As to Compensation intangible altruistic full and fair equivalent of protection and public
feeling that the person the property expropriated improvements for the
has contributed to the taxes paid
general welfare

Limitations:

Bill of Rights

Page 11 of 236
Constitutional Law 2 Bill of Rights Armando Santiago Jr

FUNDAMENTAL POWERS OF THE STATE (POLICE POWER, EMINENT DOMAIN,


TAXATION)

POLICE POWER
is the most essential, insistent and the least limitable of powers, extending as it does to all the great public
needs. It is the power vested in the legislature by the constitution to make, ordain, and establish all manner of
wholesome and reasonable laws, statutes, and ordinances, either with penalties or without not repugnant to the
constitution, as they shall judge to be for the good and welfare of the commonwealth, and of the subject of the
same.

SCOPE: Police power rests upon public necessity and upon the right of the state and of the public to self-
protection. it is vested to the legislative department basta kapag may batas na pinaguusapan kapag na-violate
mo yung batas na ginawa ng legislative department maga-apply ngayon ang police power.. (ex. na gawa ng
legislative yung labor code, civil code, revised penal code and etc.. maraming gawa ang legislative dahil sila
talaga ang gumagawa ng mga batas therefore, mostly papasok si police power especially kapag violation
against public moral, public health, general welfare and etc.)

REQUISITES FOR VALID EXERCISE POLICE POWER

Tests for Validity of Exercise of Police Power

LAWFUL SUBJECT
Interest of the general public (as distinguished from a particular class required exercise). This means that the
activity or property sought to be regulated affects the general welfare. [see Taxicab Operators v. Board of
Transportation, 119 SCRA 597]

LAWFUL MEANS
Means employed are reasonably necessary for the accomplishment of the purpose, and are not unduly
oppressive. [see Tablarin v. Gutierrez, 152 SCRA 730]

Least restrictions of individual rights.

Additional Limitations when police power is delegated


1. Express grant by law [e.g. Secs. 16, 391, 447, 458 and 468, R.A. 7160, for LGUs]
2. Limited within its territorial jurisdiction [for local government units]
3. Must not be contrary to law.

TO WHAT AREAS OF LIEE POLICE POWER HAS PERVASIVE REACH?


Police power has been used to justify enactments in the fields of:

Public health measures like make house repairs; compulsory connection to sewerage system; license to
practice medicine; regulation of cattle imports; sale of meat.

Public safety measures like building regulations; regulation of carrying deadly weapons; participation in
rotational patrol duty; regulation of gasoline stations and movie theaters; use of city roads.

Public moral like regulating the operation of public dance halls; prohibiting gambling; licensing of cock-pits;
prohibiting the operation of pinball machines; regulating the operation of motels and hotel; regulating
establishment of massage parlors.

Page 12 of 236
Constitutional Law 2 Bill of Rights Armando Santiago Jr

General (public) welfare like regulating slaughter of carabaos; provisions for the suppression of agricultural
pests; regulating nuisances; rules for the deportation of aliens; regulating building construction; prescribing
registration of land under the Torrens System; zoning regulations; anti-graft laws designed to curb activities of
public officials

Page 13 of 236
Constitutional Law 2 Bill of Rights Armando Santiago Jr

cases

ORTIGAS & CO. V. CA 1


Ponencia: Quisimbing J.
Topic: Police Power
Trigger of facts:
Triger of issue:
Trigger of Ruling:
Verdict: Instant petition was denied. Affirmed CA decision
End point of the case:

FACTS
Ortigas & co. sold to Emilia Hermoso a parcel of land located in greenhills with several restrictions in the
contract of sale that the said lot be used exclusively for residential purposes
A zoning ordinance was issued by MMC reclassifying the area as commercial
Private respondent leased the subject lot from hermoso and built a single storey building for greenhils
autohouse sale company
Filed a complaint which sought the demolition of the constructed sale company o against Hermosa as it
violated the terms and conditions of the deed of sale
Trial court ruled in favor of Ortigas & Co. Mathay raised the issue to the court of appeals from which he sought
favorable ruling

ISSUE
Whether or not the zoning ordinance may impair contracts entered prior to its effectivity.

HELD
Yes. The zoning ordinance, as a valid exercise of police power may be given effect over any standing contract.
Hence, petition is denied.

RATIO
A law enacted in the exercise of police power to regulate or govern certain activities or transactions could be
given retroactive effect and may reasonably impair vested rights or contracts. (TARGET OF THE CASE IN
RELATION TO THE TOPIC)

Police power legislation is applicable not only to future contracts, but equally to those already in existence.
Non-impairment of contracts or vested rights clauses will have to yield to the superior and legitimate exercise by
the State of police power to promote the health, morals, peace, education, good order, safety, and general
welfare of the people.

Moreover, statutes in exercise of valid police power must be read into every contract. Noteworthy, in Sangalang
vs. Intermediate Appellate Court, the Supreme Court already upheld subject ordinance as a legitimate police
power measure.

Police power may be applied retroactively (contract vs. Police power = Police power may prevail)

SYLLABI

1 G.R. No. 126102. December 4, 2000


Page 14 of 236
Constitutional Law 2 Bill of Rights Armando Santiago Jr

Contracts; Contract Clause; Police Power; Statutes; A later law which enlarges, abridges, or in any manner
changes the intent of the parties to the contract necessarily impairs the contract itself and cannot be given
retroactive effect without violating the constitutional prohibition against impairment of contracts.We agree that
laws are to be construed as having only prospective operation. Lex prospicit, non respicit. Equally settled, only
laws existing at the time of the execution of a contract are applicable thereto and not later statutes, unless the
latter are specifically intended to have retroactive effect. A later law which enlarges, abridges, or in any manner
changes the intent of the parties to the contract necessarily impairs the contract itself and cannot be given
retroactive effect without violating the constitutional prohibition against impairment of contracts.

PHILIPPINE PRESS INSTITUTE, INC., VS. COMMISSION ON


ELECTIONS2
Ponencia: FELICIANO J.
Topic: POLICE POWER
Trigger of facts: Comelec promulgated a resolution which directing the petitioner to allocate space to their
newspaper for the public use without just compensation.
Trigger of issue: WON Comelec resolution is unconstitutional
Trigger of Ruling: Yes, Unconstitutional for it is not a valid exercise of police power. Use of space is not urgent
which means theres no need for taking property or thing without just compensation
Verdict:
End point of the case: Police power applied in the case at the bar when there is an urgency which requires the use
of newspaper. In this case, there was no showing of urgency in mandating a private sector to give space for public
use. Intent of comelec was inconsistent with the words which is directive and not appealing as asking for a donation
from the petitioner.

FACTS
Respondent Comelec promulgated Resolution No. 2772
directing newspapers to provide free Comelec space of not less than one-half page for the common use of
political parties and candidates.
The Comelec space shall be allocated by the Commission, free of charge, among all candidates to enable
them to make known their qualifications, their stand on public Issue and their platforms of government.
The Comelec space shall also be used by the Commission for dissemination of vital election information
Petitioner Philippine Press Institute, Inc. (PPI), a non-profit organization of newspaper and magazine
publishers, asks the Supreme Court to declare Comelec Resolution No. 2772 unconstitutional and void
on the ground that it violates the prohibition imposed by the Constitution upon the government against the
taking of private property for public use without just compensation.
On behalf of the respondent Comelec, the Solicitor General claimed that the Resolution is a permissible
exercise of the power of supervision (police power) of the Comelec over the information operations of print
media enterprises during the election period to safeguard and ensure a fair, impartial and credible election.

ISSUE
Whether or not Comelec Resolution No. 2772 is unconstitutional.


HELD
The Supreme Court declared the Resolution as unconstitutional. It held that to compel print media companies to
donate Comelec space amounts to taking of private personal property without payment of the just
compensation required in expropriation cases. Moreover, the element of necessity for the taking has not been

2 G.R. No. L-119694 May 22, 1995


Page 15 of 236
Constitutional Law 2 Bill of Rights Armando Santiago Jr

established by respondent Comelec, considering that the newspapers were not unwilling to sell advertising
space. The taking of private property for public use is authorized by the constitution, but not without payment of
just compensation. Also, Resolution No. 2772 does not constitute a valid exercise of the police power of the
state. In the case at bench, there is no showing of existence of a national emergency to take private property of
newspaper or magazine publishers.

VICENTE DE LA CRUZ VS EDGARDO PARAS3


Ponencia: FERNANDO, C.J
Topic: Police power / Public Policy
Trigger of facts: Ordinance were promulgated to prohibit
Triger of issue: WON Ordinance is a constitutional
Trigger of Ruling: Ordinance was unconstitutional
Verdict: Writ is granted, RTC ruling was SET ASIDE, Ord. 84 is declared void and unconstitutional, TRO issued
hereby made permanent
End point of the case: When the act is legal/lawful, the municipality can do only is to regulate and not to prohibit

Subject Shall Be Expressed in the Title Police Power Not Validly Exercise

FACTS
Vicente De La Cruz et al were club & cabaret operators. They assail the constitutionality of Ord. No. 84, Ser.
of 1975 or the Prohibition and Closure Ordinance of Bocaue, Bulacan.
De la Cruz averred that the said Ordinance violates their right to engage in a lawful business for the
said ordinance would close out their business.
That the hospitality girls they employed are healthy and are not allowed to go out with customers.
Judge Paras however lifted the TRO he earlier issued against Ord. 84 after due hearing declaring that Ord 84.
is constitutional for it is pursuant to RA 938 which reads AN ACT GRANTING MUNICIPAL OR CITY
BOARDS AND COUNCILS THE POWER TO REGULATE THE ESTABLISHMENT, MAINTENANCE AND
OPERATION OF CERTAIN PLACES OF AMUSEMENT WITHIN THEIR RESPECTIVE TERRITORIAL
JURISDICTIONS.
Paras ruled that the prohibition is a valid exercise of police power to promote general welfare.
De la Cruz then appealed citing that they were deprived of due process.

ISSUE

Whether or not a municipal corporation, Bocaue, Bulacan can, prohibit the exercise of a lawful trade, the
operation of night clubs, and the pursuit of a lawful occupation, such clubs employing hostesses pursuant to Ord
84 which is further in pursuant to RA 938.

HELD
The SC ruled against Paras.

If night clubs were merely then regulated and not prohibited, certainly the assailed ordinance would pass the
test of validity.
The cabaret followed the test provided, which tells that they should not be prohibit
SC had stressed reasonableness, consonant with the general powers and purposes of municipal
corporations, as well as consistency with the laws or policy of the State.
It cannot be said that such a sweeping exercise of a lawmaking power by Bocaue could qualify under the term
reasonable.

3 G.R. No. L-42571-72 July 25, 1983


Page 16 of 236
Constitutional Law 2 Bill of Rights Armando Santiago Jr

The objective of fostering public morals, a worthy and desirable end can be attained by a measure that does
not encompass too wide a field.
Certainly, the ordinance on its face is characterized by overbreadth.
The purpose sought to be achieved could have been attained by reasonable restrictions rather than by an
absolute prohibition.
Pursuant to the title of the Ordinance, Bocaue should and can only regulate not prohibit the business of
cabarets.

LUCENA GRAND CENTRAL TERMINAL, INC., VS. JAC LINER,


INC.,
Ponencia: CARPIO MORALES J.
Topic: Police Power / Public Welfare
Trigger of facts: There was a resolution for the decongest the traffic through having a one terminal in lucena city
Triger of issue: WON the said ordinance is unconstitutional
Trigger of Ruling: The court ruled that it was not a valid exercise of police power for it did not satisfy the
requirements of lawful means and lawful subject
Verdict: Petition was DENIED
End point of the case:

FACTS
The city of lucena enacted an ordinance which provides that all busses, mini buses and out of town
passenger jeepneys
Shall be prohibited from entering the city and are hereby directed to proceed to the common terminal for
picking up and or dropping of their passenger
All temporary terminals in the city of lucena are hereby declared inoperable starting from the effectivity of this
ordinances
It also provides that all jeepneys, mini busses and busses shall use the grand central terminal of the city.
Jac liner assailed the city ordinances as constitutional
Undue taking of private property
Violation of the constitutional prohibition against monopolies

ISSUE
WON the ordinance satisfice the requisite of valid exercise of police power lawful subject and lawful means

HELD
The local government may be considered as having properly exercised its police power only if the following
requisites are met:

LAWFUL SUBJECT
(1) the interests of the public generally, as distinguished from those of a particular class, require the interference
of the State, and

LAWFUL MEANS
(2) the means employed are reasonably necessary for the attainment of the object sought to be accomplished
and not unduly oppressive upon individuals.

Otherwise stated, there must be a concurrence of a lawful subject and lawful method

The questioned ordinances having been enacted with the objective of relieving traffic congestion in the City of
Lucena, they involve public interest warranting the interference of the State.
Page 17 of 236
Constitutional Law 2 Bill of Rights Armando Santiago Jr

The first requisite for the proper exercise of police power is thus present hich is to invoke public interest
warranting the interference of the state. This leaves for determination the issue of whether the means employed
by the Lucena Sangguniang Panlungsod to attain its professed objective were reasonably necessary and not
unduly oppressive upon individuals.

The ordinances assailed herein are characterized by overbreadth. They go beyond what is reasonably
necessary to solve the traffic problem. Additionally, since the compulsory use of the terminal operated by
petitioner would subject the users thereof to fees, rentals and charges, such measure is unduly
oppressive, as correctly found by the appellate court. What should have been done was to determine exactly
where the problem lies and then to stop it right there.

The true role of Constitutional Law is to effect an equilibrium between authority and liberty so that rights are
exercised within the framework of the law and the laws are enacted with due deference to rights. It is its
reasonableness, not its effectiveness, which bears upon its constitutionality.

If the constitutionality of a law were measured by its effectiveness, then even tyrannical laws may be justified
whenever they happen to be effective.

ERMITA-MALATE HOTEL & MOTEL OPERATORS ASSOC.,


INC VS MAYOR OF MANILA
PONENCIA:
TOPIC: POLICE POWER
TRIGGER OF THE FACTS: Ordince was alleged to be vague for it is arbitrary and against due process because of
this ordinance they raised a standards and implementing rules to protect the general welfare from prostitution
TRIGGER OF THE ISSUE: is the ordinance valid?
TRIGGER OF THE RULING: yes it is valid. for the purpose is to scrub down the prostitution
VERDICT: POLICE power won
END POINT: ordinance was made to scrub down the prostitution and which solely vested in the congress to apply
police power.

POLICE POWER DUE PROCESS CLAUSE


FACTS
the Manila Municipal Board enacted Ord 4760 and the same was approved by then acting mayor Astorga.
Ordinance 4760 sought to regulate hotels and motels.
It classified them into 1st class (taxed at 6k/yr) and 2nd class (taxed at 4.5k/yr).
It also compelled hotels/motels to get the demographics of anyone who checks in to their rooms.
It compelled hotels/motels to have wide open spaces so as not to conceal the identity of their patrons.
Ermita-Malate impugned the validity of the law averring that such is oppressive, arbitrary and against due
process.
The lower court as well as the appellate court ruled in favor of Ermita-Malate.

ISSUE
Whether or not Ord 4760 is against the due process clause.

HELD
The SC ruled in favor of Astorga.
There is a presumption that the laws enacted by Congress (in this case Mun Board) is valid.
W/o a showing or a strong foundation of invalidity, the presumption stays.
As in this case, there was only a stipulation of facts and such cannot prevail over the presumption.
Further, the ordinance is a valid exercise of Police Power.
Page 18 of 236
Constitutional Law 2 Bill of Rights Armando Santiago Jr

There is no question but that the challenged ordinance was precisely enacted to minimize certain practices
hurtful to public morals. Intent of the ordinance
This is to minimize prostitution.
The increase in taxes not only discourages hotels/motels in doing any business other than legal but also
increases the revenue of the LGU concerned.
And taxation is a valid exercise of police power as well.
The due process contention is likewise untenable, due process has no exact definition but has reason as a
standard.
In this case, the precise reason why the ordinance was enacted was to curb down prostitution in the
city which is reason enough and cannot be defeated by mere singling out of the provisions of the said
ordinance alleged to be vague.

SYLLABI
Same; Police power; Ordinance regulating hotels, motels, etc.A Manila ordinance regulating the operation of
hotels, motels and lodging-houses is a police power measure specifically aimed to safeguard public morals. As
such, it is immune from any imputation of nullity resting purely on conjecture and unsupported by anything of
substance. To hold otherwise would be to unduly restrict and narrow the scope of police power which has been
properly characterized as the most essential, insistent and the least limitable of powers, extending as it does "to
all the great public needs.

Same; Nature of police power; Judicial inquiry.On the legislative organs of the government, whether national
or local, primarily rests the exercise of the police power, which is the power to prescribe regulations to promote
the health, morals, peace, good order, safety and general welfare of the people. In view of the requirements of
certain constitutional guarantees, the exercise of such police power, however, insofar as it may affect the life,
liberty or property of any person, is subject to judicial inquiry. Where such exercise of police power may be
considered as either capricious, whimsical, unjust or unreasonable, a denial of due process or a violation of any
other applicable constitutional guarantee may call for correction by the courts.

Same; Reasonableness of ordinance regulating hotels, etc. The provision in Ordinance No. 4760 of the City of
Manila, making it unlawful for the owner, manager, keeper or duly authorized representative of any hotel, motel,
lodging house, tavern, common inn or the like, to lease or rent any room or portion thereof more than twice every
24 hours, with a proviso that in all cases full payment shall be charged, cannot be viewed as a transgression
against the command of due process. The prohibition is neither unreasonable nor arbitrary, because there
appears a correspondence between the undeniable existence of an undesirable situation and the legislative
attempt at correction. Moreover, every regulation of conduct amounts to curtailment of liberty, which cannot be
absolute.

Same; Public interest; Government interference.The policy of laissez faire has to some extent given way to the
assumption by the government of the right of intervention even in contractual relations affected with public
interest. If the liberty invoked were freedom of the mind or the person, the standard for the validity of
governmental acts is much more rigorous and exacting, but where the liberty curtailed affects at the most rights
of property, the permissible scope of regulatory measures is wider.

Statutes; When statute is void because of ambiguity. What makes a statute susceptible to a charge that it is
void on its face for alleged vagueness or uncertainty is an enactment either forbidding or requiring the doing of
an act that men of common intelligence must necessarily guess at its meaning and diff er as to its application.

Page 19 of 236
Constitutional Law 2 Bill of Rights Armando Santiago Jr

POWER OF TAXATION
the power to raise revenue for governmental purposes.

DEFINITION AND SCOPE


It is the enforced proportional contributions from persons and property, levied by the State by virtue of its
sovereignty, for the support of the government and for all public needs.
It is as broad as the purpose for which it is given.

tax should not be confiscatory (wag ubusin ang pera ng tao)

Constitution prohibits double taxation (ex. piece of land same land taxed by the municipality and taxed by
province it is not considered double taxation)

PURPOSE
1. To raise revenue
2. Tool for regulation
3. Protection/power to keep alive

Tax for special purpose [Sec. 29 (3), Art. VI]: Treated as a special fund and paid out for such purpose only; when
purpose is fulfilled, the balance, if any shall be transferred to the general funds of the Government. See:
Osmena v. Orbos, 220 SCRA 703

SCOPE AND LIMITATION


General Limitations
1. Power to tax exists for the general welfare; should be exercised only for a public purpose
2. might be justified as for public purpose even if the immediate beneficiaries are private individuals
3. Tax should not be confiscatory: If a tax measure is so unconscionable as to amount to confiscation of
property, the Court will invalidate it. But invalidating a tax measure must be exercised with utmost caution,
otherwise, the States power to legislate for the public welfare might be seriously curtailed
4. Taxes should be uniform and equitable [Sec. 28(1), Art. VI]

The legislature has discretion to determine the nature, object, extent, coverage, and situs of taxation. But where
a tax measure becomes so unconscionable and unjust as to amount to confiscation of property, courts will not
hesitate to strike it down, for despite all its plenitude, the power to tax cannot override constitutional
prescriptions. [Tan v. del Rosario, 237 SCRA 324]

Specific Limitations
1. Uniformity of taxation

GENERAL RULE: simply geographical uniformity, meaning it operates with the same force and effect in
every place where the subject of it is found

EXCEPTION: rule does not prohibit classification for purposes of taxation, provided the ff requisites are met:
(SNAGAE)
(a) standards used are substantial and not arbitrary
(b) if the classification is germane to achieve the legislative purpose
(c) if that classification applies to both present and future conditions, other circumstances being equal
(d) applies equally to members of the same class. [Pepsi Cola v. City of Butuan].

2. Tax Exemptions

Page 20 of 236
Constitutional Law 2 Bill of Rights Armando Santiago Jr

No law granting any tax exemption shall be passed without the concurrence of a majority of all the Members of
Congress [Sec. 28 (4), Art. VI]

A corollary power but must be for a public purpose, uniform and equitable and in conformity with the equal
protection clause
Tax exemptions are granted gratuitously and may be revoked at will, except when it was granted for valuable
consideration

May either be constitutional or statutory

If statutory, it has to have been passed by majority of all the members of Congress [sec. 28 (4), Art. VI]

Constitutional exemptions [sec. 28(3), Art. VI]


A. Educational institutions (both profit and non-profit): Benefits redound to students, but only applied to
property taxes and not excise taxes

All revenues and assets of non-stock, non-profit educational institutions used actually, directly and exclusively
for educational purposes shall be exempt from taxes and duties. xxx Proprietary educational institutions,
including those co-operatively owned, may likewise be entitled to such exemptions subject to the limitations
provided by law including restrictions on dividends and provisions for reinvestment. [Sec. 4(3), Art. XIV]

Subject to conditions prescribed by law, all grants, endowments, donations, or contributions used actually,
directly and exclusively for educational purposes shall be exempt from tax.

B. Charitable institutions: Religious and charitable institutions give considerable assistance to the State in the
improvement of the morality of the people and the care of the indigent and the handicapped
C. Religious property: Charitable Institutions, churches, and parsonages or convents appurtenant thereto,
mosques, non-profit cemeteries, and all lands, buildings and improvements, actually, directly and exclusively
used for religious, charitable or educational purposes shall be exempt from taxation. [Sec. 28 (3), Art. VI]

Page 21 of 236
Constitutional Law 2 Bill of Rights Armando Santiago Jr

POWER OF EMINENT DOMAIN


The ultimate right of the sovereign power to appropriate, not only the public but private property of all citizens
within the territorial sovereignty to public purpose.

Page 22 of 236
Constitutional Law 2 Bill of Rights Armando Santiago Jr

Art. III, Sec. 9


Private property shall not be taken for public use without just
compensation.
Art. XII, Sec. 18
The State may, in the interest of national welfare or defense, establish
and operate vital industries and, upon payment of just compensation,
transfer to public ownership utilities and other private enterprises to
be operated by the government.
Art. XIII, Sec. 4
The State shall, by law, undertake an agrarian reform program
founded on the right of farmers and regular farmworkers who are
landless, to own directly or collectively the lands they till or, in the
case of other farmworkers, to receive a just share of the fruits
thereof.

To this end, the State shall encourage and undertake the just
distribution of all agricultural lands, subject to such priorities and
reasonable retention limits as the Congress may prescribe, taking
into account ecological, developmental, or equity considerations, and
subject to the payment of just compensation.

In determining retention limits, the State shall respect the right of


small landowners. The State shall further provide incentives for
voluntary land- sharing.

Art. XIII, Sec. 9


The State shall, by law, and for the common good, undertake, in
cooperation with the private sector, a continuing program of urban
land reform and housing which will make available at affordable cost,
decent housing and basic services to under-privileged and homeless
citizens in urban centers and resettlement areas.
It shall also promote adequate employment opportunities to such
citizens. In the implementation of such program the State shall
respect the rights of small property owners.

Page 23 of 236
Constitutional Law 2 Bill of Rights Armando Santiago Jr

Art XIV, Sec. 13.


The National assembly may authorize, upon payment of just
compensation, the expropriation of private lands to be subdivided
into small lots and conveyed at cost to deserving citizens.

REQUISITES
1. Private property
2. Genuine necessity - inherent/presumed in legislation, but when the power is delegated (e.g. local
government units), necessity must be proven.
3. For public use - Court has adopted a broad definition of public use, following the U.S. trend
4. Payment of just compensation
5. Due process [Manapat v. CA (2007)]

IT IS THE RIGHT OF THE GOVERNMENT TO TAKE PRIVATE PROPERTY WITH


JUST COMPENSATION
The power of eminent domain does not depend for its existence on a specific grant in the constitution. It is
inherent in sovereignty and exists in a sovereign state without any recognition of it in the constitution. The
provisions found in most of the state constitutions relating to the taking of property for the public use do not, by
implication, grant the power to the government of the state, but limit a power which would otherwise be without
limit. (citations omitted) [Visayan Refining Co. vs. Camus, G.R. No. L-15870, December 3, 1919]

Specifically (LGUs, Sec. 19, Local Government Code)


1. Ordinance by a local legislature council is enacted authorizing local chief executive to exercise eminent
domain,
2. For public use, purpose or welfare or for the benefit of the poor and of the landless,
3. Payment of just compensation,
4. Valid and definite offer has been previously made to owner of the property sought to be expropriated but such
offer was not accepted [Municipality of Paraaque vs. VM Realty (1998)]

Jurisdiction over a complaint for eminent domain is with the Regional Trial Court. While the value of the property
to be expropriated is estimated in monetary terms for the court is duty bound to determine the amount of just
compensation to be paid for the property it is merely incidental to the expropriation suit [Barangay San Roque,
Talisay, Cebu v. Heirs of Francisco Pastor, G.R. No. 138869, June 20, 2000; Bardillion v. Barangay Masili
of Calamba, Laguna, G.R. No. 146886, April 30, 2003]

The issuance of a writ of possession becomes ministerial upon the (1) filing of a complaint for expropriation
sufficient in form and substance, and (2) upon deposit made by the government of the amount equivalent to 15%
of the fair market value of the property sought to be expropriated per current tax declaration. [Biglang-Awa v.
Judge Bacalla, G.R. Nos. 139927-139936, November 22, 2000; Bardillon v. Barangay Masili of Calamba,
Laguna, Laguna, G.R. No. 146886, April 30, 2003]

SCOPE AND LIMITATIONS


All Private Property capable of ownership may be expropriated, except money and choses in action. Even
services may be subject to eminent domain. [Republic v. PLDT, 26 SCRA 620]

The exercise of the right of eminent domain, whether directly by the State or by its authorized agents, is
necessarily in derogation of private rights. Hence, strict construction will be made against the agency exercising
the power. [Jesus is Lord Christian School Foundation v. Municipality of Pasig, G.R. No. 152230, August
9, 2005]

Page 24 of 236
Constitutional Law 2 Bill of Rights Armando Santiago Jr

NECESSITY
The foundation of the right to exercise eminent domain is genuine necessity and that necessity must be of public
character. Government may not capriciously or arbitrarily choose which private property should be expropriated.
[Lagcao v. Judge Labra, G.R. No. 155746, October 13, 2004]

When the power is exercised by the legislature, the question of necessity is generally a political question.
[Municipality of Meycauyan, Bulacan v. Intermediate Appellate Court, 157 SCRA 640]

The RTC has the power to inquire into the legality of the exercise of the right of eminent domain and to
determine whether there is a genuine necessity for it. [Bardillon v. Barangay Masili of Calamba, Laguna, G.R.
No. 146886, April 30, 2003]

PRIVATE PROPERTY
Private property already devoted to public use cannot be expropriated by a delegate of legislature acting under a
general grant of authority. [City of Manila v. Chinese Community, 40 Phil 349]

TAKING
The exercise of the power of eminent does not always result in the taking or appropriation of title to the
expropriated property; it may only result in the imposition of a burden upon the owner of the condemned
property, without loss of title or possession. [National Power Corporation v. Gutierrez, 193 SCRA 1]

REQUISITES FOR A VALID TAKING


(1) The expropriator must enter a private property
(2) Entry must be for more than a momentary period
(3) Entry must be under warrant or color of legal authority
(4) Property must be devoted to public use or otherwise informally appropriated or injuriously affected
Utilization of the property must be in such a way as to oust the owner and deprive him of beneficial enjoyment of
the property. [Republic v. Castelvi, 58 SCRA 336]

DUE PROCESS
The defendant must be given an opportunity to be heard. In the case of Belen v. Court of Appeals, the Supreme
Court declared two Presidential Decrees unconstitutional for violating due process because they did not provide
for any form of hearing or procedure by which the propriety of the expropriation or the reasonableness of the
compensation.

TAKING VIA EMINENT DOMAIN VS. TAKING UNDER SOCIAL JUSTICE


CLAUSE

Agrarian Reform (Art. XIII, Sec. 4)


This provision is an exercise of the police power of the State through
eminent domain (Association of Small Landowners vs. Secretary of
Agrarian Reform) as it is a means to regulate private property.
The Comprehensive Agrarian Reform Law prescribes retention limits to the landowners, there is an exercise of
police power for the regulation of private property in accordance with the constitution. But in carrying out such
regulation, the owners are deprived of lands they own in excess of the maximum area allowed, there is also
taking under the power of eminent domain. The taking contemplated is not a mere limitation on the use of the
land, but the surrender of the title to and physical possession of the excess and all beneficial rights accruing to
the owner in favor of the beneficiary. [Sta. Rosa Realty & Development Corp. v. Court of Appeals, G.R. No.
112526, October 12, 2001]

Page 25 of 236
Constitutional Law 2 Bill of Rights Armando Santiago Jr

EXPANSIVE CONCEPT OF PUBLIC USE

Definition
The idea that "public use" means "use by the public" has been discarded. At present, whatever may be
beneficially employed for the general welfare satisfies the requirement of public use. [Heirs of Juancho Ardona
vs. Reyes, 123 SCRA 220]

That only a few benefit from the expropriation does not diminish its public-use character, inasmuch as pubic use
now includes the broader notion of indirect public benefit or advantage [Filstream International vs. CA, 284
SCRA 716]

Public use is the general concept of meeting public need or public exigency. It is not confined to actual use by
the public in its traditional sense. The idea that public use is strictly limited to clear cases of use by the
public has been abandoned. The term public use has now been held to be synonymous with public
interest, public benefit, public welfare and public convenience. [Reyes v. National Housing Authority,
G.R. No. 147511, January 20, 2003]

The practical reality that greater benefit may be derived by Iglesia ni Cristo members than most others could well
be true, but such peculiar advantage still remains merely incidental and secondary in nature. That only few
would benefit from the expropriation of the property does not necessarily diminish the essence and character of
public use [Manosca v. Court of Appeals, 252 SCRA 412]

JUST COMPENSATION

Definition
It is the just and complete equivalent of the loss which the owner of the thing expropriated has to suffer by
reason of the expropriation.

Full and fair equivalent of the property taken; it is the fair market value of the property. It is settled that the market
value of the property is that the sum of money which a person, desirous but not compelled to buy, and an
owner, willing but not compelled to sell, would agree on as a price to be given and received therefor [Province
of Tayabas vs. Perez (1938)]

DETERMINATION
BASIS: Fair Market Value
Price fixed by a buyer desirous but not compelled to buy and a seller willing but not compelled to sell.
Must include consequential damages (damages to other interest of the owner attributable to the expropriation)
and deduct consequential benefits (increase of value of other interests attributable to new use of the former
property).

CHOICE OF PROPERTY TO BE EXPROPRIATED IS SUBJECT TO JUDICIAL REVIEW AS TO


REASONABLENESS:
Under Section 2, Article IV of the Philippine Constitution, the Republic of the Philippines can take private
property upon payment of just compensation. However, private property to be taken cannot be chosen arbitrarily
and capriciously, as the landowner is entitled to due process. The Department of Public Highways originally
established the extension in Cuneta Avenue, and it is assumed that they made extensive studies regarding it.
The change from Cuneta Avenue to Fernando Rein-Del Pan Streets cannot be justified on the ground of social
impact, as the properties to be affected along Cuneta Avenue are mostly motels. [De Knecht vs. Bautista
(1980)]

The Presidential Decrees merely serve as a guide or a factor for the courts in determining amount of just
compensation (which should be the fair and full value of the property at time of taking). The courts have the
power and authority to determine just compensation, independent of what the decrees state, and thus may
appoint commissioners to help in determining just compensation. [EPZA vs. Dulay, 148 SCRA 305]

Page 26 of 236
Constitutional Law 2 Bill of Rights Armando Santiago Jr

While commissioners are to be appointed by the court for the determination of just compensation, the latter is not
bound by the commissioners findings. [Republic v. Santos, 141 SCRA 30; Republic (MECS) v. IAC, 185
SCRA 572]

The court may substitute its own estimate of the value of the property only for valid reasons: (a) the
commissioners have applied illegal principles to the evidence submitted to them; (b) they have disregarded a
clear preponderance of evidence; or (c) where the amount allowed is either grossly inadequate or excessive.
[National Power Corporation v. De la Cruz, G.R. No. 156093, February 2, 2007]

NON-PAYMENT OF JUST

expropriation proceeding does not entitle the private landowners to recover possession of the expropriated lots,
but only to demand payment of the fair market value of the property. [Republic of the Philippines v. Court of
Appeals, G.R. No. 146587, July 2, 2002; Reyes v. National Housing Authority, G.R. No. 147511, Janaury
29, 2003]

The Republic was ordered to pay just compensation twice: first, in the expropriation and then, in the action for
recovery of possession but it never did. 57 years have lapsed since the expropriation case was terminated but
the Republic never paid the owners. The court construed the failure to pay as a deliberate refusal on the part of
the Republic. When the government fails to pay just compensation within five years from the finality of the
judgment in the expropriation proceedings, the owners concerned shall have the right to recover possession of
their property. [Republic of the Philippines v. Vicente Lim, G.R. No. 161656, June 29, 2005]

EFFECT OF DELAY
Just compensation means not only the correct amount to be paid to the owner of the land but also payment
within a reasonable time from its taking [Eslaban v. De Onorio, G.R. No. 146062, June 28, 2001]

The filing of the case generally coincides with the taking. When the filing of the case coincides with the taking,
and the value of the property has increased because of the use to which the expropriator has put it, the value is
that of the time of the earlier taking. Otherwise the owner would gain undeserved profit. But if the value
increased independently of what the expropriator did, then the value is that of the later filing of the case. Also,
between the time payment is due and the actual payment, legal interest (6%) accrues. [NAPOCOR v. CA
(1996)]

ABANDONMENT OF INTENDED USE AND RIGHT OF REPURCHASE


If the expropriator (government) does not use the property for a public purpose, the property reverts to the owner
in fee simple. [Heirs of Moreno vs. Mactan-Cebu International Airport (2005)]

MISCELLANEOUS APPLICATION
What the due process clause requires is that the landowner must be given reasonable opportunity to be heard
and to present his claim or defense. Although due process does not always necessarily demand that a
proceeding be had before a court of law, it still mandates some form of proceeding wherein notice and
reasonable opportunity to be heard are given to the owner to protect his property rights. Although there are
exceptional situations when in the exercise of the power of eminent domain, the requirement does not need
judicial process, when it is alleged that the landowners right to due process of law has been violated in the
taking of his property, the courts can probe and check on the alleged violation. [Manotok vs. NHA (1987)]

The performance of the administrative acts necessary to the exercise of the power of eminent domain in behalf
of the state is lodged by tradition in the Sovereign or other Chief Executive.

Where the Legislature has expressly conferred the authority to maintain expropriation proceedings upon the
Chief Executive, the right of the latter to proceed therein is clear.

Page 27 of 236
Constitutional Law 2 Bill of Rights Armando Santiago Jr

Once authority is given to exercise the power of eminent domain, the matter ceases to be wholly legislative. The
executive authorities may then decide whether the power will be invoked and to what extent. (citations omitted)
[Visayan Refining Co. vs. Camus, G.R. No. L-15870, December 3, 1919]

The particular mention in the Constitution of agrarian reform and the transfer of utilities and other private
enterprises to public ownership merely underscores the magnitude of the problems sought to be remedied by
these programs. They do not preclude nor limit the exercise of the power of eminent domain for such purposes
like tourism and other development programs.

There can be no doubt that expropriation for such traditions' purposes as the construction of roads, bridges,
ports, waterworks, schools, electric and telecommunications systems, hydroelectric power plants, markets and
slaughterhouses, parks, hospitals, government office buildings, and flood control or irrigation systems is valid.
However, the concept of public use is not limited to traditional purposes. Here as elsewhere the Idea that "public
use" is strictly limited to clear cases of "use by the public" has been discarded.

Private bus firms, taxicab fleets, roadside restaurants, and other private businesses using public streets and
highways do not diminish in the least bit the public character of expropriations for roads and streets. The lease of
store spaces in underpasses of streets built on expropriated land does not make the taking for a private purpose.
Airports and piers catering exclusively to private airlines and shipping companies are still for public use. The
expropriation of private land for slum clearance and urban development is for a public purpose even if the
developed area is later sold to private homeowners, commercial firms, entertainment and service companies,
and other private concerns. [Heirs of Ardona vs. Reyes, G.R. Nos. L-60549, 60553 to 60555, October 26,
1983)

CASES

REPUBLIC OF THE PHILIPPINES VS. VDA. DE CASTELLVI4


PONENCIA:
TOPIC:
TRIGGER OF THE FACTS:
TRIGGER OF THE ISSUE:
TRIGGER OF THE RULING:
VERDICT:
END POINT:

FACTS
In 1947, the republic, through the Armed Forces of the Philippines (AFP), entered into a lease agreement over a
land in Pampanga with Castellvi on a year-to-year basis. When Castellvi gave notice to terminate the lease in
1956, the AFP refused because of the permanent installations and other facilities worth almost P500,000.00 that
were erected and already established on the property. She then instituted an ejectment proceeding against the
AFP. In 1959, however, the republic commenced the expropriation proceedings for the land in question.


ISSUE
Whether or not the compensation should be determined as of 1947 or 1959.

4 (GR No. L-20620)


Page 28 of 236
Constitutional Law 2 Bill of Rights Armando Santiago Jr

RULING
The Supreme Court ruled that the taking should not be reckoned as of 1947, and that just compensation
should not be determined on the basis of the value of the property as of that year.

The requisites for taking are:


1. The expropriator must enter a private property;
2. The entry must be for more than a momentary period;
3. It must be under warrant or color of authorities;
4. The property must be devoted for public use or otherwise informally appropriated or injuriously affected;
and
5. The utilization of the property for public use must be such a way as to oust the owner and deprive him of
beneficial enjoyment of the property.

Only requisites 1, 3, and 4 were present. It is clear, therefore, that the "taking" of Catellvi's property for purposes
of eminent domain cannot be considered to have taken place in 1947 when the Republic commenced to occupy
the property as lessee thereof.

Requirement No. 2 is not present


momentary cnstrued as limited period and not definite/not permanent
lease contract was for a period of one year renewable from year to year (temporary)
fact that AFP install a permanent construction. still it did not alter the fact that the entry into the land was
transitory (temporary)
the intention of the lessee was to make it permanent but his intention will not prevail over the clear and
express term of the lease contract

Requirement No. 5 is lacking


the entry of the republic into the property and its utilization of the same for public use, did not oust castellvi and
deprive her of all beneficial enjoyment of the property. castelvi remain as owner and continuously recognized
as owner by the republic under took to return the property to castellvi when the lease was terminated
neither was castellvi was deprived of all the beneficial enjoyment of the property
because the republic was bound to pay and had been paying
castellvi agreed monthly rentals, until the time when it filed complaint for eminent domain on june 20, 1959
it is clear that taking of castellvi propert for the purpose of eminent domain cannot be considered to have
taken place in 1947 when the republic commenced to occupy the property as lessee
and the just compensation to be paid for castellvi property should not be determined on the basis but the value
of the property as of the year
the lower court did not commit an error

Under Sec. 4, Rule 67 of the Rules of Court, just compensation is to be determined as of the date of the filing
of the complaint. The Supreme Court has ruled that when the taking of the property sought to be expropriated
coincides with the commencement of the expropriation proceedings, or takes place subsequent to the filing of
the complaint for eminent domain, the just compensation should be determined as of the date of the filing of the
complaint.

In the instant case, it is undisputed that the Republic was placed in possession of the Castellvi property, by
authority of court, on August 10, 1959. The taking of the Castellvi property for the purposes of determining the
just compensation to be paid must, therefore, be reckoned as of June 26, 1959 when the complaint for eminent
domain was filed.

DIDIPIO EARTH SAVERS MULTIPURPOSE ASSOCIATION ET


AL VS DENR SEC ELISEA GOZUN ET AL
Ponencia: CHICO NAZARIO J.

Page 29 of 236
Constitutional Law 2 Bill of Rights Armando Santiago Jr

Topic: POWER OF EMINENT DOMAIN / MINING LAW


Trigger of facts:
Trigger of issue:
Trigger of Ruling:
Verdict:
End point of the case:

FACTS
In 1987, Cory rolled out EO 279 w/c empowered DENR to stipulate with foreign companies when it
comes to either technical or financial large scale exploration or mining.
In 1995, Ramos signed into law RA 7942 or the Philippine Mining Act. In 1994, Ramos already signed an
FTAA with Arimco Mining Co, an Australian company.
The FTAA authorized AMC (later CAMC) to explore 37,000 ha of land in Quirino and N. Vizcaya
including Brgy Didipio. After the passage of the law, DENR rolled out its implementing RRs. Didipio petitioned to
have the law and the RR to be annulled as it is unconstitutional and it constitutes unlawful taking of property.
In seeking to nullify Rep. Act No. 7942 and its implementing rules DAO 96-40 as unconstitutional,
petitioners set their sight on Section 76 of Rep. Act No. 7942 and Section 107 of DAO 96-40 which they claim
allow the unlawful and unjust taking of private property for private purpose in contradiction with Section 9,
Article III of the 1987 Constitution
mandating that private property shall not be taken except for public use and the corresponding payment
of just compensation. They assert that public respondent DENR, through the Mining Act and its Implementing
Rules and Regulations, cannot, on its own, permit entry into a private property and allow taking of land without
payment of just compensation.
Traversing petitioners assertion, public respondents argue that Section 76 is not a taking provision but a
valid exercise of the police power and by virtue of which, the state may prescribe regulations to promote the
health, morals, peace, education, good order, safety and general welfare of the people.
This government regulation involves the adjustment of rights for the public good and that this adjustment
curtails some potential for the use or economic exploitation of private property.
Public respondents concluded that to require compensation in all such circumstances would compel the
government to regulate by purchase.

ISSUE
Whether or not RA 7942 and the DENR RRs are valid.

HELD

The SC ruled against Didipio. The SC noted the requisites of eminent domain. They are;

(1) the expropriator must enter a private property;


(2) the entry must be for more than a momentary period.
(3) the entry must be under warrant or color of legal authority;
(4) the property must be devoted to public use or otherwise informally appropriated or injuriously affected;
(5) the utilization of the property for public use must be in such a way as to oust the owner and deprive him of
beneficial enjoyment of the property.

In the case at bar, Didipio failed to show that the law is invalid. Indeed, there is taking involved but it is not w/o
just compensation. Sec 76 of RA 7942 provides for just compensation as well as section 107 of the DENR RR.
To wit,

Section 76. xxx Provided, that any damage to the property of the surface owner, occupant, or concessionaire as
a consequence of such operations shall be properly compensated as may be provided for in the implementing
rules and regulations.

Page 30 of 236
Constitutional Law 2 Bill of Rights Armando Santiago Jr

Section 107. Compensation of the Surface Owner and Occupant- Any damage done to the property of the
surface owners, occupant, or concessionaire thereof as a consequence of the mining operations or as a result of
the construction or installation of the infrastructure mentioned in 104 above shall be properly and justly
compensated.
Further, mining is a public policy and the government can invoke eminent domain to exercise entry, acquisition
and use of private lands.

MANOSCA VS. COURT OF APPEALS5


PONENCIA:
TOPIC:
TRIGGER OF THE FACTS:
TRIGGER OF THE ISSUE:
TRIGGER OF THE RULING:
VERDICT:
END POINT:

FACTS
Alejandro, Asuncion and Leonica Manosca inherited a piece of land located at P. Burgos Street, Calzada,
Taguig, Metro Manila, with an area of about 492 square meters.
When the parcel was ascertained by the National Historical Institute (NHI) to have been the birth site of Felix
Y. Manalo, the founder of Iglesia Ni Cristo
it passed Resolution 1, Series of 1986, pursuant to Section 4 of Presidential Decree 260, declaring the land to
be a national historical landmark.
The resolution was approved by the Minister of Education, Culture and Sports (MECS).
Later, the opinion of the Secretary of Justice was asked on the legality of the measure.
In his opinion the Secretary of Justice replied in the affirmative.
Accordingly, the Republic, through the office of the Solicitor-General, instituted a complaint for expropriation
before the Regional Trial Court of Pasig for and in behalf of the NHI.
At the same time, the Republic filed an urgent motion for the issuance of an order to permit it to take
immediate possession of the property.
The motion was opposed by the Manoscas. After a hearing, the trial court issued an order fixing the
provisional market (P54,120.00) and assessed (P16,236.00) values of the property and authorizing the
Republic to take over the property once the required sum would have been deposited with the Municipal
Treasurer of Taguig, Metro Manila.
The Manoscas moved to dismiss the complaint on the main thesis that the intended expropriation was not for
a public purpose and, incidentally, that the act would constitute an application of public funds, directly or
indirectly, for the use, benefit, or support of Iglesia ni Cristo, a religious entity, contrary to the provision of
Section 29(2), Article VI, of the 1987 Constitution.
The trial court issued its denial of said motion to dismiss.
The Manoscas moved for reconsideration thereafter but were denied.
The Manoscas then lodged a petition for certiorari and prohibition with the Court of Appeals.
the appellate court dismissed the petition/A motion for the reconsideration of the decision
was denied by the appellate court
The Manoscas filed a petition for review on certiorari with the Supreme Court.

ISSUE
Whether the setting up of the marker in commemoration of Felix Manalo, the founder of the religious
sect Iglesia ni Cristo, constitutes public use.

5 [GR 106440, 29 January 1996]


Page 31 of 236
Constitutional Law 2 Bill of Rights Armando Santiago Jr

HELD
YES. Petitioners ask about the so-called unusual interest that the expropriation of (Felix Manalos) birthplace
become so vital as to be a public use appropriate for the exercise of the power of eminent domain when only
members of the Iglesia ni Cristo would benefit. This attempt to give some religious perspective to the case
deserves little consideration, for what should be significant is the principal objective of, not the casual
consequences that might follow from, the exercise of the power. The purpose in setting up the marker is
essentially to recognize the distinctive contribution of the late Felix Manalo to the culture of the Philippines,
rather than to commemorate his founding and leadership of the Iglesia ni Cristo. The practical reality that
greater benefit may be derived by members of the Iglesia ni Cristo than by most others could well be true but
such a peculiar advantage still remains to be merely incidental and secondary in nature. Indeed, that only a few
would actually benefit from the expropriation of property does not necessarily diminish the essence and
character of public use.

All considered, the Court finds the assailed decision to be in accord with law and jurisprudence. The petition is
DENIED.

SYLLABI
Constitutional Law; Eminent Domain; Words and Phrases; Eminent Domain, Explained; The constitutional
qualification that private property shall not be taken for public use without just compensation is intended to
provide a safeguard against possible abuse and so to protect as well the individual against whose property the
power is sought to be enforced.Eminent domain, also often referred to as expropriation and, with less
frequency, as condemnation, is, like police power and taxation, an inherent power of sovereignty. It need not be
clothed with any constitutional gear to exist; instead, provisions in our Constitution on the subject are meant
more to regulate, rather than to grant, the exercise of the power. Eminent domain is generally so described as
the highest and most exact idea of property remaining in the government that may be acquired for some public
purpose through a method in the nature of a forced purchase by the State. It is a right to take or reassert
dominion over property within the state for public use or to meet a public exigency. It is said to be an essential
part of governance even in its most primitive form and thus inseparable from sovereignty. The only direct
constitutional qualification is that private property shall not be taken for public use without just compensation.
This proscription is intended to provide a safeguard against possible abuse and so to protect as well the
individual against whose property the power is sought to be enforced.

Same; Same; The power of eminent domain should not now be understood as being confined only to the
expropriation of vast tracts of land and landed estates.The court, in Guido, merely passed upon the issue of
the extent of the Presidents power under Commonwealth Act No. 539 to, specifically, acquire private lands for
subdivision into smaller home lots or farms for resale to bona fide tenants or occupants. It was in this particular
context of the statute that the Court had made the pronouncement. The guidelines in Guido were not meant to
be preclusive in nature and, most certainly, the power of eminent domain should not now be understood as being
confined only to the expropriation of vast tracts of land and landed estates.

Same; Same; Words and Phrases; Public Use, Explained; The term public use must be considered in its
general concept of meeting a public need or a public exigency.The term public use, not having been
otherwise defined by the constitution, must be considered in its general concept of meeting a public need or a
public exigency. Black summarizes the characterization given by various courts to the term; thus: Public Use.
Eminent domain. The constitutional and statutory basis for taking property by eminent domain. For
condemnation purposes, public use is one which confers some benefit or advantage to the public; it is not
confined to actual use by public. It is measured in terms of right of public to use proposed facilities for which
condemnation is sought and, as long as public has right of use, whether exercised by one or many members of
public, a public advantage or public benefit accrues sufficient to constitute a public use. Montana Power Co.
vs. Bokma, Mont. 457 P.2d 769, 772, 773.

Same; Same; Same; Same; The validity of the exercise of the power of eminent domain for traditional purposes
is beyond questionit is not at all to be said, however, that public use should thereby be restricted to such

Page 32 of 236
Constitutional Law 2 Bill of Rights Armando Santiago Jr

traditional uses.The validity of the exercise of the power of eminent domain for traditional purposes is beyond
question; it is not at all to be said, however, that public use should thereby be restricted to such traditional uses.
The idea that public use is strictly limited to clear cases of use by the public has long been discarded.
Same; Same; Same; Same; A historical research discloses the meaning of the term public use to be one of
constant growth.It has been explained as early as Sea v. Manila Railroad Co., that: x x x A historical
research discloses the meaning of the term public use to be one of constant growth. As society advances, its
demands upon the individual increase and each demand is a new use to which the resources of the individual
may be devoted. x x x for whatever is beneficially employed for the community is a public use.

Same; Same; Separation of Church and State; Freedom of Religion; An attempt to give some religious
perspective to the case deserves little consideration, for what should be significant is the principal objective of,
not the casual consequences that might follow from, the exercise of the power.Petitioners ask: But (w)hat is
the so-called unusual interest that the expropriation of (Felix Manalos) birthplace become so vital as to be a
public use appropriate for the exercise of the power of eminent domain when only members of the Iglesia ni
Cristo would benefit? This attempt to give some religious perspective to the case deserves little consideration,
for what should be significant is the principal objective of, not the casual consequences that might follow from,
the exercise of the power. The purpose in setting up the marker is essentially to recognize the distinctive
contribution of the late Felix Manalo to the culture of the Philippines, rather than to commemorate his founding
and leadership of the Iglesia ni Cristo.

Same; Same; Same; That only a few would actually benefit from the expropriation of property does not
necessarily diminish the essence and character of public use.The practical reality that greater benefit may be
derived by members of the Iglesia ni Cristo than by most others could well be true but such a peculiar advantage
still remains to be merely incidental and secondary in nature. Indeed, that only a few would actually benefit from
the expropriation of property does not necessarily diminish the essence and character of public use.
Same; Same; Just Compensation; Due Process; There is no denial of due process where the records of the
case are replete with pleadings that could have dealt with the provisional value of the propertywhat the law
prohibits is the lack of opportunity to be heard.Petitioners contend that they have been denied due process in
the fixing of the provisional value of their property. Petitioners need merely to be reminded that what the law
prohibits is the lack of opportunity to be heard; contrary to petitioners argument, the records of this case are
replete with pleadings that could have dealt, directly or indirectly, with the provisional value of the property.

REPUBLIC VS. PHILIPPINE LONG DISTANCE TELEPHONE


CO.6
PONENCIA:
TOPIC:
TRIGGER OF THE FACTS:
TRIGGER OF THE ISSUE:
TRIGGER OF THE RULING:
VERDICT:
END POINT:

FACTS
The Republic of the Philippines, is a political entity exercising governmental powers through its branches and
instrumentalities, one of which is the Bureau of Telecommunications.
That office was created under Executive Order 94, in addition to certain powers and duties formerly vested in
the Director of Posts.
Sometime, the Philippine Long Distance Telephone Company (PLDT), and the RCA Communications, Inc.,
entered into an agreement whereby telephone messages, coming from the United States and received by
RCA's domestic station, could automatically be transferred to the lines of PLDT;
and vice-versa, for calls collected by the PLDT for transmission from the Philippines to the United States.
The contracting parties agreed to divide the tolls, as follows: 25% to PLDT and 75% to RCA.

6 [GR L-18841, 27 January 1969]


Page 33 of 236
Constitutional Law 2 Bill of Rights Armando Santiago Jr

The sharing was amended in 1941 to 30% for PLDT and 70% for RCA, and again amended in 1947 to a 50-50
basis.
The arrangement was later extended to radio-telephone messages to and from European and Asiatic
countries.
Their contract contained a stipulation that either party could terminate it on a 24-month notice to the other.
PLDT gave notice to RCA to terminate their contract
Soon after its creation in 1947, the Bureau of Telecommunications set up its own Government Telephone
System by utilizing its own appropriation and equipment and by renting trunk lines of the PLDT to enable
government offices to call private parties. At that time, the Bureau was maintaining 5,000 telephones and had
5,000 pending applications for telephone connection.
The PLDT, on the other hand, was also maintaining 60,000 telephones and had also 20,000 pending
applications. Through the years, neither of them has been able to fill up the demand for telephone service.
The Bureau of Telecommunications had proposed to the PLDT that both enter into an interconnecting
agreement, with the government paying (on a call basis)
for all calls passing through the interconnecting facilities from the Government Telephone System to the PLDT.
the Republic, through the Director of Telecommunications, entered into an agreement with RCA
Communications, Inc., for a joint overseas telephone service whereby the Bureau would convey radio-
telephone overseas calls received by RCA's station to and from local residents.
They actually inaugurated this joint operation under a "provisional" agreement.
PLDT complained to the Bureau of Telecommunications that said bureau was violating the conditions under
which their Private Branch Exchange (PBX) is interconnected with the PLDT's facilities, referring to the rented
trunk lines, for the Bureau had used the trunk lines not only for the use of government offices but even to
serve private persons or the general public, in competition with the business of the PLDT;
and gave notice that if said violations were not stopped by midnight the PLDT would sever the telephone
connections.
When the PLDT received no reply, it disconnected the trunk lines being rented by the Bureau at midnight on
The result was the isolation of the Philippines, on telephone services, from the rest of the world, except the
United States.
the Republic commenced suit against PLDT, in the Court of First Instance of Manila praying in its complaint for
judgment commanding the PLDT to execute a contract with the Republic, through the Bureau, for the use of
the facilities of PLDTs telephone system throughout the Philippines under such terms and conditions as the
court might consider reasonable, and for a writ of preliminary injunction against PLDT to restrain the
severance of the existing telephone connections and/or restore those severed. After trial, the lower court
rendered judgment that it could not compel the PLDT to enter into an agreement with the Bureau because the
parties were not in agreement;
that under Executive Order 94, establishing the Bureau of Telecommunications, said Bureau was not limited
to servicing government offices alone, nor was there any in the contract of lease of the trunk lines, since the
PLDT knew, or ought to have known, at the time that their use by the Bureau was to be public throughout the
Islands, hence the Bureau was neither guilty of fraud, abuse, or misuse of the poles of the PLDT; and, in view of
serious public prejudice that would result from the disconnection of the trunk lines, declared the preliminary
injunction permanent, although it dismissed both the complaint and the counterclaims. Both parties appealed.

ISSUE
Whether interconnection between PLDT and the Government Telephone System can be an valid object
for expropriation, i.e. the exercise of eminent domain.

HELD
Although parties can not be coerced to enter into a contract where no agreement is had between them as
to the principal terms and conditions of the contract -- the freedom to stipulate such terms and conditions
being of the essence of our contractual system, and by express provision of the statute, a contract may be
annulled if tainted by violence, intimidation or undue influence -- and thus the Republic may not compel the
PLDT to celebrate a contract with it, the Republic may, in the exercise of the sovereign power of eminent
domain, require the telephone company to permit interconnection of the government telephone system and
that of the PLDT, as the needs of the government service may require, subject to the payment of just

Page 34 of 236
Constitutional Law 2 Bill of Rights Armando Santiago Jr

compensation to be determined by the court. Normally, of course, the power of eminent domain results in the
taking or appropriation of title to, and possession of, the expropriated property; but no cogent reason appears
why the said power may not be availed of to impose only a burden upon the owner of condemned property,
without loss of title and possession. It is unquestionable that real property may, through expropriation, be
subjected to an easement of right of way. The use of the PLDT's lines and services to allow interservice
connection between both telephone systems is not much different. In either case private property is subjected
to a burden for public use and benefit. If under Section 6, Article XIII, of the Constitution, the State may, in
the interest of national welfare, transfer utilities to public ownership upon payment of just compensation,
there is no reason why the State may not require a public utility to render services in the general interest,
provided just compensation is paid therefor. Ultimately, the beneficiary of the interconnecting service would
be the users of both telephone systems, so that the condemnation would be for public use.

SYLLABI
Constitutional law; Sovereign power of eminent domain; Republic of the Philippines may require telephone
company to permit interconnection of the government telephone system and that of the PLDT; Right of way;
State may require a public utility to render services in the general interest; Case at bar.The Republic may, in
the exercise of the sovereign power of eminent domain, require the telephone company to permit
interconnection of the government telephone system and that of the PLDT, as the needs of the government
service may require, subject to the payment of just compensation to be determined by the court. Normally, of
course, the power of eminent domain results. in the taking or appropriation of title to, and possession of, the
expropriated property; but no cogent reason appears why the said power may not be availed of to impose only a
burden upon the owner of condemned property, without loss of title and possession. It is unquestionable that real
property may, through expropriation, be subjected to an easement of right of way. The use of the PLDT's lines
and services to allow inter service connection between both telephone systems is not much different. In -either
case private property is subjected to a burden for public use and benefit. If, under section 6, Article XIII, of the
Constitution, the State may; in the interest of national welfare, transfer utilities to public ownership upon payment
of just compensation, there is no reason why the State may not require a public utility to render services in the
general interest, provided just compensation is paid therefor. Ultimately, the beneficiary of the interconnecting
service would be the users of both telephone systems, so that the condemnation would be for public use.

FILSTREAM INTERNATIONAL INC.7

FACTS
Filstream International, Inc., is the registered owner of the properties consisting of adjacent parcels of land
situated in Antonio Rivera Street, Tondo II, Manila, with a total area of 3,571.10 square meters
Filstream filed an ejectment suit before the Metropolitan Trial Court (MTC) of Manila against the occupants of
the parcels of land on the grounds of termination of the lease contract and non-payment of rentals.
Judgment was rendered by the MTC ordering private respondents to vacate the premises and pay back
rentals to Filstream.
Not satisfied, malit, et. al. appealed the decision to the Regional Trial Court (RTC) of Manila which in turn
affirmed the decision of the MTC.
Still not content, Malit, et. al. proceeded to the Court of Appeals via a petition for review
The result however remained the same as the appellate court affirmed the decision of the RTC in its decision
Thereafter, no further action was taken by Malit, et. al., as a result of which the decision in the ejectment suit
became final and executory.
However, during the pendency of the ejectment proceedings Malit, et. al. filed a complaint for Annulment of
Deed of Exchange against Filstream before the RTC of Manila
It was at this stage that City of Manila came into the picture when the city government approved Ordinance
7813 authorizing Mayor Alfredo S. Lim to initiate the acquisition by negotiation, expropriation, purchase, or

7 [GR 125218 and GR 128077. 23 January 1998]


Page 35 of 236
Constitutional Law 2 Bill of Rights Armando Santiago Jr

other legal means certain parcels of land which formed part of Filstream's properties then occupied by Malit,
et. al.
Subsequently, the City of Manila approved Ordinance 7855 declaring the expropriation of certain parcels of
land situated along Antonio Rivera and Fernando Ma. Guerrero streets in Tondo, Manila which were owned by
Mr. Enrique Quijano Gutierrez, Filstreams predecessor-in-interest.
The said properties were to be sold and distributed to qualified tenants of the area
pursuant to the Land Use Development Program of the City of Manila. On 23 May 1994, the City of Manila
filed a complaint for eminent domain before the RTC of Manila (Branch 42, Civil Case 94-70560), seeking to
expropriate the parcels of land owned by Filstream which are situated at Antonio Rivera Street, Tondo II,
Manila. Pursuant to the complaint filed by the City of Manila, the trial court issued a Writ of Possession in
favor of the former which ordered the transfer of possession over the disputed premises to the City of Manila.
Filstream filed a motion to dismiss the complaint for eminent domain as well as a motion to quash the writ of
possession. On 30 September 1994, the RTC of Manila issued an order denying Filstream's motion to dismiss
and the motion to quash the Writ of Possession. Filstream filed a motion for reconsideration as well as a
supplemental motion for reconsideration seeking the reversal of the order but the same were denied. Still,
Filstream filed a subsequent motion to be allowed to file a second motion for reconsideration but it was also
denied. Aggrieved, Filstream filed on 31 March 1996, a Petition for Certiorari with the Court of Appeals (CAGR
SP 36904) seeking to set aside the RTC order. On 18 March 1996, the appellate court dismissed the
petition. Filsteream filed a motion for reconsideration and attached clearer copies of the pertinent documents
and papers pursuant to Section 2(a), Rule 6 of the Revised Internal Rules of the Court of Appeals. But on 20
May 1996, the appellate court issued a resolution denying the motion as petitioner failed to submit clearer and
readable copies of the pleadings. This prompted Filstream to proceed to the Supreme Court by filing a petition
for review on certiorari.

Meanwhile, owing to the finality of the decision in the ejectment suit (Civil Case 140817-CV), the MTC of
Manila, Branch 15, upon motion of Filstream, issued a Writ of Execution as well as a Notice to Vacate the
disputed premises. Malit, et. al. filed a Motion to Recall/Quash the Writ of Execution and Notice to Vacate
alleging the existence of a supervening event in that the properties subject of the dispute have already been
ordered condemned in an expropriation proceeding in favor of the City of Manila for the benefit of the
qualified occupants thereof, thus execution shall be stayed. For its part, the City of Manila filed on 13 March
1996, a motion for intervention with prayer to stay/quash the writ of execution on the ground that it is the
present possessor of the property subject of execution. In its order dated 14 March 1996, the MTC of Manila
denied Malit, et. al.'s motion as it found the allegations therein bereft of merit and upheld the issuance of the
Writ of Execution and Notice to Vacate in Filstream's favor. Subsequently, the trial court also denied the
motion filed by the City of Manila. On 22 April 1996, the trial court issued an order commanding the
demolition of the structure erected on the disputed premises. To avert the demolition, Malit, et. al. filed before
the RTC of Manila, (Branch 14, Civil Case 96-78098) a Petition for Certiorari and Prohibition with prayer for
the issuance of a temporary restraining order and preliminary injunction . On 15 May 1996, the City of
Manila filed its Petition for Certiorari and Prohibition with prayer for the issuance of a temporary restraining
order and preliminary injunction which was raffled to Branch 23 of the RTC of Manila (Civil Case 96-78382),
seeking the reversal of the orders issued by the MTC of Manila, Branch 14. Thereafter, upon motion filed by
the City of Manila, an order was issued by the RTC of Manila, Branch 10, ordering the consolidation of Civil
Case 96-78382 with Civil Case 96-78098 pending before Branch 14 of the RTC of Manila. Injunctions were
issued. Filstream then filed a motion for reconsideration from the order of denial but pending resolution of
this motion, it filed a motion for voluntary inhibition of the presiding judge of the RTC of Manila, Branch 14.
The motion for inhibition was granted 25 and as a result, the consolidated cases (Civil Cases 96-78382 and
96-78098) were re-raffled to the RTC of Manila, Branch 33. During the proceedings before the RTC of
Manila, Branch 33, Filstream moved for the dismissal of the consolidated cases (Civil Cases 96-78382 and
96-78098) for violation of Supreme Court Circular 04-94 (forum shopping) because the same parties, causes
of action and subject matter involved therein have already been disposed of in the decision in the ejectment
case (Civil Case 140817) which has already become final and executory prior to the filing of these
consolidated cases. On 9 December 1996, the RTC of Manila, Branch 33 ordered the dismissal of Civil Cases
96-78382 and 96-78098 due to forum shopping. Immediately thereafter, Filstream filed an Ex-parte Motion
for Issuance of an Alias Writ of Demolition and Ejectment and a supplemental motion to the same dated
January 10 and 13, 1997, respectively, before the MTC of Manila, Branch 15, which promulgated the decision
in the ejectment suit (Civil Case No. 140817-CV). 23 On January 1997, the court granted the motion and

Page 36 of 236
Constitutional Law 2 Bill of Rights Armando Santiago Jr

issued the corresponding writ of demolition. As a consequence of the dismissal of the consolidated cases,
Malit, et. al. filed a Petition for Certiorari and Prohibition with prayer for the issuance of a temporary
restraining order and preliminary injunction before the Court of Appeals (CA-GR SP 43101). At the
conclusion of the hearing for the issuance of a writ of preliminary injunction, the Court of Appeals, in its
resolution dated 18 February 1997, found merit in Malit, et. al.'s allegations in support of their application of
the issuance of the writ and granted the same. Filstream filed a Petition for Certiorari under Rule 65.

ISSUE
Whether there is violation of due process against Filstream in the manner its properties were
expropriated and condemned in favor of the City of Manila.

HELD
That only a few could actually benefit from the expropriation of the property does not diminish its
public use character. It is simply not possible to provide all at once land and shelter for all who need them.
Corollary to the expanded notion of public use, expropriation is not anymore confined to vast tracts of land
and landed estates. It is therefore of no moment that the land sought to be expropriated in this case is less than
half a hectare only. Through the years, the public use requirement in eminent domain has evolved into a
flexible concept, influenced by changing conditions. Public use now includes the broader notion of indirect
public benefit or advantage, including in particular, urban land reform and housing. The Court takes judicial
notice of the fact that urban land reform has become a paramount task in view of the acute shortage of decent
housing in urban areas particularly in Metro Manila. Nevertheless, despite the existence of a serious dilemma,
local government units are not given an unbridled authority when exercising their power of eminent domain
in pursuit of solutions to these problems. The basic rules still have to be followed, which are as follows: "no
person shall be deprived of life, liberty, or property without due process of law, nor shall any person be denied
the equal protection of the laws; private property shall not be taken for public use without just compensation".
Thus, the exercise by local government units of the power of eminent domain is not without limitations. Even
Section 19 of the 1991 Local Government Code is very explicit that it must comply with the provisions of the
Constitution and pertinent laws. Very clear from Sections 9 and 10 of Republic Act 7279 (Urban Development
and Housing Act of 1992) are the limitations with respect to the order of priority in acquiring private lands
and in resorting to expropriation proceedings as a means to acquire the same. Private lands rank last in the
order of priority for purposes of socialized housing. In the same vein, expropriation proceedings are to be
resorted to only when the other modes of acquisition have been exhausted. Compliance with these conditions
must be deemed mandatory because these are the only safeguards in securing the right of owners of private
property to due process when their property is expropriated for public use. There is nothing in the records that
would indicate that City of Manila complied with Section 9 and Section 10 of RA 7279. Filstream's properties
were expropriated and ordered condemned in favor of the City of Manila sans any showing that resort to the
acquisition of other lands listed under Section 9 of RA 7279 have proved futile. Evidently, there was a
violation of Filstream's right to due process which must accordingly be rectified.

PEOPLE VS. FAJARDO8


PONENCIA:
TOPIC:
TRIGGER OF THE FACTS:
TRIGGER OF THE ISSUE:
TRIGGER OF THE RULING:
VERDICT:
END POINT:

FACTS
On 15 August 1950, during the incumbency of Juan F. Fajardo as mayor of the municipality of Baao,
Camarines Sur, the municipal council passed Ordinance 7, series of 1950, providing that "any person or
persons who will construct or repair a building should, before constructing or repairing, obtain a written

8 [GR L-12172, 29 August 1958]


Page 37 of 236
Constitutional Law 2 Bill of Rights Armando Santiago Jr

permit from the Municipal Mayor," that "a fee of not less than P2.00 should be charged for each building
permit and P1.00 for each repair permit issued," and that any violation of the provisions of the ordinance shall
make the violator liable to pay a fine of not less than P25 nor more than P50 or imprisonment of not less than
12 days nor more than 24 days or both, at the discretion of the court; and that if said building destroys the
view of the Public Plaza or occupies any public property, it shall be removed at the expense of the owner of
the building or house. 4 years later, after the term of Fajardo as mayor had expired, he and his son-in-law,
Pedro Babilonia, filed a written request with the incumbent municipal mayor for a permit to construct a
building adjacent to their gasoline station on a parcel of land registered in Fajardo's name, located along the
national highway and separated from the public plaza by a creek. On 16 January 1954, the request was
denied, for the reason among others that the proposed building would destroy the view or beauty of the public
plaza. On 18 January 1954, Fajardo and Babilonia reiterated their request for a building permit, but again the
request was turned down by the mayor. Whereupon, Fajardo and Babilonia proceeded with the construction of
the building without a permit, because they needed a place of residence very badly, their former house having
been destroyed by a typhoon and hitherto they had been living on leased property. On 26 February 1954,
Fajardo and Babilonia were charged before and convicted by the justice of the peace court of Baao,
Camarines Sur, for violation of Ordinance 7. Fajardo and Babilonia appealed to the Court of First Instance
(CDI), which affirmed the conviction, and sentenced both to pay a fine of P35 each and the costs, as well as to
demolish the building in question because it destroys the view of the public plaza of Baao. From this decision,
Fajardo and Babilonia appealed to the Court of Appeals, but the latter forwarded the records to the Supreme
Court because the appeal attacks the constitutionality of the ordinance in question.

ISSUE
Whether the refusal of the Mayor of Baao to issue a building permit on the ground that the proposed building
would destroy the view of the public plaza is an undue deprivation of the use of the property in
question, and thus a taking without due compensation.

HELD
The refusal of the Mayor of Baao to issue a building permit to Fajardo and Babilonia was predicated on
the ground that the proposed building would "destroy the view of the public plaza" by preventing its being
seen from the public highway. Even thus interpreted, the ordinance is unreasonable and oppressive, in that it
operates to permanently deprive the latter of the right to use their own property; hence, it oversteps the
bounds of police power, and amounts to a taking of the property without just compensation. But while
property may be regulated in the interest of the general welfare such as to regard the beautification of
neighborhoods as conducive to the comfort and happiness of residents), and in its pursuit, the State may
prohibit structures offensive to the sight, the State may not, under the guise of police power, permanently
divest owners of the beneficial use of their property and practically confiscate them solely to preserve or
assure the aesthetic appearance of the community. As the case now stands, every structure that may be erected
on Fajardo's land, regardless of its own beauty, stands condemned under the ordinance in question, because it
would interfere with the view of the public plaza from the highway. Fajardo would, in effect, be constrained
to let their land remain idle and unused for the obvious purpose for which it is best suited, being urban in
character. To legally achieve that result, the municipality must give Fajardo just compensation and an
opportunity to be heard.

REPUBLIC VS. CA9


PONENCIA:
TOPIC:
TRIGGER OF THE FACTS:
TRIGGER OF THE ISSUE:
TRIGGER OF THE RULING:
VERDICT:
END POINT:

9 G.R. No. 146587 July 2, 2002


Page 38 of 236
Constitutional Law 2 Bill of Rights Armando Santiago Jr

FACTS
Petitioner (PIA) instituted expropriation proceedings covering a total of 544,980 square meters of contiguous
land situated along MacArthur Highway, Malolos, Bulacan, to be utilized for the continued broadcast operation
and use of radio transmitter facilities for the Voice of the Philippines project.

Petitioner made a deposit of P517,558.80, the sum provisionally fixed as being the reasonable value of the
property. On 26 February 1979, or more than 9 years after the institution of the expropriation proceedings, the
trial court issued this order condemning the property and ordering the plaintiff to pay the defendants the just
compensation for the property.

It would appear that the National Government failed to pay the respondents the just compensation pursuant to
the foregoing decision. The respondents then filed a manifestation with a motion seeking payment for the
expropriated property. In response, the court issued a writ of execution for the implementation thereof.

Meanwhile, Pres. Estrada issued Proc. No. 22 transferring 20 hectares of the expropriated land to the Bulacan
State University.

Despite the courts order, the Santos heirs remained unpaid and no action was on their case until petitioner filed
its manifestation and motion to permit the deposit in court of the amount P4,664,000 by way of just
compensation.

The Santos heirs submitted a counter-motion to adjust the compensation from P6/sq.m. as previously fixed to its
current zonal value of P5,000/sq.m. or to cause the return of the expropriated property.

The RTC Bulacan ruled in favor of the Santos heirs declaring its 26 February 1979 Decision to be unenforceable
on the ground of prescription in accordance with Sec. 6, Rule 39 of the 1964/1997 ROC which states that a final
and executory judgment or order may be executed on motion within 5 years from the date of its entry. RTC
denied petitioners Motion to Permit Deposit and ordered the return of the expropriated property to the heirs of
Santos.

ISSUES
1. WON the petitioner may appropriate the property
2. WON the respondents are entitled to the return of the property in question

HELD
1. The right of eminent domain is usually understood to be an ultimate right of the sovereign power to
appropriate any property within its territorial sovereignty for a public purpose. Fundamental to the independent
existence of a State, it requires no recognition by the Constitution, whose provisions are taken as being merely
confirmatory of its presence and as being regulatory, at most, in the due exercise of the power. In the hands of
the legislature, the power is inherent, its scope matching that of taxation, even that of police power itself, in many
respects. It reaches to every form of property the State needs for public use and, as an old case so puts it, all
separate interests of individuals in property are held under a tacit agreement or implied reservation vesting upon
the sovereign the right to resume the possession of the property whenever the public interest so requires it.

The ubiquitous character of eminent domain is manifest in the nature of the expropriation proceedings.
Expropriation proceedings are not adversarial in the conventional sense, for the condemning authority is not
required to assert any conflicting interest in the property. Thus, by filing the action, the condemnor in effect
merely serves notice that it is taking title and possession of the property, and the defendant asserts title or
interest in the property, not to prove a right to possession, but to prove a right to compensation for the taking.

Page 39 of 236
Constitutional Law 2 Bill of Rights Armando Santiago Jr

Obviously, however, the power is not without its limits: first, the taking must be for public use, and second, that
just compensation must be given to the private owner of the property. These twin proscriptions have their origin
in the recognition of the necessity for achieving balance between the State interests, on the one hand, and
private rights, upon the other hand, by effectively restraining the former and affording protection to the latter. In
determining public use, two approaches are utilized - the first is public employment or the actual use by the
public, and the second is public advantage or benefit. It is also useful to view the matter as being subject to
constant growth, which is to say that as society advances, its demands upon the individual so increases, and
each demand is a new use to which the resources of the individual may be devoted.

The expropriated property has been shown to be for the continued utilization by the PIA, a significant portion
thereof being ceded for the expansion of the facilities of the Bulacan State University and for the propagation of
the Philippine carabao, themselves in line with the requirements of public purpose. Respondents question the
public nature of the utilization by petitioner of the condemned property, pointing out that its present use differs
from the purpose originally contemplated in the 1969 expropriation proceedings. The argument is of no moment.
The property has assumed a public character upon its expropriation. Surely, petitioner, as the condemnor and as
the owner of the property, is well within its rights to alter and decide the use of that property, the only limitation
being that it be for public use, which, decidedly, it is.

2. NO. In insisting on the return of the expropriated property, respondents would exhort on the pronouncement in
Provincial Government of Sorsogon vs. Vda. de Villaroya where the unpaid landowners were allowed the
alternative remedy of recovery of the property there in question. It might be borne in mind that the case involved
the municipal government of Sorsogon, to which the power of eminent domain is not inherent, but merely
delegated and of limited application. The grant of the power of eminent domain to local governments under
Republic Act No. 7160 cannot be understood as being the pervasive and all-encompassing power vested in the
legislative branch of government. For local governments to be able to wield the power, it must, by enabling law,
be delegated to it by the national legislature, but even then, this delegated power of eminent domain is not,
strictly speaking, a power of eminent, but only of inferior, domain or only as broad or confined as the real
authority would want it to be.

Thus, in Valdehueza vs. Republic where the private landowners had remained unpaid ten years after the
termination of the expropriation proceedings, this Court ruled -

The points in dispute are whether such payment can still be made and, if so, in what amount. Said lots have
been the subject of expropriation proceedings. By final and executory judgment in said proceedings, they were
condemned for public use, as part of an airport, and ordered sold to the government. x x x It follows that both by
virtue of the judgment, long final, in the expropriation suit, as well as the annotations upon their title certificates,
plaintiffs are not entitled to recover possession of their expropriated lots - which are still devoted to the public use
for which they were expropriated - but only to demand the fair market value of the same.

"Said relief may be granted under plaintiffs' prayer for: `such other remedies, which may be deemed just and
equitable under the premises'."

The Court proceeded to reiterate its pronouncement in Alfonso vs. Pasay City where the recovery of possession
of property taken for public use prayed for by the unpaid landowner was denied even while no requisite
expropriation proceedings were first instituted. The landowner was merely given the relief of recovering
compensation for his property computed at its market value at the time it was taken and appropriated by the
State.

The judgment rendered by the Bulacan RTC in 1979 on the expropriation proceedings provides not only for the
payment of just compensation to herein respondents but likewise adjudges the property condemned in favor of
petitioner over which parties, as well as their privies, are bound. Petitioner has occupied, utilized and, for all
intents and purposes, exercised dominion over the property pursuant to the judgment. The exercise of such
rights vested to it as the condemnee indeed has amounted to at least a partial compliance or satisfaction of the
1979 judgment, thereby preempting any claim of bar by prescription on grounds of non-execution. In arguing for
the return of their property on the basis of non-payment, respondents ignore the fact that the right of the

Page 40 of 236
Constitutional Law 2 Bill of Rights Armando Santiago Jr

expropriatory authority is far from that of an unpaid seller in ordinary sales, to which the remedy of rescission
might perhaps apply. An in rem proceeding, condemnation acts upon the property. After condemnation, the
paramount title is in the public under a new and independent title; thus, by giving notice to all claimants to a
disputed title, condemnation proceedings provide a judicial process for securing better title against all the world
than may be obtained by voluntary conveyance.

Respondents, in arguing laches against petitioner did not take into account that the same argument could
likewise apply against them. Respondents first instituted proceedings for payment against petitioner on 09 May
1984, or five years after the 1979 judgment had become final. The unusually long delay in bringing the action to
compel payment against herein petitioner would militate against them. Consistently with the rule that one should
take good care of his own concern, respondents should have commenced the proper action upon the finality of
the judgment which, indeed, resulted in a permanent deprivation of their ownership and possession of the
property.

The constitutional limitation of just compensation is considered to be the sum equivalent to the market value of
the property, broadly described to be the price fixed by the seller in open market in the usual and ordinary course
of legal action and competition or the fair value of the property as between one who receives, and one who
desires to sell, it fixed at the time of the actual taking by the government. Thus, if property is taken for public use
before compensation is deposited with the court having jurisdiction over the case, the final compensation must
include interests on its just value to be computed from the time the property is taken to the time when
compensation is actually paid or deposited with the court. In fine, between the taking of the property and the
actual payment, legal interests accrue in order to place the owner in a position as good as (but not better than)
the position he was in before the taking occurred.

The Bulacan trial court, in its 1979 decision, was correct in imposing interests on the zonal value of the property
to be computed from the time petitioner instituted condemnation proceedings and took the property in
September 1969. This allowance of interest on the amount found to be the value of the property as of the time of
the taking computed, being an effective forbearance, at 12% per annum should help eliminate the issue of the
constant fluctuation and inflation of the value of the currency over time. Article 1250 of the Civil Code, providing
that, in case of extraordinary inflation or deflation, the value of the currency at the time of the establishment of
the obligation shall be the basis for the payment when no agreement to the contrary is stipulated, has strict
application only to contractual obligations. In other words, a contractual agreement is needed for the effects of
extraordinary inflation to be taken into account to alter the value of the currency.

All given, the trial court of Bulacan in issuing its order, dated 01 March 2000, vacating its decision of 26 February
1979 has acted beyond its lawful cognizance, the only authority left to it being to order its execution. Verily,
private respondents, although not entitled to the return of the expropriated property, deserve to be paid promptly
on the yet unpaid award of just compensation already fixed by final judgment of the Bulacan RTC on 26
February 1979 at P6.00 per square meter, with legal interest thereon at 12% per annum computed from the date
of "taking" of the property, i.e., 19 September 1969, until the due amount shall have been fully paid.

Page 41 of 236
Constitutional Law 2 Bill of Rights Armando Santiago Jr

DUE PROCESS AND EQUAL


PROTECTION OF LAWS CLAUSE
Section 1: No person shall be deprived of life, liberty or property
without due process of law, nor shall any person be denied the equal
protection of the laws.

REQUISITES FOR VALID EXERCISE OF INHERENT POWERS


The yardsticks for the exercise of the inherent powers of the government are:
1. Due Process Clause
2. Equal Protection Clause

GIVE THE TWO ASPECTS OF DUE PROCESS AND DISCUSS EACH


The two aspects of due process are:

Substantive due process


simply means that the law be reasonable and not arbitrary.

Procedural due process


meant a law that hears before it condemns; that proceeds upon inquiry, and renders judgment only after trial.

DO LIFE AND PROPERTY ENJOY IDENTICAL PROTECTION FROM THE


CONSTITUTION?
NO. The primacy of human rights over property rights is recognized. Property rights can be lost through
prescription while human rights are imprescriptible.

DOES EQUAL PROTECTION OF LAW PROHIBIT CLASSIFICATION?


NO, but the classification must be reasonable. To be reasonable, it;

1. Must rest on substantial distinction;


2. Must be germane to the purpose of law;
3. Must not be limited to existing conditions only;
4. Must apply equally to all members of the same class.

GIVE THE MEANING/RELATIVITY OF DUE PROCESS ACCORDING TO THE CASE


OF US V. LING SU FAN
In the case of U.S. v. Ling Su Fan due process simply means:

That there shall be a law prescribed in harmony with the general powers of the legislative department of
the Government;
That this law shall be reasonable in its operation;
That it shall be enforced according to the regular methods of procedure prescribed; and
It shall be applicable alike to all citizens of a state or to all of a class.

Page 42 of 236
Constitutional Law 2 Bill of Rights Armando Santiago Jr

GIVE THE ESSENTIALS OF PROCEDURAL DUE PROCESS IN JUDICIAL


PROCEEDINGS ACCORDING TO THE CASE OF BANCO ESPANOL V. PALANCA
In the case of Banco Espanol Filipino v. Palanca, the essentials of procedural due process in JUDICIAL
PROCEEDINGS are:

1. There must be a court or tribunal clothed with judicial power to hear and determine the matter before it;
2. Jurisdiction must be lawfully acquired over the person of the defendant or over the property which is the
subject of the proceedings;
3. The defendant must be given opportunity to be heard; and
4. Judgment must be rendered upon lawful hearing.

GIVE THE CARDINAL PRIMARY REQUIREMENTS OF DUE PROCESS IN


ADMINISTRATIVE PROCEEDINGS
In the case of Ang Tibay v. Court of Industrial Relations, the Cardinal primary requirements in
ADMINISTRATIVE PROCEEDINGS were summarized as follows:

The right to a hearing, which includes the right to present ones case and submit evidence thereof;
The tribunal must consider the evidence presented;
The decision must have something to support itself;
The evidence must be substantial (such reasonable evidence as a reasonable mind might accept as
adequate to support a conclusion).
The decision must be based on evidence presented at the hearing, or at least contained in the record
and disclosed to the parties affected;
The tribunal or body or any of its judges must act on its independent consideration of the law and facts
of the controversy, and not simply accept the views of a subordinate; and
The Board or body should, in all controversial questions, render its decision in such manner that the
parties to the proceeding can know the various issues involved, and the reason for the decision rendered.

GIVE THE MINIMUM STANDARS WHICH MUST BE MET BY THE SCHOOLS TO


SATISFY THE DEMANDS OF PROCEDURAL DUE PROCESS
In Guzman v. National University, the Supreme Court provided the guidelines for the handling of disciplinary
cases in schools:

The students must be informed in writing of the nature and cause of an accusation against them;
They shall have the right to answer the charges against them, with the assistance of counsel, if desired;
They shall be informed of the evidence against them;
They shall have the right to adduce evidence in their own behalf; and
The evidence must be duly considered by the investigating committee or official designated by the
school authorities to hear and decide the case.

CONSTITUTIONAL AND STATUTORY DUE PROCESS


No person shall be deprived of life, liberty or property without due process of law, nor shall any person be
denied the equal protection of the laws (Section 1, Art. III).

Statutory due process: Laws shall take effect after fifteen (15) days following the completion of their publication
either in the Official Gazette or in a newspaper of general circulation, unless it is otherwise provided (Art. 2 Civil
Code of the Philippines).

EXPLAIN VOID FOR VAGUENESS DOCTRINE

Page 43 of 236
Constitutional Law 2 Bill of Rights Armando Santiago Jr

In People v. Nazario 186, 195-196 (1088) the Supreme Court said:

As a rule, a statute or act may be said to be vague when it lacks comprehensible standards that men of
common intelligence must necessarily guess at its meaning and differ in its application.

It is repugnant to the Constitution in two aspects:


it violates due process for failure to accord persons, especially the parties targeted by it, fair notice of the
conduct to avoid; and
it leaves law enforcers unbridled discretion in carrying out its provisions and becomes an arbitrary flexing
of the Government muscle.

CONCEPT OF EQUAL PROTECTION


In Tolentino v. Board of Accountancy), the Supreme Court said: The guarantee of equal protection means that
no person or class of persons shall be deprived of the same protection of the laws which is enjoyed by other
persons or other classes in the same place and in like circumstances.

STANDARDS OF JUDICIAL REVIEW ON EQUAL PROTECTION

RATIONAL BASIS TEST


A law that touches on a constitutionally protected interest must be rationally related to furthering a legitimate
government interest. In applying the rational basis test, courts begin with a strong presumption that the law or
policy under review is valid.

The classification should bear a reasonable relation to government's purpose, and the legislative classification is
presumed valid.

Notes:
! Important when there difference between the disadvantaged class and those not disadvantaged.
! Also important when the government attaches a morally irrelevant and negative significance to a difference
between the advantaged and the disadvantaged.

STRICT SCRUTINY TEST


This test is triggered when a fundamental constitutional right is limited by a law. This requires the government to
show an overriding or compelling government interest so great that it justifies the limitation of fundamental
constitutional rights (the courts make the decision of WON the purpose of the law makes the classification
necessary).

Applied also when the classification has a "suspect" basis (Suspect Classes classes subject to such a history of
purposeful unequal treatment or relegated to such a position of political powerlessness as to command
extraordinary protection from the majoritarian political process.)

To pass strict scrutiny, the law or policy must satisfy three (3) tests:
It must be justified by a compelling governmental interest;
The law or policy must be narrowly tailored to achieve the goal or interest; and
The law or policy must be the least restrictive means for achieving that interest, that is, there cannot be a
less restrictive way to effectively achieve the compelling government interest.

INTERMEDIATE SCRUTINY TEST


A third standard, denominated as heightened or immediate scrutiny, was later adopted by the U.S. Supreme
Court for evaluating classifications based on gender and legitimacy. Immediate scrutiny was adopted by the U.S.
Supreme Court in Craig. While the test may have first been articulated in equal protection analysis, it has in the
United States since been applied in all substantive due process cases as well. [White Light Corporation vs.
City of Manila (2009)]

Page 44 of 236
Constitutional Law 2 Bill of Rights Armando Santiago Jr

In Ormoc Sugar Central v. Ormoc City, Ormoc City imposes a tax on Ormoc Sugar Central by name. Ormos
Sugar Central is the only sugar central in Ormoc City. The Court held that such ordinance is not valid for it would
be discriminatory against the Ormoc Sugar Central which alone comes under the ordinance

CASES
ALFONSO C. BINCE, JR VS. COMMISSION ON ELECTIONS,
PROVINCIAL BOARD OF CANVASSERS OF PANGASINAN,
MUNICIPAL BOARDS OF CANVASSERS OF TAYUG AND SAN
MANUEL, PANGASINAN, AND EMILIANO MICU
PONENCIA:
TOPIC:
TRIGGER OF THE FACTS:
TRIGGER OF THE ISSUE:
TRIGGER OF THE RULING:
VERDICT: Respondent COMELEC did not commit grave abuse of discretion in setting aside the illegal proclamation.
END POINT: Public office is not a property or a right but a privilege which people has the power to take the s

FACTS
Petitioner Alfonso C. Bince, Jr. and private respondent Emiliano S. Micu were among the candidates in the
synchronized elections of May 11, 1992 for a seat in the Sangguniang Panlalawigan of the Province of
Pangasinan allotted to its Sixth Legislative District.

Ten (10) municipalities, including San Quintin, Tayug and San Manuel, comprise the said district. During the
canvassing of the Certificates of Canvass (COCs) for these ten (10) municipalities, respondent Provincial Board
of Canvassers (PBC) on May 20, 1992, private respondent Micu objected to the inclusion of the COC for San
Quintin on the ground that it contained false statements. Accordingly, the COCs for the remaining nine (9)
municipalities were included in the canvass. On May 21, 1992, the PBC ruled against the objection of private
respondent. From the said ruling, private respondent Micu appealed to the Commission on Elections
(COMELEC), which docketed the case as SPC No. 92-208.

ISSUE
Whether or not respondent COMELEC commit grave abuse of discretion in setting aside the illegal
proclamation.

RULING
Respondent COMELEC did not act without jurisdiction or with grave abuse of discretion in annulling the
proclamation of petitioner Alfonso Bince, Jr. and in directing the Provincial Board of Canvassers of Pangasinan
to order the Municipal Boards of Canvassers of Tayug and San Manuel to make the necessary corrections in the
SOVs and COCs in said municipalities and to proclaim the winner in the sixth legislative district of Pangasinan.
At the outset, it is worthy to observe that no error was committed by respondent COMELEC when it resolved the
"pending incidents" of the instant case pursuant to the decision of this Court in the aforesaid case of Bince, Jr. v.
COMELEC on February 9, 1993. Petitioner's contention that his proclamation has long been affirmed and
confirmed by this Court in the aforesaid case is baseless. In Bince, we nullified the proclamation of private
respondent because the same was done without the requisite due notice and hearing, thereby depriving the

Page 45 of 236
Constitutional Law 2 Bill of Rights Armando Santiago Jr

petitioner of his right to due process. In so doing, however, we did not affirm nor confirm the proclamation of
petitioner, hence, our directive to respondent COMELEC to resolve the pending incidents of the case so as to
ascertain the true and lawful winner of the said elections. In effect, petitioner's proclamation only enjoyed the
presumption of regularity and validity of an official act. It was not categorically declared valid. Neither can the
COMELEC be faulted for subsequently annulling the proclamation of petitioner Bince on account of a
mathematical error in addition committed by respondent MBCs in the computation of the votes received by both
petitioner and private respondent. The petitions to correct manifest errors were filed on time, that is, before the
petitioner's proclamation on July 21, 1992. The petition of the MBC of San Manuel was filed on June 4, 1992
while that of the MBC of Tayug was filed on June 5, 1992. Still, private respondent's petition was filed with the
MBCs of Tayug and San Manuel on June 10, 1992 and June 11, 1992, respectively, definitely well within the
period required by Section 6 (now Section 7), Rule 27 of the COMELEC Rules of Procedure. Section 6 clearly
provides that the petition for correction may be filed at any time before proclamation of a winner. The rule is plain
and simple. It needs no other interpretation contrary to petitioner's protestation.

Undoubtedly therefore, the only issue that remains unresolved is the allowance of the correction of what are
purely mathematical and/or mechanical errors in the addition of the votes received by both candidates. It does
not involve the opening of ballot boxes; neither does it involve the examination and/or appreciation of ballots.
The correction sought by private respondent and respondent MBCs of Tayug and San Manuel is correction of
manifest mistakes in mathematical addition. Certainly, this only calls for a mere clerical act of reflecting the true
and correct votes received by the candidates by the MBCs involved. In this case, the manifest errors sought to
be corrected involve the proper and diligent addition of the votes in the municipalities of Tayug and San Manuel,
Pangasinan. In Tayug, the total votes received by petitioner Bince was erroneously recorded as 2,486 when it
should only have been 2,415. Petitioner Bince, in effect, was credited by 71 votes more. In San Manuel,
petitioner Bince received 2,179 votes but was credited with 6 votes more, hence, the SOV reflected the total
number of votes as 2,185. On the other hand, the same SOV indicated that private respondent Micu garnered
2,892 votes but he actually received only 2,888, hence was credited in excess of 4 votes. Consequently, by
margin of 72 votes, private respondent indisputably won the challenged seat in the Sangguniang Panlalawigan
of the sixth district of Pangasinan. Petitioner's proclamation and assumption into public office was therefore
flawed from the beginning, the same having been based on a faulty tabulation. Hence, respondent COMELEC
did not commit grave abuse of discretion in setting aside the illegal proclamation.

TABUENA V. SANDIGANBAYAN10
PONENCIA:
TOPIC:
TRIGGER OF THE FACTS:
TRIGGER OF THE ISSUE:
TRIGGER OF THE RULING:
VERDICT:
END POINT:

FACTS
Then President Marcos instructed Luis Tabuena over the phone to pay directly to the president's office and in
cash what the Manila International Airport Authority (MIAA) owes the Philippine National Construction
Corporation (PNCC), pursuant to the 7 January 1985 memorandum of then Minister Trade and Industry Roberto
Ongpin. Tabuena agreed. About a week later, Tabuena received from Mrs. Fe Roa-Gimenez, then private
secretary of Marcos, a Presidential Memorandum dated 8 January 1986 reiterating in black and white such
verbal instruction. In obedience to President Marcos' verbal instruction and memorandum, Tabuena, with the
help of Gerardo G. Dabao and Adolfo Peralta, caused the release of P55 Million of MIAA funds by means of
three (3) withdrawals. On 10 January 1986, the first withdrawal was made for P25 Million, following a letter of
even date signed by Tabuena and Dabao requesting the PNB extension office at the MIAA the depository branch
of MIAA funds, to issue a manager's check for said amount payable to Tabuena. The check was encashed,
however, at the PNB Villamor Branch. Dabao and the cashier of the PNB Villamor branch counted the money

10 [GR 103501-03, 17 February 1997]; also Peralta v. Sandiganbayan [GR 103507]


Page 46 of 236
Constitutional Law 2 Bill of Rights Armando Santiago Jr

after which, Tabuena took delivery thereof. The P25 Million in cash was delivered on the same day to the office
of Mrs. Gimenez. Mrs. Gimenez did not issue any receipt for the money received. Similar circumstances
surrounded the second withdrawal/encashment and delivery of another P25 Million, made on 16 January 1986.
The third and last withdrawal was made on 31 January 1986 for P5 Million. Peralta was Tabuena's co-signatory
to the letter- request for a manager's check for this amount. Peralta accompanied Tabuena to the PNB Villamor
branch as Tabuena requested him to do the counting of the P5 Million. After the counting, the money was loaded
in the trunk of Tabuena's car. Peralta did not go with Tabuena to deliver the money to Mrs. Gimenez' office. It
was only upon delivery of the P5 Million that Mrs. Gimenez issued a receipt for all the amounts she received
from Tabuena. The receipt was dated January 30,1986. Tabuena and Peralta were charged for malversation of
funds, while Dabao remained at large. One of the justices of the Sandiganbayan actively took part in the
questioning of a defense witness and of the accused themselves; the volume of the questions asked were more
the combined questions of the counsels. On 12 October 1990, they were found guilty beyond reasonable doubt.
Tabuena and Peralta filed separate petitions for review, appealing the Sandiganbayan decision dated 12 October
19990 and the Resolution of 20 December 1991.

ISSUE
Whether Tabuena and Peralta were denied due process by the active participation of a Sandiganbayan
justice in the questioning witnesses in the trial.

HELD
Due process requires no less than the cold neutrality of an impartial judge. Bolstering this requirement, we have
added that the judge must not only be impartial but must also appear to be impartial, to give added assurance to
the parties that his decision will be just. The parties are entitled to no less than this, as a minimum guaranty of
due process. Our courts should refrain from showing any semblance of one-sided or more or less partial attitude
in order not to create any false impression in the minds of the litigants. For obvious reasons, it is the bounden
duty of all to strive for the preservation of the people's faith in our courts. Respect for the Constitution is more
important than securing a conviction based on a violation of the rights of the accused. The Court was struck by
the way the Sandiganbayan actively took part in the questioning of a defense witness and of the accused
themselves, as shown in the records. The volume of questions hurled by the Sandiganbayan was more the
combined questions of the counsels. More importantly, the questions of the court were in the nature of cross
examinations characteristic of confrontation, probing and insinuation. We have not adopted in this country the
practice of making the presiding judge the chief inquisitor. It is better to observe our time-honored custom of
orderly judicial procedure, even at the expense of occasional delays. The impartiality of the judge; his avoidance
of the appearance of becoming the advocate of either one side or the other of the pending controversy is a
fundamental and essential rule of special importance in criminal cases.

SON UN GIOK VS MATUSA


PONENCIA:
TOPIC:
TRIGGER OF THE FACTS:
TRIGGER OF THE ISSUE:
TRIGGER OF THE RULING:
VERDICT:
END POINT: kase nga ang nangyari diba di na siiya nabigyan ng chance to present his case dahil nga ang nangyari
parang nadisregard na yung motion to dimiss niya when he was declared in default

FACTS
Matusa filed a complaint against Son un giok alleging that he was employed by the latter as gate keeper in his
theater and that throughout the duration of said employment
he was bot given the corresponding overtime pay for all his overtime in accordance with the 8-hour Labor Law.
and that he was also [paid below the amount fixed vy the minimum wage law.
The counsel of Son un Giok filed a motion to dismiss on the ground that the action was already barred by the
statute of limitations.

Page 47 of 236
Constitutional Law 2 Bill of Rights Armando Santiago Jr

A receipt that a copy of the pleading (motion to dismiss) together with the pleading itself was submitted to the
Clerk of Cour. However, during the hearing for the said motion,
Matusa averred that he was notified for the said hearing.
Son un Giok was asked to produce proof that the adverse party was notified but he failed to do so.
The Court then decided to postpone the hearing of the said case until Son Un Giok could produce the proof of
service.
But before he could do so, Matusa file a motion to declare the former in default which was granted by the
Court.
Thereafter, Son Un Giok filed a motion to lift order of default and to vacate judgment arguing that he should
not be declared in default as there was a pending motion to dismiss which disturbed the time for the filing of an
answer and that the said motion to dismiss has not been resolved because the Court required him to produce
a proof that the adverse party had received a copy of said pleading.
Furthermore, he contends that the reason why the said requirement was not yet complied with was because of
the fact that the receipt of said pleading was not yet forwarded to him by the post office.
The said motio was denied.
The motion for reconsideration for the same was also denied.
Hence the present petition.
In the answer filed by Matusa, he was notified but he alleges that the the said pleading was considered as a
mere scrap of paper as it failed to comply with the essential requisites under the Rules of Court

ISSUE
WON the service of notice made by the counsel of Son Un Giok should be allowed

HELD
But let us grant, for the sake of argument, that the notice in question, as quoted above is defective, for failure to
specify the exact date when that motion should be heard. Even so, We believe that the Court in taking
cognizance of the motion on the date set for hearing thereof, cured whatever iota of defect such pleading may
have had, specially if it is taken into account that upon receipt of the motion to dismiss, plaintiff was properly
notified of the existence of said pleading. Counsel for plaintiff should have not relied on mere technicalities which
in the interest of justice may be relaxed for it was said that:

A litigation is not a game of technicalities in which one, more deeply schooled and skilled in the subtle art of
movement and position, entraps and destroys the other. Alonso vs. Villamor, 16 Phil. 315.)
As the motion was heard after this notice, and strictly in compliance with the Rules of Court, it cannot be said
that the hearing was held without due process of law. What the law prohibits is not the absence of previous
notice, but the absolute absence thereof and lack of opportunity to be heard. (Borja vs. Tan, 93 Phil., 167; Duran
Embate vs. Penolio, 93 Phil., 782, 49 Off. Gaz. [9] 3850)

Parties litigants shall not be deprived of their day in court as it was done in the case at bar.

PEOPLE VS. JUDGE VERA


65 Phil. 56 Political Law Constitutional Law Bill of Rights Equal Protection Probation Law

Separation of Powers Undue Delegation of Powers Power to Pardon

Constitutionality of Laws May the State Question Its Own Laws

FACTS

Page 48 of 236
Constitutional Law 2 Bill of Rights Armando Santiago Jr

Mariano Cu Unjieng was convicted in a criminal case filed against him by the Hongkong and Shanghai Banking
Corporation (HSBC). he filed for probation. The matter was referred to the Insular Probation Office which
recommended the denial of Cu Unjiengs petition for probation. A hearing was set by Judge Jose Vera
concerning the petition for probation. The Prosecution opposed the petition. Eventually, due to delays in the
hearing, the Prosecution filed a petition for certiorari with the Supreme Court alleging that courts like the Court of
First Instance of Manila (which is presided over by Judge Vera) have no jurisdiction to place accused like Cu
Unjieng under probation because under the law (Act No. 4221 or The Probation Law), probation is only meant to
be applied in provinces with probation officers; that the City of Manila is not a province, and that Manila, even if
construed as a province, has no designated probation officer hence, a Manila court cannot grant probation.

Meanwhile, HSBC also filed its own comment on the matter alleging that Act 4221 is unconstitutional for it
violates the constitutional guarantee on equal protection of the laws. HSBC averred that the said law makes it
the prerogative of provinces whether or nor to apply the probation law if a province chooses to apply the
probation law, then it will appoint a probation officer, but if it will not, then no probation officer will be appointed
hence, that makes it violative of the equal protection clause.

Further, HSBC averred that the Probation Law is an undue delegation of power because it gave the option to the
provincial board to whether or not to apply the probation law however, the legislature did not provide guidelines
to be followed by the provincial board.

Further still, HSBC averred that the Probation Law is an encroachment of the executives power to grant pardon.
They say that the legislature, by providing for a probation law, had in effect encroached upon the executives
power to grant pardon. (Ironically, the Prosecution agreed with the issues raised by HSBC ironic because their
main stance was the non-applicability of the probation law only in Manila while recognizing its application in
provinces).

For his part, one of the issues raised by Cu Unjieng is that, the Prosecution, representing the State as well as
the People of the Philippines, cannot question the validity of a law, like Act 4221, which the State itself created.
Further, Cu Unjieng also castigated the fiscal of Manila who himself had used the Probation Law in the past
without question but is now questioning the validity of the said law (estoppel).

ISSUE
1. May the State question its own laws?

2. Is Act 4221 constitutional?

HELD
1. Yes. There is no law which prohibits the State, or its duly authorized representative, from questioning the
validity of a law. Estoppel will also not lie against the State even if it had been using an invalid law.

2. No, Act 4221 or the [old] Probation Law is unconstitutional.

Violation of the Equal Protection Clause

The contention of HSBC and the Prosecution is well taken on this note. There is violation of the equal protection
clause. Under Act 4221, provinces were given the option to apply the law by simply providing for a probation
officer. So if a province decides not to install a probation officer, then the accused within said province will be
unduly deprived of the provisions of the Probation Law.

Undue Delegation of Legislative Power

Page 49 of 236
Constitutional Law 2 Bill of Rights Armando Santiago Jr

There is undue delegation of legislative power. Act 4221 provides that it shall only apply to provinces where the
respective provincial boards have provided for a probation officer. But nowhere in the law did it state as to what
standard (sufficient standard test) should provincial boards follow in determining whether or not to apply the
probation law in their province. This only creates a roving commission which will act arbitrarily according to its
whims.

Encroachment of Executive Power

Though Act 4221 is unconstitutional, the Supreme Court recognized the power of Congress to provide for
probation. Probation does not encroach upon the Presidents power to grant pardon. Probation is not pardon.
Probation is within the power of Congress to fix penalties while pardon is a power of the president to commute
penalties.

PEDRO VS PROVINCIAL BOARD OF RIZAL11


PONENCIA:
TOPIC: Due Process
TRIGGER OF THE FACTS: their pushing to make the cockpit arena binded valid law.
TRIGGER OF THE ISSUE: such cockpit license is a right
TRIGGER OF THE RULING: No it is not a right but privilege. it can be revoked by the authority.
VERDICT:
END POINT: license is not a right but privilege hindi porket may lisensya right na yun no it is privilege na

FACTS
Gregorio Pedro argues for the nullity of Ordinance No. 36, series of 1928, approved by the temporary
councillors appointed by the provincial governor of Rizal, Eligio Naval, on the ground that
(1) it impairs the acquired rights of said appellant;
(2) it was enacted on account of prejudice, because it was intended for a special and not a general purpose,
namely to prevent, at any cost, the opening, maintenance, and exploitation of the cockpit of the said petitioner-
appellant; and
(3) it provides for special committee composed of persons who are not members of the council, vested them
with powers which of their very nature, cannot be delegated by said council to that committee.

He further contends that, having obtained the proper permit to maintain, exploit, and open to the public the
cockpit in question, having paid the license fee and fulfilled all the requirements provided by Ordinance No. 35,
series of 1928, he has acquired a right which cannot be taken away from him by Ordinance No. 36, series of
1928, which was subsequently approved.

ISSUE
Whether a license authorizing the operation and exploitation of a cockpit falls under property rights which a
person may not be deprived of without due process of law

HELD
No.

The court held: (1) That a license authorizing the operation and exploitation of a cockpit is not property of which
the holder may not be deprived without due process of law, but a mere privilege which may be revoked when
the public interests so require; (2) that the work entrusted by a municipal council to a special sanitary

11 G. R. No. 34163, September 18, 1931


Page 50 of 236
Constitutional Law 2 Bill of Rights Armando Santiago Jr

committee to make a study of the sanitary effects upon the neighborhood of the establishment of a cockpit, is not
legislative in character, but only informational, and may be delegated; and (3) that an ordinance, approved by a
municipal council duly constituted, which suspends the effects of another which had been enacted to favor the
grantee of a cockpit license, is valid and legal.

ROLITO GO VS CA12
PONENCIA:
TOPIC: Preliminary investigation
TRIGGER OF THE FACTS:
TRIGGER OF THE ISSUE:
TRIGGER OF THE RULING:
VERDICT:
END POINT: Preliminary investigation is a right. even if the case

FACTS
Rolito Go while traveling in the wrong direction on a one-way street, nearly bumped Eldon Maguans car. Go
alighted from his car, shot Maguan and left the scene. A security guard at a nearby restaurant was able to take
down petitioners car plate number. The police arrived shortly thereafter at the scene of the shooting. A manhunt
ensued.

Six days after, petitioner presented himself before the San Juan Police Station to verify news reports that he was
being hunted by the police; he was accompanied by two (2) lawyers. The police forthwith detained him. An
eyewitness to the shooting, who was at the police station at that time, positively identified petitioner as the
gunman.

Petitioner posted bail, the prosecutor filed the case to the lower court, setting and commencing trial without
preliminary investigation. Prosecutor reasons that the petitioner has waived his right to preliminary investigation
as bail has been posted and that such situation, that petitioner has been arrested without a warrant lawfully, falls
under Section 5, Rule 113 and Section 7, Rule 112 of The 1985 Rules of Criminal Procedure which provides for
the rules and procedure pertaining to situations of lawful warrantless arrests.

Petitioner argues that he was not lawfully arrested without warrant because he went to the police station six (6)
days after the shooting which he had allegedly perpetrated. Thus, petitioner argues, the crime had not been just
committed at the time that he was arrested. Moreover, none of the police officers who arrested him had been an
eyewitness to the shooting of Maguan and accordingly none had the personal knowledge required for the
lawfulness of a warrantless arrest. Since there had been no lawful warrantless arrest, Section 7, Rule 112 of the
Rules of Court which establishes the only exception to the right to preliminary investigation, could not apply in
respect of petitioner.

ISSUE/S
Whether or not a lawful warrantless arrest had been effected by the San Juan Police in respect of petitioner Go;

Whether petitioner had effectively waived his right to preliminary investigation

HELD
1. No. The Court does not believe that the warrantless arrest or detention of petitioner in the instant case falls
within the terms of Section 5 of Rule 113 of the 1985 Rules on Criminal Procedure which provides as follows:

Sec. 5. Arrest without warrant; when lawful. A peace officer or a private person may, without a warrant, arrest
a person;

12 G.R. No. 101837, February 11, 1992


Page 51 of 236
Constitutional Law 2 Bill of Rights Armando Santiago Jr

(a) When, in his presence, the person to be arrested has committed, is actually committing, or is attempting to
commit an offense;

(b) When an offense has in fact just been committed, and he has personal knowledge of facts indicating that the
person to be arrested has committed it; and

(c) When the person to be arrested is a prisoner who has escaped from a penal establishment or place where he
is serving final judgment or temporarily confined while his case is pending, or has escaped while being
transferred from one confinement to another.

In cases falling under paragraphs (a) and (b) hereof, the person arrested without a warrant shall be forthwith
delivered to the nearest police station or jail, and he shall be proceeded against in accordance with Rule 112,
Section 7.

Petitioners arrest took place six (6) days after the shooting of Maguan. The arresting officers obviously were
not present, within the meaning of Section 5(a), at the time petitioner had allegedly shot Maguan. Neither could
the arrest effected six (6) days after the shooting be reasonably regarded as effected when [the shooting had]
in fact just been committed within the meaning of Section 5 (b). Moreover, none of the arresting officers had
any personal knowledge of facts indicating that petitioner was the gunman who had shot Maguan. The
information upon which the police acted had been derived from statements made by alleged eyewitnesses to the
shooting one stated that petitioner was the gunman; another was able to take down the alleged gunmans
cars plate number which turned out to be registered in petitioners wifes name. That information did not,
however, constitute personal knowledge.

It is thus clear to the Court that there was no lawful warrantless arrest of petitioner within the meaning of Section
5 of Rule 113.

2. No. In the circumstances of this case, the Court does not believe that by posting bail, petitioner had waived his
right to preliminary investigation. In People v. Selfaison, the Court held that appellants there had waived their
right to preliminary investigation because immediately after their arrest, they filed bail and proceeded to trial
without previously claiming that they did not have the benefit of a preliminary investigation.

In the instant case, petitioner Go asked for release on recognizance or on bail and for preliminary investigation in
one omnibus motion. He had thus claimed his right to preliminary investigation before respondent Judge
approved the cash bond posted by petitioner and ordered his release on 12 July 1991. Accordingly, the Court
cannot reasonably imply waiver of preliminary investigation on the part of petitioner. In fact, when the Prosecutor
filed a motion in court asking for leave to conduct preliminary investigation, he clearly if impliedly recognized that
petitioners claim to preliminary investigation was a legitimate one.

SUNTAY V. PEOPLE13
PONENCIA:
TOPIC:
TRIGGER OF THE FACTS:
TRIGGER OF THE ISSUE:
TRIGGER OF THE RULING:
VERDICT:
END POINT: No need for notice and hearing for Go because there is reasonable cause to cancel the passport of Go
so that he may be hear and try.

FACTS
Dr. Antonio Nubla, father of Alicia Nubla, a minor of 16 years, filed a verified complaint against Emilio Suntay in
the Office of the City Attorney of Quezon City, alleging that on or about the accused took Alicia Nubla from St.

13 [GR L-9430, 29 June 1957]


Page 52 of 236
Constitutional Law 2 Bill of Rights Armando Santiago Jr

Paul's College in Quezon City with lewd design and took her to somewhere near the University of the
Philippines (UP) compound in Diliman and was then able to have carnal knowledge of her.
after an investigation, an Assistant City Attorney recommended to the City Attorney of Quezon City that the
complaint be dismissed for lack of merit.
attorney for the complainant addressed a letter to the City Attorney of Quezon City wherein he took exception
to the recommendation of the Assistant City Attorney referred to and urged that a complaint for seduction be
filed against Suntay.
Suntay applied for and was granted a passport by the Department of Foreign Affairs Suntay left the Philippines
for San Francisco, California, where he is at present enrolled in school.
Alicia Nubla subscribed and swore to a complaint charging Suntay with seduction which was filed, in the Court
of First Instance (CFI) Quezon City, after preliminary investigation had been conducted the private prosecutor
filed a motion praying the Court to issue an order "directing such government agencies as may be concerned,
particularly the National Bureau of Investigation and the Department of Foreign Affairs, for the purpose of
having the accused brought back to the Philippines so that he may be dealt with in accordance with law."
the Court granted the motion.
the Secretary cabled the Ambassador to the United States instructing him to order the Consul General in San
Francisco to cancel the passport issued to Suntay and to compel him to return to the Philippines to answer the
criminal charges against him.
However, this order was not implemented or carried out in view of the commencement of this proceedings in
order that the issues raised may be judicially resolved.
Suntays counsel wrote to the Secretary requesting that the action taken by him be reconsidered, and filed in
the criminal case a motion praying that the Court reconsider its order of the Secretary denied counsel's
request and the Court denied the motion for reconsideration.
Suntay filed the petition for a writ of certiorari.

ISSUE
Whether Suntay should be accorded notice and hearing before his passport may be cancelled.

HELD
Due process does not necessarily mean or require a hearing. When discretion is exercised by an officer vested
with it upon an undisputed fact, such as the filing of a serious criminal charge against the passport holder,
hearing may be dispensed with by such officer as a prerequisite to the cancellation of his passport; lack of such
hearing does not violate the due process of law clause of the Constitution; and the exercise of the discretion
vested in him cannot be deemed whimsical and capricious because of the absence of such hearing. If hearing
should always be held in order to comply with the due process of law clause of the Constitution, then a writ of
preliminary injunction issued ex parte would be violative of the said clause. Hearing would have been proper and
necessary if the reason for the withdrawal or cancellation of the passport were not clear but doubtful. But where
the holder of a passport is facing a criminal charge in our courts and left the country to evade criminal
prosecution, the Secretary for Foreign Affairs, in the exercise of his discretion (Section 25, EO 1, S. 1946, 42 OG
1400) to revoke a passport already issued, cannot be held to have acted whimsically or capriciously in
withdrawing and cancelling such passport. Suntays suddenly leaving the country in such a convenient time, can
reasonably be interpreted to mean as a deliberate attempt on his part to flee from justice, and, therefore, he
cannot now be heard to complain if the strong arm of the law should join together to bring him back to justice.

PEOPLE VS. CAYAT


PONENCIA:
TOPIC: Equal Protection
TRIGGER OF THE FACTS: law prohibit the indigenous group to magpossess ng alcoholic
TRIGGER OF THE ISSUE: Validity of the law and violated equal protection law
TRIGGER OF THE RULING: it did not violate the equal protection because it comply the 4 requisites of reasonable
classification.
VERDICT:
END POINT:

Page 53 of 236
Constitutional Law 2 Bill of Rights Armando Santiago Jr

FACTS
Accused Cayat, a native of Baguio, Benguet, Mountain Province, and a member of the non-Christian tribes, was
found guilty of violating sections 2 and 3 of Act No. 1639 for having acquired and possessed one bottle of A-1-1
gin, an intoxicating liquor, which is not a native wine. The law made it unlawful for any native of the Philippines
who is a member of a non-Christian tribe within the meaning of Act 1397 to buy, receive, have in his possession,
or drink any ardent spirits, ale, beer, wine or intoxicating liquors of any kind, other than the so-called native wines
and liquors which the members of such tribes have been accustomed to prior to the passage of the law. Cayat
challenges the constitutionality of Act 1639 on the grounds that it is discriminatory and denies the equal
protection of the laws, violates due process clause, and is an improper exercise of police power.

HELD
It is an established principle of constitutional law that the guaranty of the equal protection of the laws is not
violated by a legislation based on reasonable classification. (1) must rest on substantial distinctions; (2) must be
germane to the purposes of the law; (3) must not be limited to existing conditions only; and (4) must apply
equally to all members of the same class.

Act No. 1639 satisfies these requirements. The classification rests on real or substantial, not merely imaginary or
whimsical distinctions. It is not based upon accident of birth or parentage, as counsel for the appellant asserts,
but upon the degree of civilization and culture. The term non-Christian tribes refers, not to religious belief but in
a way, to the geographical area and more directly, to natives of the Philippine Islands of a low grade of
civilization, usually living in tribal relationship apart from settled communities. (Rubi vs. Provincial Board of
Mindora, supra.) This distinction is unquestionably reasonable, for the Act was intended to meet the peculiar
conditions existing in the non-Christian tribes.

The prohibition enshrined in Act 1397 is designed to insure peace and order in and among non-Christian tribes.
It applies equally to all members of the class evident from perusal thereof. That it may be unfair in its operation
against a certain number of non-Christians by reason of their degree of culture, is not an argument against the
equality of its application.

RUBI VS PROVINCIAL BOARD OF MINDORO


PONENCIA:
TOPIC:
TRIGGER OF THE FACTS:
TRIGGER OF THE ISSUE:
TRIGGER OF THE RULING:
VERDICT:
END POINT:

39 Phil. 660 Political Law Delegation of Powers Liberty and due process

FACTS
Rubi and various other Manguianes (Mangyans) in the province of Mindoro were ordered by the provincial
governor of Mindoro to remove their residence from their native habitat and to established themselves on a
reservation in Tigbao, still in the province of Mindoro, and to remain there, or be punished by imprisonment if
they escaped. Manguianes had been ordered to live in a reservation made to that end and for purposes of
cultivation under certain plans. The Manguianes are a Non-Christian tribe who were considered to be of very
low culture.

One of the Manguianes, a certain Dabalos, escaped from the reservation but was later caught and was placed in
prison at Calapan, solely because he escaped from the reservation. An application for habeas corpus was made
on behalf by Rubi and other Manguianes of the province, alleging that by virtue of the resolution of the provincial

Page 54 of 236
Constitutional Law 2 Bill of Rights Armando Santiago Jr

board of Mindoro creating the reservation, they had been illegally deprived of their liberty. In this case, the
validity of Section 2145 of the Administrative Code, which provides:

With the prior approval of the Department Head, the provincial governor of any province in which non-Christian
inhabitants are found is authorized, when such a course is deemed necessary in the interest of law and order, to
direct such inhabitants to take up their habitation on sites on unoccupied public lands to be selected by him and
approved by the provincial board.

was challenged.

ISSUE
Whether or not Section 2145 of the Administrative Code constitutes undue delegation. Whether or not the
Manguianes are being deprived of their liberty.

HELD
I. No. By a vote of five to four, the Supreme Court sustained the constitutionality of this section of the
Administrative Code. Under the doctrine of necessity, who else was in a better position to determine whether or
not to execute the law but the provincial governor. It is optional for the provincial governor to execute the law as
circumstances may arise. It is necessary to give discretion to the provincial governor. The Legislature may make
decisions of executive departments of subordinate official thereof, to whom it has committed the execution of
certain acts, final on questions of fact.

II. No. Among other things, the term non-Christian should not be given a literal meaning or a religious
signification, but that it was intended to relate to degrees of civilization. The term non-Christian it was said,
refers not to religious belief, but in a way to geographical area, and more directly to natives of the Philippine
Islands of a low grade of civilization. In this case, the Manguianes were being reconcentrated in the reservation
to promote peace and to arrest their seminomadic lifestyle. This will ultimately settle them down where they can
adapt to the changing times.

The Supreme Court held that the resolution of the provincial board of Mindoro was neither discriminatory nor
class legislation, and stated among other things: . . . one cannot hold that the liberty of the citizen is unduly
interfered with when the degree of civilization of the Manguianes is considered. They are restrained for their own
good and the general good of the Philippines. Nor can one say that due process of law has not been followed. To
go back to our definition of due process of law and equal protection of the laws, there exists a law; the law
seems to be reasonable; it is enforced according to the regular methods of procedure prescribed; and it applies
alike to all of a class.

ISAE V QUISIMBING14

FACTS
The ISM, under Presidential Decree 732, is a domestic educational institution established primarily for
dependents of foreign diplomatic personnel and other temporary residents.
The local-hires union of the ISM were crying foul over the disparity in wages that they got compared to that of
their foreign teaching counterparts.

These questions are asked to qualify a teacher into a local or foreign hire.

a.....What is one's domicile?


b.....Where is one's home economy?
c.....To which country does one owe economic allegiance?

14 G.R. No. 128845. June 1, 2000


Page 55 of 236
Constitutional Law 2 Bill of Rights Armando Santiago Jr

d.....Was the individual hired abroad specifically to work in the School and was the School responsible for
bringing that individual to the Philippines?

Should any answer point to Philippines, the person is a local hire. The School grants foreign-hires certain
benefits to the foreign hires such as housing, transportation, and 25% more pay than locals under the theory of

(a) the "dislocation factor" and


(b) limited tenure. The first was grounded on leaving his home country, the second was on the lack of tenure
when he returns home.

The negotiations between the school and the union caused a deadlock between the parties.

The DOLE resolved in favor of the school, while Dole Secretary Quisimbing denied the unions mfr.

He said, The Union cannot also invoke the equal protection clause to justify its claim of parity. It is an
established principle of constitutional law that the guarantee of equal protection of the laws is not violated by
legislation or private covenants based on reasonable classification. A classification is reasonable if it is based on
substantial distinctions and apply to all members of the same class. Verily, there is a substantial distinction
between foreign hires and local hires, the former enjoying only a limited tenure, having no amenities of their own
in the Philippines and have to be given a good compensation package in order to attract them to join the
teaching faculty of the School.

The union appealed to the Supreme Court.


The petitioner called the hiring system discriminatory and racist.
The school alleged that some local hires were in fact of foreign origin. They were paid local salaries.

ISSUE
Whether or not the hiring system is violative of the equal protection clause

HELD
Yes, Petition granted

RATIO
Public policy abhors discrimination. The Article on Social Justice and Human Rights exhorts Congress to "give
highest priority to the enactment of measures that protect and enhance the right of all people to human
dignity

The very broad Article 19 of the Civil Code requires every person, "in the exercise of his rights and in the
performance of his duties, [to] act with justice, give everyone his due, and observe honesty and good faith."
International law prohibits discrimination, such as the Universal Declaration of Human Rights and the
International Covenant on Economic, Social, and Cultural Rights. The latter promises Fair wages and equal
remuneration for work of equal value without distinction of any kind.

In the workplace, where the relations between capital and labor are often skewed in favor of capital, inequality
and discrimination by the employer are all the more reprehensible.

The Constitution also directs the State to promote "equality of employment opportunities for all." Similarly, the
Labor Code provides that the State shall "ensure equal work opportunities regardless of sex, race or creed.
Article 248 declares it an unfair labor practice for an employer to discriminate in regard to wages in order to
encourage or discourage membership in any labor organization.

In this jurisdiction, there is the term equal pay for equal work, pertaining to persons being paid with equal
salaries and have similar skills and similar conditions. There was no evidence here that foreign-hires perform
25% more efficiently or effectively than the local-hires.
Page 56 of 236
Constitutional Law 2 Bill of Rights Armando Santiago Jr

The State, therefore, has the right and duty to regulate the relations between labor and capital. These relations
are not merely contractual but are so impressed with public interest that labor contracts, collective bargaining
agreements included, must yield to the common good.

For the same reason, the "dislocation factor" and the foreign-hires' limited tenure also cannot serve as valid
bases for the distinction in salary rates. The dislocation factor and limited tenure affecting foreign-hires are
adequately compensated by certain benefits accorded them which are not enjoyed by local-hires, such as
housing, transportation, shipping costs, taxes and home leave travel allowances.

In this case, we find the point-of-hire classification employed by respondent School to justify the distinction in the
salary rates of foreign-hires and local hires to be an invalid classification. There is no reasonable distinction
between the services rendered by foreign-hires and local-hires.

OBITER
However, foreign-hires do not belong to the same bargaining unit as the local-hires. It does not appear that
foreign-hires have indicated their intention to be grouped together with local-hires for purposes of collective
bargaining. The collective bargaining history in the School also shows that these groups were always treated
separately. The housing and other benefits accorded foreign hires were not given to local hires, thereby such
admixture will nbot assure any group the power to exercise bargaining rights.

The factors in determining the appropriate collective bargaining unit are


(1) the will of the employees (Globe Doctrine);
(2) affinity and unity of the employees' interest, such as substantial similarity of work and duties, or similarity of
compensation and working conditions (Substantial Mutual Interests Rule);
(3) prior collective bargaining history; and
(4) similarity of employment status.

ORMOC SUGAR COMPANY INC. VS ORMOC CITY ET AL


PONENCIA:
TOPIC:
TRIGGER OF THE FACTS:
TRIGGER OF THE ISSUE:
TRIGGER OF THE RULING:
VERDICT:
END POINT:

FACTS
Ormoc City passed a bill which read: There shall be paid to the City Treasurer on any and all productions of
centrifugal sugar milled at the Ormoc Sugar Company Incorporated, in Ormoc City a municipal tax equivalent
to one per centum (1%) per export sale to the United States of America and other foreign countries.
Though referred to as a production tax, the imposition actually amounts to a tax on the export of centrifugal
sugar produced at Ormoc Sugar Company, Inc. For production of sugar alone is not taxable; the only time the
tax applies is when the sugar produced is exported.
Ormoc Sugar paid the tax (P7,087.50) in protest averring that the same is violative of Sec 2287 of the Revised
Administrative Code which provides: It shall not be in the power of the municipal council to impose a tax in
any form whatever, upon goods and merchandise carried into the municipality, or out of the same, and any
attempt to impose an import or export tax upon such goods in the guise of an unreasonable charge for
wharfage, use of bridges or otherwise, shall be void.
And that the ordinance is violative to equal protection as it singled out Ormoc Sugar As being liable for such
tax impost for no other sugar mill is found in the city.

ISSUE
Whether or not there has been a violation of equal protection.
Page 57 of 236
Constitutional Law 2 Bill of Rights Armando Santiago Jr

HELD
The SC held in favor of Ormoc Sugar. The SC noted that even if Sec 2287 of the RAC had already been
repealed by a latter statute (Sec 2 RA 2264) which effectively authorized LGUs to tax goods and merchandise
carried in and out of their turf, the act of Ormoc City is still violative of equal protection. The ordinance is
discriminatory for it taxes only centrifugal sugar produced and exported by the Ormoc Sugar Company, Inc. and
none other. At the time of the taxing ordinances enactment, Ormoc Sugar Company, Inc., it is true, was the only
sugar central in the city of Ormoc. Still, the classification, to be reasonable, should be in terms applicable to
future conditions as well. The taxing ordinance should not be singular and exclusive as to exclude any
subsequently established sugar central, of the same class as plaintiff, from the coverage of the tax. As it is now,
even if later a similar company is set up, it cannot be subject to the tax because the ordinance expressly points
only to Ormoc Sugar Company, Inc. as the entity to be levied upon.

JAMES MIRASOL VS DEPARTMENT OF PUBLIC WORKS AND


HIGHWAYS
PONENCIA:
TOPIC:
TRIGGER OF THE FACTS: yung 4 wheels lang pwedeng dumaan sa NLEX.
TRIGGER OF THE ISSUE:
TRIGGER OF THE RULING:
VERDICT:
END POINT: Substantial distinction ay yung para sa public safety. 4 distinction was met

FACTS
On January 10, 2001, petitioners filed before the trial court a Petition for Declaratory Judgment with Application
for Temporary Restraining Order and Injunction to nullity of the following administrative issuances for being
inconsistent with the provisions of Republic Act 2000, entitled "Limited Access Highway Act" enacted in 1957: (a)
DPWH Administrative Order No. 1, Series of 1968; (b) DPWH Department Order No. 74, Series of 1993; and (c)
Art. II, Sec. 3 (a) of the Revised Rules on Limited Access Facilities promulgated in 199[8] by the DPWH thru the
Toll Regulatory Board (TRB).

ISSUE
Whether or not Administrative Order No. 1 introduces an unreasonable classification by singling-out motorcycles
from other motorized modes of transport and violates the right to travel.

RULING
No. Petitioners are not being deprived of their right to use the limited access facility. They are merely being
required, just like the rest of the public, to adhere to the rules on how to use the facility. AO 1 does not infringe
upon petitioners right to travel but merely bars motorcycles, bicycles, tricycles, pedicabs, and any non-motorized
vehicles as the mode of traveling along limited access highways.There exists real and substantial differences
exist between a motorcycle and other forms of transport sufficient to justify its classification among those
prohibited from plying the toll ways. A classification based on practical convenience and common knowledge is
not unconstitutional simply because it may lack purely theoretical or scientific uniformity.

SYLLABI
Same; Same; Equal Protection; Motorcycles; A classification can only be assailed if it is deemed invidious, that
is, it is not based on real or substantial differences.A police power measure may be assailed upon proof that it
unduly violates constitutional limitations like due process and equal protection of the law. Petitioners attempt to
seek redress from the motorcycle ban under the aegis of equal protection must fail. Petitioners contention that
AO 1 unreasonably singles out motorcycles is specious. To begin with, classification by itself is not prohibited. A

Page 58 of 236
Constitutional Law 2 Bill of Rights Armando Santiago Jr

classification can only be assailed if it is deemed invidious, that is, it is not based on real or substantial
differences.

Page 59 of 236
Constitutional Law 2 Bill of Rights Armando Santiago Jr

Search and seizures


Section 2. Art. III.
The right of the people to be secure in their persons, houses,
papers, and effects against unreasonable search and seizures of
whatever nature and for any purpose shall be inviolable, and no
search warrant or warrant of arrest shall issue except upon probable
cause to be determined personally by the Judge after examination
under oath or affirmation of the complainant and the witnesses he
may produce, and particularly describing the place to be searched
and the persons or things to be seized.

SECTION 2. DIGESTED PROVISION


GENERAL RULE: WHAT
The right of the people to be secure in their persons, houses, papers, and effects against unreasonable
searches and seizures of whatever nature and for any purpose shall be inviolable, and

EXCEPTION: HOW

no search warrant or warrant of arrest shall issue except upon probable cause to be determined personally by
the judge

REQUIREMENT FOR EXCEPTION: HOW COME

after examination under oath or affirmation of the complainant and the witnesses he may produce, and

ASCERTAINMENT:

particularly describing the place to be searched and the persons or things to be seized.

Concept: It protects the privacy and sanctity of the person himself. It is a guarantee of the right of the people to
be secure in their persons against unreasonable search and seizures. It is a guarantee against unlawful arrest
and other forms of restraint on the physical liberty of the person.
Warrant Requirement

REQUISITES OF A VALID WARRANT:


1. It must be issued upon probable cause;
2. probable cause must be determined personally by a judge;
3. such judge must examine under oath or affirmation the complainant and the witnesses he may
produce;
4. the warrant must particularly describe the place to be searched and person to be seized.

Probable cause meant such reasons, supported by facts and circumstances, as will warrant a cautious man in
the belief that his action and the means taken in prosecuting it, are legally just and proper.

Probable cause for an arrest or for the issuance of a warrant of arrest mean such facts and circumstances
which would lead a reasonably discreet and prudent man to believe that an offense has been committed by the
person sought to be arrested.

Page 60 of 236
Constitutional Law 2 Bill of Rights Armando Santiago Jr

Personally, examine the complainant and his witnesses meant the exclusive and personal responsibility of
the issuing judge to satisfy himself of the existence of probable cause

Probable cause for a search meant such facts and circumstances which would lead a reasonably discreet and
prudent man to believe that an offense has been committed and that the objects sought in connection with the
offense are in the place sought to be searched.

Particularity of description meant description that expresses a conclusion of fact not law by which the
warrant officer may be guided in making the search and seizure, or when the things described are limited to
those which bear direct relation to the offense for which the warrant is being issued

John Doe satisfies the requirement of particularity of description provided it contains a descriptio personae
such as will enable the officer to identify the accused

Warrantless Searches
IS EVERY WARRANTLESS SEARCH AN ILLEGAL SEARCH?
NO. Not every warrantless search is illegal search. As a general rule searches and seizures must be
accompanied with a valid warrant except:

1. When the right is voluntarily waived;


2. When there is valid reason to stop and frisk;
3. When the search is incidental to a lawful arrest;
4. Search of vessels and aircraft;
5. Search of moving vehicle;
6. Inspection of buildings and other premises for the enforcement of sanitary and building regulations;
7. When prohibited articles are in plain view;
8. Search and seizure under exigent and emergency;
9. Areal target zoning or saturation drive (valid exercise of military powers of the President; Searches of
passengers at airport. R.A. 6235 provides that every airline ticket shall contain a condition that hand-carried
luggage, etc., shall be subject to search and this condition shall form part of the contract between the passenger
and the air carrier.

Search incident of a lawful Arrest Arresting officer is authorized to search the arresting as long as the arrest
is valid

Search of a moving motor vehicle it must be based on probable cause (ex. There is a place where there is a
person who are known to habitually deliver drug, the police officer has a probable cause, he may search the car)

Search in violation of custom laws Pertains to Bureau of Customs (Ex. kapag naviolate mo yung laws ng
bureau customs)

Seizure of the evidence in plain view Seen without touching (ex. Robin padilla Case pagbaba nya ng kotse,
may nakatak na baril sa bewang nya, kahit hindi sya kapkapan kitang may baril sya.)

When the accused, himself waives his right against unreasonable search and seizures Gave consent to
search even if its illegally search

Stop and frisk it is a doctrine, it nevertheless holds that mere suspicion or a hunch will not validate a stop and
frisk. A genuine reason must exist, in the light of the police officers experience and surrounding conditions, to
warrant the belief that the person detained has weapons concealed about him. (Ex. Malacat vs. CA nung time na
naganap yung Miranda bombing there is certain people na parang hindi sila mapakali na parang may
kahinahinalang gagawin, the police officer approached them, with the probable cause they made a search on
these people and found out that there is a bomb inside their belongings.)

Page 61 of 236
Constitutional Law 2 Bill of Rights Armando Santiago Jr

Exigent and emergency circumstances

Additional Exceptions (Not in the Rules):


(1) Estoppel: When the right is voluntarily waived
(2) Violent insanity.

EXAMINATION OF APPLICATION (RULE 126, SECTION 4 OF THE RULES OF


COURT)
SEC. 4. Requisites for issuing search warrant. A search warrant shall not issue except upon probable cause in
connection with one specific offense to be determined personally by the judge after examination under oath or
affirmation of the complainant and the witnesses he may produce, and particularly describing the place to be
searched and the things to be seized which may be anywhere in the Philippines. (3a)

The judge, before issuing the search warrant, must personally examine in the form of searching questions and
answers, in writing and under oath the complainant and any writers he may produce on facts personally known
to then, and attach to
the record their sworn statement together with any affidavit submitted.

The evidence offered by the complainant and his witness should be based on their own personal knowledge and
not on mere information or belief.

Affiant could be charged with perjury if the allegations contained therein are found to be untrue.

PARTICULARITY OF PRESCRIPTION
Constitution requires that the place to be searched on the persons or things to be seized be described
with such particularity as to enable the person serving the warrant to identify them.
The person sought to be seized should be identified by name if the warrant is issued without a name or
with the name in blank such that it can be enforced against any person, it is unquestionably void.
Where by the nature of the articles to be seized, their description must be rather general, it is not
required that a technical description be given.
Only the articles particularly described in the warrant can be seized and no other property can be taken
thereunder unless it is prohibited law.

PROPERTIES SUBJECT TO SEIZURE (RULE 126, SECTION 2 OF THE RULES OF


COURT)
SEC. 2. Court where application for search warrant shall be filed. An application for search warrant shall be
filed with the following:

(a) Any court within whose territorial jurisdiction a crime was committed.

(b) For compelling reasons stated in the application, any court within the judicial region where the crime was
committed if the place of the commission of the crime is known, or any court within the judicial region where the
warrant shall be enforced.

However, if the criminal action has already been filed, the application shall only be made in the court where the
criminal action is pending, (n)

Property subject of the offense

Property stolen or embezzled and other proceeds or fruits of the offense

Page 62 of 236
Constitutional Law 2 Bill of Rights Armando Santiago Jr

Property used or intended to be used as the means of committing an offense where the search and seizures is
made only for the purpose of obtaining evidence to be used against the accused the warrant is unlawful as it
would violate the constitutional rights against self-incrimination

ADMISSIBILITY OF ILLEGALLY SEIZED EVIDENCE (EXCLUSIONARY RULE) ART.


III, SECTION 3 PARAGRAPH 2

Purpose of EXCLUSIONARY RULE: Kung ang gusto talaga ng authority ay makulong ang isang tao, pwedeng
nakawin nalang nya yung evidence, because of this exclusionary rule, hindi na nila pwedeng gawin yun dahil
lahat ng illegally seized evidence is inadmissible as an evidence of the case (for it is considered, fruit from the
poisonous tree)
Articles illegally seized are not admissible as evidence (EXCLUSIONARY RULE), pursuant to the doctrine
originally announced in the case of Stonehill vs. Diokno

Properties may have been seized in violation of the said provision, it does not follow that its owner shall be
entitled to recover it immediately. If the said property is the subject of litigation, like will remain incustodia legis
until the case is terminated

May nonetheless be used in the judicial or administrative action that may be filed against the officer responsible
for it illegal seizure

where the accused did not raise the issue of the admissibility of the evidence against him on ground that it had
been illegally seized, such omission constituted a waiver of the protection granted by this section

Warrantless searches and seizures (RULE 113) instance when a search or seizures may be validly made
notwithstanding noncompliance with the requisites disused above.

Peace officer or even a private person may without a warrant a person

when such person has in fact just committed, is actually committing, or is attempting to commit an offense in his
presence.

When an offense has in fact just been committed and he has personal knowledge of the facts indicating that the
person to be arrested has committed it.

When the person to be arrested is a prisoner who has escaped from penal establishment or a place where he is
serving his final judgement

DO THE ORDINARY RIGHTS AGAINST UNREASONABLE SEARCHES AND


SEIZURES APPLY TO SEARCHES CONDUCTED AT THE AIRPORT PURSUANT
TO ROUTINE AIRPORT SECURITY PROCEDURES?
NO. Searches of passengers at airport. R.A. 6235 provides that every airline ticket shall contain a condition that
hand-carried luggage, etc., shall be subject to search and this condition shall form part of the contract between
the passenger and the air carrier.

Warrant must refer to only one specific offense

Warrantless Arrest
A peace officer or private person may, without a warrant, arrest a person:
When, in his presence the person to be arrested has committed, is actually committing, or attempting to
commit an offense;

Page 63 of 236
Constitutional Law 2 Bill of Rights Armando Santiago Jr

When an offense has in fact been committed, and he has personal knowledge of facts indicating that the
person to be arrested has committed it; and
When the person to be arrested is a prisoner who has escaped from a penal establishment or place
where he is serving a final judgment or temporarily confined while his case is pending, or has escaped while
being transferred from one confinement to another.

Warrantless Arrest a writ issued by a judge after finding of judicial probable cause, that there is necessity in
placing the accused in custody

RULE 113 ARREST


Section 1. Definition of arrest. Arrest is the taking of a person into custody in order that he may be bound to
answer for the commission of an offense. (1) purpose is to bind the person to answer an accusation of a crime;
referring to a criminal warrant of arrest.

Other instances of arrest:

Arrest or contempt by administrative and legislative bodies Ronnie dayan: he was arrested in La Union,
Pangasinan not on the basis of warrant arrest issued by the court and not on the basis of criminal court, but
issued warrant for he was cited in contempt by congress: house of representatives for not showing up.

Deportation arrest - executive branch is authorized arrest securing a warrant any alien who has illegally entered
into the country or proven unfit to stay. executive department need not to apply or secure a judicial warrant to
arrest an individual
Both are subject to due process-must be exercise with the recognition of arrest

Section 2. Arrest;how made. An arrest is made by an actual restraint of a person to be arrested, or by his
submission to the custody of the person making the arrest.

No violence or unnecessary force shall be used in making an arrest. The person arrested shall not be subject to
a greater restraint than is necessary for his detention. (2a)

Section 3. Duty of arresting officer. It shall be the duty of the officer executing the warrant to arrest the
accused and to deliver him to the nearest police station or jail without unnecessary delay. (3a)

Section 4. Execution of warrant. The head of the office to whom the warrant of arrest was delivered for
execution shall cause the warrant to be executed within ten (10) days from its receipt. Within ten (10) days after
the expiration of the period, the officer to whom it was assigned for execution shall make a report to the judge
who issued the warrant. In case of his failure to execute the warrant, he shall state the reasons therefor. (4a)

Section 5. Arrest without warrant;when lawful. A peace officer or a private person may, without a warrant,
arrest a person:

(a) When, in his presence, the person to be arrested has committed, is actually committing, or is attempting to
commit an offense; Reason for allowing POLICE warrantless arrest: because there was no time to for filing such
warrant (baka kapag nagapply pa ng warrant of arrest pagbalik nya wala na yung aarestohin nakatakas na!)

(b) When an offense has just been committed, and he has probable cause to believe based on personal
knowledge of facts or circumstances that the person to be arrested has committed it; and

(c) When the person to be arrested is a prisoner who has escaped from a penal establishment or place where he
is serving final judgment or is temporarily confined while his case is pending, or has escaped while being
transferred from one confinement to another.

Page 64 of 236
Constitutional Law 2 Bill of Rights Armando Santiago Jr

In cases falling under paragraph (a) and (b) above, the person arrested without a warrant shall be forthwith
delivered to the nearest police station or jail and shall be proceeded against in accordance with section 7 of Rule
112. (5a)

Section 6. Time of making arrest. An arrest may be made on any day and at any time of the day or night. (6)

Section 7. Method of arrest by officer by virtue of warrant. When making an arrest by virtue of a warrant, the
officer shall inform the person to be arrested of the cause of the arrest and of the fact that a warrant has been
issued for his arrest, except when he flees or forcibly resists before the officer has opportunity to so inform him,
or when the giving of such information will imperil the arrest. The officer need not have the warrant in his
possession at the time of the arrest but after the arrest, if the person arrested so requires, the warrant shall be
shown to him as soon as practicable. (7a)

Section 8. Method of arrest by officer without warrant. When making an arrest without a warrant, the officer
shall inform the person to be arrested of his authority and the cause of the arrest, unless the latter is either
engaged in the commission of an offense, is pursued immediately after its commission, has escaped, flees or
forcibly resists before the officer has opportunity so to inform him, or when the giving of such information will
imperil the arrest. (8a)

Section 9. Method of arrest by private person. When making an arrest, a private person shall inform the
person to be arrested of the intention to arrest him and cause of the arrest, unless the latter is either engaged in
the commission of an offense, is pursued immediately after its commission, or has escaped, flees, or forcibly
resists before the person making the arrest has opportunity to so inform him, or when the giving of such
information will imperil the arrest. (9a)

Section 10. Officer may summon assistance. An officer making a lawful arrest may orally summon as many
persons as he deems necessary to assist him in effecting the arrest. Every person so summoned by an officer
shall assist him in effecting the arrest when he can render such assistance without detriment to himself. (10a)

Section 11. Right of officer to break into building or enclosure. An officer, in order to make an arrest either by
virtue of a warrant, or without a warrant as provided in section 5, may break into any building or enclosure where
the person to be arrested is or is reasonably believed to be, if he is refused admittance thereto, after announcing
his authority and purpose. (11a)

Section 12. Right to break out from building or enclosure.Whenever an officer has entered the building or
enclosure in accordance with the preceding section, he may break out therefrom when necessary to liberate
himself. (12a)

Section 13. Arrest after escape or rescue. If a person lawfully arrested escapes or is rescued, any person
may immediately pursue or retake him without a warrant at any time and in any place within the Philippines. (13)

Section 14. Right of attorney or relative to visit person arrested. Any member of the Philippine Bar shall, at
the request of the person arrested or of another acting in his behalf, have the right to visit and confer privately
with such person in the jail or any other place of custody at any hour of the day or night. Subject to reasonable
regulations, a relative of the person arrested can also exercise the same right. (14a)

BOND (PYANSADOR)
Section 23. Arrest of accused out on bail. For the purpose of surrendering the accused, the bondsmen may
arrest him or, upon written authority endorsed on a certified copy of the undertaking, cause him to be arrested by
a police officer or any other person of suitable age and discretion.

An accused released on bail may be re-arrested without the necessity of a warrant if he attempts to depart from
the Philippines without permission of the court where the case is pending. (23a)

Page 65 of 236
Constitutional Law 2 Bill of Rights Armando Santiago Jr

Ex. Accused He may maintain a premium to a surety company to assures the court that I will appear when so
requires,
failure to appear, the court will confiscate the bond. The bondsman may arrest the accused without obtaining a
warrant of arrest in court. (This bond serve a guarantor)

SEARCH WARRANT- An order signed by a judge directed to a peace officer to search seize and to bring to
court

2 kinds of search:
1. Searches with warrant
2. Searches Without any warrant (but only under certain circumstances)

Section 1. Search warrant defined. A search warrant is an order in writing issued in the name of the People of
the Philippines, signed by a judge and directed to a peace officer, commanding him to search for personal
property described therein and bring it before the court. (1)

WHO MAY ISSUE WARRANT OF ARREST


1987 Constitution - Only the JUDGE may issue warrant of arrest (exclusive prerogative of Judge)
1973 (Martial Law Constitution) - A warrant of arrest may be issued by the JUDGE AND SUCH OTHER
OFFICER AS MAY BE AUTHORIZED BY LAW Secretary of Defense may issue warrant without obtaining
from

ADMINISTRATIVE ARRESTS
General Rule: Only the judge has the power to issue a warrant after the proper procedure has been duly taken.

Exceptions:
(1) In cases of deportation of illegal and undesirable aliens, whom the President or the Commissioner of
Immigration may order arrested, following a final order of deportation, for the purpose of deportation. [Salazar v.
Achacoso (1990)]

(2) Warrant of arrest may be issued by administrative authorities only for the purpose of carrying out a final
finding of a violation of law and not for the sole purpose of investigation or prosecution. It may be issued only
after the proceeding has taken place as when there is already a final decision of the administrative authorities.

Peace officer Police, NBI or PDEA

The warrant cannot be a "general warrant" meaning, it cannot be a fishing expedition. It must state the
suspected offense and only be limited to:

SUBJECT OF THE OFFENSE;


Stolen or embezzled and other proceeds, or fruits of the offense; o
Used or intended to be used as the means of committing an offense.
(Section 2, Rule 126, Rules of Criminal Procedure)

It cannot be issued if there is no suspected offense, like "I have not heard from him in over a month now."
Stone hill Case: Applies the use of general warrant which is not allowed by the law, as provides by our
constitution that it must be specific, otherwise considered fishing expedition which requires ascertainment of the
offense committed.

Page 66 of 236
Constitutional Law 2 Bill of Rights Armando Santiago Jr

CASES

PEOPLE VS. ANDRE MARTI15


Ponencia: BIDIN, J
Topic: Search and Seizures
Trigger of facts: Package of Marijuana to be sent abroad by unknown person for certain amount of 2000php
Triger of issue: Whether or not the items admitted in the searched illegally searched and seized
Trigger of Ruling: inspection was reasonable and a standard operating procedure on the part of Mr. Reyes as a
precautionary measure before delivery of packages to the Bureau of Customs or the Bureau of Posts.
Verdict: Appellant guilty beyond reasonable doubt of the crime charged is hereby affirmed
End point of the case: Inspection is different from search. It is valid it was just an inspection and not search

FACTS
Certain Shirley Reyes, went to the booth of he Manila Packing and Export Forwarders
Carrying with them four (4) gif wrapped packages
Anita Reyes, Proprietress attended them
Appellant informed Anita Reyes that he was sending the packages to a friend in Switzerland
Appellant filled up the contract necessary for the transaction, writing therein his name, passport number and
other information needed for forwarding and named certain Walter Fierz
Anita Reyes asked the appellant if she could examine and inspect the packages
Appellant however, refused and state that the packages are simply books, cigars and gloves were gift to his
friend Zurich
Package were sealed by Anita and made ready for shipment

INTERACTION OF THE CASE


Before delivery of the appellants box to the Bureau of customs, certain Mr. Job Reyes (proprietor) and
husband Anita Reyes
Following the standard operating procedure
They had open the package for final inspection
Upon inspection, they got curious with the smell of the package with sting of dried leaves inside
Mr. Job Reyes forthwith made a letter reporting the shipment to the NBI and requested to undergone a
laboratory examination
Thereafter, examination result was positive of dried marijuana
Which leads to filing of information violation of R.A 6425 Dangerous Drugs Act

ISSUES
Whether or not the items admitted in the searched illegally searched and seized.
Whether or not custodial investigation was not properly applied.
Whether or not the trial court did not give credence to the explanation of the appellant on how said packages
came to his possession.


HELD
No. The case at bar assumes a peculiar character since the evidence sought to be excluded was primarily
discovered and obtained by a private person, acting in a private capacity and without the intervention and
participation of State authorities. Under the circumstances, can accused/appellant validly claim that his
constitutional right against unreasonable searches and seizure has been violated. Stated otherwise, may an act
of a private individual, allegedly in violation of appellant's constitutional rights, be invoked against the State. In
the absence of governmental interference, the liberties guaranteed by the Constitution cannot be invoked
against the State. It was Mr. Job Reyes, the proprietor of the forwarding agency, who made search/inspection of
the packages. Said inspection was reasonable and a standard operating procedure on the part of Mr. Reyes as
a precautionary measure before delivery of packages to the Bureau of Customs or the Bureau of Posts. Second,

15 G.R. 81561 Jan. 18, 1991


Page 67 of 236
Constitutional Law 2 Bill of Rights Armando Santiago Jr

the mere presence of the NBI agents did not convert the reasonable search effected by Reyes into a warrantless
search and seizure proscribed by the Constitution. Merely to observe and look at that which is in plain sight is
not a search. Having observed that which is open, where no trespass has been committed in aid thereof, is not
search.

No. The law enforcers testified that accused/appellant was informed of his constitutional rights. It is presumed
that they have regularly performed their duties (See. 5(m), Rule 131) and their testimonies should be given full
faith and credence, there being no evidence to the contrary.

No. Appellant signed the contract as the owner and shipper thereof giving more weight to the presumption that
things which a person possesses, or exercises acts of ownership over, are owned by him (Sec. 5 [j], Rule 131).
At this point, appellant is therefore estopped to claim otherwise.

STONEHILL VS. DIOKNO


Ponencia: CONCEPCION, C.J.
Topic: Seacrh Warrant / General Warrant
Trigger of facts: General Warrant / Exclusionary Rule / 42 warrants to seized corporations and residence
Triger of issue:
Trigger of Ruling: general warrant is unconstitutional. it must be specific and listed on the search warrant to make
the search valid.
Verdict: Order was illegal and inadmissible as an evidence in the case
End point of the case: General Warrant is unconstitutional, seized articles were inadmissible as an evidence to the
crime.

Search and Seizure General Warrants Abandonment of the Moncado Doctrine

FACTS
Stonehill et al and the corporation they form were alleged to have committed acts in violation of Central Bank
Laws, Tariff and Customs Laws, Internal Revenue (Code) and Revised Penal Code. By the strength of this
allegation a search warrant was issued against their persons and their corporation. The warrant provides
authority to search the persons above-named and/or the premises of their offices, warehouses and/or
residences, and to seize and take possession of the following personal property to wit:

Books of accounts, financial records, vouchers, correspondence, receipts, ledgers, journals,


portfolios, credit journals, typewriters, and other documents and/or papers showing all business
transactions including disbursements receipts, balance sheets and profit and loss statements and
Bobbins (cigarette wrappers).

The documents, papers, and things seized under the alleged authority of the warrants in question may be split
into (2) major groups, namely:

those found and seized in the offices of the aforementioned corporations and
those found seized in the residences of petitioners herein.
Stonehill averred that the warrant is illegal for:
they do not describe with particularity the documents, books and things to be seized;
cash money, not mentioned in the warrants, were actually seized;
the warrants were issued to fish evidence against the aforementioned petitioners in deportation cases filed
against them;
the searches and seizures were made in an illegal manner; and
the documents, papers and cash money seized were not delivered to the courts that issued the warrants, to
be disposed of in accordance with law.
The prosecution counters, invoking the Moncado doctrine, that the defects of said warrants, if any, were cured
by petitioners consent; and (3) that, in any event, the effects seized are admissible in evidence against them.
In short, the criminal cannot be set free just because the government blunders.

Page 68 of 236
Constitutional Law 2 Bill of Rights Armando Santiago Jr

ISSUE
Whether or not the search warrant issued is valid.

HELD
The SC ruled in favor of Stonehill et al. The SC emphasized however that Stonehill et al cannot assail the validity
of the search warrant issued against their corporation for Stonehill are not the proper party hence has no cause
of action. It should be raised by the officers or board members of the corporation.
The constitution protects the peoples right against unreasonable search and seizure. It provides;

(1) that no warrant shall issue but upon probable cause, to be determined by the judge in
the manner set forth in said provision; and

(2) that the warrant shall particularly describe the things to be seized. In the case at bar,
none of these are met. The warrant was issued from mere allegation that Stonehill et
al committed a violation of Central Bank Laws, Tariff and Customs Laws, Internal
Revenue (Code) and Revised Penal Code.

In other words, no specific offense had been alleged in said applications. The averments thereof with respect to
the offense committed were abstract. As a consequence, it was impossible for the judges who issued the
warrants to have found the existence of probable cause, for the same presupposes the introduction of
competent proof that the party against whom it is sought has performed particular acts, or committed specific
omissions, violating a given provision of our criminal laws.

As a matter of fact, the applications involved in this case do not allege any specific acts performed by herein
petitioners. It would be a legal heresy, of the highest order, to convict anybody of a violation of Central Bank
Laws, Tariff and Customs Laws, Internal Revenue (Code) and Revised Penal Code, as alleged in the
aforementioned applications without reference to any determinate provision of said laws or codes.

The grave violation of the Constitution made in the application for the contested search warrants was
compounded by the description therein made of the effects to be searched for and seized, to wit:

Books of accounts, financial records, vouchers, journals, correspondence, receipts,


ledgers, portfolios, credit journals, typewriters, and other documents and/or papers
showing all business transactions including disbursement receipts, balance sheets and
related profit and loss statements.

Thus, the warrants authorized the search for and seizure of records pertaining to all business transactions of
Stonehill et al, regardless of whether the transactions were legal or illegal.

The warrants sanctioned the seizure of all records of Stonehill et al and the aforementioned corporations,
whatever their nature, thus openly contravening the explicit command of the Bill of Rights that the things to be
seized be particularly described as well as tending to defeat its major objective: the elimination of general
warrants.

The Moncado doctrine is likewise abandoned and the right of the accused against a defective search warrant is
emphasized.

PEOPLE VS. SARAP


Ponencia: YNARES-SANTIAGO J
Topic: Search / Stop and Frisk / Drugs
Trigger of facts:
Triger of issue:
Trigger of Ruling:
Verdict: Sarap was acquitted
Page 69 of 236
Constitutional Law 2 Bill of Rights Armando Santiago Jr

End point of the case:

FACTS
Armed with a search warrant
raided the house of Conrado Ricaforte
relative to the reported sale of marijuana by its occupants
certain Jonalyn Duran, Joysie Duran and Pepe Casabuena. The three were apprehended for illegal
possession of marijuana and were detained
In the course of their investigation, the police learned that a certain Melly from Capiz and Roger Amar were
the suppliers of marijuana and that they will be back on March 4, 1996
Thereafter, caretaker of the house of Ricaforte informed Guarino that there were two strangers looking for the
Duran sisters.
Then they proceeded to the house and saw a woman, who turned out to be accused-appellant Melly Sarap.
Melly saw Guarino and Navida in police uniform and immediately threw away her black canvass bag, which
Roger Amar picked up.
Guarino Blocked Saraps path and grabbed from her the green plastic bag she was holding.
The plastic bag was found to contain two blocks of marijuana fruiting tops.
Navida pursued Amar and arrested him. The accused-appellant denied the accusations against her. The
Court fines accused guilty beyond reasonable doubt of violation of Dangerous Drugs Act.
The accused appealed the decision of the trial court.

ISSUES
Whether the warrantless search and arrest conducted is legal.
Whether the evidence presented by the prosecution is sufficient to find the accused guilty beyond reasonable
doubt.

HELD
A search may be conducted by law enforcers only on the strength of a warrant validly issued by a judge as
provided in Article III, Section 2 of the Constitution. Articles which are the product of unreasonable searches and
seizures are inadmissible as evidence, pursuant to Article III, Section 3 (2) of the Constitution.

Warrantless searches and seizures may be made without a warrant in the following instances:

(1) search incident to a lawful arrest,


(2) search of a moving motor vehicle,
(3) search in violation of custom laws,
(4) seizure of the evidence in plain view,
(5) when the accused himself waives his right against unreasonable searches and seizures,
(6) stop and frisk and
(7) exigent and emergency circumstances.

These instances, however do not dispense with the requisite of probable cause before a warrantless search and
seizure can be lawfully conducted. In warrantless search cases, probable cause must only be based on
reasonable ground of suspicion or belief that a crime has been committed or is about to be committed. Without
the illegally seized prohibited drug, the appellants conviction cannot stand. There is simply no sufficient
evidence to convict her. That the search disclosed marijuana fruiting tops in appellants possession, and thus
confirmed the police officers initial information and suspicion, did not cure its patent illegality. An illegal search
cannot be undertaken and then an arrest effected on the strength of the evidence yielded by the search for being
a fruit of a poisonous tree.

All told, the guilt of the accused-appellant was not proven beyond reasonable doubt measured by the required
moral certainty of conviction. The evidence presented by the prosecution was not enough to overcome the
presumption of innocence as constitutionally ordained

Page 70 of 236
Constitutional Law 2 Bill of Rights Armando Santiago Jr

Wherefore the Accused-appellant Melly Sarap y Arcangeles is ACQUITTED of the crime charged on the ground
of reasonable doubt.

SYLLABI
Constitutional Law; Searches and Seizures; Instances where warrantless searches and seizures may be made
without a warrant; In warrantless search cases, probable cause must only be based on reasonable ground of
suspicion or belief that a crime has been committed or is about to be committed. A search may be conducted
by law enforcers only on the strength of a warrant validly issued by a judge as provided in Article III, Section 2 of
the Constitution. Articles which are the product of unreasonable searches and seizures are inadmissible as
evidence, pursuant to Article III, Section 3 (2) of the Constitution. Warrantless searches and seizures may be
made without a warrant in the following instances: (1) search incident to a lawful arrest, (2) search of a moving
motor vehicle, (3) search in violation of custom laws, (4) seizure of the evidence in plain view (5) when the
accused himself waives his right against unreasonable searches and seizures, (6) stop and frisk and (7) exigent
and emergency circumstances. These instances, however do not dispense with the requisite of probable cause
before a warrantless search and seizure can be lawfully conducted. In warrantless search cases, probable cause
must only be based on reasonable ground of suspicion or belief that a crime has been committed or is about to
be committed.

Same; Same; The search must be incidental to a lawful arrest in order that the search itself may likewise be
considered legal. The Banga Police could not effect a warrantless search and seizure since there was no
probable cause and Sarap was not lawfully arrested. The law requires that the search must be incidental to a
lawful arrest in order that the search itself may likewise be considered legal.

Same; Same; Plain View Doctrine; Conditions in order for the doctrine to apply. Contrary to the finding of the
trial court, the instant case did not come within the purview of the plain view doctrine. In order for the doctrine to
apply, the following conditions must be present: (a) a prior valid intrusion based on the valid warrantless arrest in
which the police are legally present in the pursuit of their official duties; (b) the evidence was inadvertently
discovered by the police who had the right to be where they are; (c) the evidence must be immediately apparent;
and (d) plain view justified mere seizure of evidence without further search.

Same; Same; An illegal search cannot be undertaken and then an arrest effected on the strength of the evidence
yielded by the search for being a fruit of a poisonous tree. Without the illegally seized prohibited drug, the
appellants conviction cannot stand. There is simply no sufficient evidence to convict her. That the search
disclosed marijuana fruiting tops in appellants possession, and thus confirmed the police officers initial
information and suspicion, did not cure its patent illegality. An illegal search cannot be undertaken and then an
arrest effected on the strength of the evidence yielded by the search for being a fruit of a poisonous tree.

ALIB VS. JUDGE LABAYEN


Ponencia: Gonzaga-Reyes J.
Topic: Invalid issuance of Warrant of Arrest
Trigger of facts: Judge labayen erroneously signed a warrant of arrest without her jurisdiction
Triger of issue: was the said warrant is valid.
Trigger of Ruling: judge failed to investigate and check the jurisdiction which he failed to issue a warrant of arrest
Verdict: Judge was Fined 10,000 php
End point of the case: provided by the constitution that issuance of warrant of arrest solely vested in the judiciary
through judge of the court within their jurisdiction. if its not within its jurisdiction it is erroneous.

FACTS
Charge was filed against Judge Emma Labayen for issuing a warrant of arrest against the accused
The accused filed a motion for re-investigation and recall of warrant of arrest
Alleging that the court has no jurisdiction as the crime of perjury is within the jurisdiction of MTC
Judge Labayen denied the motion thereof and ordered the remand of the case to the MTC
Considering that the case falls under the jurisdiction of the said court

Page 71 of 236
Constitutional Law 2 Bill of Rights Armando Santiago Jr

Judge labayen is administrative liable for issuing an illegal warrant of arrest after admitting that she had no
jurisdiction over the case
Judge Labayen alleges that there was no malice nor bad faith when she signed the warrant of arrest and in
fact, she ordered the remand of the case to the lower court upon a finding that the case falls within the
jurisdiction of the MTC.
Judge prays for dismissal of the case

ISSUE
WON Judge Labayen there was a valid issuance of warrant of arrest against the accused

HELD
She is administratively liable for gross ignorance of the law for refusing to withdraw the warrant of arrest she
issued despite having admitted in her order that case was within the jurisdiction of the MTCC. He recommends
to fine in the amount of 20,000. Onset of criminal proceedings, Judge Labayen had no jurisdiction to hear and
decide the criminal case as the crime of perjury falls under the jurisdiction of the MTCC, respondent cannot be
totally absolved, the issuance of warrant of arrest is ministerial function of the court.

Before issuing a warrant of arrest, a judge must not rely solely on the report or resolution of the prosecutor, he
must evaluate the report and the supporting documents which will assist him to make his determination of
probable cause. A finding of the existence of a probable cause is a pre-requisite to issuance of a warrant of
arrest and strict compliance therewith is required of judge.

Issues are so simple and the facts so evident as to be beyond permissible margins of error, to still err thereon
amounts to ignorance of the law. The fine was found excessive and reduced to 10,000 pesos.

MALACAT V CA
Ponencia: Davide Jr. J.
Topic: Search without warrant
Trigger of facts:
Miranda Bombing alert
Trigger of issue:
Trigger of Ruling:
Verdict:
End point of the case:

FACTS
allegedly in response to bomb threats reported seven days earlier, Rodolfo Yu of the Western Police in
Quiapo, Manila, was on foot patrol with three other police officers (all of them in uniform).
They chanced upon two groups of Muslim-looking men, with each group, comprised of three to four men,
posted at opposite sides of the corner of Quezon Boulevard near the Mercury Drug Store.
These men were acting suspiciously with "their eyes moving very fast." Yu and his companions positioned
themselves at strategic points and observed both groups for about 30 minutes.
The police officers then approached one group of men, who then fled in different directions. As the policemen
gave chase, Yu caught up with and apprehended Sammy Malacat y Mandar
Upon searching Malacat, Yu found a fragmentation grenade tucked inside the latter's "front waist line." Yu's
companion, police officer Rogelio Malibiran, apprehended Abdul Casan from whom a .38 caliber revolver was
recovered.
Malacat and Casan were then brought to Police Station 3 where Yu placed an "X" mark at the bottom of the
grenade and thereafter gave it to his commander.
Yu did not issue any receipt for the grenade he allegedly recovered from Malacat. On 30 August 1990,
Malacat was charged with violating Section 3 of Presidential Decree 1866.
At arraignment on 9 October 1990, petitioner, assisted by counsel de officio, entered a plea of not guilty.

Page 72 of 236
Constitutional Law 2 Bill of Rights Armando Santiago Jr

Malacat denied the charges and explained that he only recently arrived in Manila. However, several other
police officers mauled him, hitting him with benches and guns. Petitioner was once again searched, but
nothing was found on him. He saw the grenade only in court when it was presented.
In its decision dated 10 February 1994 but promulgated on 15 February 1994, the trial court ruled that the
warrantless search and seizure of Malacat was akin to a "stop and frisk," where a "warrant and seizure can be
effected without necessarily being preceded by an arrest" and "whose object is either to maintain the status
quo momentarily while the police officer seeks to obtain more information"; and that the seizure of the
grenade from Malacat was incidental to a lawful arrest. The trial court thus found Malacat guilty of the crime of
illegal possession of explosives under Section 3 of PD 1866, and sentenced him to suffer the penalty of not
less than 17 years, 4 months and 1 day of Reclusion Temporal, as minimum, and not more than 30 years of
Reclusion Perpetua, as maximum.
Malacat filed a notice of appeal indicating that he was appealing to the Supreme Court. However, the record
of the case was forwarded to the Court of Appeals
Court of Appeals affirmed the trial court.
Manalili filed a petition for review with the Supreme Court.

ISSUE
Whether the search made on Malacat is valid, pursuant to the exception of stop and frisk.

HELD
The general rule as regards arrests, searches and seizures is that a warrant is needed in order to validly effect
the same. The Constitutional prohibition against unreasonable arrests, searches and seizures refers to those
effected without a validly issued warrant, subject to certain exceptions. As regards valid warrantless arrests,
these are found in Section 5, Rule 113 of the Rules of Court. A warrantless arrest under the circumstances
contemplated under Section 5(a) has been denominated as one "in flagrante delicto," while that under Section
5(b) has been described as a "hot pursuit" arrest.

Turning to valid warrantless searches, they are limited to the following: (1) customs searches; (2) search of
moving vehicles; (3) seizure of evidence in plain view; (4) consent searches; (5) a search incidental to a lawful
arrest; and (6) a "stop and frisk." The concepts of a "stop-and-frisk" and of a search incidental to a lawful arrest
must not be confused. These two types of warrantless searches differ in terms of the requisite quantum of proof
before they may be validly effected and in their allowable scope. In a search incidental to a lawful arrest, as the
precedent arrest determines the validity of the incidental search. Here, there could have been no valid in
flagrante delicto or hot pursuit arrest preceding the search in light of the lack of personal knowledge on the part
of Yu, the arresting officer, or an overt physical act, on the part of Malacat, indicating that a crime had just been
committed, was being committed or was going to be committed. Plainly, the search conducted on Malacat could
not have been one incidental to a lawful arrest. On the other hand, while probable cause is not required to
conduct a "stop and frisk," it nevertheless holds that mere suspicion or a hunch will not validate a "stop and
frisk." A genuine reason must exist, in light of the police officer's experience and surrounding conditions, to
warrant the belief that the person detained has weapons concealed about him. Finally, a "stop-and-frisk" serves
a two-fold interest: (1) the general interest of effective crime prevention and detection, which underlies the
recognition that a police officer may, under appropriate circumstances and in an appropriate manner, approach a
person for purposes of investigating possible criminal behavior even without probable cause; and (2) the more
pressing interest of safety and self-preservation which permit the police officer to take steps to assure himself
that the person with whom he deals is not armed with a deadly weapon that could unexpectedly and fatally be
used against the police officer. Here, there are at least three (3) reasons why the "stop-and-frisk" was invalid:

First, there is grave doubts as to Yu's claim that Malacat was a member of the group which attempted to bomb
Plaza Miranda 2 days earlier. This claim is neither supported by any police report or record nor corroborated by
any other police officer who allegedly chased that group.

Second, there was nothing in Malacat's behavior or conduct which could have reasonably elicited even mere
suspicion other than that his eyes were "moving very fast" an observation which leaves us incredulous since
Page 73 of 236
Constitutional Law 2 Bill of Rights Armando Santiago Jr

Yu and his teammates were nowhere near Malacat and it was already 6:30 p.m., thus presumably dusk. Malacat
and his companions were merely standing at the corner and were not creating any commotion or trouble.

Third, there was at all no ground, probable or otherwise, to believe that Malacat was armed with a deadly
weapon. None was visible to Yu, for as he admitted, the alleged grenade was "discovered" "inside the front
waistline" of Malacat, and from all indications as to the distance between Yu and Malacat, any telltale bulge,
assuming that Malacat was indeed hiding a grenade, could not have been visible to Yu. What is unequivocal
then are blatant violations of Malacat's rights solemnly guaranteed in Sections 2 and 12(1) of Article III of the
Constitution.

PADILLA VS CA
Ponencia: Francisco J.
Topic: Plain view doctrine
Trigger of facts: Applies Plain View Doctrine / Established Rule 113 section 5 (b) How SC applied this requirement
Triger of issue:
Trigger of Ruling:
Verdict: Convicted
End point of the case:

FACTS
High-powered firearms with live ammunitions were found in the possession of petitioner Robin Padilla:

(1)One .357 Caliber revolver, Smith and Wesson, SN-32919 with six (6) live ammunitions;
(2) One M-16 Baby Armalite rifle, SN-RP 131120 with four (4) long and one (1) short magazine with
ammunitions;
(3)One .380 Pietro Beretta, SN-A 35723 Y with clip and eight (8) ammunitions; and
(4)Six additional live double action ammunitions of .38 caliber revolver.
Appellant voluntarily surrendered item no. 3.and a black bag containing two additional long magazines and one
short magazine.

PNP Chief Espino, Record Branch of the Firearms and Explosives Office issued a Certification which stated that
the three firearms confiscated from appellant, an M-16 Baby armalite rifle SN-RP 131280, a .357 caliber revolver
Smith and Wesson SN 32919 and a .380 Pietro Beretta SN-A35720, were not registered in the name of Robin C.
Padilla.A second Certification stated that the three firearms were not also registered in the name of Robinhood
C. Padilla.

ISSUE
Whether or not his arrest was illegal and consequently, the firearms and ammunitions taken in the course thereof
are inadmissible in evidence under the exclusionary rule

HELD
No. There is no dispute that no warrant was issued for the arrest of petitioner, but thatper sedid not make his
apprehension at the Abacan bridge illegal.

Warrantless arrests are sanctioned in the following instances:


Sec. 5.Arrest without warrant; when lawful. A peaceofficeror aprivate personmay, without a warrant, arrest
a person:
(a) When, in his presence, the person to be arrested has committed, is actually committing, or is attempting to
commit an offense;
(b)When an offense has in fact just been committed, and he has personal knowledge of facts indicating that the
person to be arrested has committed it.

Page 74 of 236
Constitutional Law 2 Bill of Rights Armando Santiago Jr

(c) When the person to be arrested is a prisoner who has escaped from a penal establishment or place where
he is serving final judgment or temporarily confined while his case is pending, or has escaped while being
transferred from one confinement to another.
Paragraph

(a) requires that the person be arrested

(i) after he has committed or while he is actually committing or is at least attempting to commit an offense,
(ii) in the presence of the arresting officer or private person.

Both elements concurred here, as it has been established that petitioners vehicle figured in a hit and run an
offense committed in the presence of Manarang, a private person, who then sought to arrest petitioner.It must
be stressed at this point that presence does not only require that the arresting person sees the offense, but
also when he hears the disturbance created thereby AND proceeds at once to the scene. As testified to by
Manarang, he heard the screeching of tires followed by a thud, saw the sideswiped victim (balut vendor),
reported the incident to the police and thereafter gave chase to the erring Pajero vehicle using his motorcycle in
order to apprehend its driver.After having sent a radio report to the PNP for assistance, Manarang proceeded to
the Abacan bridge where he found responding policemen SPO2 Borja and SPO2 Miranda already positioned
near the bridge who effected the actual arrest of petitioner.

Petitioner would nonetheless insist on the illegality of his arrest by arguing that the policemen who actually
arrested him were not at the scene of the hit and run.We beg to disagree.That Manarang decided to seek the
aid of the policemen (who admittedly were nowhere in the vicinity of the hit and run) in effecting petitioners
arrest, did not in any way affect the propriety of the apprehension. It was in fact the most prudent action
Manarang could have taken rather than collaring petitioner by himself, inasmuch as policemen are
unquestionably better trained and well-equipped in effecting an arrest of a suspect (like herein petitioner) who, in
all probability, could have put up a degree of resistance which an untrained civilian may not be able to contain
without endangering his own life.Moreover, it is a reality that curbing lawlessness gains more success when law
enforcers function in collaboration with private citizens. It is precisely through this cooperation, that the offense
herein involved fortunately did not become an additional entry to the long list of unreported and unsolved crimes.

It is appropriate to state at this juncture that a suspect, like petitioner herein, cannot defeat the arrest which has
been set in motion in a public place for want of a warrant as the police was confronted by an urgent need to
render aid or take action.The exigent circumstances of hot pursuit,a fleeing suspect, a moving vehicle, the
public place and the raining nighttime all created a situation in which speed is essential and delay
improvident.The Court acknowledges police authority to make the forcible stop since theyhad more than mere
reasonable and articulable suspicion that the occupant of the vehicle has been engaged in criminal
activity.Moreover, when caught inflagrante delictowith possession of an unlicensed firearm (Smith & Wesson)
and ammunition (M-16 magazine), petitioners warrantless arrest was proper as he was again actually
committing another offense (illegal possession of firearm and ammunitions) and this time in the presence of a
peace officer.

Besides, the policemens warrantless arrest of petitioner could likewise be justified under paragraph
as he had in fact just committed an offense.

There was no supervening event or a considerable lapse of time between the hit and run and the actual
apprehension. Moreover, after having stationed themselves at the Abacan bridge in response to Manarangs
report, the policemen saw for themselves the fast approaching Pajero of petitioner, its dangling plate number
(PMA 777 as reported by Manarang), and the dented hood and railings thereof. These formed part of the
arresting police officers personal knowledge of the facts indicating that petitioners Pajero was indeed the
vehicle involved in the hit and run incident.Verily then, the arresting police officers acted upon verified personal
knowledge and not on unreliable hearsay information.

Furthermore, in accordance with settled jurisprudence, any objection, defect or irregularity attending an arrest
must be made before the accused enters his plea.Petitioners belated challenge thereto aside from his failure to
quash the information, his participation in the trial and by presenting his evidence, placed him in estoppel to

Page 75 of 236
Constitutional Law 2 Bill of Rights Armando Santiago Jr

assail the legality of his arrest.Likewise, by applying for bail, petitioner patently waived such irregularities and
defects.

We now go to the firearms and ammunitions seized from petitioner without a search warrant, the admissibility in
evidence of which, we uphold.

The five (5) well-settled instances when a warrantless search and seizure of property is valid,are as follows:

warrantless search incidental to a lawful arrest recognized under Section 12, Rule 126 of the Rules of Courtand
by prevailing jurisprudence,

Seizure of evidence in plain view, the elements ofwhich are:


aprior valid intrusionbased on the valid warrantless arrest in which the police are legally present in the pursuit of
their official duties;

the evidence wasinadvertentlydiscoveredby the police who had the right to be where they are;
the evidence must be immediately apparent, and
plain view justified mereseizureof evidence without further search.
search of a moving vehicle. Highly regulated by the government, the vehicles inherent mobility reduces
expectation of privacy especially when its transit in public thoroughfares furnishes a highly reasonable suspicion
amounting to probable cause that the occupant committed a criminal activity.
consented warrantless search, and
customs search.

In conformity with respondent courts observation, it indeed appears that the authorities stumbled upon
petitioners firearms and ammunitions without even undertaking any active search which, as it is commonly
understood, is a prying into hidden places for that which is concealed. The seizure of the Smith & Wesson
revolver and an M-16 rifle magazine was justified for they came within plain view of the policemen
who inadvertently discovered the revolver and magazine tucked in petitioners waist and back pocket
respectively, when he raised his hands after alighting from his Pajero. The same justification applies to the
confiscation of the M-16 arm

alite rifle which wasimmediately apparentto the policemen as they took a casual glance at the Pajero and saw
said rifle lying horizontally near the drivers seat.Thus it has been held that:

(W)hen in pursuing an illegal action or in the commission of a criminal offense, the . . . police officers should
happen to discover a criminal offense being committed by any person, they are not precluded from performing
their duties as police officers for the apprehension of the guilty person and the taking of thecorpusdelicti.

Objects whose possession are prohibited by law inadvertently found in plain view are subject to seizure even
without a warrant.

With respect to the Berreta pistol and a black bag containing assorted magazines, petitioner voluntarily
surrendered them to the police.This latter gesture of petitioner indicated a waiver of his right against the alleged
search and seizure, and that his failure to quash the information estopped him from assailing any purported
defect.

Even assumingthat the firearms and ammunitions were products of an active search done by the authorities on
the person and vehicle of petitioner, their seizure without a search warrant nonetheless can still be justified
under a searchincidental to a lawful arrest (first instance). Once the lawful arrest was effected, the police may
undertake a protective search of the passenger compartment and containers in the vehicle which are within
petitioners grabbing distance regardless of the nature of the offense. This satisfied the two-tiered test of an
incidental search: (i) the item to be searched (vehicle) was within the arrestees custody or area of immediate
control and (ii) the search was contemporaneous with the arrest. The products of that search are admissible
evidence not excluded by the exclusionary rule. Another justification is a search of a moving vehicle (third
instance).In connection therewith, a warrantless search is constitutionally permissible when, as in this case, the

Page 76 of 236
Constitutional Law 2 Bill of Rights Armando Santiago Jr

officers conducting the search have reasonable or probable cause to believe, before the search, that either the
motorist is a law-offender (like herein petitioner with respect to the hit and run) or the contents or cargo of the
vehicle are or have been instruments or the subject matter or the proceeds of some criminal offense.

Page 77 of 236
Constitutional Law 2 Bill of Rights Armando Santiago Jr

Privacy of communication and


correspondence
Section 3 Article III.

(1) The privacy of communication and correspondence shall be


inviolable except upon lawful order of the court, or when public
safety or order requires otherwise as prescribed by law.

(2) Any evidence obtained in violation of this or the preceding


section shall be inadmissible for any purpose in any proceeding.

SECTION 3. DIGESTED PROVISION


GENERAL RULE: WHAT
The privacy of communication and correspondence shall be inviolable

EXCEPTION: HOW

except upon lawful order of the court, or when public safety or order requires otherwise as prescribed by law.

TO STRENGTHEN THE GR:

Any evidence obtained in violation of this or the preceding section shall be inadmissible for any purpose in any
proceeding.

US Supreme Court applicable only to the unlawful taking of tangible objects


Rules out eves dropping on private gadgets and thus covers with its protection even intangible things
US Supreme Court, letters and seated packages in the mails may be examined only as to their external
appearance and right and may not be opened except in accordance with the constitutional requirement of a
lawful search and seizures.

WHAT IS THE WRIT OF HABEAS DATA?


It is a remedy available to any person whose right to privacy in life, liberty or security is violated or threatened by
an unlawful act or omission of a public official or employee, or of a private individual or entity engaged in the
gathering, collecting or storing of data or information regarding the person, family, home and correspondence of
the aggrieved party.

WHO MAY FILE A PETITION FOR THE ISSUANCE OF A WRIT OF HABEAS DATA?
The petition may be filed by the aggrieved party. However, in cases of extralegal killings and enforced
disappearances, the petition may be filed by:

Any member of the immediate family of the aggrieved party, namely: the spouse, children and parents;
Any ascendant, descendant or collateral relative of the aggrieved party within the fourth civil degree of
consanguinity or affinity, in default of those mentioned in the preceding paragraph.

Page 78 of 236
Constitutional Law 2 Bill of Rights Armando Santiago Jr

WHERE CAN THE PETITION BE FILED?


Regional Trial Court where the petitioner or respondent resides, or that which has jurisdiction over the place
where the data or information is gathered, collected or stored, at the option of the petitioner;
Supreme Court;
Court of Appeals; or
Sandiganbayan, when the action concerns public data files of government offices.

Page 79 of 236
Constitutional Law 2 Bill of Rights Armando Santiago Jr

Right to privacy
DISCOVERY AND REVELATION OF SECRETS (CHAPTER THREE)
Art. 290. Discovering secrets through seizure of correspondence. The penalty of prision correccional in
its minimum and medium periods and a fine not exceeding 500 pesos shall be imposed upon any private
individual who in order to discover the secrets of another, shall seize his papers or letters and reveal the
contents thereof.

If the offender shall not reveal such secrets, the penalty shall be arresto mayor and a fine not exceeding 500
pesos

The provision shall not be applicable to parents, guardians, or persons entrusted with the custody of minors with
respect to the papers or letters of the children or minors placed under their care or study, nor to spouses with
respect to the papers or letters of either of them.

Art. 291. Revealing secrets with abuse of office. The penalty of arresto mayor and a fine not exceeding
500 pesos shall be imposed upon any manager, employee, or servant who, in such capacity, shall learn the
secrets of his principal or master and shall reveal such secrets.

Art. 292. Revelation of industrial secrets. The penalty of prision correccional in its minimum and medium
periods and a fine not exceeding 500 pesos shall be imposed upon the person in charge, employee or workman
of any manufacturing or industrial establishment who, to the prejudice of the owner thereof, shall reveal the
secrets of the industry of the latter.

R.A. 4200 - Anti-Wiretapping Law

R.A. 10175 - Cybercrime law (Privacy to the internet)

Page 80 of 236
Constitutional Law 2 Bill of Rights Armando Santiago Jr

Freedom of expression
No law shall be passed abridging the freedom of speech, of
expression, or of the press, or the right of the people peaceably to
assemble and petition the Government for redress of grievances
(Section 4, Article III)
EXPLAIN PRIOR RESTRAINT AND SUBSEQUENT PUNISHMENT
PRIOR RESTRAINT
(THIS IS NOT ALLOWED) means governmental restrictions on the press or other forms of expression in
advance of actual publication like licensing, judicial restraint in the form of injunction or taxes measured by gross
receipts for the privilege or flat license fee for the privilege of selling religious books. (Hindi mo pa ginagawa
yung expression mo pinigilan ka na which is UNCONSTITUTIONAL)

SUBSEQUENT PUNISHMENT
(THIS IS ALLOWED) is an assurance so that an individual may not hesitate to speak for fear that he might be
held to account for his speech or that he may suffer the retaliation of the officials he may have criticized or cited.
(Ginawa mo muna yung expression mo tapos nung may nagawa kang violation tsaka ka binigyan ng punishment
which is CONSTITUTIONAL)

4 ASPECTS OF FREEDOM OF PRESS


Freedom from prior restraint
Freedom from punishment subsequent to publication
Freedom of access to information
Freedom of circulation

TESTS: EXPLAIN DANGEROUS TENDENCY RULE, CLEAR AND PRESENT


DANGER, AND THE BALANCING OF INTEREST TEST

DANGEROUS TENDENCY RULE


For speech to be punishable, there should be a rational connection between the speech and the evil
apprehended. Simply put, the determination by the question: Is the statute reasonable? (Exisiting dapat yung
evil sought or yung masamang mangyayari hindi pwedeng bumase sa BAKA dapat malinaw)

CLEAR AND PRESENT DANGER RULE


The question in every case is whether the words used in such circumstances are of such nature as to create a
clear and present danger that they will bring about the substantive evils that Congress has a right to prevent. It is
a question of proximity and degree. (Kapag may malinaw na basahan na makakasama sa public dun pa lang
pwedeng pigilan)

BALANCING OF INTEREST TEST


If on balance it appears that the public interest served by restrictive legislation is of such a character that it
outweighs the abridgment of freedom, then the Court will find the legislation valid (may 2 Interest katulad sa
case ni Hello Garci, may individual interest at public people interest. In application sa case ni hello garci, public
interest na makakasira at ma-violate yung right to privacy since it was taken from wiretapping and the other
interest is interest of the people since they are the one who voted and have the right over election because we
are in a democratic country. On that case explain how SC apply this test.)

Page 81 of 236
Constitutional Law 2 Bill of Rights Armando Santiago Jr

FACIAL CHALLENGE AND THE OVER BREADTH DOCTRINE


The rule prohibits one from challenging the constitutionality of the statute based solely on the violation of the
rights of third persons not before the court. This rule is also known as the prohibition against third-party
standing.

The over breadth doctrine is an exception to the prohibition against third-party standing. It permits a person to
challenge a statute on the ground that it violates the First Amendment (free speech) rights of third parties not
before the court, even though the law is constitutional as applied to that defendant. In other words, the over
breadth doctrine provides that: Given a case or controversy, a litigant whose own activities are unprotected may
nevertheless challenge a statute by showing that it substantially abridges the First Amendment rights of other
parties not before the court.

COMMERCIAL SPEECH
The Supreme Court of the United States has prescribed the four-prong Central Hudson test to determine
whether a governmental regulation of commercial speech is constitutional. This test asks initially:

whether the commercial speech at issue is protected by the First Amendment (that is, whether it concerns a
lawful activity and is not misleading) and
whether the asserted governmental interest in restricting it is substantial. If both inquiries yield positive
answers, then to be constitutional the restriction must;
directly advance the governmental interest asserted, and
be not more extensive than is necessary to serve that interest.

PRIVATE VS. GOVERNMENT SPEECH


The Government Speech doctrine
establishes that the government may advance its own speech without requiring viewpoint neutrality when the
government itself is the speaker.

CASES

PRIMICIAS VS. FUGOSO


PRIOR RESTRAINT
Ponencia: Feria J.
Topic: Application of Prior Restraint in relation to clear and present danger / Necessity of obtaining permit
Trigger of facts: petitioner has no permit to rally because allegedly there is Clear and present danger / permit is
indispensable dahil ang pinagraralihan ay street/bridge and etc. which is used by public kasi dapat sila
makasagabal sa daan.
Triger of issue:
Trigger of Ruling:
Verdict:
End point of the case: the permit is needed to regulate the rally. permit is not made to avoid them from excersing
their right.

FACTS
Cipriano Primicias, a campaign manager for the coalesced parties, filed a petition for mandamus to compel
Fugoso, the mayor, to issue a permit for the holding of a public meeting at Plaza Miranda for the purpose of
petitioning the government for redress of grievances.
The Philippine legislature has delegated the exercise of the police power to the Municipal Board of the City of
Manila, which according to Administrative Code, grants the Municipal Board, among others

Page 82 of 236
Constitutional Law 2 Bill of Rights Armando Santiago Jr

the following legislative power, to wit: "(p) to provide for the prohibition and suppression of riots, affrays,
disturbances, and disorderly assemblies
Under the above delegated power, the Municipal Board of the City of Manila, enacted Revised Ordinances of
1927 which prohibits as an offense against public peace,
Revised Ordinance penalizes as a misdemeanour,
which provides "any act, in any public place, meeting, or procession, tending to disturb the peace or excite a
riot; or collect with other persons in a body or crowd for any unlawful purpose; or disturb or disquiet any
congregation engaged in any lawful assembly.
Other section also contains, that the holding of any parade or procession in any streets or public places is
prohibited
unless a permit therefor is first secured from the Mayor who shall, on every such occasion, determine or
specify the streets or public places
As there is no express and separate provision in the Revised Ordinance of the City regulating the holding of
public meeting or assembly at any street or public places, the provisions of said section 1119 regarding the
holding of any parade or procession in any street or public paces may be applied by analogy to meeting and
assembly in any street or public places.


ISSUE
WON Mayor has the right to refuse to issue permit hence violating freedom of assembly

HELD
No. Petition granted.

RATIO
A construction of the law giving the mayor the power to prohibit the use of the streets for assemblies for the
reason that would make the ordinance invalid and void or violative of the constitutional limitations.
As the Municipal Boards is empowered only to regulate the use of streets, parks, and the other public places,
and the word "regulate," as used in the said code, means and includes the power to control, to govern, and to
restrain
but cannot be construed a synonymous with construed "suppressed" or "prohibit" the Municipal Board cannot
grant the Mayor a power that it does not have.
Besides, the powers and duties of the Mayor as the Chief Executive of the City are executive and one of them
is "to comply with and enforce and give the necessary orders for the faithful performance and execution of
laws and ordinances
a part of the Charter of the City of Manila, which provides that the Mayor shall have the power to grant and
refuse municipal licenses or permits of all classes
cannot be cited as an authority for the Mayor to deny the application of the petitioner, for the simple reason
that said general power is based upon the ordinances enacted by the Municipal Board requiring licenses or
permits to be issued by the Mayor
Besides, assuming arguendo that the Legislature has the power to confer, and in fact has conferred, upon the
Mayor the power to grant or refuse licenses and permits of all classes.
The respondents reason for not granting the permit was because he believed that the participants would
speak against the government.
Hague vs. Committee on Industrial Organization, supra, "It does not make comfort and convenience in the
use of streets or parks the standard of official action. It enables the Director of Safety to refuse the permit on
his mere opinion that such refusal will prevent riots, disturbances or disorderly assemblage. It can thus, as the
record discloses,
be made the instrument of arbitrary suppression of free expression of views on national affairs, for the
prohibition of all speaking will undoubtedly 'prevent' such eventualities.
"Fear of serious injury cannot alone justify suppression of free speech and assembly.
To justify suppression of free speech there must be reasonable ground to fear that serious evil will result if
free speech is practiced.
There must be reasonable ground to believe that the danger apprehended is imminent. There must be
reasonable ground to believe that the evil to be prevented is a serious one.

Page 83 of 236
Constitutional Law 2 Bill of Rights Armando Santiago Jr

MIRIAM COLLEGE FOUNDATION INC. VS. COURT OF


APPEALS
SUBSEQUENT PUNISHMENT
Ponencia: Kapunan J.
Topic: Freedom of press / Sex Articles / who have jurisdiction over the said case = Mariam College
Trigger of facts: May publication ng school newspaper pero yung content nya ay orientation ng sex, inexpell ang
mga editors ng newspaper and alleged na may journalism act at walang jurisdiction ang school to expel the
students.
Triger of issue:
Trigger of Ruling:
Verdict: School has the authority to hear and decide the cases filed against respondent student / CA decision was
reversed and set aside / Mariam college ordered to readmit Joel Tan
End point: the school has the power to regulate against violation of their student. Freedom of Expression is a right,
even if its not absolute, which should be limited with decency
End point of the case: the school has the power to regulate against violation of their student. Freedom of
Expression is a right, even if its not absolute, which should be limited with decency.

FACTS
Following the publication of Miriam College's school paper (Chi-Rho), and magazine (Ang Magasing
Pampanitikan ng Chi-Rho),
the members of the editorial board, and Relly Carpio, author of Libog,
all students of Miriam College, received a letter signed by the Chair of the Miriam College Discipline
Committee.
The Letter informed them that letters of complaint were "filed against you by members of the Miriam
Community and a concerned Ateneo grade five students have been forwarded to the Discipline Committee for
inquiry and investigation.
As expressed in their complaints you have violated regulations in the student handbook and compelled to
submit a written statement in answer to the charge/s on or before the initial date.
None of the students submitted their respective answers.
They instead requested Dr. Sevilla to transfer the case to the Regional Office of the Department of Education,
Culture and Sports (DECS) which supposedly had jurisdiction over the case.
In a Dr. Sevilla again required the students to file their written answers.
In response, Atty. Ricardo Valmonte, lawyer for the students, submitted a letter to the Discipline Committee
reiterating his clients' position that said Committee had no jurisdiction over them.
According to the students lawyer, the Committee was "trying to impose discipline on his clients on account of
their having written articles and poems in their capacity as campus journalists."
Hence, he argued that "what applies is Republic Act No. 7079 The Campus Journalism Act and its
implementing rules and regulations." He also questioned the partiality of the members of said Committee who
allegedly "had already articulated their position" against his clients. The Discipline Committee proceeded with
its investigation ex parte.
Thereafter, the Discipline Board, after a review of the Discipline Committee's report, imposed disciplinary
sanctions upon the students, to wit:

(1) Jasper Briones [Editor-in-Chief of ChiRho, 4th year student]: Expulsion;


(2) Daphne Cowper: Suspension up to (summer) March 1995;
(3) Imelda Hilario: suspension for 2 weeks to expire on 2 February 1995;
(4) Deborah Ligon [4th year student and could graduate as summa cum laude]: suspension up to May 1995;
(5) Elizabeth Valdezco: suspension up to (summer) March 1995;
(6) Camille Portuga [Octoberian]: graduation privileges withheld, including diploma;
(7) Joel Tan: suspension for 2 weeks to expire on 2 February 1995;
(8) Gerald Gary Renacido [2nd year student]: Expelled and given transfer credentials;
(9) Relly Carpio [3rd year student]: Dismissed and given transfer credentials;
(10) Jerome Gomez [3rd year student]: Dismissed and given transfer credentials; and
(11) Jose Mari Ramos [Art editor of Chi-Rho, 2nd year student]: Expelled and given transfer papers.

Said students thus filed a petition before the Regional Trial Court of Quezon City questioning the jurisdiction of
the Discipline Board of Miriam College over them.
In the long run of appeal
The students, excluding the 3 students elevate the matter to the Supreme Court
Page 84 of 236
Constitutional Law 2 Bill of Rights Armando Santiago Jr

Issue: Whether Section 7 of the Campus Journalism Act precludes the schools right to discipline its students.


HELD
the Supreme Court has upheld the right of the students to free speech in school premises.
The right of the students to free speech in school premises, however, is not absolute.
The right to free speech must always be applied in light of the special characteristics of the school
environment. Thus, the Court did not rule out disciplinary action by the school for "conduct by the student, in
class or out of it, which for any reason - whether it stems from time, place, or type of behaviour - which
materially disrupts classwork or involves substantial disorder or invasion of the rights of others."
Provisions of law (such as Section 7 of the Campus Journalism Act) should be construed in harmony with
those of the Constitution; acts of the legislature should be construed, wherever possible, in a manner that
would avoid their conflicting with the fundamental law.
A statute should not be given a broad construction if its validity can be saved by a narrower one. Thus,
Section 7 should be read in a manner as not to invade upon the school's right to discipline its students. At the
same time, however, said provision should not be construed as to improperly restrict the right of the students
to free speech.
Consistent with jurisprudence, Section 7 of the Campus Journalism Act is read to mean that the school cannot
suspend or expel a student solely on the basis of the articles he or she has written, except when such article
materially interrupt class work or involve substantial disorder or invasion of the rights of others. Further, the
power of the school to investigate is an assistant of its power to suspend or expel. It is a necessary result to
the enforcement of rules and regulations and the maintenance of a safe and orderly educational environment
conducive to learning.
That power, like the power to suspend or expel, is an inherent part of the academic freedom of institutions of
higher learning guaranteed by the Constitution. the Court therefore rule that Miriam College has the authority
to hear and decide the cases filed against the students.
Academic Freedom; Schools and Colleges; Academic freedom includes the right of the school or college to
decide for itself its aims and objectives, and how best to attain them free from outside coercion or interference
save possibly when the overriding public welfare calls for some restraint. Section 5 (2), Article XIV of the
Constitution guarantees all institutions of higher learning academic freedom. This institutional academic
freedom includes the right of the school or college to decide for itself, its aims and objectives, and how best to
attain them free from outside coercion or interference save possibly when the overriding public welfare calls
for some restraint. The essential freedoms subsumed in the term academic freedom encompasses the
freedom to determine for itself on academic grounds:

(1) Who may teach,


(2) What may be taught,
(3) How it shall be taught, and
(4) Who may be admitted to study.

The right of the school to discipline its students is at once apparent in the third freedom, i.e., how it shall be
taught. A school certainly cannot function in an atmosphere of anarchy.

FRANCISCO CHAVEZ V. RAUL M. GONZALES


Ponencia: PUNO,C.J.
Topic: Hello Garci / explanation of 3 test / content based expression and content neutral expression
Trigger of facts:
Triger of issue:
Trigger of Ruling:
Verdict: petition was granter / nullifying the official statement made by the respondent
End point of the case: 2 interest to be balance

FACTS
A year following the 2004 national and local elections, Press Secretary Ignacio Bunye
Page 85 of 236
Constitutional Law 2 Bill of Rights Armando Santiago Jr

disclosed to the public how the opposition planned to destabilize the administration by releasing an audiotape of
a mobile phone conversation allegedly between President Gloria Macapagal Arroyo and Commissioner
Garcillano of the Commission on Elections (COMELEC).
The conversation was alleged taken through wire-tapping.
respondent Secretary Raul Gonzales of the Department of Justice (DOJ) warned reporters who are in
possession of copies of the said conversation, as well as those broadcasting companies and/or publishers that
they may be held liable under the Anti-Wiretapping Act.
NTC issued a press release strengthening the prohibition on the dissemination of the same that the
broadcasting/airing of such information shall be just cause for the suspension, revocation and/or cancellation
of the licenses or authorizations issued by the Commission.
Petitioner Francisco Chavez filed a petition against respondent Chavez and NTC, praying for the issuance of
the said writs, issuances and orders of respondents
as they were complete violations of the freedom of expression and of the press, and the right of the people to
information on matters of public concern.

ISSUE
Whether or not the acts of the respondents abridge freedom of speech and of the press.

HELD
Yes. Generally, restraints on freedom of speech and expression are evaluated by either or a combination of
three tests,i.e.,

thedangerous tendency doctrine, which limits speech once a rational connection has been established between
the speech controlled and the danger intended;

the balancing of interests tests, a standard when courts balance conflicting social values and individual
interests,and

theclear and present danger rulewhich rests on the premise that speech may be controlled because there is
substantial danger that the speech will likely lead to an evil the government has a right to prevent.

It appears that the great evil which government wants to prevent is the airing of a tape recording in alleged
violation of the anti-wiretapping law. However, respondents evidence falls short of satisfying the clear and
present danger test.

Firstly, the various statements of the Press Secretary obfuscate the identity of the voices in the tape recording.

Secondly, the integrity of the taped conversation is also suspect. The Press Secretary showed to the public two
versions, one supposed to be a complete version and the other, an altered version.

Thirdly, the evidence of the respondents on the whos and the hows of the wiretapping act is uncertain,
especially considering the tapes different versions. The identity of the wire-tappers, the manner of its
commission and other related and relevant proofs are some of the invisibles of this case.

Fourthly, given all these unsettled aspects of the tape, it is even arguable whether its airing would violate the
anti-wiretapping law.

We rule thatnot every violation of a law will justify straitjacketing the exercise of freedom of speech and of the
press.

The need to prevent their violation cannot per setrump the exercise of freespeechandfreepress,a preferred
right whose breach can leadtogreater evils.For this failure of the respondents alone to offer proof to satisfy the
clear and present danger test, the Court has no option but to uphold the exercise of free speech and free press.

Page 86 of 236
Constitutional Law 2 Bill of Rights Armando Santiago Jr

There is no showing that the feared violation of the anti-wiretapping law clearly endangers thenational security
of the State.

What are the four aspects of freedom of the press?

SUGGESTED ANSWER:

1. freedom from prior restraint


2. freedom from punishment subsequent to publication
3. freedom of access to information
4. freedom of circulation

What is the difference between content-neutral regulation and a content-based restraint?

SUGGESTED ANSWER:

A content-neutral regulation merely concerned with the incidents of the speech or one that controls the time,
place or manner and under well define standards (example BP 880). pwedeng i-restrict o pwedeng gawin ng
government para macontrol ang expression. in consideration of government interest

A content-based restraint or censorship is based on the subject matter of the utterance or speech. dapat
maprove na may clear and present danger test para makontrol yung content. pwede rin i-restrict basta pumasa
dun sa clear and present danger test

The first is subjected to an intermediate review. The other bears a heavy presumption of invalidity and is
measured against the clear and present danger rule.

Hello Garci CDs Case

In 2006, following the public release of the Hello Garci tapes, DOJ Secretary Raul Gonzalez and the NTC
issued a warning against reporters from airing the same. Gonzales warned reporters that those who had copies
of the CD and those broadcasting or publishing its contents could be held liable under the Anti-Wiretapping Act.
Are the official statements of the Gonzales and the NTC constitutional?

SUGGESTED ANSWER:

No. The SC nullified the official statements made by DOJ Secretary Gonzalez and the NTC for constituting
unconstitutional prior restraint on the exercise of freedom of speech and of the press. The Court held that the
challenged acts in the case at bar need to be subjected to the clear and present danger rule, as they are
content-based restrictions. The acts of respondents focused solely on but one object a specific content fixed
as these were on the alleged taped conversations between President Arroyo and Garcillano.

The Court said that a governmental action that restricts freedom of speech or of the press based on content is
given the strictest scrutiny, with the government having the burden of overcoming the presumed
unconstitutionality by the clear and present danger rule. visit fellester.blogspot.com This rule applies equally to
all kinds of media, including broadcast media. Prior restraint on speech based on its content cannot be justified
by hypothetical fears, said the Court.

Same; Freedom of Expression; In line with the liberal policy of the Supreme Court on locus standi when a case
involves an issue of overarching significance to our society, the Court brushes aside technicalities of procedure
and takes cognizance of the instant petition, seeing as it involves a challenge to the most exalted of all the civil
rights, the freedom of expression.In line with the liberal policy of this Court on locus standi when a case
involves an issue of overarching significance to our society, we therefore brush aside technicalities of procedure
and take cognizance of this petition, seeing as it involves a challenge to the most exalted of all the civil rights,
the freedom of expression. The petition raises other issues like the extent of the right to information of the public.

Page 87 of 236
Constitutional Law 2 Bill of Rights Armando Santiago Jr

It is fundamental, however, that we need not address all issues but only the most decisive one which in the case
at bar is whether the acts of the respondents abridge freedom of speech and of the press. Chavez vs. Gonzales,
545 SCRA 441, G.R. No. 168338 February 15, 2008

BAYAN, ET AL., VS. EDUARDO ERMITA


PRIOR RESTRAINT
Ponencia: AZCUNA J
Topic: Obtaining permit / freedom park / B.P 880 alleged to be unconstitutional
Trigger of facts:
Triger of issue:
Trigger of Ruling:
Verdict: the petitions are GRANTED in part, and respondents, more particularly the Secretary of the Interior and
Local Governments, are DIRECTED to take all necessary steps for the immediate compliance with Section 15 of
Batas Pambansa No. 880 through the establishment or designation of at least one suitable freedom park or plaza in
every city and municipality of the country. After thirty (30) days from the finality of this Decision, subject to the
giving of advance notices, no prior permit shall be required to exercise the right to peaceably assemble and petition
in the public parks or plaza in every city or municipality that has not yet complied with section 15 of the law.
Furthermore, calibrated pre-emptive response (CPR), insofar as it would purport to differ from or be in lieu of
maximum tolerance, is NULL and VOID and respondents are ENJOINED to REFRAIN from using it and to STRICTLY
OBSERVE the requirements of maximum tolerance, the petitions are DISMISSED in all other respects, and the
constitutionality of Batas Pambansa No. 880 is SUSTAINED
End point of the case: it is a right which is not absolute may be regulated. they are not prohibit but they have to
obtain a permit. permit is just merely to regulate it and not to prohibit

FACTS
The petitioners, Bayan, et al., alleged that they are citizens and taxpayers of the Philippines
and that their right as organizations and individuals were violated when the rally they participated was violently
dispersed by policemen implementing Batas Pambansa No. 880.
Petitioners contended that Batas Pambansa No. 880 is clearly a violation of the Constitution and the
International Covenant on Civil and Political Rights and other human rights treaties of which the Philippines is
a signatory.
They argue that B.P. No. 880 requires a permit before one can stage a public assembly regardless of the
presence or absence of a clear and present danger.
It also restricts the choice of venue and is thus repugnant to the freedom of expression clause as the time and
place of a public assembly form part of the message which the expression is sought.
Furthermore, it is not content-neutral as it does not apply to mass actions in support of the government. The
words lawful cause, opinion, protesting or influencing suggest the explanation of some cause not
supported by the government.
Also, the phrase maximum tolerance shows that the law applies to assemblies against the government
because they are being tolerated.
As a content-based legislation, it cannot pass the strict scrutiny test.
the petitioners, in the interest of a speedy resolution of the petitions, withdrew the portions of their petitions
raising factual issues
particularly those raising the issue of whether B.P. No. 880 and/or CPR is void as applied to the rallies.

ISSUE
Whether the Calibrated Pre-emptive response and the Batas Pambansa No. 880, specifically Sections 4, 5, 6,
12, 13(a) and 14(a) violates Art. III Sec. 4 of the Philippine Constitution as it causes a disturbing effect on the
exercise by the people of the right to peaceably assemble.

HELD

Page 88 of 236
Constitutional Law 2 Bill of Rights Armando Santiago Jr

Section 4 of Article III of the Philippine Constitution provides that no law shall be passed abridging the freedom
of speech, of expression, or of the press, or the right of the people peaceably to assemble and petition the
government for redress of grievances.
The right to peaceably assemble and petition for redress of grievances, together with freedom of speech, of
expression, and of the press, is a right that enjoys authority in the range of constitutional protection. For this
rights represent the very basis of a functional democratic organization, without which all the other rights would
be meaningless and unprotected.
However, it must be remembered that the right, while sacrosanct, is not absolute.
It may be regulated that it shall not be injurious to the equal enjoyment of others having equal rights, nor
injurious to the rights of the community or society.
The power to regulate the exercise of such and other constitutional rights is termed the sovereign police
power, which is the power to prescribe regulations, to promote the health, morals, peace, education, good
order or safety, and general welfare of the people.
B.P. No 880 is not an absolute ban of public assemblies but a restriction that simply regulates the time, place
and manner of the assemblies.
B.P. No. 880 thus readily shows that it refers to all kinds of public assemblies that would use public places.
The reference to lawful cause does not make it content-based because assemblies really have to be for
lawful causes, otherwise they would not be peaceable and entitled to protection. Neither the words opinion,
protesting, and influencing in of grievances come from the wording of the Constitution, so its use cannot be
avoided.
Finally, maximum tolerance is for the protection and benefit of all rallyist and is independent of the content of
the expression in the rally.
Furthermore, the permit can only be denied on the ground of clear and present danger to public order, public
safety, public convenience, public morals or public health. This is a recognized exception to the exercise of the
rights even under the Universal Declaration of Human Rights and The International Covenant on Civil and
Political Rights.

BURGOS, SR. VS. CHIEF OF STAFF, AFP


Ponencia: ESCOLIN J.
Topic: We FORUM / Closure of premise constitute virtual denial of press freedom / GENERAL WARRANT IS NOT
VAID WARRANT
Trigger of facts: may warrant tapos sinara yung newpaper building which means nahinto yung transaksyon ng
pagawa ng newspaper
Triger of issue: lack of probable cause
Trigger of Ruling: the warrant was not valid because it is a general warrant. yung nature ng pagpadlock ng building
ay prior restraint dahil alleged pa lang yung subversive act. meaning hindi alam ni judge kung ano talaga yung
ginamit nung mga accused to commit the crime.. therefore, they are restraint from their press freedom.
Verdict: Search warrants issued were NULL and VOID, Prayer were granted to rescind all articles seized
End point of the case: Mere discontinuance due to the erroneous action of the state may constitute deprivation of
their press freedom.

FACTS
Executive Judge of Quezon City, issued 2 search warrants
where the premises building in Quezon Avenue, business addresses of the Metropolitan Mail and We
Forum newspapers respectively, were searched, and office and printing machines, equipment, paraphernalia,
motor vehicles and other articles used in the printing, publication and distribution of the said newspapers
as well as numerous papers, documents, books and other written literature alleged to be in the possession
and control of Jose Burgos, Jr. publisher-editor of the We Forum newspaper, were seized.
A petition was filed after 6 months following the raid to question the validity of said search warrants, and to
enjoin the Judge Advocate General of the AFP, the city fiscal of Quezon City, et.al. from using the articles
seized as evidence in Criminal Case Q-022782 of the RTC Quezon City (People v. Burgos).

ISSUE
Page 89 of 236
Constitutional Law 2 Bill of Rights Armando Santiago Jr

Whether allegations of possession and printing of seditious materials may be the basis of the issuance of search
warrants.

HELD
Section 3 provides that no search warrant or warrant of arrest shall issue except upon probable cause to be
determined by the judge, or such other responsible officer as may be authorized by law, after examination
under oath or affirmation of the complainant and the witnesses he may produce, and particularly describing
the place to be searched and the persons or things to be seized.
Probable cause for a search is defined as such facts and circumstances which would lead a reasonably
discreet and practical man to believe that an offense has been committed and that the objects sought in
connection with the offense are in the place sought to be searched.
In mandating that no warrant shall issue except upon probable cause to be determined by the judge, after
examination under oath or affirmation of the complainant and the witnesses he may produce; the Constitution
requires no less than personal knowledge by the complainant or his witnesses of the facts upon which the
issuance of a search warrant may be justified.
Herein, a statement in the effect that Burgos is in possession or has in his control printing equipment and
other paraphernalia, news publications and other documents which were used and are all continuously being
used as a means of committing the offense of subversion punishable under PD 885, as amended is a mere
conclusion of law and does not satisfy the requirements of probable cause. Bereft of such particulars as would
justify a finding of the existence of probable cause, said allegation cannot serve as basis for the issuance of a
search warrant.
Further, when the search warrant applied for is directed against a newspaper publisher or editor in connection
with the publication of subversive materials, the application and/or its supporting affidavits must contain a
specification, stating with particularity the alleged subversive material he has published or is intending to
publish. Mere generalization will not suffice.

FREEDOM OF PRESS Yung mga kinuhang gamit for printing is considered deprivation of their freedom of
press.

Same; Same; Closure of the premises of a news publishing house constitutes a virtual denial of press freedom.
Such closure is in the nature of previous restraint or censorship abhorrent to the freedom of the press
guaranteed under the fundamental law, and constitutes a virtual denied of petitioners freedom to express
themselves in print. This state of being is patently anathematic to a democratic framework where a free, alert
and even militant press is essential for the political enlighten and growth of the citizenry.

ABAD SANTOS, J.: CONCURRING


I am glad to give my concurrence to the ponencia of Mr. Justice Escolin. At the same time, I wish to state my
own reasons for holding that the search warrants which are the subject of the petition are utterly void.
The action against WE FORUM was a naked suppression of press freedom for the search warrants were issued
in gross violation of the Constitution.

The Constitutional requirement which is expressed in Section 3, Article IV, stresses two points, namely: (1) that
no warrant shall issue but upon probable cause, to be determined by the judge in the manner set forth in said
provision; and (2) that the warrant shall particularly describe the things to be seized. (Stonehill vs. Diokno, 126
Phil. 738, 747: 20 SCRA 383 [1967].)

Any search warrant is conducted in disregard of the points mentioned above will result in wiping out completely
one of the most fundamental rights guaranteed in our Constitution, for it would place the sanctity of the domicile
and the privacy of communication and correspondence at the mercy of the whims, caprice or passion of peace
officers. Burgos, Sr. vs. Chief of Staff, AFP, 133 SCRA 800, No. L-64261 December 26, 1984

Page 90 of 236
Constitutional Law 2 Bill of Rights Armando Santiago Jr

Page 91 of 236
Constitutional Law 2 Bill of Rights Armando Santiago Jr

Freedom of Religion
(SEPARATION OF CHURCH AND
STATE)
No law shall be made respecting an establishment of religion, or
prohibiting the free exercise thereof. The free exercise and enjoyment
of religious profession and worship, without discrimination or
preference, shall forever be allowed. No religious test shall be
required for the exercise of civil or political rights
(Section 5, Article III)
SUPPORTED PROVISIONS
Art. 6 section 28 exemption of church taxation
Art. 14 section 3 Allowed catechism
Art. 6 section 24 (2) - Non-establishment clause means that the State cannot establish or sponsor an official
religion. The non-establishment clause prohibits the State from passing laws which aid one religion, aid all
religions, or prefer one religion over another

The values non-establishment seeks to protect are voluntarism and insulation of the political process from
interfaith dissension. Such voluntarism cannot be achieved unless the political process is insulated from religion
and unless religion is insulated from politics.

Benevolent neutrality is summarized into four propositions or tests:

1. Government must not prefer one religion over another religion or irreligion because such preference would
violet voluntarism and breed dissension; walang kinikilingan
2. Government funds must not be applied to religious purposes because this too would violate voluntarism and
breed interfaith dissension; not allowed to build a church
3. Government action must not aid religion because this too can violate voluntarism and breed interfaith
dissension; and giving funds to a church or religion under public funds
4. Government action must not result in excessive entanglement with religion because this too can violate
voluntarism and breed interfaith dissension. Forcing and prohibiting to join or not in a religion

FREE EXERCISE CLAUSE


Freedom of conscience and freedom to obey to such religious organization or form of worship as the individual
may choose cannot be restricted by law. On the other hand, it safeguards the free exercise of the chosen for of
religion. Thus the Amendment embraces two concepts freedom to believe and freedom to act.

The judicial task in free exercise cases is one of balancing the secular interest of the state with the interest of
religion. The tests are:

Clear and present danger test - The question in every case is whether the action used in such circumstances
and are of such nature as to create a clear and present danger that they will bring about the substantive evils
that Congress has a right to prevent.

Page 92 of 236
Constitutional Law 2 Bill of Rights Armando Santiago Jr

Compelling state interest test - effectuates the First Amendments command that religious liberty is an
independent liberty, that it occupies a preferred position, and that the Court will not permit encroachments upon
this liberty, whether direct or indirect, unless required by clear and compelling government interest of the highest
order

BENEVOLENT NEUTRALITY
With religion looked upon with benevolence and not anger, benevolent neutrality allows accommodation of
religion under certain circumstances. Accommodations are government policies that take religion specifically
into account not to promote the governments favoured form of religion, but to allow individuals and groups to
exercise their religion without hindrance. Their purpose or effect therefore is to remove a burden on, or facilitate
the exercise of, a persons or institutions religion.

BALANCING OF INTERESTS
In weighing the states interest and religious freedom, when these collide, three questions are answered in this
process.

First, has the statute or government action created a burden on the free exercise of religion? The courts often
look into the sincerity of the religious belief, but without inquiring into the truth of the belief because the Free
Exercise Clause prohibits inquiring about its truth;

Second, the court asks: is there a sufficiently compelling state interest to justify this infringement of religious
liberty? In this step, the government has to establish that its purposes are legitimate for the state and that they
are compelling; and

Third, the court asks: has the state in achieving its legitimate purposes used the least intrusive means possible
so that the free exercise is not infringed any more than necessary to achieve the legitimate goal of the state?
The analysis requires the state to show that the means in which it is achieving its legitimate state objective is the
least intrusive means, i.e., it has chosen a way to achieve its legitimate state end that imposes as little as
possible on religious liberties.

STRICT NEUTRALITY
Strict neutrality holds that government should base public policy solely on secular considerations, without regard
to the religious consequences of its actions.

CASES

TARUC VS. DELA CRUZ


Ponencia: CORONA, J.
Topic: separation of church and state.
Trigger of facts: nagfile si taruc ng case sa RTC which the court has no jurisdiction
Triger of issue:
Trigger of Ruling:
Verdict: Petition were denied for lack of merit
End point of the case: it is not for the court to exercise control over church authorities because of the sepration of
church and state except for the protection of civil and property rights.

FACTS
The petitioners are lay members of the Philippine Independent Church (PIC) in Socorro, Surigao City.

Page 93 of 236
Constitutional Law 2 Bill of Rights Armando Santiago Jr

Petitioners led by Taruc shouted for the transfer of parish priest Rustom Florano for the reason that Fr.
Floranos wifes family belonged to a political party opposed to petitioner Tarucs. Bishop De la Cruz found this
reason too weak so he did not give in to the request.
Things worsened when Taruc conducted an open mass for the town Fiesta celebrated by Fr. Ambong who was
not a member of the clergy of the diocese of Surigao.
Petitioners were then expelled/excommunicated from the PIC for the reason of (1) disobedience to duly
constituted authority, (2) inciting dissension resulting in division of the Parish of Our Mother of Perpetual Help
and (3) threatening to forcible occupy the Parish Church causing anxiety among the General Membership.
Petitioners filed a complaint for damages with preliminary injunction against Bishop De la Cruz and impleaded
Fr. Florano and a certain Delfin Bordas for conspiring with the Bishop. They said that their rights to due
process were violated because they were not heard before the order of expulsion was made.

ISSUE
Whether or not the courts have jurisdiction to hear a case involving the expulsion/excommunication of members
of a religious institution NO


RATIO
Section 5 of Article III A form of government where the complete separation of civil and ecclesiastical authority is
insisted upon, the civil courts must not allow themselves to intrude unduly in matters of an ecclesiastical in
nature.
In disputes involving religious institutions or organizations, there is one area, which the Court should not touch:
doctrinal and disciplinary differences.

To the power of excluding form the church those allegedly unworthy of membership, are unquestionably
ecclesiastical matters, which are outside the province of civil courts.

Comments: records show that Bishop De la Cruz pleaded with petitioners several times not to commit acts
inimical to the best interests of PIC. They were also warned of the consequences of their actions yet these pleas
and warnings fell on deaf ears.

Remedial Law; Actions; Jurisdictions; Religious Organizations; It is not for the courts to exercise control over
church authorities in the performance of their discretionary and official functions. We agree with the Court of
Appeals that the expulsion/excommunication of members of a religious institution/organization is a matter best
left to the discretion of the officials, and the laws and canons, of said institution/organization. It is not for the
courts to exercise control over church authorities in the performance of their discretionary and official functions.
Rather, it is for the members of religious institutions/organizations to conform to just church regulations.

Same; Same; Same; Same; In disputes involving religious institutions or organizations, there is one area which
the Court should not touch: doctrinal and disciplinary differences. In the leading case of Fonacier v. Court of
Appeals, we enunciated the doctrine that in disputes involving religious institutions or organizations, there is one
area which the Court should not touch: doctrinal and disciplinary differences. Thus, the amendments of the
constitution, restatement of articles of religion and abandonment of faith or abjuration alleged by appellant,
having to do with faith, practice, doctrine, form of worship, ecclesiastical law, custom and rule of a church and
having reference to the power of excluding from the church those allegedly unworthy of membership, are
unquestionably ecclesiastical matters which are outside the province of the civil courts.

AGLIPAY VS. RUIZ


Ponencia: LAUREL, J.
Topic:
Trigger of facts:alleged that these actions favour a religion which is violation of Separation of church and state
Triger of issue: is it constitutional?
Trigger of Ruling:

Page 94 of 236
Constitutional Law 2 Bill of Rights Armando Santiago Jr

Verdict:
End point of the case: yung benefit ng church are merely incidental and not intended, hindi binabandera ang
catholic church but rather using it to advertise and encourage tourist.

FACTS
Petitioner Aglipay, the head of Phil. Independent Church, filed a writ against respondent Ruiz
the Director of Post, enjoining the latter from issuing and selling postage stamps commemorative of the 33rd Intl
Eucharistic Congress organized by the Roman Catholic.
The petitioner invokes that such issuance and selling, as authorized by Act 4052 by the Phil. Legislature,
contemplates religious purpose for the benefit of a particular sect or church. Hence, this petition.

ISSUE
Whether or not the issuing and selling of commemorative stamps is constitutional?

HELD
The Court said YES, the issuing and selling of commemorative stamps by the respondent does not
contemplate any favour upon a particular sect or church
but the purpose was only to advertise the Philippines and attract more tourist and the government just took
advantage of an event considered of international importance
thus, not violating the Constitution on its provision on the separation of the Church and State. Moreover,
the Court stressed that Religious freedom, as a constitutional mandate is not inhibition of profound reverence
for religion and is not denial of its influence in human affairs. Emphasizing that, when the Filipino people
implored the aid of Divine Providence,
they thereby manifested reliance upon Him who guides the destinies of men and nations. The elevating
influence of religion in human society is recognized here as elsewhere.
In fact, certain general concessions are indiscriminately accorded to religious sects and denominations.

GERMAN VS. BARANGAN


Ponencia: LAUREL, J.
Topic:
Trigger of facts: dinisperse dahil sabi ni barangan may imminent dangeat ginamit lang nilang rason yung freedom of
religion para maka-access sila pero ang plano talaga ay magrally, ang duty ni barangan ay to protect the leader from
anyone.
Triger of issue:
Trigger of Ruling: NO. it is not violative to the freedom of religion.
Verdict: Petition were published
End point of the case: may clear and present danger sa intent ng mga ralihista. iba yung pinapakita nila sa intent
nila which is to rally. it was necessary to maintain the functions of the presidents.

135 SCRA 514 Political Law Religious Freedom vs. Clear and Present Danger Doctrine

FACTS
Reli German et al went to JP Laurel Sreet to pray and worship at the St. Luke Chapel.
But they were barred by General Santiago Barangan from entering the church because the same is within the
vicinity of the Malacaang.
And considering that Germans group is expressively known as the August Twenty-One Movement who were
wearing yellow shirts with compress fists,
Barangan deemed that they were not really there to worship but rather they are there to disrupt the ongoing
within the Malacaang.

Page 95 of 236
Constitutional Law 2 Bill of Rights Armando Santiago Jr

ISSUE
Whether or not the bar disallowing petitioners to worship and pray at St. Lukes is a violation of their freedom to
worship and locomotion.

HELD
No.In the case at bar, German et al were not denied or restrained of their freedom of belief or choice of their
religion
but only in the manner by which they had attempted to translate the same into action.
There has been a clear manifestation by Barangan et al that they allow German et al to practice their religious
belief but not in the manner that German et al impressed.
Such manner impresses clear and present danger to the executive of the state hence the need to curtail it
even at the expense of curtailing ones freedom to worship.

DISSENTING OPINIONS
J. Fernando It would be an unwarranted departure then from what has been unanimously held in the J.B.L.
Reyes decision if on such a basic right as religious freedom -clearly the most fundamental and thus entitled to
the highest priority among human rights, involving as it does the relationship of man to his Creator -this Court will
be less vigilant in upholding any rightful claim. More than ever, in times of stress -and much more so in times of
crisis -it is that deeply-held faith that affords solace and comfort if not for everyone at least for the majority of
mankind. Without that faith, mans very existence is devoid of meaning, bereft of significance.

J. Teehankee The right to freely exercise ones religion is guaranteed in Section 8 of our Bill of Rights. 7
Freedom of worship, alongside with freedom of expression and speech and peaceable assembly along with the
other intellectual freedoms, are highly ranked in our scheme of constitutional values. It cannot be too strongly
stressed that on the judiciary -even more so than on the other departments -rests the grave and delicate
responsibility of assuring respect for and deference to such preferred rights. No verbal formula, no sanctifying
phrase can, of course, dispense with what has been so felicitously termed by Justice Holmes as the sovereign
prerogative of judgment. Nonetheless, the presumption must be to incline the weight of the scales of justice on
the side of such rights, enjoying as they do precedence and primacy.

J. Makasiar With the assurances aforestated given by both petitioners and respondents, there is no clear and
present danger to public peace and order or to the security of persons within the premises of Malacaang and
the adjacent areas, as the respondents has adopted measures and are prepared to insure against any public
disturbance or violence.

Constitutional Law; Exercise of right to religious freedom must be done in good faith without any ulterior motive,
e.g., political. The foregoing cannot but cast serious doubts on the sincerity and good faith of petitioners in
invoking the constitutional guarantee of freedom of religious worship and of locomotion. While it is beyond
debate that every citizen has the undeniable and inviolable right to religious freedom, the exercise thereof, and
of all fundamental rights for that matter, must be done in good faith. As Article 19 of the Civil Code admonishes:
Every person must, in the exercise of his rights and in the performance of his duties x x x x x observe honesty
and good faith.

Same; Reasonable restrictions in use of thoroughfares near the Malacaang Palace are valid as threats to lives
of heads of states are constant and real. Even assuming that petitioners claim to the free exercise of religion
is genuine and valid, still respondents reaction to the October 2, 1984 mass action may not be characterized as
violative of the freedom of religious worship. Since 1972, when mobs of demonstrators crashed through the
Malacaang gates and scaled its perimeter fence, the use by the public of J.P. Laurel Street and the streets
approaching it have been restricted. While travel to and from the affected thoroughfares has not been absolutely
prohibited, passers-by have been subjected to courteous, unobtrusive security checks. The reasonableness of
this restriction is readily perceived and appreciated if it is considered that the same is designed to protect the
lives of the President and his family, as well as other government officials, diplomats
Legislative Fun and foreign guests transacting business with Malacaang. The need to secure the safety of
heads of state and other government officials cannot be overemphasized. The threat to their lives and safety is
Page 96 of 236
Constitutional Law 2 Bill of Rights Armando Santiago Jr

constant, real and felt throughout the world Vivid illustrations of this grave and serious problem are the gruesome
assassinations, kidnappings and other acts of violence and terrorism that have been perpetrated against heads
of state and other public officers of foreign nations.

Same; Same. Said restriction is moreover intended to secure the several executive offices within the
Malacaang grounds from possible external attacks and disturbances. These offices include communications
facilities that link the central government to all places in the land. Unquestionably. the restriction imposed is
necessary to maintain the smooth functioning of the executive branch of the government, which petitioners
mass action would certainly disrupt.

Same; Freedom to translate religious belief into action may be curtailed. In the case at bar, petitioners are not
denied or restrained of their freedom of belief or choice of their religion, but only in the manner by which they
had attempted to translate the same into action. This curtailment is in accord with the pronouncement of this
Court in Gerona v. Secretary of Education. German vs. Barangan, 135 SCRA 514, No. L-68828 March 27, 1985
a

Same; Freedom of locomotion may be curtailed. Suffice it to say that the restriction imposed on the use of J.P.
Laurel Street, the wisdom and reasonableness of which have already been discussed, is allowed under the
fundamental law, the same having been established in the interest of national security. German vs. Barangan,
135 SCRA 514, No. L-68828 March 27, 1985

EBRALINAG ET AL VS. DIV. SUPT. OF SCHOOL OF CEBU


SUBSEQUENT PUNISHMENT
Ponencia: KAPUNAN J.
Topic:
Trigger of facts: Jehovah Member was allegedly deprived with their right to free education
Triger of issue:
Trigger of Ruling:
Verdict: Instant motion was DENIED
End point of the case: walang masamang tumayo at umatend ng flag ceremony hindi na sila kailangang i-expell as
long as they do not disrupt other in the performance of their freedom or as long as it did not violate the right of
others.

CAMEL STORY: una pinasok yung ilong sa tent, tapos sabi nung tao sa loob ng tent okay lang ilong lang naman.
Tapos pinasok nung camel yung ulo, hanggang sa pati leeg at buong katawan, ang ending ung tao na yung nasa
labas ng tent yung camel nasa loob. Yung ang negative side ng ruling of this case

Note: Please read GERONA, ET AL. vs. THE HON. SEC. OF EDUCATION and BALBUNA, ET AL. vs. THE HON. SEC.
OF EDUCATION first before reading this case.

FACTS
DECS Regional Office in Cebu received complaints about teachers and pupils belonging to the Jehovahs
Witness, and enrolled in various public and private schools, which refused to sing the Phil. National Anthem,
salute the flag and recite the patriotic pledge.
Division Superintendent of schools and her Assistant issued a Memorandum directing District Supervisors,
High School Principals and Heads of Private Educational institutions to remove from service
after due process, teachers and school employees, and to deprive the students and pupils from the benefit of
public education, if they do not participate in daily flag ceremony and doesnt obey flag salute rule.
Members of the Jehovahs Witness sect find such memorandum to be contrary to their religious belief and
choose not to obey.
Despite a number of appropriate arguments made by the Cebu officials to let them obey the directives, still
they chose to follow their conviction to their belief.
As a result, an order was issued by the district supervisor of Daan Bantayan District of Cebu, ordering the
dropping from the list in the school register of all Jehovahs Witness teachers and pupils from Grade 1 to
Grade 6 who chose to follow their belief which is against the Flag Salute Law

Page 97 of 236
Constitutional Law 2 Bill of Rights Armando Santiago Jr

however, given a chance to be re-accepted if they change their mind.


Some Jehovahs Witness members appealed to the Secretary of Education but the latter did not answer to
their letter.
students and their parents filed an action alleging that the respondents acted without or in excess of their
jurisdiction and with grave abuse of discretion in ordering their expulsion without prior notice and hearing,
hence, in violation of their right to due process, their right to free public education and their right to freedom of
speech, religion and worship.
Petitioners prayed for the voiding of the order of expulsion or dropping from the rolls issued
and enjoining respondent from barring them from classes; and compelling the respondent and all persons
acting for him to admit and order their(Petitioners) re-admission I their respective schools.
Court lifted the TRO and immediately re-admit the petitioners to their respective classes until further orders.
the Solicitor General filed a consolidated comment to the petitions defending the expulsion orders issued by
the respondents.
Petitioners stressed that while they do not take part in the compulsory flag ceremony, they do not engage in
external acts or behaviour that would offend their countrymen who believe in expressing their love of country
through observance of the flag ceremony.
They quietly stand at attention during the flag ceremony to show their respect for the right of those who
choose to participate in the solemn proceedings. Since they do not engage in disruptive behaviour, there is no
warrant for their expulsion.

ISSUE
Whether or not the expulsion of the members of Jehovahs Witness from the schools violates right receive free
education.

HELD
The expulsion of the members of Jehovahs Witness from the schools where they are enrolled will violate their
right as Philippine citizens, under the 1987 Constitution
to receive free education, for it is the duty of the state to protect and promote the right of all citizens to quality
education, and to make such education accessible to all (Sec. I, Art XIV).
Nevertheless, their right not to participate in the Flag Ceremony does not give them a right to disturb such
patriotic exercises.
If they quietly stand at attention during flag ceremony while their classmates and teachers salute the flag, sing
the national anthem and recite the patriotic pledge,
we do not see how such conduct may possibly disturb the peace, or pose a grave and present danger of a
serious evil to public safety, public morals, public health or any legitimate public interest that the state has a
right and duty to prevent.
It is appropriate to recall the Japanese occupation of our country in 1942-1944 when every Filipino, regardless
of religious persuasion, in fear of the invader, saluted the Japanese flag and bowed before every Japanese
soldier, perhaps if petitioners had lived through that dark period of our history, they would not object now about
saluting the Phil. Flag.
The petitions for certiorari and prohibition are granted and expulsion orders are hereby annulled and set aside.

Same; Same; Same; Two-fold aspect of right to religious profession and worship; Scope. The right to religious
profession and worship has a two-fold aspect, vis., freedom to believe and freedom to act on one's belief. The
first is absolute as long as the belief is confined within the realm of thought. The second is subject to regulation
where the belief is translated into external acts that affect the public welfare" (J. Cruz, Constitutional Law 1991
Ed pp 176-177).

Same; Same; Same; Prior restraint or limitation on the exercise of religious freedom, sole justification thereof,
explained."The sole justification for a prior restraint or limitation on the exercise of religious freedom
(according to the late Chief Justice Claudio Teehankee in his dissenting opinion in German vs. Barangan, 135
SCRA 514, 517) is the existence of a grave and present danger of a character both grave and imminent, of a
serious evil to public safety, public morals, public health or any other legitimate public interest', that the State has

Page 98 of 236
Constitutional Law 2 Bill of Rights Armando Santiago Jr

a right (and duty) to prevent." Absent such a threat to public safety, the expulsion of the petitioners from the
schools is not justified.

Same; Same; Freedom of Speech; Administrative Code; Flag Salute Law; Compulsion to observe flag salute law
on pain of dismissal from one's job or expulsion from school is alien to the conscience of present generation of
Filipinos, being violative of their constitutional rights to free speech and free exercise of religious profession and
worship.Our task here is extremely difficult, for the 30-year-old decision of this Court in Gerona upholding the
flag salute law and approving the expulsion of students who refuse to obey it, is not lightly to be trifled with. It is
somewhat ironic however, that after the Gerona ruling had received legislative cachet by its incorporation in the
Administrative Code of 1987, the present Court believes that the time has come to re-examine it. The idea that
one may be compelled to salute the flag, sing the national anthem, and recite the patriotic pledge, during a flag
ceremony on pain of being dismissed from one's job or of being expelled from school, is alien to the conscience
of the present generation of Filipinos who cut their teeth on the Bill of Rights which guarantees their rights to free
speech and the free exercise of religious profession and worship (Sec. 5, Article III, 1987 Constitution; Article IV,
Section 8, 1973 Constitution; Article III, Section 1[7], 1935 Constitution).

Same; Same; Same; Same; Same; Jehovah's witnesses are accorded exemption to the observance of flag
ceremony in deference to their religious beliefs but said right not to participate in the flag ceremony does not
give them the right to disrupt such patriotic exercises. Exemption may be accorded to the Jehovah's
Witnesses with regard to the observance of the flag ceremony out of respect for their religious beliefs, however
"bizarre" those beliefs may seem to others. Nevertheless, their right not to participate in the flag ceremony does
not give them a right to disrupt such patriotic exercises. Paraphrasing the warning cited by this Court in Non vs.
Dames II, 185 SCRA 523, 535, while the highest regard must be afforded their right to the free exercise of their
religion, "this should not be taken to mean that school authorities are powerless to discipline them" if they should
commit breaches of the peace by actions that offend the sensibilities, both religious and patriotic, of other
persons. If they quietly stand at attention during the flag ceremony while their classmates and teachers salute
the flag, sing the national anthem and recite the patriotic pledge, we do not see how such conduct may possibly
disturb the peace, or pose "a grave and present danger of a serious evil to public safety, public morals, public
health or any other legitimate public interest that the State has a right (and duty) to prevent" (German vs.
Barangan, 135 SCRA 514, 517).

Same; Same; Same; Same; Same; Right to free education; Expulsion from school by reason of one's religious
belief considered a violation of a citizen's right to free education.Moreover, the expulsion of members of
Jehovah's Witnesses from the schools where they are enrolled will violate their right as Philippine citizens, under
the 1987 Constitution, to receive free education, for it is the duty of the State to "protect and promote the right of
all citizens to quality education x x x and to make such education accessible to all" (Sec. 1, Art. XIV).

Same; Same; Same; Same; Same; Same; To force a religious group, through statutory compulsion, to
participate in a ceremony violative of its religious belief is not conducive to love of country or respect for duly
constituted authorities. Expelling or banning the petitioners from Philippine schools will bring about the very
situation that this Court had feared in Gerona. Forcing a small religious group, through the iron hand of the law,
to participate in a ceremony that violates their religious beliefs, will hardly be conducive to love of country or
respect for duly constituted authorities. Ebralinag vs. The Division Superintendent of Schools of Cebu, 219
SCRA 256, G.R. No. 95770, G.R. No. 95887 March 1, 1993

(OTHER DIGEST VERSION) EBRALINAG ET AL VS. DIV.


SUPT. OF SCHOOL OF CEBU
Ponencia:KAPUNAN J.
Topic: Conflict with the freedom of religion to the power of the state
Trigger of facts: Jehovah Member was allegedly deprived with their right to free education
Triger of issue:
Trigger of Ruling:
Verdict:
End point of the case:
Page 99 of 236
Constitutional Law 2 Bill of Rights Armando Santiago Jr

Note: Please read GERONA, ET AL. vs. THE HON. SEC. OF EDUCATION and BALBUNA, ET AL. vs. THE HON. SEC.
OF EDUCATION first before reading this case.

FACTS
1. The facts of these two consolidated cases are the same with the cases of Gerona and Balbuna, students
who are members of Jehovas Witnesses who refuse to salute the flag, sing the national anthem and recite
the patriotic pledge based on their belief that the same are acts of worship or religious devotion which they
cannot conscientiously give to anyone or anything except God.

2. Sometime in 1989, Division Memorandum No. 108 was issued by the Division Superintendent of Cebu.
Because of reports received, teachers and school employees who choose not to participate in the daily flag
ceremony or to obey the flag salute regulation were considered removed from the service after due process.

3. Also, they quoted the rulings that if students refuse to comply, they forfeited their right to attend public
schools. Cebu school officials resorted to a number of ways to persuade the children to obey the
memorandum. They were asked to sign an agreement and conduct confrontation meetings, however, to no
avail.

4. This led to the expulsion of the petitioning students. Appeal to the Sec. of Education was also unsuccessful
as he did not answer the letter. The succeeding Division Superintendent of School also did not recall the
orders and in fact, ordered verbally the expulsion of more students who refused to comply.

5. Hence, this present petition was commenced and a temporary restraining order and writ of preliminary
mandatory injunction was issued commanding the respondents to immediately re-admit the petitioners to
their respective classes until further orders from this Court.

ISSUE
Should the Gerona and Balbuna rulings be reversed?

HELD
Yes. The petition was granted and the students were permanently reinstated.

1. The idea that one may be compelled to salute the flag, sing the national anthem, and recite the patriotic
pledge, during a flag ceremony on pain of being dismissed from ones job or of being expelled from school, is
alien to the conscience of the present generation of Filipinos who cut their teeth on the Bill of Rights which
guarantees their rights to free speech and the free exercise of religious profession and worship.

2. Religious freedom is a fundamental right which is entitled to the highest priority and the amplest protection
among human rights, for it involves the relationship of man to his Creator.

3. The right to religious profession and worship has a two-fold aspect, vis., freedom to believe and freedom to
act on ones belief. The first is absolute as long as the belief is confined within the realm of thought. The
second is subject to regulation where the belief is translated into external acts that affect the public welfare.

4. While Petitioners do not take part in the compulsory flag ceremony, they do not engage in external acts or
behavior that would offend their countrymen who believe in expressing their love of country. They quietly
stand at attention to show their respect for the right of those who choose to participate in the solemn
proceedings. Since they do not engage in disruptive behavior, there is no warrant for their expulsion.

5. The sole justification for a prior restraint or limitation on the exercise of religious freedom is the existence of
a grave and present danger of a character both grave and imminent, of a serious evil to public safety, public

Page 100 of 236


Constitutional Law 2 Bill of Rights Armando Santiago Jr

morals, public health or any other legitimate public interest, that the State has a right and duty to prevent.
Absent such a threat to public safety, the expulsion of the petitioners from the schools is not justified.

6. We are not persuaded that by exempting the Jehovahs Witnesses from saluting the flag, singing the national
anthem and reciting the patriotic pledge, this religious group which admittedly comprises a small portion of
the school population will shake up our part of the globe and suddenly produce a nation untaught and
uninculcated in and unimbued with reverence for the flag, patriotism, love of country and admiration for
national heroes.

7. After all, what the petitioners seek only is exemption from the flag ceremony, not exclusion from the public
schools where they may study the Constitution, the democratic way of life and form of government, and learn
not only the arts, sciences, Philippine history and culture but also receive training for a vocation of profession
and be taught the virtues of patriotism, respect for human rights, appreciation for national heroes, the rights
and duties of citizenship, and moral and spiritual values as part of the curricula.

8. Expelling or banning the petitioners from Philippine schools will bring about the very situation that this Court
had feared in Gerona. Forcing a small religious group, through the iron hand of the law, to participate in a
ceremony that violates their religious beliefs, will hardly be conducive to love of country or respect for dully
constituted authorities.

9. To believe that patriotism will not flourish if patriotic ceremonies are voluntary and spontaneous instead of a
compulsory routine is to make an unflattering estimate of the appeal of our institutions to free minds. When
they are so harmless to others or to the State as those we deal with here, the price is not too great. (West
Virginia vs. Barnette, Justice Jackson)

10. Furthermore, let it be noted that coerced unity and loyalty even to the country assuming that such unity
and loyalty can be attained through coercion is not a goal that is constitutionally obtainable at the expense
of religious liberty. A desirable end cannot be promoted by prohibited means. (Meyer vs. Nebraska)

11. The expulsion of members of Jehovahs Witnesses from the schools where they are enrolled will violate their
right as Philippine citizens, under the 1987 Constitution, to receive free education, for it is the duty of the
State to protect and promote the right of all citizens to quality education and to make such education
accessible to all.

12. However, while the highest regard must be afforded to their right to the free exercise of their religion, this
should not be taken to mean that school authorities are powerless to discipline them if they should commit
breaches of the peace by actions that offend the sensibilities, both religious and patriotic, of other persons.
(German vs. Barangan)

13. Before we close this decision, it is appropriate to recall the Japanese occupation of our country in 1942-1944
when every Filipino, regardless of religious persuasion, in fear of the invader, saluted the Japanese flag and
bowed before every Japanese soldier. Perhaps, if petitioners had lived through that dark period of our history,
they would not quibble now about saluting the Philippine flag. For when liberation came in 1944 and our own
flag was proudly hoisted aloft again, it was a beautiful sight to behold that made our hearts pound with pride
and joy over the newly-regained freedom and sovereignty of our nation.

14. Although the Court upholds in this decision the petitioners right under our Constitution to refuse to salute the
Philippine flag on account of their religious beliefs, we hope, nevertheless, that another foreign invasion of
our country will not be necessary in order for our countrymen to appreciate and cherish the Philippine flag.

CRUZ, J., concurring: In my humble view, Gerona was based on an erroneous assumption. The Court that
promulgated it was apparently laboring under the conviction that the State had the right to determine what was
religious and what was not and to dictate to the individual what he could and could not worship. In pronouncing
that the flag was not a religious image but a symbol of the nation, it was implying that no one had the right to
worship it or as the petitioners insisted not to worship it. This was no different from saying that the cult that

Page 101 of 236


Constitutional Law 2 Bill of Rights Armando Santiago Jr

reveres Rizal as a divinity should not and cannot do so because he is only a civic figure deserving honor but not
veneration.

GERONA VS. SEC OF EDUCATION


Ponencia: MONTEMAYOR
Topic: Origin of the decision of Ebralinag case / EXPELLED FOR FAILURE TO FOLLOW FLAG CEREMONY
Trigger of facts:
Triger of issue:
Trigger of Ruling:
Verdict: Appealed decision was affirmed, Writ issued is ordered dissolved
End point of the case: Beliefs should not be conflict to the laws and constitution, even if its religious violation / SC
required them to salute and give respect as patriotism otherwise may be deprived of using public benefits like the
right to study in a public school.

FACTS
1. When RA 1265 (An Act Making Flag Ceremony Compulsory in All Educational Institutions) took effect, the
Sec. of Education issued Dept. Order No. 8 prescribing the rules and regulations for the proper conduct of the
flag ceremony.

2. The said order mandates that a proper salute must be given, or at least standing still with arms and hands
straight at sides along with the singing of the National Anthem and recital of the pledge. However, petitioners
children attending the Buenavista Community School in Uson, Masbate refused to do so.

3. This was because, as members of Jehovas Witnesses, they believe that the obligation imposed by law of
God is superior to that of laws enacted by the State. This is based on a verse which states:

Thou shalt not make unto thee any graven image, or any likeness of anything that is in heaven above, or that is
in the earth beneath, or that is in the water under the earth; thou shalt not bow down thyself to them, nor serve
them.

They consider that the flag is an image within this command and thus refuse to salute it. Because of this, they
were expelled from the school.

4. The counsel of petitioners wrote to the Sec. of Education that the children be allowed to just remain silent and
stand still with their arms and hands straight at their sides. This was, however, denied along with the childrens
reinstatement.

5. An action was then filed before the CFI with prayer for a writ of preliminary injunction but the complaint was
dismissed. Hence, the present petition with the SC issuing a temporary writ subject to the result of the case.

ISSUE
Should the department order be upheld?

HELD
Yes. The CFI decision was affirmed and the writ of preliminary injunction was dissolved.

1. First, there was no question with the act of saluting since the department order allows that students can just
stand still with their arms and hands straight at their sides. The issue was focused on the singing of the
national anthem and the recital of pledge.

2. The court eventually held that if the exercise of said religious belief clashes with the established institutions of
society and with the law, then the former must yield and give way to the latter. The reasons are:

Page 102 of 236


Constitutional Law 2 Bill of Rights Armando Santiago Jr

a. the flag is not an image nor the flag ceremony a religious rite; the flag is a symbol of the Republic of the
Philippines, an emblem of national sovereignty, unity and cohesion and of freedom and liberty.

b. the wordings of the patriotic pledge or the national anthem does not have anything that is religiously
objectionable as they speak only of love of country, patriotism, liberty and the glory of suffering and dying for
it.

c. the State was merely carrying out its constitutional duty to supervise and regulate educational institutions
and see to it that all schools aim to develop civic conscience and teach the duties of citizenship. (Art. XIV,
section 5 of the Constitution).

d. considering the separation of the State and Church, the flag does not have any religious significance.

e. also, the determination of whether a certain ritual is or is not a religious ceremony must rest with the court;
it cannot be left to a religious group or sector to its follower as there would be confusion and
misunderstanding for there might be as many interpretations and meaning to be given as there are religious
groups or sects or followers.

f. as emphatically stated, if a man lived on an island, alone and all by himself, he would normally have
complete and absolute rights as to the way he lives, his religion, including the manners he practices his
religious beliefs with no laws to obey, no rules and regulations to follow; but since man is gregarious by
nature and instinct and he gravitates toward community life, to receive and enjoy the benefits of society, he
becomes a member of a community or nation; thus, he has to give up rights for the benefit of his fellow
citizens and for the general welfare, just as his fellow men and companions also agree to a limitation of their
rights in his favor.

g. also, exempting the children will disrupt school discipline and demoralize the rest of the school population
which by far constitutes the great majority; other pupils would naturally ask for the same privilege because
they might want to do something else such as play or study; if this exemption is extended, then the flag
ceremony would soon be a thing of the past or perhaps conducted with very few participants, and the time
will come when we would have citizens untaught and uninculcated in and not imbued with reverence for the
flag and love of country, admiration for national heroes, and patriotism a pathetic, even tragic situation,
and all because a small portion of the school population imposed its will, demanded and was granted an
exemption.

3. US jurisprudence made as basis:



a. Reynolds vs. US the law prohibited polygamy which was allowed for Mormons

Can a man excuse his practices to the contrary because of his religious belief? To permit this would be to
make the professed doctrines of religious belief superior to the law of the land, and in effect to permit every
citizen to become a law unto himself.

b. Hamilton vs. University of California the university requires military science and tactics training but the
objectioners believe that war and preparation for war is a violation of their religious belief

it was held untenable. The Court stated that California did not call them. They sought education in the
university and the due process clause secured by law will be violated if they are to be exempted from the
training.

In this case, having elected not to comply with the regulations about the flag salute, they forfeited their right
to attend public schools.

c. Minersville School District vs. Gobitis same facts with present case; the US Supreme Court upheld the
conduct of flag ceremony but after 3 years, it was reversed in West Virginia State Board of Education vs.
Bernette. This was only because in the latter case, the parents are to be prosecuted criminally if their children

Page 103 of 236


Constitutional Law 2 Bill of Rights Armando Santiago Jr

are not in school. It turned out as a dilemma with the authority against individual rights so the Court then
approved the exemption. However, it is not the rulingin the present case.

Mr. Justice Frankfurter dissented in the latter case stating:

The constitutional protection of religious freedom gave religious equality, not civil immunity. Its essence is
freedom from conformity to religious dogma, not freedom from conformity to law because of religious
dogma

Page 104 of 236


Constitutional Law 2 Bill of Rights Armando Santiago Jr

Liberty of Abode and Freedom of


Movement
The liberty of abode and of changing the same within the limits
prescribed by law shall not be impaired except upon lawful order of
court. Neither shall the right to travel, be impaired except in the
interest of national security, public safety, or public health, as may be
provided by law
(Section 6, Art. III)
THE LIMITATIONS ON THE RIGHT TO TRAVEL

1. National security
2. public safety; and
3. public health

RIGHTS GUARANTEED UNDER SECTION 6


1. Freedom to choose and change ones place of abode.
2. Freedom to travel within the country and outside.

RIGHT CURTAILMENT/LIMITATIONS OF RIGHT


1. Liberty of abode
Lawful order of the court and within the limits prescribed by law.

2. Right to travel
May be curtailed even by administrative officers (ex. passport officers) in the interest of national security, public
safety, or public health, as may be provided by law.

NOTE: The right to travel and the liberty of abode are distinct from the right to return to ones country, as shown
by the fact that the Declaration of Human Rights and the Covenant on Human Rights have separate guarantees
for these. Hence, the right to return to ones country is not covered by the specific right to travel and liberty of
abode. (Marcos v.Manglapus, 177 SCRA 668).

FREEDOM OF MOVEMENT: LIBERTY OF ABODE AND OF TRAVEL

THE LIBERTY GUARANTEED BY THIS PROVISION INCLUDES


(1) freedom to choose and change one's place of abode and;
(2) freedom to travel both within the country and outside.

Freedom of movement is not absolute.

Universal Declaration of Human Rights, Art. 13(2): everyone has the right to leave any country, including
ones own, and to return to that country.

Covenant on Civil and Political Rights, Art. 12(4): no one shall be deprived of the right to enter his own
country.

Page 105 of 236


Constitutional Law 2 Bill of Rights Armando Santiago Jr

The right to return to one's country is not among the rights specifically guaranteed in the Bill of Rights, which
treats only of the liberty of abode and the right to travel, but it is the Court's well considered view that the right to
return may be considered as a generally accepted principle of international law, and under our Constitution, is
part of the law of the land. However, it is distinct and separate from the right to travel and enjoys a different
protection under the Intl. Covenant of Civil and Political Rights, i.e. against being arbitrarily deprived thereof.
[Marcos v. Manglapus, 177 SCRA 668 & 178 SCRA 760 (1989)]

Dissenting opinion of Justice Gutierrez in the Marcos case: Sec. 6 of the Bill of Rights states categorically
that the liberty of abode and of changing the same within limits prescribed by law may be impaired only upon
lawful order of the court. Not by an executive officer. Not even by the President. Sec. 6 further provides that the
right to travel, and this obviously includes the right to travel out of or back into the Philippines, cannot be
impaired except in the interest of national security, public safety, or public health, as may be provided by law.
The right to change abode and travel within the Philippines are not absolute rights. It can be regulated by lawful
order. The order of the CA releasing petitioner on bail constitutes such lawful order as contemplated by Section
6. The condition imposed by the CA is simply consistent with the nature and function of a bail bond, which is to
ensure that petitioner will make himself available at all times whenever the Court requires his presence. (Yap, Jr.
v. CA, 2001)

The liberty of abode may be impaired only upon lawful order of the court, and the court is to be guided by the
limits prescribed by law on the liberty itself.

The liberty of travel may be impaired even without court order, but the appropriate executive officer is not armed
with arbitrary discretion to impose limitations. He can impose limits only on the basis of national security, public
safety, or public health and as may be provided by law.

The right to travel should not be construed as delimiting the inherent power of the courts to use all means
necessary to carry their orders into effect in criminal cases pending before them. When by law jurisdiction is
conferred on a court or judicial officer, all auxiliary writs, process and other means necessary to carry it into
effect may be employed by such court or officer.

ALIENS AND RIGHT TO ENTRY


While the right to travel of citizens covers both exit from and entry into the country, aliens cannot claim the same
right.

REFERENCE AND CITATION


In Caunca vs. Salazar, 82 Phil 851, a maid has the right to transfer to another residence even if she had not yet
paid the amount advanced for her transportation from the province by an employment agency which was then
effectively detaining her.

Villavicencio vs. Lukban, 39 Phil 778, the Mayor of Manila was not sustained by the SC when he deported
some 170 women of ill-repute to Davao, for the admittedly commendable purpose of ridding the city for serious
moral and health problems. These women are nevertheless not chattels but Philippine citizens protected by the
same constitutional guarantees as are other citizensto change their domicile from Manila to another locality.

Rubi vs. Board of Mindoro, 39 Phil 660, the respondents were justified in requiring the members of certain
non-Christian tribes to reside in a reservation, for their better education, advancement and protection. The
measure was held to be a legitimate exercise of police power.

Lorenzo vs. Director of Health, 50 Phil 595, health officers may restrict access to contaminated areas and also
quarantine those already exposed to the disease sought to be contained.

Zemel vs. Rusk, 381 US 1, the Secretary of State may regulate or even prohibit the travel of citizens to hostile
countries to prevent possible international misunderstanding and conflict.

Page 106 of 236


Constitutional Law 2 Bill of Rights Armando Santiago Jr

Section 26 of HAS of 2007cases where evidence of guilt is not strong, and the person charged with the crime
of terrorism as therein defined is entitled to bail and if granted the same, the court, upon application by the
prosecutor, shall limit the right to travel of the accused to within the municipality or city where he resides or
where the case is pending, in the interest of national security and public safety.

CASES

VILLAVICENCIO V LUKBAN
Ponencia: Malcom J.
Topic: Right to Abode
Trigger of facts: Ill reputate was transferred to davao
Triger of issue: they were deprived of their right to abode
Trigger of Ruling: the court ruled against the mayor for he was no jurisdiction and right to choose for others where
to abode.
Verdict:
End point of the case: Mayor has no power to transport and exercise police power because police power is vested
in legislature

FACTS
The Mayor of the city of Manila, Justo Lukban, for the best of all reasons,
to exterminate vice, ordered the segregated district for women of ill repute, which had been permitted for a
number of years in the city of Manila, closed.
the women were kept confined to their houses in the district by the police.
The women (who were prostitutes) were given no opportunity to collect their belongings, and apparently were
under the impression that they were being taken to a police station for an investigation. They had no
knowledge that they were destined for a life in Mindanao.
They had not been asked if they wished to depart from that region and had neither directly nor indirectly given
their consent to the deportation. The involuntary guests were received on board the steamers by a
representative of the Bureau of Labor and a detachment of Constabulary soldiers. The two steamers with their
unwilling passengers sailed for Davao during the night of October
Just about the time the Corregidor and the Negros were putting in to Davao, the attorney for the relatives and
friends of a considerable number of the deportees presented an application for habeas corpus to a member of
the Supreme Court.

Writ of Habeas Corpus - An order issued by a court to a person detaining another, to produce the body of the
prisoner at a certain time and place, and to show sufficient cause for holding in custody the individual. - Habeas
Corpus: to bring the body

City fiscal (for the respondent): Admitted certain facts relative to sequestration and deportation, and prayed that
the writ should not be granted because the petitioners were not proper parties, because the action should have
been begun in the Court of First Instance for Davao, Department of Mindanao and Sulu, because the
respondents did not have any of the women under their custody or control, and because their jurisdiction did not
extend beyond the boundaries of the city of Manila.

According to an exhibit attached to the answer of the fiscal, the 170 women were destined to be laborers, at
good salaries, on the haciendas of Yigo and Governor Sales.

SC first order: The court awarded the writ that directed Justo Lukban, Mayor of the city of Manila, Anton
Hohmann, chief of police of the city of Manila, Francisco Sales, governor of the province of Davao, and Feliciano
Yigo, an hacendero of Davao, to bring before the court the persons therein named, alleged to be deprived of
their liberty.
Page 107 of 236
Constitutional Law 2 Bill of Rights Armando Santiago Jr

None of the persons in whose behalf the writ was issued were produced in court by the respondents.

Fiscal: offered certain affidavits showing that the women were contained with their life in Mindanao and did not
wish to return to Manila.

Sales: answered alleging that it was not possible to fulfill the order of the Supreme Court because the women
had never been under his control, because they were at liberty in the Province of Davao, and because they had
married or signed contracts as laborers.

Yigo: answered alleging that he did not have any of the women under his control and that therefore it was
impossible for him to obey the mandate.

SC second order: Since respondents did had not complied nor explained their failure to do so the 1st order, court
directed that those of the women not in Manila be brought before the court by respondents unless:

1.) women should, in written statements voluntarily made before the judge of first instance of Davao or the clerk
of that court, renounce the right, or
2.) the respondents should demonstrate some other legal motives that made compliance impossible.

ISSUE
WON judiciary permit a government of the men instead of a government of laws to be set up in the Philippine
Islands.

By authority of what law did the Mayor and the Chief of Police presume to act in deporting by duress these
persons from Manila to another distant locality within the Philippine Islands?

HELD
1.) Government of laws.

"No freeman shall be taken, or imprisoned, or be disseized of his freehold, or liberties, or free customs, or be
outlawed, or exiled, or any other wise destroyed; nor will we pass upon him nor condemn him, but by lawful
judgment of his peers or by the law of the land. We will sell to no man; we will not deny or defer to any man
either justice or right."

No official, no matter how high, is above the law.

"The law," said Justice Miller, delivering the opinion of the Supreme Court of the United States, "is the only
supreme power in our system of government, and every man who by accepting office participates in its functions
is only the more strongly bound to submit to that supremacy, and to observe the limitations which it imposes
upon the exercise of the authority which it gives."

2.) There is no law or ordinance permitting/authorizing such Mayor and Chief of police in deporting these
women.

These women despite their being in a sense lepers of society are nevertheless not chattels but Philippine
citizens protected by the same constitutional guaranties as are other citizens to change their domicile from
Manila to another locality. On the contrary, Philippine penal law specifically punishes any public officer who, not
being expressly authorized by law or regulation, compels any person to change his residence.
Mayor Lukban was primarily responsible for the unlawful deportation, who ordered the police to accomplish the
same, who made arrangements for the steamers and the constabulary, who conducted the negotiations with the
Bureau of Labor, and who later, as the head of the city government, had it within his power to facilitate the return
of the unfortunate women to Manila.

Page 108 of 236


Constitutional Law 2 Bill of Rights Armando Santiago Jr

His intention to suppress the social evil was commendable. His methods were unlawful. His regard for the writ of
habeas corpus issued by the court was only tardily and reluctantly acknowledged.

The respondents Hohmann, Rodriguez, Ordax, Joaquin, Yigo, and Diaz are found not to be in contempt of
court. Respondent Lukban is found in contempt of court and shall pay into the office of the clerk of the Supreme
Court within five days the sum of one hundred pesos (P100).

SYLLABI
1. CONSTITUTIONAL LAW; RlGHT OF DOMICILE; LlBERTY; HABEAS CORpus; CONTEMPT.One hundred
and seventy women, who had lived in the segregated district for women of ill repute in the city of Manila, were by
orders of the Mayor of the city of Manila and the chief of police of that city isolated from society and then at night,
without their consent and without any opportunity to consult with friends or to defend their rights, were forcibly
hustled on board steamers for transportation to regions unknown. No law, order, or regulation authorized the
Mayor of the city of Manila or the chief of the police of that city to force citizens of the Philippine Islands to
change their domicile from Manila to another locality. Held: That the writ of habeas corpus was properly granted,
and that the Mayor of the city of Manila who was primarily responsible for the deportation, is in contempt of court
for his failure to comply with the order of the court.

2.ID.; ID.; ID.; ID.The remedies of the unhappy victims of official oppression are three: (1) Civil action; (2)
criminal action, and (3) habeas corpus. A civil action was never intended effectively and promptly to meet a
situation in which there is restraint of liberty. That the act may be a crime and that the person may be proceeded
against is also no bar to the institution of habeas corpus proceedings. Habeas corpus is the proper remedy.

3.ID.; ID.; ID.; ID.These women, despite their being in a sense lepers of society, are nevertheless not chattles,
but Philippine citizens protected by the same constitutional guaranties as are other citizens.

4.ID.; ID.The privilege of domicile is a principle often protected by constitutions and deeply imbedded in
American jurisprudence.

5.HABEAS CORPUS; NATURE. The writ of habeas corpus was devised and exists as a speedy and effectual
remedy to relieve persons from unlawful restraint, and as the best and only sufficient defense of personal
freedom.

6.ID.; PARTIES. Where it is impossible for a party to sign an application for the writ of habeas corpus, it is
proper for the writ to be submitted by some person in his behalf.

7.ID.; JURISDICTION. It is a general rule of good practice that, to avoid unnecessary expense and
inconvenience,
petitions for habeas corpus should be presented to the nearest judge of the Court of First Instance.

8. ID.; ID.The writ of habeas corpus may be granted by the Supreme Court or any judge thereof enforceable
anywhere in the Philippine Islands.

9.ID.; ID.Whether the writ shall be made returnable before the Supreme Court or before an inferior court rests
in the discretion of the Supreme Court and is dependent on the particular circumstances.

10.ID.; RESTRAINT OF LIBERTY. A prime specification of an application for a writ of habeas corpus is
restraint of liberty. The essential objects and purpose of the writ of habeas corpus is to inquire into all manner of
involuntary restraint as distinguished from voluntary, and to relieve a person therefrom if such restraint is illegal.
Any restraint which will preclude freedom of action is sufficient.

11.ID.; ID.The forcible taking of these women from Manila by officials of that city, who handed them over to
other parties, who deposited them in a distant region, deprived these women of freedom of locomotion just as

Page 109 of 236


Constitutional Law 2 Bill of Rights Armando Santiago Jr

effectively as if they had been imprisoned. The restraint of liberty which began in Manila continued until the
aggrieved parties were returned to Manila and released or until they freely and truly waived this right.

12.ID.; ID.The true principle should be that if the respondent is within the jurisdiction of the court and has it in
his power to obey the order of the court, and thus to undo the wrong that he has inflicted, he should be
compelled to do so.

13.ID.; ID.Even if the party to whom the writ is addressed has illegally parted with the custody of a person
before the application for the writ, is no reason why the writ should not issue.

14.ID.; ID.The place of confinement is not important to the relief if the guilty party is within the reach of process
so that by the power of the court he can be compelled to release his grasp.

15.ID.; COMPLIANCE WITH WRIT. For respondents to fulfill the order of the court granting the writ of habeas
corpus, three courses were open: (1) They could have produced the bodies of the persons according to the
command of the writ; (2) they could have shown by affidavit that on account of sickness or infirmity these,
persons could not safely be brought before the Court; or (3) they could have. presented affidavits to show that
the parties in question or their attorney waived the right to be present. (Code of Criminal Procedure, sec. 87.)

16.CONTEMPT OF COURT. The power to punish for contempt of court should be exercised on the
preservative and not on the vindictive principle. Only occasionally should a court invoke its inherent power in
order to retain that respect without which the administration of justice must falter or fail.

17. ID.When one is commanded to produce a certain person and does not do so, and does not offer a valid
excuse, a court must, to vindicate its authority, adjudge the respondent to be guilty of contempt, and must order
him either imprisoned or fined.

18.ID.An officer's failure to produce the body of a person in obedience to a writ -of habeas corpus, when he
has power to do so, is contempt committed in the face of the court.

19.GOVERNMENT OF THE PHILIPPINE ISLANDS; A GOVERNMENT OF LAWS. The Government of the


Philippine Islands is a. government of laws. The court will assist in retaining it as a government of laws and not
of men.

20.ID, ID.No official, however high, is above the law.

21.ID.; ID.The courts are the forum which functionate to safeguard individual liberty and to punish official
transgressors. Villavicencio vs. Lukban., 39 Phil., 778, No. 14639 March 25, 1919

MARCOS,PETITIONERVS.MANGLAPUS,RESPONDENT
(PART 2)
Ponencia: CORTES J.
Topic: Right to Domicile
Trigger of facts: Marcos, wanted to go back to the country after he was deposed and exiled to the country and cory
did not allow him to go back and die in the Philippines for there was a treat against the country.
Triger of issue: marcos was deprived of his right to travel back to the Philippines
Trigger of Ruling: the court ruled disallowing the former president to go back to the philippines for there was a clear
and present danger and applying
Verdict:
End point of the case:

FACTS
Page 110 of 236
Constitutional Law 2 Bill of Rights Armando Santiago Jr

Former President Ferdinand E. Marcos was deposed from the presidency via the non-violent people power
revolution and was forced into exile. Marcos, in his deathbed, has signified his wish to return to the Philippines to
die. But President Corazon Aquino, considering the dire consequences to the nation of his return at a time when
the stability of government is threatened from various directions and the economy is just beginning to rise and
move forward, has stood firmly on the decision to bar the return of Marcos and his family.

Aquino barred Marcos from returning due to possible threats & following supervening events:

failed Manila Hotel coup in 1986 led by Marcos leaders


channel 7 taken over by rebels & loyalists
plan of Marcoses to return w/ mercenaries aboard a chartered plane of a Lebanese arms dealer. This is to
prove that they can stir trouble from afar
Honasans failed coup
Communist insurgency movements
secessionist movements in Mindanao
devastated economy because of accumulated foreign debt
plunder of nation by Marcos & cronies

Marcos filed for a petition of mandamus and prohibition to order the respondents to issue them their travel
documents and prevent the implementation of President Aquinos decision to bar Marcos from returning in the
Philippines. Petitioner questions Aquinos power to bar his return in the country. He also questioned the claim of
the President that the decision was made in the interest of national security, public safety and health. Petitioner
also claimed that the President acted outside her jurisdiction.

According to theMarcoses, such act deprives them of their right to life, liberty, property without due process and
equal protection of the laws. They also said that it deprives them of their right to travel which according to
Section 6, Article 3 of the constitution, may only be impaired by a court order.

ISSUE
1. Whether or not, in the exercise of the powers granted by the Constitution, the President may prohibit the
Marcoses from returning to the Philippines.
2. Whether or not the President acted arbitrarily or with grave abuse of discretion amounting to lack or excess of
jurisdiction when she determined that the return of the Marcoses to the Philippines poses a serious threat to
national interest and welfare and decided to bar their return.

HELD
Noto both issues. Petition dismissed.

RATIO
Separation of power dictates that each department has exclusive powers. According to Section 1, Article VII of
the 1987 Philippine Constitution, the executive power shall be vested in the President of the Philippines.
However, it does not define what is meant by executive power although in the same article it touches on
exercise of certain powers by the President, i.e., the power of control over all executive departments, bureaus
and offices, the power to execute the laws, the appointing power to grant reprieves, commutations and
pardons (art VII secfs. 14-23). Although the constitution outlines tasks of the president, this list is not defined &
exclusive. She has residual & discretionary powers not stated in the Constitution which include the power to
protect the general welfare of the people. She is obliged to protect the people, promote their welfare & advance
national interest. (Art. II, Sec. 4-5 of the Constitution). Residual powers, according to Theodore Roosevelt,
dictate that the President can do anything which is not forbidden in the Constitution (Corwin, supra at 153),
inevitable to vest discretionary powers on the President (Hyman, American President) and that the president has
to maintain peace during times of emergency but also on the day-to-day operation of the State.

The rights Marcoses are invoking are not absolute. Theyre flexible depending on the circumstances. The
request of theMarcosesto be allowed to return to the Philippines cannot be considered in the light solely of the
Page 111 of 236
Constitutional Law 2 Bill of Rights Armando Santiago Jr

constitutional provisions guaranteeing liberty of abode and the right to travel, subject to certain exceptions, or of
case law which clearly never contemplated situations even remotely similar to the present one. It must be treated
as a matter that is appropriately addressed to those residual unstated powers of the President which are implicit
in and correlative to the paramount duty residing in that office to safeguard and protect general welfare. In that
context, such request or demand should submit to the exercise of a broader discretion on the part of the
President to determine whether it must be granted or denied.

For issue number 2, the question for the court to determine is whether or not there exist factual basis for the
President to conclude that it was in the national interest to bar the return of the Marcoses in the Philippines. It is
proven that there are factual bases in her decision. The supervening events that happened before her decision
are factual. The President must take preemptive measures for the self-preservation of the country & protection of
the people. She has to uphold the Constitution.

Fernan, Concurring
The presidents power is not fixed. Limits would depend on the imperatives of events and not on abstract
theories of law. We are undergoing a critical time and the current problem can only be answerable by the
President.
Threat is real. Return of the Marcoses would pose a clear & present danger. Thus, its the executives
responsibility & obligation to prevent a grave & serious threat to its safety from arising.
We cant sacrifice public peace, order, safety & our political & economic gains to give in to Marcos wish to die in
the country. Compassion must give way to the other state interests.

Cruz, Dissenting
As a citizen of this country, it is Marcos right to return, live & die in his own country. It is a right guaranteed by
the Consti to all individuals, whether patriot, homesick, prodigal, tyrant, etc.
Military representatives failed to show that Marcos return would pose a threat to national security. Fears were
mere conjectures.

Residual powers but the executives powers were outlined to limit her powers & not expand.

Paras, Dissenting
AFP has failed to prove danger which would allow State to impair Marcos right to return to the Philippines. .
Family can be put under house arrest & in the event that one dies, he/she should be buried w/in 10 days.
Untenable that without a legislation, right to travel is absolute & state is powerless to restrict it. Its w/in police
power of the state to restrict this right if national security, public safety/health demands that such be restricted. It
cant be absolute & unlimited all the time. It cant be arbitrary & irrational.

No proof that Marcos return would endanger national security or public safety. Fears are speculative & military
admits that its under control. Filipinos would know how to handle Marcos return.

Padilla, Dissenting

Sarmiento, Dissenting
Presidents determination that Marcos return would threaten national security should be agreed upon by the
court. Such threat must be clear & present.

MARCOS,PETITIONERVS.MANGLAPUS (PART 2)
Ponencia: CORTES J.
Topic:
Trigger of facts:
Triger of issue:
Trigger of Ruling:
Verdict:
End point of the case:

Page 112 of 236


Constitutional Law 2 Bill of Rights Armando Santiago Jr

FACTS
In its decision dated September 15, 1989, the Court by a vote of eight to seven, dismissed the petition, after
finding that the President did not act arbitrarily or with grave abuse of discretion in determining that the return of
former President Marcos and his family pose a threat to national interest and welfare and in prohibiting their
return to the Philippines. On September 28, 1989, Marcos died in Honolulu, Hawaii.

President Corazon Aquino issued a statement saying that in the interest of the safety of those who will take the
death of Marcos in widely and passionately conflicting ways, and for the tranquility and order of the state and
society, she did not allow the remains of Marcos to be brought back in the Philippines.

A motion for Reconsideration was filed by the petitioners raising the following arguments:

Barring their return would deny them their inherent right as citizens to return to their country of birth and all other
rights guaranteed by the Constitution to all Filipinos.

The President has no power to bar a Filipino from his own country; if she has, she had exercised it arbitrarily.
There is no basis for barring the return of the family of former President Marcos.

ISSUE
Whether or not the motion for reconsideration that the Marcoses be allowed to return in the Philippines be
granted.

HELD
No. TheMarcoseswere not allowed to return. Motion for Reconsideration denied because of lack of merit.

RATIO
Petitioners failed to show any compelling reason to warrant reconsideration.
Factual scenario during the time Court rendered its decision has not changed. The threats to the government, to
which the return of the Marcoses has been viewed to provide a catalytic effect, have not been shown to have
ceased. Imelda Marcos also called President Aquino illegal claiming that it is Ferdinand Marcos who is the legal
president.

President has unstated residual powers implied from grant of executive power. Enumerations are merely for
specifying principal articles implied in the definition; leaving the rest to flow from general grant that power,
interpreted in conformity with other parts of the Constitution (Hamilton). Executive unlike Congress can exercise
power from sources not enumerates so long as not forbidden by constitutional text (Myers vs. US). This does not
amount to dictatorship. Amendment No. 6 expressly granted Marcos power of legislation whereas 1987
Constitution granted Aquino with implied powers.

It is within Aquinos power to protect & promote interest & welfare of the people. She bound to comply w/ that
duty and there is no proof that she acted arbitrarily

SYLLABI
Same; Same; The constitutional guarantees invoked by petitioners are not absolute and inflexible, they admit of
limits and must be adjusted to the requirements of equally important public interests. The resolution of the
problem is made difficult because the persons who seek to return to the country are the deposed dictator and his
family at whose door the travails of the country are laid and from whom billions of dollars believed to be ill-gotten
wealth are sought to be recovered. The constitutional guarantees they invoke are neither absolute nor inflexible.
For the exercise of even the preferred freedoms of speech and of expression, although couched in absolute
terms, admits of limits and must be adjusted to the requirements of equally important public interests [Zaldivar v.

Page 113 of 236


Constitutional Law 2 Bill of Rights Armando Santiago Jr

Sandiganbayan, G.R. Nos. 79690-707, October 7, 1988].

Same; Separation of Powers; Executive Powers; The grant of executive power means a grant of all executive
powers. The 1987 Constitution has fully restored the separation of powers of the three great branches of
government. To recall the words of Justice Laurel in Angara v. Electoral Commission [63 Phil. 139 (1936)], the
Constitution has blocked out with deft strokes and in bold lines, allotment of power to the executive, the
legislative and the judicial departments of the government. [At 157]. Thus, the 1987 constitution explicitly
provides that [t]he legislative power shall be vested in the Congress of the Philippines [Art. VI, Sec. 1], [t]he
executive power shall be vested in the President of the Philippines [Art. VII, Sec. 1], and [t]he judicial power
shall be vested in one Supreme Court and in such lower courts as may be established by law [Art. VIII, Sec. 1].
These provisions not only establish a separation of powers by actual division [Angara v. Electoral Commission,
supra] but also confer plenary legislative, executive and judicial powers subject only to limitations provided in the
Constitution. For as the Supreme Court in Ocampo v. Cabangis [15 Phil. 626, (1910)] pointed out a grant of the
legislative power means a grant of all legislative power; and a grant of the judicial power means a grant of all the
judicial power which may be exercised under the government. [At 631-632.] If this can be said of the legislative
power which is exercised by two chambers with a combined membership of more than two hundred members
and of the judicial power which is vested in a hierarchy of courts, it can equally be said of the executive power
which is vested in one officialthe President.

Same; Same; Same; The President; The powers granted to the President are not limited to those powers
specifically enumerated in the Constitution. It would not be accurate, however, to state that executive power
is the power to enforce the laws, for the President is head of state as well as head of government and whatever
powers in here in such positions pertain to the office unless the Constitution itself withholds it. Furthermore, the
Constitution itself provides that the execution of the laws is only one of the powers of the President. It also grants
the President other powers that do not involve the execution of any provision of law, e.g., his power over the
countrys foreign relations. On these premises, we hold the view that although the 1987 Constitution imposes
limitations on the exercise of specific powers of the President, it maintains intact what is traditionally considered
as within the scope of executive power. Corollary, the powers of the President cannot be said to be limited only
to the specific powers enumerated in the Constitution. In other words, executive power is more than the sum of
specific powers so enumerated.

Same; Same; Same; Same; Commander-In-Chief Powers: The President can exercise Commander-In-Chief
powers in order to keep the peace and maintain public order and security even in the absence of an emergency.
More particularly, this case calls for the exercise of the Presidents powers as protector of the peace. [Rossiter,
The American Presidency.] The power of the President to keep the peace is not limited merely to exercising the
commander-in-chief powers in times of emergency or to leading the State against external and internal threats to
its existence. The President is not only clothed with extraordinary powers in times of emergency, but is also
tasked with attending to the day-to-day problems of maintaining peace and order and ensuring domestic
tranquility in times when no foreign foe appears on the horizon. Wide discretion, within the bounds of law, in
fulfilling presidential duties in times of peace is not in any way diminished by the relative want of an emergency
specified in the commander-in-chief provision. For in making the President commander-in-chief the enumeration
of powers that follow cannot be said to exclude the Presidents exercising as Commander-in-Chief powers short
of the calling the armed forces, or suspending the privilege of the writ of habeas corpus or declaring martial law,
in order to keep the peace, and maintain public order and security.

Same; Same; Same; Same; The President has the power under the Constitution to bar the Marcoses from
returning to our country.That the President has the power under the Constitution to bar the Marcoses from
returning has been recognized by members of the Legislature, and is manifested by the Resolution proposed in
the House of Representatives and signed by 103 of its members urging the President to allow Mr. Marcos to
return to the Philippines as a genuine unselfish gesture for true national reconciliation and as irrevocable proof
of our collective adherence to uncompromising respect for human rights under the Constitution and our
laws. [House Resolution No. 1342, Rollo, p. 321.] The Resolution does not question the Presidents power to
bar the Marcoses from returning to the Philippines, rather, it appeals to the Presidents sense of compassion to
allow a man to come home to die in his country. What we are saying in effect is that the request or demand of
the Marcoses to be allowed to return to the Philippines cannot be considered in the light solely of the
constitutional provisions guaranteeing liberty of abode and the right to travel, subject to certain exceptions, or of

Page 114 of 236


Constitutional Law 2 Bill of Rights Armando Santiago Jr

case law which clearly never contemplated situations even remotely similar to the present one. It must be treated
as a matter that is appropriately addressed to those residual unstated powers of the President which are implicit
in and correlative to the paramount duty residing in that office to safeguard and protect general welfare. In that
context, such request or demand should submit to the exercise of a broader discretion on the part of the
President to determine whether it must be granted or denied.

Same; Same; Same; Power of Judicial Review; Political Question Doctrine; The present Constitution limits resort
to the political question doctrine and broadens the scope of judicial inquiry. Under the Constitution, judicial
power includes the duty to determine whether or not there has been a grave abuse of discretion amounting to
lack or excess of jurisdiction on the part of any branch or instrumentality of the Government. [Art. VIII, Sec. 1.]
Given this wording, we cannot agree with the Solicitor General that the issue constitutes a political question
which is beyond the jurisdiction of the Court to decide. The present Constitution limits resort to the political
question doctrine and broadens the scope of judicial inquiry into areas which the Court, under previous
constitutions, would have normally left to the political departments to decide. But nonetheless there remain
issues beyond the Courts jurisdiction the determination of which is exclusively for the President, for Congress or
for the people themselves through a plebiscite or referendum. We cannot, for example, question the Presidents
recognition of a foreign government, no matter how premature or improvident such action may appear. We
cannot set aside a presidential pardon though it may appear to us that the beneficiary is totally undeserving of
the grant. Nor can we amend the Constitution under the guise of resolving a dispute brought before us because
the power is reserved to the people.

Same; Same; Same; Same; In the exercise of the power of judicial review, the function of the court is merely to
check, not to supplant the Executive. There is nothing in the case before us that precludes our determination
thereof on the political question doctrine. The deliberations of the Constitutional Commission cited by petitioners
show that the framers intended to widen the scope of judicial review but they did not intend courts of justice to
settle all actual controversies before them. When political questions are involved, the Constitution limits the
determination to whether or not there has been a grave abuse of discretion amounting to lack or excess of
jurisdiction on the part of the official whose action is being questioned. If grave abuse is not established, the
Court will not substitute its judgment for that of the official concerned and decide a matter which by its nature or
by law is for the latter alone to decide. In this light, it would appear clear that the second paragraph of Article VIII,
Section 1 of the Constitution, defining judicial power, which specifically empowers the courts to determine
whether or not there has been a grave abuse of discretion on the part of any branch or instrumentality of the
government, incorporates in the fundamental law the ruling in Lansang v. Garcia [G.R. No. L-33964, December
11, 1971, 42 SCRA 448] that: Article VII of the [1935] Constitution vests in the Executive the power to suspend
the privilege of the writ of habeas corpus under specified conditions. Pursuant to the principle of separation of
powers underlying our system of government, the Executive is supreme within his own sphere. However, the
separation of powers, under the Constitution, is not absolute. What is more, it goes hand in hand with the system
of checks and balances, under which the Executive is supreme, as regards the suspension of the privilege, but
only if and when he acts within the sphere allotted to him by the Basic Law, and the authority to determine
whether or not he has so acted is vested in the Judicial Department, which, in this respect, is, in turn,
constitutionally supreme. In the exercise of such authority, the function of the Court is merely to checknot to
supplantthe Executive, or to ascertain merely whether he has gone beyond the constitutional limits of his
jurisdiction, not to exercise the power vested in him or to determine the wisdom of his act. . . . [At 479-480.]

Same; Same; Same; Same; The President did not act arbitrarily, capriciously and whimsically in determining that
the return of the Marcoses poses a serious threat to national interest and welfare, and in prohibiting their return.
We find that from the pleadings filed by the parties, from their oral arguments, and the facts revealed during
the briefing in chambers by the Chief of Staff of the Armed Forces of the Philippines and the National Security
Adviser, wherein petitioners and respondents were represented, there exist factual basis for the Presidents
decision. The Court cannot close its eyes to present realities and pretend that the country is not besieged from
within by a wellorganized communist insurgency, a separatist movement in Mindanao, rightist conspiracies to
grab power, urban terrorism, the murder with impunity of military men, police officers and civilian officials, to
mention only a few. The documented history of the efforts of the Marcoses and their followers to destabilize the
country, as earlier narrated in thisponenciabolsters the conclusion that the return of the Marcoses at this time
would only exacerbate and intensify the violence directed against the State and instigate more chaos. As
divergent and discordant forces, the enemies of the State may be contained. The military establishment has

Page 115 of 236


Constitutional Law 2 Bill of Rights Armando Santiago Jr

given assurances that it could handle the threats posed by particular groups. But it is the catalytic effect of the
return of the Marcoses that may prove to be the proverbial final straw that would break the camels back. With
these before her, the President cannot be said to have acted arbitrarily and capriciously and whimsically in
determining that the return of the Marcoses poses a serious threat to the national interest and welfare and in
prohibiting their return. Marcos vs. Manglapus, 177 SCRA 668, G.R. No. 88211 September 15, 1989

DEFENSOR-SANTIAGO VS. VASQUEZ, 217 SCRA 633 (1993)


Ponencia: REGALADO J.
Topic: Liberty of Abode and Freedom of Movement (Right to travel)
Trigger of facts: Miriam was charged of graft and corruption and order of arrest was lifted to serve and Miriam post a
cash bond thereafter cancelled posted bond and did not recognized sandiganbayan but apparently, she filed a
motion which may deem recognition of jurisdiction and filed in the supreme court a TRO for sandinganbayan action,
hence it was denied by SC. Miriam was about to fly in the U.S to purse further studies in Harvard university in US.
And she was not allowed to travel and alleged that she her freedom to travel was deprived.
Triger of issue: WON Miriams right was impaired.
Trigger of Ruling: SC ruled in the negative. Such right is not an absolute right. Provided under art. 3 section 6 which
state the exception of lawful order of the court. Courts have jurisdiction over the case for These inherent powers are
such powers as are necessary for the ordinary and efficient exercise of jurisdiction; or essential to the existence,
dignity and functions of the courts, as well as to the due administration of justice; or are directly appropriate,
convenient and suitable to the execution of their granted powers; and include the power to maintain the court's
jurisdiction and render it effective in behalf of the litigants.
Verdict: WHEREFORE, with respect to and acting on the motion now before us for resolution, the same is hereby
DENIED for lack of merit.
End point of the case: Right to travel is not an absolute right. In addition, with the criminal cases person to be
prosecuted are not allowed to go out of the country.

Court possess cartain inherent power which may be said to implied from a general grant of jurisdiction

1. these inherent powers are such powers as are necessary for the ordinary and efficient of jurisdiction
2. or are directly appropriate, convinient and suitable to the execution of their granted powers
3. and conclude the power to maintain the courts jurisdiction and render it effective in behalf of the litigants

FACTS
Miriam Defensor-Santiago was charged with violation of Section 3(e), Republic Act No. 3019, otherwise known
as the Anti-Graft and Corrupt Practices Act before the Sandiganbayan.
An order of arrest was issued against her with bail for her release fixed at P15,000.00.
She filed an "Urgent Ex-parte Motion for Acceptance of Cash Bail Bond".
The Sandiganbayan issued a resolution authorizing the Santiago to post cash bond which the later filed in the
amount of P15,000.00.
Her arraignment was set, but she asked for the cancellation of her bail bond and that she
be allowed provisional release on recognizance.
The Sandiganbayan postponed the arraignment.
Meanwhile, it issued a hold departure order against Santiago by reason of the announcement she made,
which was widely publicized in both print and broadcast media, that she would be leaving for the U.S. to
accept a fellowship at Harvard University.
She directly filed a "Motion to Restrain the Sandiganbayan from Enforcing its Hold Departure Order with
Prayer for the Issuance of a Temporary Restraining Order and/or Preliminary Injunction" with the SC.
She argued that the Sandiganbayan acted without or in excess of jurisdiction and with grave abuse of
discretion in issuing the hold departure order considering that it had not acquired jurisdiction over her person
as she has neither been arrested nor has she voluntarily surrendered.
The hold departure order was also issued sua sponte without notice and hearing.
She likewise argued that the hold departure order violates her right to due process, right to travel and freedom
of speech.

ISSUES
Page 116 of 236
Constitutional Law 2 Bill of Rights Armando Santiago Jr

1. Has the Sandiganbayan acquired jurisdiction over the person of Santiago?


2. Did the Sandiganbayan err when it issued the hold departure order without any motion from the prosecution
and without notice and hearing?
3. Has Santiago's right to travel been impaired?
4. How the court acquires jurisdiction over the person of the accused.

HELD
It has been held that where after the filing of the complaint or information a warrant for the arrest of the accused
is issued by the trial court and the accused either voluntarily submitted himself to the court or was duly arrested,
the court thereby acquires jurisdiction over the person of the accused. The voluntary appearance of the accused,
whereby the court acquires jurisdiction over his person, is accomplished either by his pleading to the merits
(such as by filing a motion to quash or other pleadings requiring the exercise of the court's jurisdiction thereover,
appearing for arraignment, entering trial) or by filing bail. On the matter of bail, since the same is intended to
obtain the provisional liberty of the accused, as a rule the same cannot be posted before custody of the accused
has been acquired by thejudicialauthorities either by his arrest or voluntary surrender.

Santiago is deemed to have voluntarily submitted herself to the jurisdiction of respondent court upon the filing of
her "Urgent Ex-parte Motion for Acceptance of Cash Bail Bond" wherein she expressly sought leave "that she be
considered as having placed herself under the jurisdiction of (the Sandiganbayan) for purposes of the required
trial and other proceedings," and categorically prayed "that the bail bond she is posting in the amount of
P15,000.00 be duly accepted" and that by said motion "she be considered as having placed herself under the
custody" of said court. Santiago cannot now be heard to claim otherwise for, by her own representations, she is
effectively estopped from asserting the contrary after she had earlier recognized the jurisdiction of the court and
caused it to exercise that jurisdiction over the aforestated pleadings she filed therein.

2.The ex parteissuanceof a hold-departure order was a valid exercise of the presiding courts inherent power to
preserve and to maintain the effectiveness of its jurisdiction over the case and the person of the accused.

Santiago does not deny and, as a matter of fact, even made a public statement that she had every intention of
leaving the country allegedly to pursuehigher studiesabroad. We uphold the course of action adopted by the
Sandiganbayan in takingjudicialnotice of such fact of petitioner's plan to go abroad and in thereafter issuing sua
sponte the hold departure order. To reiterate, the hold departure order is but an exercise of respondent court's
inherent power to preserve and to maintain the effectiveness of its jurisdiction over the case and the person of
the accused.

3. By posting bail, an accused holds himself amenable at all times to the orders and processes of the court,
thus, he may legally be prohibited from leaving the country during the pendency of the case.

Since under the obligations assumed by petitioner in her bail bond she holds herself amenable at all times to the
orders and processes of the court, she may legally be prohibited from leaving the country during the pendency of
the case. Parties with pending cases should apply for permission to leave the country from the very same courts
which, in the first instance, are in the best position to pass upon suchapplicationsand toimposethe appropriate
conditions therefor since they are conversant with the facts of the cases and the ramifications or implications
thereof.

Page 117 of 236


Constitutional Law 2 Bill of Rights Armando Santiago Jr

Right of Association
The right of the people, including those employed in the public and
private sectors, to form unions, associations, or societies for
purposes not contrary to law shall not be abridged
(Section 8, Article III)
RELATED PROVISIONS

Sec 2(5), Art. IX-B


The right to self-organization shall not be denied to government
employees.

Sec. 3, Art. XIII.


x x x. It shall guarantee the rights of all workers to self-organization,
collective bargaining and negotiations, and peaceful concerted
activities, including the right to strike in accordance with law. They
shall be entitled to security of tenure, humane conditions of work,
and a living wage. They shall also participate in policy and decision-
making processes affecting their rights and benefits as may be
provided by law.

LABOR UNIONISM
The right to form associations does not necessarily include the right to be given legal personality. However, if the
law itself should make possession of legal personality a pre-condition for effective associational action, involved
would be not just the right to have legal personality but also the right to be an association. Philippine
Association of Free Labor Unions vs Secretary of Labor (1969)

The right of association of managerial employees is denied because of Article 245 of the Labor Code which
provides that managerial employees are not eligible to join, assist or form any labor organization. This is
because Art III Sec 8 is subject to the condition that its exercise is for the purposes not contrary to law. [United
Pepsi-Cola Supervisory Union (UPSU) vs Laguesma (1998)]

COMMUNIST AND SIMILAR ORGANIZATIONS


This is a question of the constitutionality of the Anti- Subversion Act which declares the Communist Party of the
Philippines (CPP) and similar organizations illegal and outlawed. Although the Supreme Court upheld the
validity of the statute, we cannot overemphasize the need for prudence and circumspection in its enforcement,
operating as it does in the sensitive area of freedom of expression and belief. The basic guidelines for
prosecution under the Act, are the following elements for the crime to be established:

(1) In case of subversive organizations other than the CPP,


(a) that the purpose of the organization is to overthrow the present Government of the
Philippines and to establish in this country a totalitarian regime under the domination of a
foreign power;

Page 118 of 236


Constitutional Law 2 Bill of Rights Armando Santiago Jr

(b) that the accused joined such organization; and


(c) that he did so knowingly, willfully and by overt acts; and
(1) In the case of the CPP,
(a) that the CPP continues to pursue the objectives which led Congress in 1957 to
declare it to be an organized conspiracy for the overthrow of the Government by illegal
means for the purpose of placing the country under the control of a foreign power;
(b) that the accused joined the CPP;
(c) that he did so willfully, knowingly and by overt acts. [People vs Ferrer (1972)]

INTEGRATED BAR OF THE PHILIPPINES


Compulsory membership of a lawyer in the integrated bar of the Philippines does not violate the constitutional
guarantee. In Re: Edillon, 84 SCRA 554

Right to Form associationsshall not be impaired without due process of law; guarantees the right not to join
an association. Sta. Clara Homeowners Association vs. Gaston16

This right is especially meaningful in a free society because a man is by nature gregarious. His disposition to mix
with others of the same persuasions, interests or objectives is guaranteed by this provision. It also expressly
guarantees to those employed in the public and private sectors the right to form unions.

This right is available also to those in the government sectors. It is a settled in jurisprudence that, in general,
workers in the public sectors do not enjoy the right to strike. The general rule in the past and up to present is that
the terms and conditions of employment in the Government, including any political subdivision or instrumentality
thereof are governed by law. x x x. Since the terms and conditions of government employment are fixed by law,
government workers cannot use the same weapons employed by the workers in the private sector to secure
concessions from their employers. The principle behind labor unionism in private industry is that industrial peace
cannot be secured through compulsion by law.

Relations between private employers and their employees rest on an essentially voluntary basis. x x x In
government employment, however, it is the legislature and, where properly given delegated power, the
administrative heads of government which fix the terms and conditions of employment. And this is effected
through statutes or administrative circulars, rules and regulations, not through collective bargaining agreements.
Alliance of Concerned Government Workers vs. Ministry of Labor and Employment17

In the case of Jacinto vs. CA18, the SC held that petitioners were not penalized for the exercise of their right to
assemble peacefully and to petition the government for a redress of grievances. Rather, the Civil Service
Commission found them guilty of conduct prejudicial to the best interest of the service for having absented
themselves without proper authority, from their school during regular school days, in order to participate in the
mass protest, their absence ineluctably resulting in the non holding of classes and in the deprivation of students
of education, for which they were responsible. Had petitioners availed themselves of their free timerecess,
after classes, weekends or holidaysto dramatize their grievances and to dialogue with the proper authorities
within the bounds of law, no onenot the DECS, the CSC or even the SCcould have held them liable for the
valid exercise of their constitutionally guaranteed rights. As it was, the temporary stoppage of classes resulting
from their activity necessarily disrupted public services, the very evil sought to be forestalled by the prohibition
against strikes by government workers. Their act by their nature was enjoined by the Civil Service Law, rules and
regulations, for which they must, therefore, be made answerable.

GSIS vs. Kapisanan ng mga Manggagawa sa GSIS,19 it was against the backdrop of the provisions of the
Constitution that the Court resolved that employees in the public service may not engage in strikes or in
concerted and unauthorized stoppage of work; that the right of government employees to organize is limited to

16G.R. No. 141961, January 23, 2002

17 124 SCRA 1

18 281 SCRA 657

19 G. R. No. 170132, December 6, 2006,


Page 119 of 236
Constitutional Law 2 Bill of Rights Armando Santiago Jr

the formation of unions or associations, without including the right to strike. It may be, as the appellate court
urged, that the freedom of expression and assembly and the right to petition the government for a redress of
grievances stand on a level higher than economic and other liberties. The right to form associations shall not be
impaired without due process of law and is thus an aspect of the right of liberty. It is also an aspect of the
freedom of contract. In addition, insofar as the associations may have for their object the advancement of beliefs
and ideas, the freedom of association is an aspect of the freedom of speech and expression, subject to the
same limitation.

- The right also covers the right not to join an association.


- Government employees have the right to form unions. They also have the right to strike, unless there is a
statutory ban on them (i.e. ban on public school teachers).

The right to form associations shall not be impaired without due process of law.
The instinct to organize is a very basic human drive.
The right to form an association does not include the right to compel others to form an association. But there
may be situations in which, by entering into a contract, one may also be agreeing to join an association.

A land buyer who buys a lot with an annotated lien that the lot owner becomes an automatic member of
a homeowners association thereby voluntarily joins the association.

One who becomes an employee of an establishment that has a closed shop agreement with the union
thereby becomes a member of the union.

The right to form associations includes the right to unionize.


The right of civil servants to unionize is also recognized in Article IX, B, Section 2(5) which says: The right to
self-organization shall not be denied to government employees. The right is recognized whether such
employees perform governmental or proprietary functions.
The right of labor in general to unionize is also recognized in Article XIII, Section 3: The State shall
guarantee the rights of all workers to self-organization, collective bargaining and negotiations, and
peaceful concerted activities, including the right to strike in accordance with law.
Employees of the SSS and public school teachers do not have a constitutional right to strike. But the current
ban on them against strikes is statutory and may be lifted by statute.

Philippine Association of Free Labor Unions v. Secretary of Labor: Sec. 23 of RA 875 requiring the
registration of labor unions was challenged as violative of the Bill of Rights. The SC held that this contention was
untenable. The registration is not a limitation to the right of assembly or association, which may be exercised
with or without said registration. It is merely a condition sine qua non to for the acquisition of legal personality by
labor organizations, associations, or unions and the possession of the rights and privileges granted by law to
legitimate labor organizations.

Mere membership in the Communist Party and similar organizations may not be punished as a criminal
offense. Membership does not render the members either of rebellion or of conspiracy to commit
rebellion, because mere membership and nothing more implies advocacy of abstract theory or principle
without any action being induced thereby; and that such advocacy becomes criminal only if it if coupled
with action or advocacy of action. (People v. Hernandez)

(Note: The Anti-Subversion Act has been repealed.)

CASES

PEOPLE VS. FERRER20

PONENCIA: CASTRO
TOPIC: Right to association

20 GR L-32613-14, 27 December 1972 First Division, Castro (J): 5 concur, 12 took no part, 1 dissented in a separate opinion
Page 120 of 236
Constitutional Law 2 Bill of Rights Armando Santiago Jr

TRIGGER OF THE FACTS: alleged that anti-subversive act is a bill of attainder considering as a ranking leader of
communist party.
TRIGGER OF THE ISSUE: Anti-subversive law as bill of attainder
TRIGGER OF THE RULING: Anti-subversive law is constitutional. Anti-subversive Law is not a bill of attainder
because it does not render penalty to those who are identified as members of the Communist Party of the
Philippines. Rather, the mention of the CPP serves a mere definitional purpose. The law mentions the CCP and
"other subversive organization" ultimately, to hinder people from joining the said groups.

Definition: a bill of attainder is a legislative act which renders punishment without judicial proceeding. It infringes on
the jurisdiction of the Judicial Department. It violates the Doctrine of Separation of Powers.

VERDICT: the questioned resolution of September15, 1970 is set aside, and these two cases are hereby remanded to
the court a quo for trial on the merits.
END POINT: Mere membership is not violation but doing subversive activities, like attempt against government and
etc. is violative of Anti-subversive act.

FACTS
a criminal complaint for violation of section 4 of the Anti-Subversion Act was filed against Feliciano Co in the
Court of First Instance (CFI) of Tarlac.
Judge Jose C. de Guzman conducted a preliminary investigation and, finding a prima facie case against Co,
directed the Government prosecutors to file the corresponding information.
The twice-amended information recites
"That an officer and/or ranking leader of the Communist Party of the Philippines, an outlawed and illegal
organization aimed to takeover the Government of the Philippines by means of force, violence, deceit,
subversion, or any other illegal means for the purpose of establishing in the Philippines a totalitarian
regime and placing the government under the control and domination of an alien power, by being an
instructor in the Mao Tse Tung University
the training school of recruits of the New People's Army, the military arm of the said Communist
Party of the Philippines. That in the commission of the above offense, the following aggravating
circumstances are present, to wit:

B. That the crime has been committed in contempt of or with insult to public authorities;
C. That the crime was committed by a band; and
D. With the aid of armed men or persons who insure or afford impunity." Co moved to quash on
the ground that the Anti-Subversion Act is a bill of attainder. Meanwhile, on 25 May 1970,
another criminal complaint was filed with the same court, charging Nilo Tayag and five
others with subversion. After preliminary investigation was had, an information was filed. On
21 July 1970 Tayag moved to quash, impugning the validity of the statute on the grounds
that
(1) Republic Act 1700 is a bill of attainder;
(2) it is vague;
(3) it embraces more than one subject not expressed in the title thereof; and
(4) it denies him the equal protection of the laws. Resolving the constitutional
issues raised, the trial court, in its resolution of 15 September 1970, declared the statute void on the grounds
that it is a bill of attainder and that it is vague and overbroad, and dismissed the informations against the two
accused.
The Government appealed. The Supreme Court resolved to treat its appeal as a special civil action for
certiorari.

ISSUE
Whether the Anti-Subversion Law partakes of the nature of a Bill of Attainder.

HELD
Page 121 of 236
Constitutional Law 2 Bill of Rights Armando Santiago Jr

Article III, section 1 (11) of the Constitution states that "No bill of attainder or ex post facto law shall be
enacted." A bill of attainder is a legislative act which inflicts punishment without trial.
Its essence is the substitution of a legislative for a judicial determination of guilt.
The constitutional ban against bills of attainder serves to implement the principle of separation of powers by
confining legislatures to rule-making and thereby forestalling legislative usurpation of the judicial function.
History in perspective, bills of attainder were employed to suppress unpopular causes and political minorities,
and it is against this evil that the constitutional prohibition is directed.
The singling out of a definite class, the imposition of a burden on it, and a legislative intent, suffice to
stigmatize a statute as a bill of attainder.
Herein, when the Anti-Subversion Act is viewed in its actual operation, it will be seen that it does not specify
the Communist Party of the Philippines or the members thereof for the purpose of punishment.
What it does is simply to declare the Party to be an organized conspiracy for the overthrow of the
Government for the purposes of the prohibition, stated in section 4, against membership in the outlawed
organization.
The term "Communist Party of the Philippines" is used solely for definitional purposes.
In fact, the Act applies not only to the Communist Party of the Philippines but also to "any other organization
having the same purpose and their successors."
Its focus is not on individuals but on conduct. Were the Anti-Subversion Act a bill of attainder, it would be
totally unnecessary to charge Communists in court, as the law alone, without more, would suffice to secure
their punishment. But the undeniable fact is that their guilt still has to be judicially established.
The Government has yet to prove at the trial that the accused joined the Party knowingly, willfully and by
overt acts, and that they joined the Party, knowing its subversive character and with specific intent to further
its basic objective
i.e., to overthrow the existing Government by force, deceit, and other illegal means and place the
country under the control and domination of a foreign power. Further, the statute specifically requires
that membership must be knowing or active, with specific intent to further the illegal objectives of the
Party.
That is what section 4 means when it requires that membership, to be unlawful, must be shown to have been
acquired "knowingly, willfully and by overt acts." The ingredient of specific intent to pursue the unlawful goals
of the Party must be shown by "overt acts." This constitutes an element of "membership" distinct from the
ingredient of guilty knowledge. The former requires proof of direct participation in the organization's unlawful
activities, while the latter requires proof of mere adherence to the organization's illegal objectives.
Even assuming, however, that the Act specifies individuals and not activities, this feature is not enough to
render it a bill of attainder.
It is only when a statute applies either to named individuals or to easily ascertainable members of a group in
such a way as to inflict punishment on them without a judicial trial does it become a bill of attainder. Nor is it
enough that the statute specifies persons or groups in order that it may fall within the ambit of the prohibition
against bills of attainder. It is also necessary that it must apply retroactively and reach past conduct.
This requirement follows from the nature of a bill of attainder as a legislative adjudication of guilt. Indeed, if
one objection to the bill of attainder is that Congress thereby assumes judicial magistracy, then it must be
demonstrated that the statute claimed to be a bill of attainder reaches past conduct and that the penalties it
imposes are inescapable.
Section 4 of Anti-Subversion Act expressly states that the prohibition therein applies only to acts committed
"After the approval of this Act." Only those who "knowingly, willfully and by overt acts affiliate themselves with,
become or remain members of the Communist Party of the Philippines and/or its successors or of any
subversive association" after 20 June 1957, are punished.
Those who were members of the Party or of any other subversive association at the time of the enactment of
the law, were given the opportunity of purging themselves of liability by renouncing in writing and under oath
their membership in the Party.
The law expressly provides that such renunciation shall operate to exempt such persons from penal liability.
The penalties prescribed by the Act are therefore not inescapable.

SYLLABI

Page 122 of 236


Constitutional Law 2 Bill of Rights Armando Santiago Jr

Constitutional law; Bill of Attainder, defined.A bill of attainder is a legislative act which inflicts punishment
without trial. Its essence is the substitution of a legislative for a judicial determination of guilt. The constitutional
ban against bills of attainder serves to implement the principle of separation of powers by confining legislatures
to rule-making and thereby forestalling legislative usurpation of the judicial function.

Same; Bill of Attainder, history of. History in perspective, bills of attainder were employed to suppress
unpopular causes and political minorities, and it is against this evil that the -constitutional prohibition is directed.
The singling out of a definite class, the imposition of a burden on it, and a legislative intent, suffice to stigmatize
a statute as a bill of attainder. People vs. Ferrer, 48 SCRA 382, Nos. L-32613-14 December 27, 1972

Same; Same; To be Bill of Attainder statute must not only specify persons or groups but also it must reach past
conduct. Nor is it enough that the statute specify persons or groups in order that it may fall within the ambit of
the prohibition against bills of attainder. It is also necessary that it must apply retroactively and reach past
conduct. This requirement ement follows from the nature of a bill of attainder as a legislative legislative
adjudication of guilt.

R.A. 1700, other known as The Anti-Subversion Act, not a bill of attainder.When the Act is viewed in its actual
operation, it will be seen that it does not specify the Communist Party of the Philippines or the members thereof
for the purpose of punishment. What it does is simply to declare the Party to be an organized conspiracy for the
overthrow of the Government for the purposes of the prohibition, stated in section 4, against membership in the
outlawed organization. The term "Communist Party of the Philippines" is used solely for definition purposes. In
fact the Act applies not only to the Communist Party of the Philippines but also to "any other organization having
the same purpose and their successors." Its focus is not on individuals but on conduct.

Same;Same;Under the Anti-Subversion Act guilt of accused must be judicially established.Indeed, were the
AntiSubversion Act a bill of attainder, it would be totally unnecessary to charge Communists in court, as the law
alone, without more would suffice to secure their punishment. But the undeniable fact is that their guilt still has to
be judicially established. The Government has yet to prove at the trial that the accused joined the Party
knowingly, willfully and by overt acts, and that they joined the Party, knowing its subversive character and with
specific intent to further its basic, objective, i.e., to overthrow the existing Government by force, deceit, and other
illegal means and place the country under the control and domination of a foreign power.

Same; Same; Same; Mere membership in Communist Party not punished.As to the claim that under the
statute organizational guilt is nonetheless imputed despite the requirement of proof of knowing membership in
the Party, suffice it to say that that is precisely the nature of conspiracy, which has been referred to as a "dragnet
device" whereby all who participate in the criminal covenant are liable. The contention would be correct if the
statute were construed as punishing mere membership devoid of any specific intent to further the unlawful goals
of the Party. But the statute specifically requires that membership must beknowingor active, with specific intent
to further the illegal objectives of the Party. That is what section 4 means when it requires that membership, to
be unlawful, must be shown to have been acquired "knowingly, willfully and by overt acts." The ingredient of
specific intent to pursue the unlawful goals of the Party must be shown by "overt acts." This constitutes an
element of "membership" distinct from the ingredient of guilty knowledge. The former requires proof of direct
participation in the organizations unlawful activities, while the latter requires proof of mere adherence to the
organization's illegal objectives.

Same; Same; Even if Anti-Subversion Act specifies individuals it will not be Bill of AttainderEven assuming,
however, that the Act specifies individuals and not activities, this feature is not enough to render it a bill of
attainder. A statute prohibiting partners or employees of securities underwriting firms from serving as officers or
employees of national banks on the basis of a legislative f inding that the persons mentioned would be subject to
the temptation to commit acts deemed inimical to the national economy, has been declared not to be a bill of
attainder. Similarly, a statute requiring every secret, oath-bound society having a membership of at least twenty
to register, and punishing any person who becomes a member of such society which fails to register or remains
a member thereof, was declared valid even if in its operation it was shown to apply only to the members of the
Ku Klux Klan, In the Philippines, the validity of section 23(b) of the Industrial Peace Act, requiring labor unions to
file with the Department of Labor affidavits of union officers "to the effect that they are not members of the

Page 123 of 236


Constitutional Law 2 Bill of Rights Armando Santiago Jr

Communist Party and that they are not members of any organization which teaches the overthrow of the
Government by force or by any illegal or unconstitutional methods," was upheld by this Court.

Same; Same.Indeed, it is only when a statute applies either to named individuals or to easily ascertainable
members of a group in such a way as to inflict punishment on them without a judicial trial does it become a bill of
attainder. But when the judgment expressed in legislation is so universally acknowledged to be certain as to be
"judicially noticeable," the legislature may apply its own rules, and judicial hearing is not needed fairly to make
such determination.

Same; Same; Character of Communist Party as construed by Court.In the Philippines the character of the
Communist Party has been the object of continuing scrutiny by this Court. In 1932 we found the Communist
Party of the Philippines to be an illegal association. In 1969 we again found that the objective of the Party was
the "overthrow of the Philippine Government by armed struggle and to establish in the Philippines a communist
form of government similar to that of Soviet Russia and Red China." More recently, inLansang vs. Garcia,we
noted the growth of the Communist Party of the Philippines and the organization of Communist fronts among
youth organizations such the Kabataang Makabayan (KM) and the emergence of the New People's Army. After
meticulously reviewing the evidence, we said: "We entertain, therefore, no doubts about the existence of a
sizeable group of men who have publicly risen in arms to overthrow the government and have thus been and still
are engaged in rebellion against the Government of the Philippines."

Same;Same;To be Bill of Attainder statute must not only specify persons or groups but also it must reach past
conduct. Nor is it enough that the statute specifies persons or groups in order that it may fall within the ambit
of the prohibition against bills of attainder. It is also necessary that it must apply retroactively and reach past
conduct. This requirement ement follows from the nature of a bill of attainder as a legeslative legislative
adjudication of guilt.

Same; Anti-Subversion Act not violative of constitutional freedom 'of speech and association.The legislative
declaration in section 2 of the Act that the Communist Party of the Philippines is an organized conspiracy for the
overthrow of the Government is intended not to provide the basis for a legislative finding of guilt of the members
of the Party but rather to justify the proscription spelled out in section 4. Freedom of expression and freedom of
association are so fundamental that they are thought by some to occupy a "preferred position" in the hierarchy of
constitutional values. Accordingly, any limitation on their exercise must be justified by the existence of a
substantive evil. This is the reason why before enacting the statute in question Congress conducted careful
investigations and then stated its findings in the preamble of the Act. In truth, the constitutionality of the Act
would be open to question if, instead of making those findings in enacting the statute, Congress omitted to do
so.
Same;Act does not infringe freedoms of expression and association.Whatever interest in freedom of speech
and freedom of association is infringed by the prohibition against knowing membership in the Communist Party
of the Philippines, is so indirect and so insubstantial as to be clearly and heavily outweighed by the overriding
considerations of national security and the preservation of democratic institutions in this country.

BANGALISAN VS. HON. COURT OF APPEALS21

PONENCIA: REGALADO
TOPIC: Right to association
TRIGGER OF THE FACTS: 800 public school teachers had a mass action for their benefits, they were dismissed by
DECS for the neglect of their duty as a teacher.
TRIGGER OF THE ISSUE: WON CA committed grave abuse of discretion and petitioners exonerated that they are
deprived of their constitutional right.
TRIGGER OF THE RULING: No. CA did not commit grave abuse of discretion. Public employees are not allowed to
engage in a strike.
VERDICT: the decision of the Court of Appeals is hereby AFFIRMED, but with the MODIFICATION that petitioner
Rodolfo Mariano shall be given back wages without deduction or qualification from the time he was suspended until

21 G.R. No. 124678. July 31, 1997


Page 124 of 236
Constitutional Law 2 Bill of Rights Armando Santiago Jr

his actual reinstatement which, under prevailing jurisprudence, should not exceed five years.
END POINT: The right of government employees to organize is limited only to the formation of unions or
associations, without including the right to strike, mass leaves and demonstrations. because public teachers action
disrupts the public rights to service. they deal with public services.

FACTS
Petitioners were among the 800 public school teachers who staged mass actions
to dramatize their protests concerning the alleged failure of the public authorities to implement in a just and
correct manner certain laws and measures intended for their material benefit.
the Secretary of the Department of Education, Culture and Sports (DECS) issued a Return-to-Work Order.
Petitioners failed to comply with said order, hence they were charged by the Secretary with grave
misconduct; gross neglect of duty; gross violation of Civil Service law, rules and regulations and reasonable
office regulations; refusal to perform official duty; gross insubordination; conduct prejudicial to the best
interest of the service; and absence without official leave in violation of PD 807, otherwise known as the Civil
Service Decree of the Philippines.
They were simultaneously placed under preventive suspension.
Thereafter, the DECS Secretary rendered a decision finding petitioners guilty as charged and dismissing
them from the service effective immediately.
Acting on the motions for reconsideration filed by some of the petitioners the Secretary subsequently
modified the penalty of dismissal to suspension for nine months without pay.
The other petitioners also filed individual appeals to the MSPB, but all of their appeals were dismissed for
lack of merit.
Petitioners then appealed to the Civil Service Commission (CSC) where it ruled that respondents were guilty
of conduct prejudicial to the best interest of the service.
It, however, modified the penalty of nine months suspension previously meted to them to six months
suspension with automatic reinstatement in the service but without payment of back wages.
All the petitioners moved for reconsideration of the CSC resolutions but these were all denied,
except that of petitioner Rodolfo Mariano who was found guilty only of a violation of reasonable office rules
and regulations because of his failure to inform the school of his intended absence and to file an application
for leave therefor.
This petitioner was accordingly given only a reprimand.
On appeal, the Court of Appeals dismissed the petition for lack of merit

ISSUE
Whether or not the Court of Appeals committed grave abuse of discretion when it upheld the resolutions of the
CSC that penalized petitioners whose only offense was to exercise their constitutional right to peaceably
assemble and petition the government for redress of grievances

HELD
No, the CA did not commit grave abuse of discretion.

It is the settled rule in this jurisdiction that employees in the public service may not engage in strikes. While the
Constitution recognizes the right of government employees to organize, they are prohibited from performing
strikes, demonstrations, mass leaves, walk-outs and other forms of mass action which will result in temporary
work stoppage or disruption of public services. The right of government employees to organize is limited only to
the formation of unions or associations, without including the right to strike.

It is an undisputed fact that there was a work stoppage and that petitioners purpose was to realize their
demands by withholding their services. The fact that the conventional term strike was not used by the striking
employees to describe their common course of action is inconsequential, since the substance of the situation,
and not its appearance, will be deemed to be controlling.

Page 125 of 236


Constitutional Law 2 Bill of Rights Armando Santiago Jr

The ability to strike is not essential to the right of association. To grant employees of the public sector the right to
strike, there must be a clear and direct legislative authority therefor. In the absence of any express legislation
allowing government employees to strike, recognizing their right to do so, or regulating the exercise of the right,
employees in the public service may not engage in strikes, walkouts and temporary work stoppages like workers
in the private sector

It is not the exercise by the petitioners of their constitutional right to peaceably assemble that was punished, but
the manner in which they exercised such right which resulted in the temporary stoppage or disruption of public
service and classes in various public schools in Metro Manila. For, indeed, there are efficient but non-disruptive
avenues, other than the mass actions in question, whereby petitioners could petition the government for redress
of grievances.

It bears stressing that suspension of public services, however temporary, will inevitably derail services to the
public, which is one of the reasons why the right to strike is denied government employees. It may be conceded
that the petitioners had valid grievances and noble intentions in staging the mass actions, but that will not
justify their absences to the prejudice of innocent school children. Their righteous indignation does not legalize
an illegal work stoppage. The right of the sovereign to prohibit strikes or work stoppages by public employees
was clearly recognized at common law.

SYLLABI
Civil Service; Administrative Law; Strikes; The right of government employees to organize is limited only to the
formation of unions or associations, without including the right to strike.It is the settled rule in this jurisdiction
that employees in the public service may not engage in strikes. While the Constitution recognizes the right of
government employees to organize, they are prohibited from staging strikes, demonstrations, mass leaves, walk-
outs and other forms of mass action which will result in temporary stoppage or disruption of public services. The
right of government employees to organize is limited only to the formation of unions or associations, without
including the right to strike.

Same; Same; Same; Same; The fact that the conventional term strike was not used by the striking employees
to describe their common course of action is inconsequential, since the substance of the situation, and not its
appearance, will be deemed to be controlling. It is an undisputed fact that there was a work stoppage and that
petitioners purpose was to realize their demands by withholding their services. The fact that the conventional
term strike was not used by the striking employees to describe their common course of action is
inconsequential, since the substance of the situation, and not its appearance, will be deemed to be controlling.
The ability to strike is not essential to the right of association. In the absence of statute, public employees do not
have the right to engage in concerted work stoppages for any purpose.

Same; Same; Same; Right of Assembly; The public school teachers who went on strike are penalized not
because they exercised their right of peaceable assembly and petition for redress of grievances but because
their successive unauthorized and unilateral absences produced adverse effects upon their students for whose
education they are responsible.Further, herein petitioners, except Mariano, are being penalized not because
they exercised their right of peaceable assembly and petition for redress of grievances but because of their
successive unauthorized and unilateral absences which produced adverse effects upon their students for whose
education they are responsible. The actuations of petitioners definitely constituted conduct prejudicial to the best
interest of the service, punishable under the Civil Service law, rules and regulations.

Same; Same; Same; Suspension of public services, however temporary, will inevitably derail services to the
public, which is one of the reasons why the right to strike is denied government employees. It bears stressing
that suspension of public services, however temporary, will inevitably derail services to the public, which is one
of the reasons why the right to strike is denied government employees. It may be conceded that the petitioners
had valid grievances and noble intentions in staging the mass actions, but that will not justify their absences to
the prejudice of innocent school children. Their righteous indignation does not legalize an illegal work stoppage.

Page 126 of 236


Constitutional Law 2 Bill of Rights Armando Santiago Jr

Same; Same; Same; Even in the absence of express statutory prohibition like Memorandum Circular No. 6,
public employees are denied the right to strike or engage in a work stoppage against a public employer. As a
general rule, even in the absence of express statutory prohibition like Memorandum Circular No. 6, public
employees are denied the right to strike or engage in a work stoppage against a public employer. The right of the
sovereign to prohibit strikes or work stoppages by public employees was clearly recognized at common law.
Indeed, it is frequently declared that modern rules which prohibit such strikes, either by statute or by judicial
decision, simply incorporate or reassert the common-law rule.
Same; Same; Same; To grant employees of the public sector the right to strike, there must be a clear and direct
legislative authority therefor. To grant employees of the public sector the right to strike, there must be a clear
and direct legislative authority therefor. In the absence of any express legislation allowing government
employees to strike, recognizing their right to do so, or regulating the exercise of the right, employees in the
public service may not engage in strikes, walkouts and temporary work stoppages like workers in the private
sector.

Same; Same; Preventive Suspension; Under Section 51 of Executive Order No. 292, it is the nature of the
charge against an officer or employee which determines whether he may be placed under preventive
suspension.Section 51 of Executive Order No. 292 provides that (t)he proper disciplining authority may
preventively suspend any subordinate officer or employee under his authority pending an investigation, if the
charge against such officer or employee involves dishonesty, oppression or grave misconduct, or neglect in the
performance of duty, or if there are reasons to believe that the respondent is guilty of charges which would
warrant his removal from the service. Under the aforesaid provision, it is the nature of the charge against an
officer or employee which determines whether he may be placed under preventive suspension. In the instant
case, herein petitioners were charged by the Secretary of the DECS with grave misconduct, gross neglect of
duty, gross violation of Civil Service law, rules and regulations, and reasonable office regulations, refusal to
perform official duty, gross insubordination, conduct prejudicial to the best interest of the service and absence
without official leave (AWOL), for joining the teachers mass actions held at Liwasang Bonifacio on September
17 to 21, 1990. Hence, on the basis of the charges against them, it was within the competence of the Secretary
to place herein petitioners under preventive suspension.

UNITED PEPSI-COLA SUPERVISORY UNION (UPSU) VS HON.


BIENVENIDO E. LAGUESMA22
PONENCIA: Mendoza
TOPIC: Non-Impairment clause
TRIGGER OF THE FACTS: Petitioners impugn the intent of Art. 245 of the labor code, further, alleged that
TRIGGER OF THE ISSUE: WON managers forming, joining and assisting labor unions is unconstitutional
TRIGGER OF THE RULING: Route manager is ineligible union membership
VERDICT: WHEREFORE, I vote, given all the foregoing, for the reversal of the resolution and the order of 22
September 1995, of public respondent. Petition dismissed.
END POINT: Route manager is ineligible union membership because even if they are not belong to the top managers
which their role is to lay down policies, route managers are the one who is carrying the policy made by the top
managers to supervise if these policies are followed properly and another reason was, they have a direct
connection with the company which have a conflict of interest. Prohibition was provided by the labor code.

FACTS
The petitioner is a union of supervisory employees.
Sometime of march, the union filed a petition for certification on behalf of the route managers at Pepsi-Cola
Products Philippines, Inc.
However, its petition was denied by the med-arbiter and, on appeal, by the Secretary of Labor and
Employment, on the ground that the route managers are managerial employees and, therefore, ineligible for
union membership under the first sentence of Art 245 of the Labor Code, which provides:

22 G.R. No. 122226, March 25, 1998


Page 127 of 236
Constitutional Law 2 Bill of Rights Armando Santiago Jr

Ineligibility of managerial employees to join any labor organization; right of supervisory Employees-
Managerial employees are not eligible to join, assist or form any labor organization. Supervisory
employees shall not be eligible for membership in a labor organization of the rank-and-file
employees but may join, assist or form separate labor organizations of their own.

Petitioner filed a motion for reconsideration, pressing for resolution its contention that the first sentence of Art.
245 of the Labor Code, so far declares managerial employees to be ineligible to form, assit or join unions,
contravenes Article III, Section 8 of the 1987 Constitution which provides:

The right of the people, including those employed in the public and private sectors to form unions,
associations, or societies for purposes not contrary to law shall not be abridged.

ISSUE
Whether or not Art. 245, insofar as it prohibits managerial employees from forming, joining or assisting labor
unions, violates Article III, Section 8 of the Constitution.

RULING
Art. 245 do not violate Article III, Section 8 of the Constitution.
The real intent of Article III, section 8 is evident in Lerums proposal.
The Commission intended the absolute right to organize of government workers, supervisory employees and
security guards to be constitutionally guaranteed.
By implication, no similar absolute constitutional rights to organize for labor purposes should be deemed to
have been granted to top-level and middle managers.
Nor is the guarantee of organizational right in Art. III, Section 8 of the Constitution encroach on by a ban
against managerial employees forming a union.
The guaranteed right in Art. III, Section 8 is subject to the condition that its exercise should be for the
purposes not contrary to law.
In the case of Art 245, there is rational basis for prohibiting managerial employees from forming or joining
labor organization.
For the reason that these managerial employees would belong to or be affiliated with a Union, the latter might
not be assured of their loyalty to the Union in view of evident conflict of interest. The union also becomes
company-dominated with the presence of managerial employees in Union membership.

SYLLABI
Same; Same; The guarantee of organizational right in Art. 111, 8 not infringed by a ban against managerial
employees forming a union; There is a rational basis for prohibiting managerial employees from forming or
joining labor organizations. Nor is the guarantee of organizational right in Art. III, 8 infringed by a ban against
managerial employees forming a union. The right guaranteed in Art. III, 8 is subject to the condition that its
exercise should be for purposes not contrary to law. In the case of Art. 245, there is a rational basis for
prohibiting managerial employees from forming or joining labor organizations. As Justice Davide, Jr., himself a
constitutional commissioner, said in his ponencia in Philips Industrial Development, Inc. v. NLRC: In the first
place, all these employees, with the exception of the service engineers and the sales force personnel, are
confidential employees. Their classification as such is not seriously disputed by PEO-FFW; the five (5) previous
CBAs between PIDI and PEO-FFW explicitly considered them as confidential employees. By the very nature of
their functions, they assist and act in a confidential capacity to, or have access to confidential matters of,
persons who exercise managerial functions in the field of labor relations. As such, the rationale behind the
ineligibility of managerial employees to form, assist or joint a labor union equally applies to them.

Supreme Court; Judgments; Prospective Application of Judgments; Natural Resources; Reliance on De


Agbayani v. PNB, 38 SCRA 429 (1971) and Benzonan v. Court of Appeals, 205 SCRA 515 (1992), is misplaced,
as these cases would apply if the prevailing law or doctrine at the time of the signing of the Amended Joint
Venture Agreement (JVA) was that a private corporation could acquire alienable lands of the public domain, and

Page 128 of 236


Constitutional Law 2 Bill of Rights Armando Santiago Jr

the Decision annulled the law or reversed this doctrinethe prevailing law before, during and after the signing of
the Amended JVA is that private corporations cannot hold, except by lease, alienable lands of the public domain.
Amaris reliance on De Agbayani and Spouses Benzonan is misplaced. These cases would apply if the
prevailing law or doctrine at the time of the signing of the Amended JVA was that a private corporation could
acquire alienable lands of the public domain, and the Decision annulled the law or reversed this doctrine.
Obviously, this is not the case here. Under the 1935 Constitution, private corporations were allowed to acquire
alienable lands of the public domain. But since the effectivity of the 1973 Constitution, private corporations were
banned from holding, except by lease, alienable lands of the public domain. The 1987 Constitution continued this
constitutional prohibition. The prevailing law before, during and after the signing of the Amended JVA is that
private corporations cannot hold, except by lease, alienable lands of the public domain. The Decision has not
annulled or in any way changed the law on this matter. The Decision, whether made retroactive or not, does not
change the law since the Decision merely reiterates the law that prevailed since the effectivity of the 1973
Constitution. Thus, De Agbayani, which refers to a law that is invalidated by a decision of the Court, has no
application to the instant case.

Page 129 of 236


Constitutional Law 2 Bill of Rights Armando Santiago Jr

Contract Clause / Non-Impairment


clause
No law impairing the obligation of contracts shall be passed
(Section 10, Article III)
REQUISITES OF VALID IMPAIRMENT
1. Substantial impairment
A. change the terms of legal contract either in time or mode of performance
B. impose new conditions
C. dispenses with expressed conditions
D. authorizes for its satisfaction something different from that provided in the terms
2. Affects rights of parties with reference to each other, and not with respect to non-parties.23

WHEN DOES, A LAW IMPAIR THE OBLIGATION OF CONTRACTS?


1. If it changes the terms and conditions of a legal contract either as to the time or mode of performance
2. If it imposes new conditions or dispenses with those expressed
3. If it authorizes for its satisfaction something different from that provided in its terms.

WHEN NON-IMPAIRMENT CLAUSE PREVAILS


(1) against power of taxation
(2) regulation on loans

New regulations on loans making redemption of property sold on foreclosure stricter are not allowed to apply
retroactively. Co v. Philippine National Bank24

Civil Code, Art. 1306.


The contracting parties may establish such stipulations, clauses,
terms and conditions as they may deem convenient, provided they
are not contrary to law, morals, good customs, public order, or public
policy.

APPLICATION OF THE CONTRACT CLAUSE


Impairment is anything that diminishes the efficacy of the contract. There is substantial impairment when the law
changes the terms of a legal contact between the parties, either in the time or mode of performance, or imposes
new conditions, or dispenses with those expressed, or authorizes for its satisfaction something different from that
provided in its terms. [Clements v. Nolting25]

23 [Philippine Rural Electric Cooperatives Association v. Secretary, DILG (2003)]

24 [Co v. Philippine National Bank (1982)]

25 42 Phil. 702
Page 130 of 236
Constitutional Law 2 Bill of Rights Armando Santiago Jr

CONTEMPORARY APPLICATION OF THE CONTRACT CLAUSE


As to Tax:

GENERAL RULE: Power of taxation may not be used to violate the constitutional right of every person to be
secured against any statute that impairs the obligation of contracts.

EXCEPTION: But if the statute exempts a party from any one class of taxes, the imposition of a different tax is
not an impairment of the obligation of contracts.

A law enacted in the exercise of police power to regulate or govern certain activities or transactions could be
given retroactive effect and may reasonably impair vested rights or contracts. Police power legislation is
applicable not only to future contracts, but equally to those already in existence. Non-impairment of contracts or
vested rights clauses will have to yield to the superior and legitimate exercise by the State of the police power.
[Ortigas & Co. v. CA26]

New regulations on loans making redemption of property sold on foreclosure more strict are not allowed to apply
retroactively. [Co v. Philippine National Bank (1982)]

Limitations on the use of land imposed by a contract to yield to a reasonable exercise of police power are
affirmed. Thus, zoning regulations are superior to contractual restrictions on the use of property. [Presley v. Bel-
Air Village Association (1991)]

A separation pa law can be given retroactive effect to apply to existing contracts. [Abella v. National Labor
Regulations Commission (1987)]

The charter of a bank, even if a contract, is no obstacle to liquidation done under police power. [Philippine
Veterans Bank Employees Union v. Philippine Veterans Bank (1990)]

Contracts also yield to the requirements of the freedom of religion. [Victoriano v. Elizalde Rope Workers (1974)]
Timber licenses, permits, and license agreements are the principal instruments by which the State regulates the
utilization and disposition of forest resources to the end that public welfare is promoted. They may be validly
amended, modified, replaced, or rescinded by the Chief Executive when national interests so require. Thus, they
are not deemed contracts within the purview of the due process of law clause. [Oposa v. Factoran (1993)]

A rehabilitation plan approved by statute which merely suspends the actions for claims does not violate the
contract clause. [GSIS v. Kapisanan (2006)]

The SECs approval of the Rehabilitation Plan did not impair BPIs right to contract. The impairment clause is a
limit on the exercise of legislative power and not of judicial or quasi-judicial power. The SEC...was acting as a
quasi-judicial body, and its order approving the plan cannot constitute an impairment of the right and the freedom
to contract.

Besides, the mere fact that the Rehabilitation Plan proposes a dacion en pago approach does not render it
defective on the ground of impairment of the right to contract. The undertaking really partakes in a sense of the
nature of sale. As such, the essential elements of a contract of sale must be present. Being a form of contract,
the dacion en pago agreement cannot be perfected without the consent of the parties involved. [China Banking
Corporation v. ADB Holdings (2008)]

The amount of rental is an essential condition of any lease contract. The change of its rate in the Rehabilitation
Plan is not justified as it impairs the stipulation between the parties. [Leca Realty v. Manuela Corporation (2007)]
The non-impairment clause is a limit on legislative power, and not of judicial or quasi-judicial power. The
approval of the Rehabilitation Plan by the Securities and Exchange Commission is an exercise of adjudicatory
power by an administrative agency and thus the non-impairment clause does not apply. Neither does it impair
the power to contract. [BPI v. SEC (2007)]

26 G.R. No. 126102, December 4, 2000


Page 131 of 236
Constitutional Law 2 Bill of Rights Armando Santiago Jr

Laws prohibiting premature campaigning are intended to level the playing field for candidates to public office, to
equalize the situation between popular or rich candidates, on one hand, and lesser- known or poorer candidates,
on the other, by preventing undue advantage in exposure and publicity on account of their resources and
popularity. Such laws might affect advertising contracts, the non-impairment clause of the Constitution must yield
to the loftier purposes targeted by the Government. [Chavez v. COMELEC (2004)]

The Court has imposed 2 essential requisites in order that RA 7641 (Retirement Law) may be given retroactive
effect. First, the claimant for retirement benefits must still be in the employ of the employer at the time the statute
took effect. Second, the claimant must have complied with the requirements for eligibility for such retirement
benefits under the statute. [Universal Robina Sugar v. Cabaleda (2008)]

LIMITATIONS
It is ingrained in jurisprudence that the constitutional prohibition does not prohibit every change in existing laws.
To fall within the prohibition, the change must not only impair the obligation of the existing contract, but the
impairment must be substantial. Moreover, the law must effect a change in the rights of the parties with
reference to each other, and not with respect to non-parties. [Philippine Rural Electric Cooperatives
Association v. Secretary, DILG27]

NATURE OF PROTECTION
Purpose: Safeguard the integrity of valid contractual agreements against unwarranted
interference by the State.
General Rule: Contracts should be respected by the legislature and not tampered with by subsequent
laws that will change the intention of the parties or modify their rights and obligations. The will of the parties to
the contract must prevail.

CONTRACTS AFFECTED
Affects legal, executed and executory contracts, which must be valid.

LIMITATIONS
1. A contract valid at the time of its execution may be legally modified or even completely invalidated by a
subsequent law.
2. Strict construction. Charters, franchises and licenses granted by the Government are strictly construed
against the grantees.
3. If a law is a proper exercise of the police power, it will prevail over the contract. This is because public
welfare is superior to private rights (PNB v. Remigio, G.R. 78508, March 21, 1995). Into each contract
are read the provisions of existing law and, always, a reservation of the police power as long as the
agreement deals with a matter affecting the public welfare.
4. Eminent domain and taxation may also validly limit the impairment clause.

EFFECT OF EMERGENCY LEGISLATION ON CONTRACTS


In a national emergency, such as a protracted economic depression, the police power may be exercised to the
extent of impairing some of the rights of parties arising from contracts. However, such emergency laws are to
remain in effect only during the continuance of the emergency.

CURRENCY LEGISLATION AND CONTRACTS


1. The legislative department has complete authority to determine the currency of the state and to
prescribe what articles shall be used and accepted as legal tender in the payment of lawful obligations.
2. Private parties are bound to observe this governmental authority over the nation's currency in the
execution of their contracts.

27 G.R. No. 143076, June 10, 2003


Page 132 of 236
Constitutional Law 2 Bill of Rights Armando Santiago Jr

IMPAIRMENT
In order to come within the meaning of the constitutional provision, the obligation of contract must be
impaired by some legislative act (statute, ordinance, etc.). The act need not be by a legislative office; but it
should be legislative in nature. Furthermore, the impairment must be substantial (Philippine Rural Electric
Cooperatives Assoc. v. DILG Secretary, G.R. 143076, June 10, 2003).

A mere administrative order, not legislative in nature, may not be considered a cause of impairment within
the scope of the constitutional guarantee. The guarantee is also not violated by court decisions.

The act of impairment is anything which diminishes the value of the contract. The legislature may,
however, change the remedy or may prescribe new modes of procedure. The change does not impair the
obligation of contracts so long as another remedy, just as efficacious, is provided for the adequate
enforcement of the rights under the contract. (Manila Trading Co v. Reyes, 1935)

NOTE:
A mere change in PROCEDURAL REMEDIES which does not change the substance of the contract, and
which still leaves an efficacious remedy for enforcement does NOT impair the obligation of contracts.
A valid exercise of police power is superior to obligation of contracts.
The freedom to contract is not absolute; all contracts and all rights are subject to the police power of the
State and not only may regulations which affect them be established by the State, but all such regulations
must be subject to change from time to time, as the general well-being of the community may require, or
the circumstances may change, or as experience may demonstrate the necessity.
The purpose of the impairment clause is to safeguard the integrity of valid contractual agreements against
unwarranted interference by the State. As a rule, they should be respected by the legislature and not
tampered with by subsequent laws that will change the intention of the parties or modify their rights and
obligations. The will of the obligor and the obligee must be observed; the obligation of their contract must
not be impaired. However, the protection of the impairment clause is not absolute. There are instances
when contracts valid at the time of their conclusion may become invalid, or some of their provisions may
be rendered inoperative or illegal, by virtue of supervening legislation.

Limitations:
1. Police powerprevails over contracts;
2. Eminent domainmay weaken obligation of contracts; and
3. Taxationcannot weaken obligation of contracts.

Impairmentanything that diminishes the efficacy of a contract There is impairment when there is change in the
terms of a legal contract between parties, either in the time or mode of performance, or imposes new conditions,
or dispenses with those expressed, or authorizes for its satisfaction something different from that provided in its
terms. (Clemons vs. Nolting, No. 17959, January 24, 1922)

Hon. Heherson Alvarez vs. PICOP Resources, Inc., G.R. No. 162243, November 29, 2006, in unequivocal
terms, the SC have consistently held that such licenses concerning the harvesting of timber in the countrys
forests cannot be considered contracts that would bind the Government regardless of changes in policy and the
demands of public interest and welfare. Since timber licenses are not contracts, the non-impairment clause
cannot be invoked.

The law relating to the obligation of contracts does not prohibit every change in existing laws. To fall within
the prohibition the change must impair the obligation of the existing contract, and the impairment must be
substantial.

A law which changes the terms of a legal contract between parties, either in the time or mode of performance,
or imposes new conditions, or dispenses with those expressed, or authorizes for its satisfaction something
different from that provided in its terms, is law which impairs the obligation of a contract and is therefore null
and void. (Clemons v. Nolting)

Page 133 of 236


Constitutional Law 2 Bill of Rights Armando Santiago Jr

Any law which enlarges, abridges, or in any manner changes the intention of the parties, necessarily impairs
the contract itself. (U.S. v. Diaz Conde)

Government of the Philippine Islands v. Frank: The terms of a statute had been made a part of the contract
between Frank and the Government. The Court said that a subsequent amendment of the statute could not have
the effect of amending the terms of the contract.

To come under the prohibition, the law must effect a change on the rights of the parties with reference to each
other and not with reference to non-parties. Moreover, what the prohibition envisions are enactments passed
by a governmental law-making body.

An additional tax imposed upon goods already contracted to be sold does not impair the contract
between the buyer and seller.
A change in the Catholic Churchs canon law which affects the contractual relation between parties with
reference to internal affairs of the church is not covered by the prohibition.

During the period of transition, a statute was passed providing that money judgments rendered in court
should be stated in terms of the new Philippine currency in an amount equivalent to the value of the old. The
law, when applied to contractual debts, was challenged as an impairment of the obligation of contracts. In
Gaspar v. Molina, the Court upheld the statute since it did not require the debtor to pay more nor the creditor
to receive less than they were required to pay or receive under the former law.

Clemons v. Nolting: An attempt by the government to satisfy an obligation to pay in U.S. dollars by the payment
of Philippine pesos at the rate of P2 = $1 when the commercial value of the U.S. dollar was much more, was
deemed an impairment of the contractual obligation.

The non-impairment clause is a limit on the exercise of legislative power and not of judicial or quasi-judicial
power.

Police power is superior over the sanctity of contracts.

Statutes enacted for the regulation of public utilities, being a proper exercise by the state of police power, are
applicable not only to those public utilities coming into existence after its passage, but likewise to those
already established and in operation. (Pangasinan Transportation Co. v. Public Service Commission)

In Abe v. Foster Wheeler Corporation, in upholding a statute granting to workers rights which they did not
have under existing contracts, the Court said that the constitutional guaranty of non-impairment is limited by
the exercise of police power of the State, in the interest of public health, safety, morals and general welfare.

The contract clause did not preclude remedial legislation in the interest of the general welfare.
Primero v. Court of Agrarian Relations: The security of tenure guaranteed to an agricultural tenant by a
statute was challenged as impairing existing tenancy relations. The Court ruled that the law was unquestionably
a remedial legislation promulgated pursuant to the social justice precepts of the Constitution and in the exercise
of the police power of the State to promote the common weal.

New regulations on loans making redemption of property sold on foreclosure stricter were not allowed to
apply retroactively. (Co v. Philippine National Bank)

Limitations on the use of land imposed by contract yield to a reasonable exercise of police power and, hence,
zoning regulations are superior to contractual restrictions on the use of property. (Ortigas & Co. v. FEATI
Bank)

Contracts also yield to the requirements of freedom of worship.

Page 134 of 236


Constitutional Law 2 Bill of Rights Armando Santiago Jr

A license or permit is not a contract between the sovereignty and the licensee or permittee, and is not a property
in the constitutional sense, as to which the constitutional prescription against the impairment of contracts may
extend. (Gonzalo Sy Trading v. Central Bank)

CASES

ORTIGAS & COMPANY VS COURT OF APPEALS28


PONENCIA: QUISUMBING, J.:
TOPIC: Non-Impairment clause
TRIGGER OF THE FACTS: ortigas sold to hermoso only as a residencial area then hermosa lease to mathay who
build a commercial building which is not stated in the contract which cause of filing a suit, thereafter, the said area
was converted by MMC to become a commercial area.
TRIGGER OF THE ISSUE: retroactive effect may impair contracts
TRIGGER OF THE RULING: Zoning ordinance is a valid exercise of police power. As a general rule, contracts
between parties cannot be impaired by subsequent ordinances, law, etc. Exception to the rule, it may be impaired
pursuant to police power.
VERDICT: the instant petition is DENIED. The challenged decision of the Court of Appeals
as well as the assailed resolution of August 13, 1996, in CAG.R. SP No. 39193 is AFFIRMED. Costs against petitioner.
END POINT: through police power a contract may be impaired and may be applied retroactively

FACTS
Ortigas & Company sold to Emilia Hermoso, a parcel of land located in Greenhills Subdivision with conditions
duly annotated on the certificate of title issued to Emilia
the Metropolitan Manila Commission (now MMDA) enacted MMC Ordinance 81-01 (Comprehensive Zoning
Area for the National Capital Region),
which reclassified as a commercial area a portion of Ortigas Avenue where the lot is located.
Ismael Mathay III leased the lot from Emilia Hermoso and J.P. Hermoso Realty Corp.
The lease contract did not specify the purposes of the lease.
Thereupon, Mathay constructed a single story commercial building for Greenhills Autohaus, Inc., a car sales
company.
Ortigas filed a complaint against Emilia Hermoso with the RTC Pasig
seeking the demolition of the said commercial structure for having violated the terms and conditions of
the Deed of Sale.
The complaint was later amended to implead Ismael G. Mathay III and J.P. Hermoso Realty Corp.,
which have 10% interest in the lot.
In his answer, Mathay III denied any knowledge of the restrictions on the use of the lot and filed a cross-claim
against the Hermosos.
the trial court issued the writ of preliminary injunction.
Mathay III moved to set aside the injunctive order,
but the trial court denied the motion.
Mathay III then filed with the CA a special civil action for certiorari
ascribing to the trial court grave abuse of discretion in issuing the writ of preliminary injunction.
He claimed that MMC Ordinance 81-01 classified the area where the lot was located as
commercial area and said ordinance must be read into the Deed of Sale as a concrete exercise
of police power.
Ortigas & Company averred that restrictions duly annotated on the title must prevail over the ordinance.
the appellate court granted the petition, nullified and set aside the assailed orders.
The appellate court held that the MMC Ordinance effectively nullified the restrictions allowing only residential
use of the property in question.

28 GR 126102 04 December2000
Page 135 of 236
Constitutional Law 2 Bill of Rights Armando Santiago Jr

Ortigas seasonably moved for reconsideration, but the appellate court denied it.
Ortigas filed the petition for review.

ISSUE
Whether or notMMC 81-01 be given retroactive effect and impair vested rights and contracts?

HELD
Yes,in general, laws are to be construed as having only prospective operation.
Lex prospicit, non respicit.
Equally settled, only laws existing at the time of the execution of a contract are applicable thereto and not
later statutes, unless the latter are specifically intended to have retroactive effect.
A later law which enlarges, abridges, or in any manner changes the intent of the parties to the contract
necessarily impairs the contract itself and cannot be given retroactive effect without violating the
constitutional prohibition against impairment of contracts.
One exception involves police power.
A law enacted in the exercise of police power to regulate or govern certain activities or transactions could be
given retroactive effect and may reasonably impair vested rights or contracts.
Police power legislation is applicable not only to future contracts, but equally to those already in existence.
Non-impairment of contracts or vested rights clauses will have to yield to the superior and legitimate exercise
by the State of police power to promote the health, morals, peace, education, good order, safety, and general
welfare of the people.
Moreover, statutes in exercise of valid police power must be read into every contract. MMC Ordinance 81-01
is a legitimate police power measure as previously held in Sangalang vs. IAC. Thus, following the ruling in
Ortigas & Co., Ltd. vs. Feati Bank & Trust Co., 94 SCRA 533 (1979), the contractual stipulations annotated on
the Torrens Title must yield to the ordinance.
When that stretch was reclassified, the restrictions in the contract of sale were deemed extinguished by the
retroactive operation of the zoning ordinance and could no longer be enforced.
While Philippine legal system upholds the sanctity of contract so that a contract is deemed law between the
contracting parties, nonetheless, stipulations in a contract cannot contravene law, morals, good customs

SYLLABI
Contracts; Contract Clause; Police Power; Statutes; A later law which enlarges, abridges, or in any manner
changes the intent of the parties to the contract necessarily impairs the contract itself and cannot be
given retroactive effect without violating the constitutional prohibition against impairment of contracts.
We agree that laws are to be construed as having only prospective operation. Lex prospicit, non respicit.
Equally settled, only laws existing at the time of the execution of a contract are applicable thereto and not later
statutes, unless the latter are specifically intended to have retroactive effect. A later law which enlarges,
abridges, or in any manner changes the intent of the parties to the contract necessarily im pairs the contract
itself and cannot be given retroactive effect without violating the constitutional prohibition against impairment of
contracts.

Same; Same; Same; Same; A law enacted in the exercise of police power to regulate or govern certain activities
or transactions could be given retroactive effect and may reasonably impair vested rights or contractspolice
power legislation is applicable not only to future contracts, but equally to those already in existence. The
foregoing principles do admit of certain exceptions. One involves police power. A law enacted in the exercise of
police power to regulate or govern certain activities or transactions could be given retroactive effect and may
reasonably impair vested rights or contracts. Police power legislation is applicable not only to future contracts,
but equally to those already in existence. Nonimpairment of contracts or vested rights clauses will have to yield
to the superior and legitimate exercise by the State of police power to promote the health, morals, peace,
education, good order, safety, and general welfare of the people. Moreover, statutes in exercise of valid police
power must be read into every contract. Noteworthy, in Sangalang vs. Intermediate Appellate Court, we already
upheld MMC Ordinance No. 81-01 as a legitimate police power measure.

Page 136 of 236


Constitutional Law 2 Bill of Rights Armando Santiago Jr

Same; Same; Same; Same; Restrictions in a contract of sale limiting all constructions on the subject lot to single-
family residential buildings deemed extinguished by the retroactive operation of a zoning ordinance reclassifying
the area as a commercial zone. Following our ruling in Ortigas & Co., Ltd. vs. Feati Bank & Trust Co., 94
SCRA 533 (1979), the contractual stipulations annotated on the Torrens Title, on which Ortigas relies, must yield
to the ordinance. When that stretch of Ortigas Avenue from Roosevelt Street to Madison Street was reclassified
as a commercial zone by the Metropolitan Manila Commission in March 1981, the restrictions in the contract of
sale between Ortigas and Hermoso, limiting all construction on the disputed lot to single-family residential
buildings, were deemed extinguished by the retroactive operation of the zoning ordinance and could no longer
be enforced. While our legal system upholds the sanctity of contract so that a contract is deemed law between
the contracting parties, nonetheless, stipulations in a contract cannot contravene law, morals, good customs,
public order, or public policy. Otherwise such stipulations would be deemed null and void.

MARIVELES VS. CA
PONENCIA: QUISUMBING, J.:
TOPIC: Non-Impairment clause
TRIGGER OF THE FACTS: Private respondents were illegally dismissed from their job because the petitioner was
unsatisfied with their work. (Mariveles is an agency wherein the persons to be dismissed was employed and not employed to
the company who want to dismiss them from service)
TRIGGER OF THE ISSUE: WON petitioner has the liability over respondents
TRIGGER OF THE RULING: the court ruled in the negative. di pwedeng idismiss ng petitioner dahil wala silang right na
idismiss ito dahil under sila ng agency. si agency lang ang pwedeng magtangal sa kanila. accord to the SC ruling, they are
SOLIDARILY LIABLE which is not stated in the contract but this is under the labor code with valid police power they can
impair the contracts as long as it is against public police, order and etc.
VERDICT: the Resolution of the Court of Appeals AFFIRMED with MODIFICATION. Petitioner and Longest Force are held
liable jointly and severally for underpayment of wages and overtime pay of the security guards, without prejudice to
petitioners right of reimbursement from Longest Force Investigation and Security Agency, Inc. The amounts payable to
complaining security guards, herein private respondents, by way of total back wages and attorneys fees are hereby set at
P3,926,100.40 and P392,610.04, respectively. Costs against petitioner.
END POINT: GENERAL RULE Impairment of contract is PROHIBITED. pero inamend ng labor code yung laman ng contract
pero valid yung ginawa dito dahil sabi ng labor code solidarily liablity which is not stated on the contracts. (POLICE POWER
PREVAILS OVER CONTRACT)

*POLICE POWER - inapply dahil may violation ng law which is created by legislative which police power solely vested to the
legislative.

FACTS
Mariveles Shipyard Corporation engaged the services of Longest Force Investigation and Security Agency,
Inc.
to render security services at its premises.
Pursuant to their agreement, Longest Force deployed its security guards,
the private respondents herein, at the petitioners shipyard in Mariveles, Bataan.
According to petitioner, it complied with the terms of the security contract with the petitioner
promptly paying its bills and the contract rates of the latter.
However, it found the services being rendered by the assigned guards unsatisfactory and inadequate,
causing it to terminate its contract with Longest Force.
Longest Force, in turn, terminated the employment of the security guards it had deployed at petitioners
shipyard.
private respondents filed a case for illegal dismissal
underpayment of wages pursuant to the PNPSOSIA-PADPAO rates,
non-payment of overtime pay,
premium pay for holiday and rest day, service incentive leave pay, 13th month pay and attorneys fees,
against both Longest Force and petitioner,
before the Labor Arbiter.
The case sought the guards reinstatement with full back wages and without loss of seniority rights.
Longest Force admitted that it employed private respondents and assigned them as security guards at the
premises of petitioner rendering a 12 hours duty per shift for the said period.
Page 137 of 236
Constitutional Law 2 Bill of Rights Armando Santiago Jr

It likewise admitted its liability as to the non-payment of the alleged wage differential in the total amount of
P2,618,025 but passed on the liability to petitioner
The petitioner denied any liability on account of the alleged illegal dismissal,
stressing that no employer-employee relationship existed between it and the security guards.
It further pointed out that it would be the height of injustice to make it liable again for monetary claims which it
had already paid.
Petitioner averred that Longest Force had benefited from the contract;
The Labor Arbiter rendered judgment that Longest Force and Mariveles Shipping be jointly and severally
liable to pay the money claims of the complainants.
Petitioner appealed the foregoing to the NLRC.
The labor tribunal, affirmed the decision of the Labor Arbiter. Petitioner moved for reconsideration, but this
was denied by the NLRC.
The petitioner then filed a special civil action for certiorari assailing the NLRC judgment for having been
rendered with grave abuse of discretion with the Court of Appeals.
The Court of Appeals denied due course to the petition and dismissed it outright.

ISSUE
WON Longest Force should be held solely and ultimately liable.

HELD
Petitioners liability is joint and several with that of Longest Force, pursuant to Articles 106, 107 and 109 of the
Labor Code which provide as follows:

ART. 106. CONTRACTOR OR SUBCONTRACTOR. Whenever an employer enters into a contract with
another person for the performance of the formers work, the employees of the contractor and of the latters
subcontractor, if any, shall be paid in accordance with the provisions of this Code.

In the event that the contractor or subcontractor fails to pay the wages of his employees in accordance with this
Code, the employer shall be jointly and severally liable with his contractor or subcontractor to such employees to
the extent of the work performed under the contract, in the same manner and extent that he is liable to
employees directly employed by him.

ART. 107. INDIRECT EMPLOYER. The provisions of the immediately preceding Article shall likewise apply to
any person, partnership, association or corporation which, not being an employer, contracts with an independent
contractor for the performance of any work, task, job or project.

ART. 109. SOLIDARY LIABILITY. The provisions of existing laws to the contrary notwithstanding, every
employer or indirect employer shall be held responsible with his contractor or subcontractor for any violation of
any provision of this Code. For purposes of determining the extent of their civil liability under this Chapter, they
shall be considered as direct employers. (jointly - parehas silang pwedeng magbayad whereas in SOLIDARITY
pwedeng isa sa kanila ang magbayad. pero may right sila for reinbursement)

In this case, when petitioner contracted for security services with Longest Force as the security agency that hired
private respondents to work as guards for the shipyard corporation, petitioner became an indirect employer of
private respondents pursuant to Article 107 above cited. Following Article 106, when the agency as contractor
failed to pay the guards, the corporation as principal becomes jointly and severally liable for the guards wages.
This is mandated by the Labor Code to ensure compliance with its provisions, including payment of statutory
minimum wage.

The security agency is held liable by virtue of its status as direct employer, while the corporation is deemed the
indirect employer of the guards for the purpose of paying their wages in the event of failure of the agency to pay
them. This statutory scheme gives the workers the ample protection consonant with labor and social justice
provisions of the 1987 Constitution. Petitioner cannot evade its liability by claiming that it had religiously paid the
compensation of guards as stipulated under the contract with the security agency. Labor standards are enacted
by the legislature to alleviate the plight of workers whose wages barely meet the spiralling costs of their basic
Page 138 of 236
Constitutional Law 2 Bill of Rights Armando Santiago Jr

needs. Labor laws are considered written in every contract. Stipulations in violation thereof are
considered null. Similarly, legislated wage increases are deemed amendments to the contract. Thus,
employers cannot hide behind their contracts in order to evade their (or their contractors or
subcontractors) liability for noncompliance with the statutory minimum wage.

However, the court emphasizes that the solidary liability of petitioner with that of Longest Force does not
preclude the application of the Civil Code provision on the right of reimbursement from his co-debtor by the one
who paid. As held in Del Rosario & Sons Logging Enterprises, Inc. v. NLRC, the joint and several liabilities
imposed on petitioner is without prejudice to a claim for reimbursement by petitioner against the security agency
for such amounts as petitioner may have to pay to complainants, the private respondents herein. The security
agency may not seek exculpation by claiming that the principals payments to it were inadequate for the guards
lawful compensation. As an employer, the security agency is charged with knowledge of labor laws; and the
adequacy of the compensation that it demands for contractual services is its principal concern and not any
others.

SYLLABI
Same; Indirect Employers; Security Guards; Joint and Several Obligations; When the security agency fails to pay
its guards, the corporation serviced, as principal, becomes jointly and severally liable for the guards wages.In
this case, when petitioner contracted for security services with Longest Force as the security agency that hired
private respondents to work as guards for the shipyard corporation, petitioner became an indirect employer of
private respondents pursuant to Article 107 above cited. Following Article 106, when the agency as contractor
failed to pay the guards, the corporation as principal becomes jointly and severally liable for the guards wages.
This is mandated by the Labor Code to ensure compliance with its provisions, including payment of statutory
minimum wage. The security agency is held liable by virtue of its status as direct employer, while the corporation
is deemed the indirect employer of the guards for the purpose of paying their wages in the event of failure of the
agency to pay them. This statutory scheme gives the workers the ample protection consonant with labor and
social justice provisions of the 1987 Constitution.

CLEMONS VS. NOLTING


PONENCIA: JOHNSON, J.:
TOPIC: Non-Impairment clause
TRIGGER OF THE FACTS: there was this law changes the evaluation to something lower
TRIGGER OF THE ISSUE: does the law impaired the contract
TRIGGER OF THE RULING: yes. invalid impairment.
VERDICT:For all of the foregoing facts and the law, we are fully persuaded that the remedy prayed for should be, and
is hereby, granted. And it is hereby ordered and decreed that the writ of mandamus be issued to the defendant
herein, commanding him to countersign, or cause to the countersigned the original of the warrant set forth in
paragraph 9 of the complaint, and to deliver the same to the plaintiff so that he may present it to the Treasurer of the
Philippine Islands and receive payment of said sum of P73.33 due him as averred in the complaint and without any
finding as to costs. So ordered.
END POINT: impairment of contract because it is unfair and less to what is written on the contract. lugi si clemons.

FACTS
1. Clemons was hired in the Philippines;
2. His contract says he would be paid S333;
3. He was paid in Pesos and less than S333 (in exchange rate).

ISSUE
WON there was impairment of contract?

HELD
Page 139 of 236
Constitutional Law 2 Bill of Rights Armando Santiago Jr

Yes. He should have received S333 as stipulated in the contract or at least the Peso equivalent which would
allow him to purchase $333 worth in the US.

SYLLABI
Contracts; Use of Dollar Sign ($), Significance of; A Contract Payable in Dollars Must Be Paid in Dollars or Their
Equivalent. When the dollar sign ($) is used in a written contract made in the United States, it signifies dollars
in the money of the United States, and the contract can be discharged only by the payment of the required
amount in United States money or in Philippine pesos of an equivalent commercial value, unless otherwise
specifically provided in the contract. It would be ruinous to the commercial interests of the Philippine Islands to
declare that the payment of debts of money could be made in other specie than that stipulated in the contract.

PHILRECA VS DILG
PONENCIA: PUNO, J.:
TOPIC: Non-Impairment clause
TRIGGER OF THE FACTS: loan contract
TRIGGER OF THE ISSUE: does PD impaired the contract
TRIGGER OF THE RULING: no it did not because there was no exemption of taxes in the contract
VERDICT:the instant petition is DENIED and the temporary restraining order heretofore issued is LIFTED.
End point of the case: ang issue talaga ng kaso kung sino ang magbabayad ng taxes pero walang exemption di sila
tatamaan nung pd 269.

PHILRECA CASE WAS EXPLAINED BY JUDGE REYES


*No tax exemption, tax remains due
*but lender pays nothing, as borrower payment of taxes
*law which removes tax exemption of electric cooperatives does not impair loan contract

*there is no tax exemption kahit na may PD 269 amendment which granted sa LGU CODE which removes the
tax exemptions enjoyed by electric corporation
*walang impairment of contract kasi wala namang exemption of taxes stated in contract ang meron lang ay kung
sino ang magbabayad ng taxes. dito sa case RP ang magbabayad.

FACTS
Under Presidential Decree (PD) 269, as amended, or the National Electrification Administration Decree, it is the
declared policy of the State to provide the total electrification of the Philippines on an area coverage basis the
same being vital to the people and the sound development of the nation.

Pursuant to this policy, PD 269 aims to promote, encourage and assist all public service entities engaged in
supplying electric service, particularly electric cooperatives by giving every tenable support and assistance to
the electric cooperatives coming within the purview of the law.

From 1971 to 1978, in order to finance the electrification projects envisioned by PD 269, as amended, the
Philippine Government, acting through the National Economic Council (now National Economic Development
Authority) and the NEA (National Electrification Administration), entered into 6 loan agreements with the
government of the United States of America through the United States Agency for International Development
(USAID) with electric cooperatives, including Agusan Del Norte Electric Cooperative, Inc. (ANECO); Iloilo I
Electric Cooperative, Inc. (ILECO I); and Isabela I Electric Cooperative, Inc. (ISELCO I), as beneficiaries.

The 6 loan agreements involved a total amount of approximately US$86,000,000.00. These loan agreements are
existing until today.

Page 140 of 236


Constitutional Law 2 Bill of Rights Armando Santiago Jr

The loan agreements contain similarly worded provisions on the tax application of the loan and any property or
commodity acquired through the proceeds of the loan.

On 23 May 2000, a class suit was filed by the Philippine Rural Electric Cooperatives Association, Inc.
(PHILRECA); ANECO, ILECO I and ISELCO I; in their own behalf and in behalf of other electric cooperatives
organized and existing under PD 269, against the Secretary of the Department of Interior and Local Government
(DILG) and the Secretary of the Department of Finance, through a petition for prohibition, contending that
pursuant to the provisions of PD 269, as amended, and the provision in the loan agreements, they are exempt
from payment of local taxes, including payment of real property tax.

With the passage of the Local Government Code, however, they allege that their tax exemptions have been
invalidly withdrawn, in violation of the equal protection clause and impairing the obligation of contracts between
the Philippine Government and the United States Government.

ISSUE
Whether or not the Local Government Code unduly discriminated against electric cooperatives organized and
existing under PD 269 on the ground that it violated the equal protection clause.

HELD
The equal protection clause under the Constitution means that no person or class of persons shall be deprived
of the same protection of laws which is enjoyed by other persons or other classes in the same place and in like
circumstances. Thus, the guaranty of the equal protection of the laws is not violated by a law based on
reasonable classification.

Classification, to be reasonable, must (1) rest on substantial distinctions; (2) be germane to the purposes of the
law; (3) not be limited to existing conditions only; and (4) apply equally to all members of the same class.

There is reasonable classification under the Local Government Code to justify the different tax treatment
between electric cooperatives covered by PD 269, as amended, and electric cooperatives under RA 6938
(Cooperative Code of the Philippines).

First, nowhere in PD 269, as amended, does it require cooperatives to make equitable contributions to capital.
Under the Cooperative Code, the articles of cooperation of a cooperative applying for registration must be
accompanied with the bonds of the accountable officers and a sworn statement of the treasurer elected by the
subscribers showing that at least 25% of the authorized share capital has been subscribed and at least 25% of
the total subscription has been paid and in no case shall the paid-up share capital be less than P2,000.00.

Second, another principle adhered to by the Cooperative Code is the principle of subsidiarity. Pursuant to this
principle, the government may only engage in development activities where cooperatives do not possess the
capability nor the resources to do so and only upon the request of such cooperatives. In contrast, PD 269, as
amended by PD 1645, is replete with provisions which grant the NEA, upon the happening of certain events, the
power to control and take over the management and operations of cooperatives registered under it. The extent
of government control over electric cooperatives covered by PD 269, as amended, is largely a function of the
role of the NEA as a primary source of funds of these electric cooperatives. It is crystal clear that NEA incurred
loans from various sources to finance the development and operations of the electric cooperatives.

Consequently, amendments to PD 269 were primarily geared to expand the powers of the NEA over the electric
cooperatives to ensure that loans granted to them would be repaid to the government. In contrast, cooperatives
under RA 6938 are envisioned to be self-sufficient and independent organizations with minimal government
intervention or regulation.

Lastly, the transitory provisions of RA 6938 are indicative of the recognition by Congress of the fundamental
distinctions between electric cooperatives organized under PD 269, as amended, and cooperatives under the
new Cooperative Code.

Page 141 of 236


Constitutional Law 2 Bill of Rights Armando Santiago Jr

Article 128 of the Cooperative Code provides that all cooperatives registered under previous laws shall be
deemed registered with the CDA upon submission of certain requirements within one year. However,
cooperatives created under PD 269, as amended, are given three years within which to qualify and register with
the CDA, after which, provisions of PD 1645 which expand the powers of the NEA over electric cooperatives,
would no longer apply.

SYLLABI
Same; Same; Constitutional Law; Non-Impairment Clause; It is ingrained in jurisprudence that the constitutional
prohibition on the impairment of the obligation of contracts does not prohibit every change in existing laws. To fall
within the prohibition, the change must not only impair the obligation of the existing contract, but the impairment
must be substantial. It is ingrained in jurisprudence that the constitutional prohibition on the impairment of the
obligation of contracts does not prohibit every change in existing laws. To fall within the prohibition, the change
must not only impair the obligation of the existing contract, but the impairment must be substantial. What
constitutes substantial impairment was explained by this Court in Clemons v. Nolting: A law which changes the
terms of a legal contract between parties, either in the time or mode of performance, or imposes new conditions,
or dispenses with those expressed, or authorizes for its satisfaction something different from that provided in its
terms, is law which impairs the obligation of a contract and is therefore null and void. Moreover, to constitute
impairment, the law must affect a change in the rights of the parties with reference to each other and not with
respect to non-parties.

Page 142 of 236


Constitutional Law 2 Bill of Rights Armando Santiago Jr

Access/Right to information
The right of the people to information on matters of public concern
shall be recognized. Access to official records, and to documents,
and papers pertaining to official acts, transactions, or decisions, as
well as to government research data used as basis for policy
development, shall be afforded the citizen, subject to such limitations
as may be provided by law
(Section 7, Art. III)
The standards that have been developed for the regulation of speech and press and assembly and petition and
of association are applicable to the right of access to information. Those are the dangerous tendency rule, the
clear and present danger and the balancing of interest test.

SUBIDO V. OZAETA
the citizen obiously has a right to know what is going on his country and in his government.

i.e. if he is denied examination of official vouchers. A citizen may not expose anomaly if those responsible for it
may validly prevent him from investigating their activities.

Art. 6, section 16(4)


Requiring publication of the legislative journals from time to time excepting such parts as may, in the judgement
from time to time excepting such parts as may, in the judgement of the house, effect national national security.

Art. 6, section 20
The records and books of accounts of the congress shall be open to the public in accordance with law, and such
books shall be audited by the comission on Audit which shall publish annually an itemized list of amounts paid
and expenses incurred for each member.

Page 143 of 236


Constitutional Law 2 Bill of Rights Armando Santiago Jr

Art. II Section 28

Subject to reasonable conditions prescribed by law, the State adopts
and implements a policy of full public disclosure of all its
transactions involving public interest.

Art. XVI Section 10


The State shall provide the policy environment for the full
development of Filipino capability and the emergence of
communication structures suitable to the needs and aspirations of
the nation and the balanced flow of information into, out of, and
across the country, in accordance with a policy that respects the
freedom of speech and of the press.

SCOPE
Right to information contemplates inclusion of negotiations leading to the consummation of the transaction.
Otherwise, the people can never exercise the right if no contract is consummated, or if one is consummated, it
may be too late for the public to expose its defects. However, if the right only affords access to records,
documents and papers, which means the opportunity to inspect and copy them at his expense. The exercise is
also subject to reasonable regulations to protect the integrity of public records and to minimize disruption of
government operations. [Chavez v. PEA and Amarii29 ]

LIMITATIONS
The right does not extend to matters recognized as privileged information rooted in separation of powers, nor to
information on military and diplomatic secrets, information affecting national security, and information on
investigations of crimes by law enforcement agencies before the prosecution of the accused. [Chavez v. PEA
and Amari30 ]

Media practitioners requested information from the GM of GSIS regarding clean loans granted to certain
members of the defunct Batasang Pambansa on the guaranty of Imelda Marcos shortly before the Feb 1986
elections. Request was refused on the ground of confidentiality.

The right to information is not absolute. It is limited to matters of public concern and is subject to such limitations
as may be provided by law. That the GSIS was exercising a proprietary function would not justify its exclusion of
the transactions from the coverage of the right to info. But although citizens have such right and, pursuant
thereto, are entitled to access to official records, the Constitution does not accord them the right to compel
custodians of official records to prepare lists, summaries and the like in their desire to get info on matters of
public concern. [Valmonte vs. Belmonte (1989)]

Confidential information means information not yet made a matter of public record relating to pending cases, as
well as information not yet made public concerning the work of any justice or judge relating to pending cases,

29 G.R. No. 133250, July 9, 2002

30 Supra
Page 144 of 236
Constitutional Law 2 Bill of Rights Armando Santiago Jr

including notes, drafts, research papers, internal discussions, internal memoranda, records of internal
deliberations and similar papers.

The notes, drafts, research papers, internal discussions, internal memoranda, records of internal deliberations
and similar papers that a justice or judge uses in preparing a decision, resolution or order shall remain
confidential even after the decision, resolution or order is made public.

Decisions are matters of public concern and interest. Pleadings and other documents filed by parties to a case
need not be matters of public concern or interest. They are filed for the purpose of establishing the basis upon
which the court may issue an order or a judgment affecting their rights and interest. Access to court records may
be permitted at the discretion and subject to the supervisory and protective powers of the court, after considering
the actual use or purpose for which the request for access is based and the obvious prejudice to any of the
parties. [Hilado, et al vs Judge (2006)]

RIGHT TO INFORMATION RELATIVE TO GOVERNMENT CONTRACT


NEGOTIATIONS
The constitutional right to information includes official information on on-going negotiations before a final contract.
The information, however, must constitute definite propositions by the government, and should not cover
recognized exceptions. [Chavez v. Philippine Estate Authority (2002)]
The limitations recognized to the right of information are:

(1) National security matter including state secrets regarding military and diplomatic matters, inter-
government exchanges prior to the conclusion of treaties and executive agreements.

(2) Trade secrets and banking transactions

(3) Criminal Matters

(4) Other confidential matters. [Neri vs Senate

(2008) citing Chavez vs President Commission on Good Government]

DIPLOMATIC NEGOTIATIONS
Diplomatic negotiations have a privileged character. [Akbayan vs Aquino cited in Neri vs Senate (2008)]

COURT HEARINGS
When the constitutional guarantees of freedom of the press and the right to public information, on the one hand,
and the fundamental rights of the accused, on the other hand, along with the constitutional power of a court to
control its proceedings in ensuring a fair and impartial trial race against another, jurisprudence tells us that the
right of the accused must be preferred to win. With the possibility of losing not only the precious liberty but also
the very life of an accused, it behooves all to make absolutely certain that an accused receives a verdict solely
on the basis of a just and dispassionate judgment, a verdict that would come only after the presentation of
credible evidence testified to by unbiased witnesses unswayed by any kind of pressure, whether open or subtle,
in proceedings that are devoid of histrionics that might detract from its basic aim to ferret veritable facts free from
improper influence, and decreed by a judge with an unprejudiced mind unbridled by running emotions or
passions. [Re: Request for Live Radio-TV Coverage of the Trial in the Sandiganbayan of the Plunder
Cases against former President Joseph Ejercito Estrada, Secretary of Justice Hernando Perez v. Joseph
Ejercito Estrada, A.M. No. 00-1-4-03-SC, June 29, 2001]

Page 145 of 236


Constitutional Law 2 Bill of Rights Armando Santiago Jr

CASES

CHAVEZ VS. PUBLIC ESTATES AUTHORITY31


PONENCIA: CARPIO, J.:
TOPIC: Right to Information
TRIGGER OF THE FACTS: PEA and AMARI entered into a contract which was not disclosed in the public.
TRIGGER OF THE ISSUE:
TRIGGER OF THE RULING: right to information may be applied when the agreement was binded by both private and
government
VERDICT: WHEREFORE, the petition is GRANTED. The Public Estates Authority and Amari Coastal Bay
Development Corporation are PERMANENTLY ENJOINED from implementing the Amended Joint Venture Agreement
which is hereby declared NULL and VOID ab initio.
End point of the case: Public has the right to information. contracts or agreement between the government and
private corporation shall be accessible to the public. so that public may see if the law was violated.

CHAVES CASE WAS EXPLAINED BY JUDGE REYES

Government office 1 and 2 will have an exchange evaluation of the practicality and substance of the contract.
Public has no right to demand an information from the evaluation contemplated by government office 1 and 2.
for it is confidential and information solely for government.
after the said evaluation, they will give a concrete proposal to the contractor
on this part. this concrete proposal can be seen by the public and demand for an information to see if this
contract did not violate any law or etc. dahil dito daw nagsisimula ang corruption kaya public has the right to
information.
allegedly, PEA at AMARI did not expose those information by public.

FACTS
The President Ferdinand E. Marcos issued Presidential Decree No. 1084 creating PEA32.
tasked PEA "to reclaim land, including foreshore and submerged areas," and "to develop, improve, acquire,
lease and sell any and all kinds of lands."
On the same date, then President Marcos issued Presidential Decree No. 1085 transferring to PEA the "lands
reclaimed in the foreshore and offshore of the Manila Bay" under the MCCRRP33
President Corazon C. Aquino issued Special Patent No. 3517,
granting and transferring to PEA "the parcels of land so reclaimed under the MCCRRP containing a total area
of 1,915,894 square meters.
Subsequently, the Register of Deeds of the Municipality of Paraaque issued Transfer Certificates of Title in
the name of PEA
covering the three reclaimed islands known as the "Freedom Islands" located at the southern portion of the
MCCRP

31 403 SCRA 1, G.R. No. 133250 May 6, 2003

32 Public Estate Authority

33 Manila-Cavite Coastal Road and Reclamation Project


Page 146 of 236
Constitutional Law 2 Bill of Rights Armando Santiago Jr

PEA and AMARI entered into the JVA 34 through negotiation without public bidding.
the Board of Directors of PEA confirmed the JVA, then President Fidel V. Ramos approved the JVA.

INTERACTION
The Senate Committees reported the results of their investigation in Senate Committee Report
Among the conclusions of their report are:
(1) the reclaimed lands PEA seeks to transfer to AMARI under the JVA are lands of the public domain
which the government has not classified as alienable lands and therefore PEA cannot alienate these
lands;
(2) the certificates of title covering the Freedom Islands are thus void, and
(3) the JVA itself is illegal.
President Fidel V. Ramos issued Presidential Administrative Order creating a Legal Task Force to conduct a
study on the legality of the JVA in view of Senate Committee Report
The Legal Task Force upheld the legality of the JVA, contrary to the conclusions reached by the Senate
Committees.
petitioner Frank I. Chavez as a taxpayer, filed the instant Petition35.
Petitioner contends the government stands to lose billions of pesos in the sale by PEA of the reclaimed
lands to AMARI.
Petitioner prays that PEA publicly disclose the terms of any renegotiation of the JVA, invoking Section 28,
Article II, and Section 7, Article III, of the 1987 Constitution on the right of the people to information on
matters of public concern.
Due to the approval of the Amended JVA by the Office of the President, petitioner now prays that on
"constitutional and statutory grounds the renegotiated contract be declared null and void.

ISSUE
The issues raised by petitioner, PEA and AMARI are as follows:
1. Whether the reliefs prayed for are moot and academic because of subsequent events;
2. Whether the petition should be dismissed for failing to observe the principle of governing the heirarchy of
courts;
3. Whether the petition should be dismissed for non-exhaustion of administrative remedies;
4. Whether petitioner has locus standi;
5. Whether the constitutional right to information includes information on on-going neogtiations
BEFORE a final agreement;
6. Whether the stipulations in the amended joint venture agreement for the transfer to AMARI of certain
lands, reclaimed and still to be reclaimed violate the 1987 Constitution; and
7. Whether the Court has jurisdiction over the issue whether the amended JVA is grossly disadvantageous
to the government


HELD

ISSUE 1
We rule that the signing and of the Amended JVA by PEA and AMARI and its approval by the President cannot
operate to moot the petition and divest the Court of its jurisdiction.

PEA and AMARI have still to implement the Amended JVA. The prayer to enjoin the signing of the Amended
JVA on constitutional grounds necessarily includes preventing its implementation if in the meantime PEA and
AMARI have signed one in violation of the Constitution. Petitioner's principal basis in assailing the renegotiation

34 Joint Venture Agreement

35 Petition for Mandamus with Prayer for the Issuance of a Writ of Preliminary Injunction and Temporary Restraining Order
Page 147 of 236
Constitutional Law 2 Bill of Rights Armando Santiago Jr

of the JVA is its violation of the Section 3, Article XII of the Constitution, which prohibits the government from
alienating lands of the public domain to private corporations. The Amended JVA is not an ordinary commercial
contract but one which seeks to transfer title and ownership to 367.5 hectares of reclaimed lands and
submerged areas of Manila Bay to a single private corporation.

Also, the instant petition is a case of first impression being a wholly government owned corporation
performing public as well as proprietary functions. All previous decisions of the Court involving Section 3, Article
XII of the 1987 Constitution, or its counterpart provision in the 1973 Constitution, covered agricultural lands sold
to private corporations which acquired the lands from private parties.

Lastly, there is a need to resolve immediately the constitutional issue raised in this petition because of the
possible transfer at any time by PEA to AMARI of title and ownership to portions of the reclaimed lands. Under
the Amended JVA, PEA is obligated to transfer to AMARI the latter's seventy percent proportionate share in the
reclaimed areas as the reclamation progresses, The Amended JVA even allows AMARI to mortgage at any time
the entire reclaimed area to raise financing for the reclamation project.

ISSUE 2
The instant case, however, raises constitutional issues of transcendental importance to the public. The Court can
resolve this case without determining any factual issue related to the case. Also, the instant case is a petition for
mandamus which falls under the original jurisdiction of the Court under Section 5, Article VIII of the Constitution.
We resolve to exercise primary jurisdiction over the instant case.

ISSUE 3
PEA was under a positive legal duty to disclose to the public the terms and conditions for the sale of its lands.
The law obligated PEA make this public disclosure even without demand from petitioner or from anyone. PEA
failed to make this public disclosure because the original JVA, like the Amended JVA, was the result of a
negotiated contract, not of a public bidding. Considering that PEA had an affirmative statutory duty to make the
public disclosure, and was even in breach of this legal duty, petitioner had the right to seek direct judicial
intervention.

The principle of exhaustion of administrative remedies does not apply when the issue involved is a purely
legal or constitutional question. The principal issue in the instant case is the capacity of AMARI to acquire lands
held by PEA in view of the constitutional ban prohibiting the alienation of lands of the public domain to private
corporations. We rule that the principle of exhaustion of administrative remedies does not apply in the instant
case.

The petitioner has standing to bring this taxpayer's suit because the petition seeks to compel PEA to comply
with its constitutional duties. There are two constitutional issues involved here. First is the right of citizens to
information on matters of public concern. Second is the application of a constitutional provision intended to
insure the equitable distribution of alienable lands of the public domain among Filipino Citizens.
The thrust of the second issue is to prevent PEA from alienating hundreds of hectares of alienable lands of the
public domain in violation of the Constitution, compelling PEA to comply with a constitutional duty to the nation.

ISSUE 4
Ordinary taxpayers have a right to initiate and prosecute actions questioning the validity of acts or orders of
government agencies or instrumentalities, if the issues raised are of 'paramount public interest,' and if they
'immediately affect the social, economic and moral well-being of the people.'

We rule that since the instant petition, brought by a citizen, involves the enforcement of constitutional rights
to information and to the equitable diffusion of natural resources matters of transcendental public
importance, the petitioner has the requisite locus standi.

ISSUE 5
Page 148 of 236
Constitutional Law 2 Bill of Rights Armando Santiago Jr

The State policy of full transparency in all transactions involving public interest reinforces the people's right to
information on matters of public concern. This State policy is expressed in Section 28, Article II of the
Constitution, thus: Subject to reasonable conditions prescribed by law, the State adopts and implements a
policy of full public disclosure of all its transactions involving public interest."

Contrary to AMARI's contention, the commissioners of the 1986 Constitutional Commission understood that
the right to information "contemplates inclusion of negotiations leading to the consummation of the transaction."
Certainly, a consummated contract is not a requirement for the exercise of the right to information. Otherwise,
the people can never exercise the right if no contract is consummated, and if one is consummated, it may be too
late for the public to expose its defects.

Requiring a consummated contract will keep the public in the dark until the contract, which may be grossly
disadvantageous to the government or even illegal, becomes a fait accompli.

However, the right to information does not compel PEA to prepare lists, abstracts, summaries and the like
relating to the renegotiation of the JVA. 34 The right only affords access to records, documents and papers,
which means the opportunity to inspect and copy them. One who exercises the right must copy the records,
documents and papers at his expense. The exercise of the right is also subject to reasonable regulations to
protect the integrity of the public records and to minimize disruption to government operations, like rules
specifying when and how to conduct the inspection and copying.

ISSUE 6
Article 339 of the Civil Code of 1889 defined property of public dominion as follows:
"Art. 339. Property of public dominion is
1. That devoted to public use, such as roads, canals, rivers, torrents, ports and bridges constructed by the
State, riverbanks, shores, roadsteads, and that of a similar character;
2. That belonging exclusively to the State which, without being of general public use, is employed in some
public service, or in the development of the national wealth, such as walls, fortresses, and other works for the
defense of the territory, and mines, until granted to private individuals.

Property devoted to public use referred to property open for use by the public. In contrast, property devoted
to public service referred to property used for some specific public service and open only to those authorized to
use the property.Property of public dominion referred not only to property devoted to public use, but also to
property not so used but employed to develop the national wealth. This class of property constituted property of
public dominion although employed for some economic or commercial activity to increase the national wealth.

"Art. 341. Property of public dominion, when no longer devoted to public use or to the defense of the
territory, shall become a part of the private property of the State." This provision, however, was not self-
executing. The legislature, or the executive department pursuant to law, must declare the property no longer
needed for public use or territorial defense before the government could lease or alienate the property to private
parties.

Act No. 2874 of the Philippine Legislature


Sec. 55. Any tract of land of the public domain which, being neither timber nor mineral land, shall be
classified as suitable for residential purposes or for commercial, industrial, or other productive purposes other
than agricultural purposes, and shall be open to disposition or concession, shall be disposed of under the
provisions of this chapter, and not otherwise.

The rationale behind this State policy is obvious. Government reclaimed, foreshore and marshy public lands
for non-agricultural purposes retain their inherent potential as areas for public service. This is the reason the
government prohibited the sale, and only allowed the lease, of these lands to private parties. The State always
reserved these lands for some future public service.

However, government reclaimed and marshy lands, although subject to classification as disposable public
agricultural lands, could only be leased and not sold to private parties because of Act No. 2874.

Page 149 of 236


Constitutional Law 2 Bill of Rights Armando Santiago Jr

The 1987 Constitution continues the State policy in the 1973 Constitution banning private corporations from
acquiring any kind of alienable land of the public domain. Like the 1973 Constitution, the 1987 Constitution
allows private corporations to hold alienable lands of the public domain only through lease. As in the 1935 and
1973 Constitutions, the general law governing the lease to private corporations of reclaimed, foreshore and
marshy alienable lands of the public domain is still CA No. 141.

Without the constitutional ban, individuals who already acquired the maximum area of alienable lands of the
public domain could easily set up corporations to acquire more alienable public lands. An individual could own as
many corporations as his means would allow him. An individual could even hide his ownership of a corporation
by putting his nominees as stockholders of the corporation. The corporation is a convenient vehicle to
circumvent the constitutional limitation on acquisition by individuals of alienable lands of the public domain.

PD No. 1085, coupled with President Aquino's actual issuance of a special patent covering the Freedom
Islands, is equivalent to an official proclamation classifying the Freedom Islands as alienable or disposable lands
of the public domain. Being neither timber, mineral, nor national park lands, the reclaimed Freedom Islands
necessarily fall under the classification of agricultural lands of the public domain. Under the 1987 Constitution,
agricultural lands of the public domain are the only natural resources that the State may alienate to qualified
private parties. All other natural resources, such as the seas or bays, are "waters . . . owned by the State"
forming part of the public domain, and are inalienable pursuant to Section 2, Article XII of the 1987 Constitution.

In short, DENR is vested with the power to authorize the reclamation of areas under water, while PEA is
vested with the power to undertake the physical reclamation of areas under water whether directly or through
private contractors. DENR is also empowered to classify lands of the public domain into alienable or disposable
lands subject to the approval of the President. On the other hand, PEA is tasked to develop, sell or lease the
reclaimed alienable lands of the public domain.

Clearly, the mere physical act of reclamation by PEA of foreshore or submerged areas does not make the
reclaimed lands alienable or disposable lands of the public domain, much less patrimonial lands of PEA.
Likewise, the mere transfer by the National Government of lands of the public domain to PEA does not make the
lands alienable or disposable lands of the public domain, much less patrimonial lands of PEA.

There is no express authority under either PD No. 1085 or EO No. 525 for PEA to sell its reclaimed lands. PD
No. 1085 merely transferred "ownership and administration" of lands reclaimed from Manila Bay to PEA, while
EO No. 525 declared that lands reclaimed by PEA "shall belong to or be owned by PEA." PEA's charter,
however, expressly tasks PEA "to develop, improve, acquire, administer, deal in, subdivide, dispose, lease and
sell any and all kinds of lands . . . owned, managed, controlled and/or operated by the government." 87
(Emphasis supplied) There is, therefore, legislative authority granted to PEA to sell its lands, whether patrimonial
or alienable lands of the public domain. PEA may sell to private parties its patrimonial properties in accordance
with the PEA charter free from constitutional limitations. The constitutional ban on private corporations from
acquiring alienable lands of the public domain does not apply to the sale of PEA's patrimonial lands.

Moreover, under Section 79 of PD No. 1445, otherwise known as the Government Auditing Code, the
government is required to sell valuable government property through public bidding. Section 79 of PD No. 1445
mandates that:... "In the event that the public auction fails, the property may be sold at a private sale at such
price as may be fixed by the same committee or body concerned and approved by the Commission."

However, the original JVA dated April 25, 1995 covered not only the Freedom Islands and the additional 250
hectares still to be reclaimed, it also granted an option to AMARI to reclaim another 350 hectares. The original
JVA, a negotiated contract, enlarged the reclamation area to 750 hectares. The failure of public bidding on
December 10, 1991, involving only 407.84 hectares, is not a valid justification for a negotiated sale of 750
hectares, almost double the area publicly auctioned.

Jurisprudence holding that upon the grant of the patent or issuance of the certificate of title the alienable land
of the public domain automatically becomes private land cannot apply to government units and entities like PEA.

Page 150 of 236


Constitutional Law 2 Bill of Rights Armando Santiago Jr

The grant of legislative authority to sell public lands in accordance with Section 60 of CA No. 141 does not
automatically convert alienable lands of the public domain into private or patrimonial lands. The alienable lands
of the public domain must be transferred to qualified private parties, or to government entities not tasked to
dispose of public lands, before these lands can become private or patrimonial lands. Otherwise, the
constitutional ban will become illusory if Congress can declare lands of the public domain as private or
patrimonial lands in the hands of a government agency tasked to dispose of public lands.

To allow vast areas of reclaimed lands of the public domain to be transferred to PEA as private lands will
sanction a gross violation of the constitutional ban on private corporations from acquiring any kind of alienable
land of the public domain. This scheme can even be applied to alienable agricultural lands of the public domain
since PEA can "acquire . . . any and all kinds of lands."

The 157.84 hectares of reclaimed lands comprising the Freedom Islands, now covered by certificates of title
in the name of PEA, are alienable lands of the public domain. PEA may lease these lands to private corporations
but may not sell or transfer ownership of these lands to private corporations.

ISSUE 7
Considering that the Amended JVA is null and void ab initio, there is no necessity to rule on this last issue.
Besides, the Court is not the trier of facts, and this last issue involves a determination of factual matters.

SYLLABI
Right to Information; Nothing can be more empowering than to compel access to all information relevant to the
negotiation of government contracts including but not limited to the negotiation of government contracts including
but not limited to evaluation reports, recommendations, legal and expert opinions, minutes of meetings, terms of
reference and other documents attached to such reports or minutes, all relating to any proposed undertaking.
First, my concurrence. I am happy that this Court has stuck to a civil libertarians honesty and transparency in
government service when interpreting the ambit of the peoples right to information on matters of public concern.
Nothing can be more empowering on this aspect than to compel access to all information relevant to the
negotiation of government contracts including but not limited to evaluation reports, recommendations, legal and
expert opinions, minutes of meetings, terms of reference and other documents attached to such reports or
minutes, all relating to any proposed undertaking. This to me encourages our people to watch closely the
proprietary acts of State functionaries which more often than not, because they have been cloaked in technical
jargon and speculation due to the absence of verifiable resource materials, have been left unaccounted for
public debate and searching inquiry.

Page 151 of 236


Constitutional Law 2 Bill of Rights Armando Santiago Jr

Legal Assistance and Free Access


to Courts
Art. 3, Section 11
Free access to the courts and quasi-judicial bodies and adequate
legal assistance shall not be denied to any person by reason of
poverty.

Art. VIII, Section 5 (5) (Mandated by Constitution)


Promulgate rules concerning the protection and enforcement of
constitutional rights, pleading, practice, and procedure in all courts,
the admission to the practice of law, the integrated bar, and legal
assistance to the under- privileged. Such rules shall provide a
simplified and inexpensive procedure for the speedy disposition of
cases, shall be uniform for all courts of the same grade, and shall not
diminish, increase, or modify substantive rights. Rules of procedure
of special courts and quasi-judicial bodies shall remain effective
unless disapproved by the Supreme Court.

RA 9406 (PUBLIC ATTORNEYS OFFICE LAW - PAO)

Section 14 of chapter 5 title 3 book 4


PAO shall be the principle law office for the government in extending free legal assistance indigent persons
ONLY in criminal, civil, labor, administrative and other quasi judicial cases.

PAO clients - exempted from paying docket fees36

Privileges of PAO Lawyers have given the frunking privileges (in other words libreng mail, pwede silang
magsend ng registered mail hindi sila magbabayad) include legal representation in litigation, assistance and
counselling even if they do not hold the case.

QUALIFIED FOR PAO SERVICE (LIMITATION)


Indigent - Metero manila earning 14,000 a month
other cities (ex. Malolos) - earning below 13,000 a month
other town - earning below 12,000 a month

MERIT TEST
Only if there handle of the case furthers the interest of justice (if the case is guilty mostly the PAO will not handle
the case)

36 Docket fees are the court records


Page 152 of 236
Constitutional Law 2 Bill of Rights Armando Santiago Jr

Rules of Court Rule 141 Sec. 18


Indigent litigants exempt from payment of legal fees.Indigent
litigants (a) whose gross income and that of their immediate family do
not exceed four thousand (P4,000.00) pesos a month if residing in
Metro Manila, and three thousand (P3,000.00) pesos a month if
residing outside Metro Manila, and (b) who do not own real property
with an assessed value of more than fifty thousand (P50,000.00)
pesos shall be exempt from the payment of legal fees.
The legal fees shall be a lien on any judgment rendered in the case favorably to the indigent litigant, unless the
court otherwise provides.

To be entitled to the exemption herein provided, the litigant shall execute an affidavit that he and his immediate
family do not earn a gross income abovementioned, nor they own any real property with the assessed value
aforementioned, supported by an affidavit of a disinterested person attesting to the truth of the litigant's affidavit.
Any falsity in the affidavit of a litigant or disinterested person shall be sufficient cause to strike out the pleading of
that party, without prejudice to whatever criminal liability may have been incurred.

Rules of Court Rule 3 Sec. 21


Indigent party.A party may be authorized to litigate his action, claim
or defense as an indigent if the court, upon an ex parte application
and hearing, is satisfied that the party is one who has no money or
property sufficient and available for food, shelter and basic
necessities for himself and his family.

Such authority shall include an exemption from payment of docket and other lawful fees, and of transcripts of
stenographic notes (TSN) which the court may order to be furnished him.

The amount of the docket and other lawful fees which the indigent was exempted from paying shall be a lien on
any judgment rendered in the case favourable to the indigent, unless the court otherwise provides.

Any adverse party may contest the grant of such authority at any time before judgment is rendered by the trial
court. If the court should determine after hearing that the party declared as an indigent is in fact a person with
sufficient income or property, the proper docket and other lawful fees shall be assessed and collected by the
clerk of court. If the payment is not made within the time fixed by the court, execution shall issue or the payment
thereof, without prejudice to such other sanctions as the court may impose.

Those protected include low paid employees, domestic servants and laborers. [Cabangis v. Almeda Lopez
(1940)]

The difference between p aupers and i ndigent persons is that the latter are persons who have no property or
sources of income sufficient for their support aside from their own labor though self- supporting when able to
work and in employment.[Acar v. Rosal (1067)]

The new rule applies even to litigation pending at the time of its enactment. The retroactive application of the
new rule has been found to be more in keeping with Section 11 of Article III. The previous rule, denied the right
to litigate as paupers in appellate courts. [Martinez v. People (2000)]

Page 153 of 236


Constitutional Law 2 Bill of Rights Armando Santiago Jr

Note: The significance of having an explicit free access provisions in the Constitution may be gathered from
the rocky road which free access seems to have traveled in American jurisprudence. The American
constitution does not have an explicit free access provision and, hence, its free access doctrine has been
developed as implicit from both the equal protection clause and the due process clause. [BERNAS]

Assistance of Counsel
ELEMENTS OF THE RIGHT TO COUNSEL
(1) Courts duty to inform the accused of right to counsel before being arraigned;
(2) It must ask him if he desires the services of counsel;
(3) If he does, and is unable to get one, the Court must give him one; if the accused wishes to procure
private counsel, the Court must give him time to obtain one.
(4) Where no lawyer is available, the Court may appoint any person resident of the province and of good
repute for probity and ability.

Page 154 of 236


Constitutional Law 2 Bill of Rights Armando Santiago Jr

RIGHT TO BAIL
Section 13
All persons, except those charged with offenses punishable by
reclusion perpetua when evidence of guilt is strong, shall, before
conviction, be bailable by sufficient sureties, or be released on
recognizance37 as may be provided by law. The right to bail shall not
be impaired even when the privilege of the writ of habeas corpus is
suspended. Excessive bail shall not be required.
Bail is the security (guarantee or deposit) and not a payment given for the release of a person in custody of the
law, furnished by him or a bondsman, conditioned upon his appearance before any court as may be required.

only the person under detention or in custody may petition for bail for the purpose of bail is to secure
their provisional release.

EXCEPTION TO THE RIGHT TO BAIL


1. Accused charged of offense punishable by reclusion perpetua AND the evidence of guilt is strong
(EXCEPTION TO THE EXCEPTION: even if the accused was charged of punishment reclusion perpetua" if
the evidence of guilt is NOT STRONG that person may petition for bail or he may be given the right to bail)
2. convicts has no right to bail (EXCEPTIONS TO THE EXCEPTION: MTC convicts (convicted in first level
courts MTC, MCTC, MeTC, MTCC) they may petition for bail as a matter of RIGHT, a person may demand a
bail even id that person was convicted because the person convicted may only be given by the discretion of
the court and the convicted person can NOT demand for it)
3. Crimes punishable by reclusion perpetua (the constitution is silent when it comes to death penalty for it was
suspended) DILLEMA would be, if the death penalty will be reinstated the constitution should be amended
because 1987 constitution did not provide bail for death penalty. 1987 was specific and before it was
CAPITAL PUNISHMENT comprises of death and RP it must be bailable for it is more serious)

FORMS OF BAIL
1. Surety Bail - Pyansador (payment of premium)
2. offer of the property (property bond) - kapag di ka sumipot kukunin ng court yung property na inoffer mo
3. Cash Bond - magbabayad ng cash sa pay clerk to the clerk of court (magre-retrive yung nakadeposit na cash
bond whether
4. recognizance - it is allowed in certain instances (under custody of a prominent person) it is a species of
bailbond or security, given by the prisoner either on being bound over for trial on his taking an appeal

PRESUMPTION OF INNOCENCE

37 Recognizance - a legal promise made by someone before a court of law that must be kept to avoid being punished
Page 155 of 236
Constitutional Law 2 Bill of Rights Armando Santiago Jr

Section 14 (2) 1st Premise

In all criminal prosecutions, the accused shall be presumed innocent


until the contrary is proved,
Accusation is not synonymous with guilt38 the accused is presumed innocent until the contrary is proved

RIGHT TO BE HEARD

Section 14 (2) 2nd Premise


and shall enjoy the right to be heard by himself and counsel, to be
informed of the nature and cause of the accusation against him, to
have a speedy, impartial, and public trial, to meet the witnesses face
to face, and to have compulsory process to secure the attendance of
witnesses and the production of evidence in his behalf. However,
after arraignment, trial may proceed notwithstanding the absence of
the accused provided that he has been duly notified and his failure to
appear is unjustifiable.
This right is indispensable in any criminal prosecution where the stakes are the liberty or even the life of the
accused, who must for this reason be given a chance to defend himself
In the case of People vs. Lamague, the supreme court set a side the conviction of 3 co-accused after finding that
they were denied due process because they were not given a chance to testify and to present an evidence in
their behalf.

ASSISTANCE OF COUNSEL

it has previously been observed that the right to counsel now begins from the time a person is taken into custody
and placed under investigation for the commission of a crime.

The duty of the court is not ended with such appointment, as it should also see to it that the counsel does his
duty by the defendant. counsel de officio should not merely make the motions of defending the accused but
exert his utmost efforts as if he were representing a paying client.

TRIAL IN ABSENTIA

in the course of his trial the defendant escaped or otherwise failed or refused to appear.

REQUISITE OF TRIAL IN ABSENTIA

1. the accused has already been arraigned


2. he has been duly notified of the trial
3. his failure to appear is unjustified

38 people vs. dramayo, 42 SCRA 59


Page 156 of 236
Constitutional Law 2 Bill of Rights Armando Santiago Jr

RIGHT TO CONFRONTATION
it has also been held that if a prosecution witness dies before his cross-examination can be completed, his direct
testimony cannot be stricken off the record, provided the material points of his direct testimony has been covered
on cross.

COMPULSORY PROCESS
the accused is entitled under the constitution to issuance of subpoena and subpoena duces tecum for the
purpose of compelling the attendance of witness and the production of evidence that he may need for his
defense.

PROHIBITED PUNISHMENT

Page 157 of 236


Constitutional Law 2 Bill of Rights Armando Santiago Jr

CASES
PEOPLE VS. ORDOO
PONENCIA:
TOPIC:
TRIGGER OF THE FACTS:
TRIGGER OF THE ISSUE:
TRIGGER OF THE RULING:
VERDICT:
END POINT:

FACTS
1. A decomposing body of a child was found. Evidence pointed to 2 suspects;
2. The two suspects were questioned but eventually let go for lack of evidence to pin them to the crime of murder
and rape;
3. Eventually, the 2 went back to the police and admitted to their crime;
4. Then and there, their confession was written and they were locked up;
5. After a while, they were made to confess publicly in front of the Mayor, Parish Priest, their family;
6. A radio host interviewed them and again, they confessed to their crimes;

ISSUE
WON their uncounselled extrajudicial confession is invalid?

HELD
Their uncouselled extrajudicial confession in public in front of the Mayor, Priest, family, etc was not valid because
they cannot replace a lawyer with whom a confession before him would render the confession valid. However,
their confession before a radio announcer is admissible because it was done, in public and voluntarily.

DOCENA-CASPE VS BUGTAS
PONENCIA:
TOPIC:
TRIGGER OF THE FACTS:
TRIGGER OF THE ISSUE:
TRIGGER OF THE RULING:
VERDICT:
END POINT: hindi pwedeng magissue ng bail without a hearing

FACTS
An administrative case was filed against Judge Bugtos for granting a bail bond to the accused in a criminal case
without a hearing. The said case stemmed from a criminal case wherein the predecessor of Judge Bugtos,
Judge alvarez granted the accused in the said case a bail while the latter were at large. Subsequently, one of the
accused was apprehended. The provincial prosecutor then filed before the RTC, now presided by Judge Bugtos,
a motion that an alias warrant be issued against the other accused and that both accused be not allowed to bail.
Thereafter, the accused. filed a motion to allow him to bail on the account that the said murder was committed at
the time wherein the death penalty was suspended. The said request was denied. The accused then filed a
motion for reconsideration. Judge Bugtos asked for the comment of the prosecution. However the latter failed to
file a comment. The said motion for reconsideration for a bail was then granted without conducting a hearing
based solely on the fact that a bail was previously granted by his predecessor.

Page 158 of 236


Constitutional Law 2 Bill of Rights Armando Santiago Jr

ISSUE
whether or not a bail may be granted without conducting a hearing

RULING
Jurisprudence is replete with decisions on the procedural necessity of a hearing, whether summary or otherwise,
relative to the grant of bail especially in cases involving offenses punishable by death, reclusion perpetua, or life
imprisonment, where bail is a matter of discretion. Under the present rules, a hearing is required in granting bail
whether it is a matter of right or discretion. It must be stressed that the grant or the denial of bail in cases where
bail is a matter of discretion hinges on the issue of whether or not the evidence on the guilt of the accused is
strong, and the determination of whether or not the evidence is strong is a matter of judicial discretion which
remains with the judge. In order for the latter to properly exercise his discretion, he must first conduct a hearing
to determine whether the evidence of guilt is strong.

COMENDADOR VS. DE VILLA39


(as chief of staff of the AFP) Right to bail of Military Personnel

PONENCIA:
TOPIC:
TRIGGER OF THE FACTS:
TRIGGER OF THE ISSUE:
TRIGGER OF THE RULING:
VERDICT: CCORDINGLY, in G.R. No. 93177, the petition is DISMISSED for lack of merit. In G.R. No. 96948, the
petition is GRANTED, and the respondents are DIRECTED to allow the petitioners to exercise the right of
peremptory challenge under Article 18 of the Articles of War. In G.R. Nos. 95020 and 97454, the petitions are also
GRANTED, and the orders of the respondent courts for the release of the private respondents are hereby
REVERSED and SET ASIDE. No costs.
END POINT:

FACTS
The case involves 4 consolidated cases of the officers of the AFP who are facing prosecution for their
alleged participation in the failed coup d etat on December 1-9, 1989:

G.R. No. 93177-petition for certiorari, prohibition, mandamus- questioning the conduct of the pre-trial panel and
the creation of General Court Martial (GMC No. 14)

G.R. No. 96948-certiorari against the ruling denying them the right to pre-emptory challenge (or that the
Members of general or special courts-martial may be challenged by the accused or the trial judge advocate
for cause stated to the court. The court shall determine the relevancy and validity thereof.)

G.R. No. 95020-certiorari- against the respondent judge on the ground that he has no jurisdiction of GCM No. 14
and no authority to set aside its ruling of denying bail to private respondents

G.R. No. 97454-certiorari- against the decision of RTC in a petition for habeas corpus directing the release of the
private respondents. Jurisdictional objection are also raised.

Charges against them include mutiny, conduct unbecoming an officer and a gentleman, and various
crimes in relation to murder

39 G.R. No. 93177, G.R. No. 96948,G.R. No. 95020,G.R. No. 97454
Page 159 of 236
Constitutional Law 2 Bill of Rights Armando Santiago Jr

The pre-trail investigation (PTI) panel issued several letters of notice to the petitioners for counter-
affidavit and of the affidavits of their witnesses. All were moved to delay and the petitioners contend that there
was no pre-trail investigation done
In G.R. No. 95020, Ltc. Jacinto Ligot applied for bail and it was denied by GMC No. 14. The RTC
granted him provisional liberty but he was not released immediately, pending the final resolution of the appeal to
be taken. Then the RTC ruled that the right to bail covers military men facing court-martial proceedings

ISSUES
Whether there was violation of due process
Whether or not the military personnel are entitled to bail, thus, WON there was a violation of the right to
bail

HELD
The petitioners in G.R. Nos. 93177 and 96948 were given several opportunities to be heard when they
were asked to submit their counter-affidavits to the PTI. They cannot claim that they were denied due process.
Failure to submit the aforementioned counter-affidavits on the date above specified shall be deemed a waiver of
(their) right to submit controverting evidence."
"even a failure to conduct a pre-trial investigation does not deprive a general court- martial of
jurisdiction."
We find that the right to bail invoked by the private respondents in G.R. Nos. 95020 has traditionally not
been recognized and is not available in the military, as an exception to the general rule embodied in the Bill of
Rights.
However, a right to speedy trial is given more emphasis in the military, where the right to bail does not
exist.
Solicitor Generals explanation of the exception:
The unique structure of the military should be enough reason to exempt military men from the
constitutional coverage on the right to bail.
soldiers operate within the framework of democratic system, are allowed the fiduciary use of firearms by the
government for the discharge of their duties and responsibilities and are paid out of revenues collected from the
people.
the truly disquieting thought is that they could freely resume their heinous activity which could very well result
in the overthrow of duly constituted authorities,
Neither does it violate equal protection because the military is not similarly situated with others.
Dispositive part of the case:
As in that case, we find that the respondents in G.R. No. 93177 have not acted with grave abuse of
discretion or without or in excess of jurisdiction to justify the intervention of the Court and the reversal of the acts
complained of by the petitioners. Such action is indicated, however, in G.R. No. 96948, where we find that the
right to peremptory challenge should not have been denied, and in G.R. Nos. 95020 and 97454, where the
private respondents should not have been ordered released.
ACCORDINGLY, in G.R. No. 93177, the petition is DISMISSED for lack of merit. In G.R. No. 96948, the
petition is GRANTED, and the respondents are DIRECTED to allow the petitioners to exercise the right of
peremptory challenge under Article 18 of the Articles of War. In G.R. Nos. 95020 and 97454, the petitions are
also GRANTED, and the orders of the respondent courts for the release of the private respondents are hereby
REVERSED and SET ASIDE. No costs.

SYLLABI
Constitution; Bail; Constitution grants the right to bail to all persons with the defined exception is applicable and
covers all military men facing court-martial proceedings.On August 22, 1990, the trial court rendered judgment
inter alia: (a) Declaring, that Section 13, Article III of the Constitution granting the right to bail to all persons with
the defined exception is applicable and covers all military men facing court-martial proceedings. Accordingly, the
assailed orders of General Court-Martial No. 14 denying bail to petitioner and intervenors on the mistaken

Page 160 of 236


Constitutional Law 2 Bill of Rights Armando Santiago Jr

assumption that bail does not apply to military men facing court-martial proceedings on the ground that there is
no precedent, are hereby set aside and declared null and void. Respondent General Court-Martial No. 14 is
hereby directed to conduct proceedings on the applications of bail of the petitioner, intervenors and which may
as well include other persons facing charges before General Court-Martial No. 14.

Constitution; Bill of Rights; Bail; Equal Protection; That denial from the military of the right to bail would violate
the equal protection clause is not acceptable.The argument that denial from the military of the right to bail
would violate the equal protection clause is not acceptable. This guaranty requires equal treatment only of
persons or things similarly situated and does not apply where the subject of the treatment is substantially
different from others. The accused officers can complain if they are denied bail and other members of the military
are not. But they cannot say they have been discriminated against because they are not allowed the same right
that is extended to civilians.

PEOPLE VS. RIVERA40


PONENCIA:
TOPIC:
TRIGGER OF THE FACTS:
TRIGGER OF THE ISSUE:
TRIGGER OF THE RULING:
VERDICT:
END POINT: The trial court appointed Atty. Bansil a counsel de oficio to represent accused-appellant on October 6,
1998 because his regular counsel, Atty. Anselmo Mangalindan, was absent without any explanation. Atty.
Mangalindan had previously been granted several postponements.

Atty. Bansil was present and heard the testimony of Dr. Barin, the prosecution witness, on that day. Dr. Barins
testimony on direct examination was simple, containing primarily a discussion of her findings on the hymenal
laceration sustained by complainant. Her testimony did not require considerable study and extraordinary
preparation on the part of defense counsel for the purpose of cross-examination. It seems Atty. Bansil no longer
found it necessary to cross-examine Dr. Barin.

Moreover, beyond stating that Dr. Barin was a vital witness, accused-appellant has not indicated what questions his
counsel wanted to ask from Dr. Barin. It may well be that these questions do not exist at all and that the importance
given by accused-appellant to counsel de oficios failure to cross-examine the witness is exaggerated. Indeed, a
medical examination of the victim, together with the medical certificate, is merely corroborative and is not an
indispensable element of rape.[43] The primordial issue in this case remains to be whether the complainants
testimony, not Dr. Barins, established beyond reasonable doubt the crime of rape.

FACTS
Erlanie Rivera's younger sister, Zaira
was taken by their parents to the Hospital in Pampanga
Erlanie's mother stayed with her sister in the hospital
but her father, Rolando Rivera, went back home to pampanga
Erlanie was awakened as Rolando started kissing her and fondling her breasts.
Erlanie tried to resist by kicking and pushing Rolando, but her efforts were to no avail
Rolando removed her shorts and panty, touched her private parts, and then had sexual intercourse with her.
After he was through with her, Rolando told complainant not to tell anyone what had happened or he would kill
Erlanie's mother and sister.
when her mother came home the following day, Erlanie did not tell her what had happened because she was
afraid of Rolando.
Erlanie, in the presence of her mother, told her aunt, Marietta Pagtalunan, and her grandmother, Maxima
Payumo, that she had been raped by Rolando.
she was referred to Dr. Barin for physical examination.

40 GR 139180, 31 July 2001 En Banc, Mendoza (J): 9 concur, 4 on official business, 1 on leave
Page 161 of 236
Constitutional Law 2 Bill of Rights Armando Santiago Jr

filed an action against rivera by means of violence, threat and intimidation, did then and there willfully,
unlawfully and feloniously, and maliciously succeeded in having carnal knowledge [of] his 13 year old
daughter, Erlanie D. Rivera, against the latter's will and without her consent.
rivera pleaded not guilty to the crime charged
trial court rendered a decision, finding Rolando Rivera guilty beyond reasonable doubt of the crime of rape as
charged.
For having violated Article 335 of the Revised Penal Code, as amended by Republic Act 7659, with the
attendant circumstances that the victim is under 18 years of age and the offender is the father of the victim
and absent any circumstance that could mitigate the commission thereof,
the Court sentenced Rolando to suffer the supreme penalty of death by lethal injection, and ordered him, in
line with established jurisprudence, to indemnify the offended party Erlanie Rivera in the sum of P75,000.00 as
compensatory damages and P50,000.00 as moral damages.
Rolando appealed.

ISSUE
Whether the right to speedy and adequate justice of one party necessary limits the right to competent
and independent counsel of choice of another, and whether the speedy disposition of the case (a day after the
memorandum was filed) denies due process to the accused.

HELD
While the Constitution recognizes the accused's right to competent and independent counsel of his own choice,
his option to secure the services of a private counsel is not absolute. For considering the State's and the
offended party's right to speedy and adequate justice, the court may restrict the accused's option to retain a
private counsel if the accused insists on an attorney he cannot afford, or if the chosen counsel is not a member
of the bar, or if the attorney declines to represent the accused for a valid reason. The trial court appointed Atty.
Bansil a counsel de oficio to represent Rolando on 6 October 1998 because his regular counsel, Atty. Anselmo
Mangalindan, was absent without any explanation. Atty. Mangalindan had previously been granted several
postponements. As held in People v. Serzo, Jr. (274 SCRA 553, 568 [1997]), the Courts are not required to wait
indefinitely the pleasure and convenience of the accused as they are also mandated to promote the speedy and
orderly administration of justice. Nor should they countenance such an obvious trifling with the rules. Indeed,
public policy requires that the trial continue as scheduled, considering that appellant was adequately
represented by counsels who were not shown to be negligent, incompetent or otherwise unable to represent
him. Atty. Bansil was present and heard the testimony of Dr. Barin, the prosecution witness, on that day. Dr.
Barin's testimony on direct examination was simple, containing primarily a discussion of her finding son the
hymenal laceration sustained by complainant. Her testimony did not require considerable study and
extraordinary preparation on the part of defense counsel for the purpose of cross-examination. It seems Atty.
Bansil no longer found it necessary to cross-examine Dr. Barin. Further, Rolando was not denied due process
considering the speed with which the trial court rendered judgment against him, which judgment was
promulgated one day after he filed his memorandum. The decision rendered by the trial court gives a clear
account of the facts and the law on which it is based. It discusses in full the court's findings on the credibility of
both the prosecution and defense witnesses and its evaluation of the evidence of both parties. As held in the
analogous case of People v. Mercado (GR. 116239,

29 November 2000.), the speed with which the trial court disposed of the case cannot be attributed to the
injudicious performance of its function. Indeed, a judge is not supposed to study a case only after all the
pertinent pleadings have been filed. It is a mark of diligence and devotion to duty that a judge studies a case
long before the deadline set for the promulgation of his decision has arrived. The one-day period between the
filing of accused-appellants' memorandum and the promulgation of the decision was sufficient time to consider
their arguments and to incorporate these in the decision. As long as the trial judge does not sacrifice the orderly
administration of justice in favor of a speedy but reckless disposition of a case, he cannot be taken to task for
rendering his decision with due dispatch.

SYLLABI
Page 162 of 236
Constitutional Law 2 Bill of Rights Armando Santiago Jr

Right of Confrontation; The cross-examination of a witness is essential to test his or her credibility, expose
falsehoods or half-truths, uncover the truth which rehearsed direct examination testimonies may successfully
suppress, and demonstrate inconsistencies in substantial matters which create reasonable doubt as to the guilt
of the accused and thus give substance to the constitutional right of the accused to confront the witnesses
against him.The right of a party to cross-examine a witness is embodied in Art. III, 14(2) of the Constitution
which provides that the accused shall have the right to meet the witnesses face to face and in Rule 115, l(f) of
the Revised Rules of Criminal Procedure which states that, in all criminal prosecutions, the accused shall have
the right to confront and cross-examine the witnesses against him. The cross-examination of a witness is
essential to test his or her credibility, expose falsehoods or half-truths, uncover the truth which rehearsed direct
examination testimonies may successfully suppress, and demonstrate inconsistencies in substantial matters
which create reasonable doubt as to the guilt of the accused and thus give substance to the constitutional right
of the accused to confront the witnesses against him.

Same; The right of the accused to cross-examine a witness is, however, not without limits but is subject to the
rules on the admissibility and relevance of evidence.The right of the accused to cross-examine a witness is,
however, not without limits but is subject to the rules on the admissibility and relevance of evidence. Thus, in
People v. Zheng Bai Hui, this Court upheld the ruling of the trial judge disallowing the questions propounded by
the accuseds counsel on the ability of the arresting officer to distinguish between tawas and shabu without a
laboratory examination, the academic degree of his training instructor, and the officers authorship of books on
drug identity and analysis for being irrelevant, improper, and impertinent.

Right to Counsel; Considering the States and the offended partys right to speedy and adequate justice, the
court may restrict the accuseds option to retain a private counsel if the accused insists on an attorney he cannot
afford, or if the chosen counsel is not a member of the bar, or if the attorney declines to represent the accused
for a valid reason.While the Constitution recognizes the accuseds right to competent and independent
counsel of his own choice, his option to secure the services of a private counsel is not absolute. For considering
the States and the offended partys right to speedy and adequate justice, the court may restrict the accuseds
option to retain a private counsel if the accused insists on an attorney he cannot afford, or if the chosen counsel
is not a member of the bar, or if the attorney declines to represent the accused for a valid reason.

Alibis and Denials; Denial, when unsubstantiated by clear and convincing evidence, constitutes negative self-
serving evidence which deserves no greater evidentiary value than the testimony of a credible witness who
testified on affirmative matters.Accused-appellant also raises the defense of denial and alibi. But the bare
denial of accused-appellant cannot overcome the positive declarations of complainant. Denial, when
unsubstantiated by clear and convincing evidence, constitutes negative self-serving evidence which deserves no
greater evidentiary value than the testimony of a credible witness who testified on affirmative matters.

Same; Witnesses; The defense of alibi cannot prosper if it is established mainly by the accused and his relatives,
and not by credible persons.Accused-appellants sister, Concepcion Sayo, testified that accused-appellant
lived with her family in Bulacan at the time of the rape. No other witness not related to accused-appellant,
however, was called to corroborate her claim. We have already held that the defense of alibi cannot prosper if it
is established mainly by the accused and his relatives, and not by credible persons. It is not improbable that
these witnesses would freely perjure themselves for the sake of their loved ones. Accused-appellants defense
thus fails to convince this Court.

HONG KONG V. OLALIA41


PONENCIA:
TOPIC:
TRIGGER OF THE FACTS:
TRIGGER OF THE ISSUE:
TRIGGER OF THE RULING:
VERDICT:
END POINT:

41 G.R. No. 153675


Page 163 of 236
Constitutional Law 2 Bill of Rights Armando Santiago Jr

FACTS
The Philippines and Hong Kong signed an Agreement for the Surrender of Accused and Convicted Persons.

Private respondent Muoz was charged before the Hong Kong Court. Department of Justice (DOJ) received
from the Hong Kong Department of Justice a request for the provisional arrest of private respondent Muoz. The
DOJ then forwarded the request to the National Bureau of Investigation (NBI) which, in turn, filed with the RTC of
Manila, Branch 19 an application for the provisional arrest of private respondent. The NBI agents arrested and
detained him.

Muoz filed a petition for bail which was denied by Judge Bernardo, Jr. holding that there is no Philippine law
granting bail in extradition cases and that private respondent is a high flight risk. After Judge Bernardo, Jr.
inhibited himself from further hearing the case, it was then raffled off to Branch 8 presided by respondent judge.
Private respondent filed a motion for reconsideration of the Order denying his application for bail and this was
granted by respondent judge.

ISSUE
Whether or not the trial court committed grave abuse of discretion amounting to lack or excess of jurisdiction in
allowing private respondent to bail?

HELD
No, the trial court did not commit grave abuse of discretion amounting to lack or excess of jurisdiction in allowing
private respondent to bail.

Accordingly, although the time-honored principle of pacta sunt servanda demands that the Philippines honor its
obligations under the Extradition Treaty it entered into with the Hong Kong Special Administrative Region it does
not necessarily mean that in keeping with its treaty obligations, the Philippines should diminish a potential
extraditees rights to life, liberty, and due process guaranteed by the Constitution. More so, where these rights
are guaranteed, not only by our Constitution, but also by international conventions, particularly the Universal
Declaration of Human Rights, to which the Philippines is a party.

We should not, therefore, deprive an extraditee of his right to apply for bail, provided that a certain
standard for the grant is satisfactorily met. In his Separate Opinion in Purganan, then Associate Justice, now
Chief Justice Reynato S. Puno, proposed that a new standard which he termed clear and convincing evidence
should be used in granting bail in extradition cases. According to him, this standard should be lower than proof
beyond reasonable doubt but higher than preponderance of evidence. The potential extraditee must prove by
clear and convincing evidence that he is not a flight risk and will abide with all the orders and processes of the
extradition court.

In this case, there is no showing that private respondent presented evidence to show that he is not a flight risk.
Consequently, this case should be remanded to the trial court to determine whether private respondent may be
granted bail on the basis of clear and convincing evidence.

WHEREFORE, we DISMISS the petition. This case is REMANDED to the trial court to determine whether private
respondent is entitled to bail on the basis of clear and convincing evidence. If not, the trial court should order
the cancellation of his bail bond and his immediate detention; and thereafter, conduct the extradition proceedings
with dispatch.

Page 164 of 236


Constitutional Law 2 Bill of Rights Armando Santiago Jr

SYLLABI
Same; Same; Bail; Human Rights; The modern trend in public international law is the primacy placed on the
worth of the individual person and the sanctity of human rights.At first glance, the above ruling applies
squarely to private respondents case. However, this Court cannot ignore the following trends in international
law: (1) the growing importance of the individual person in public international law who, in the 20th century, has
gradually attained global recognition; (2) the higher value now being given to human rights in the international
sphere; (3) the corresponding duty of countries to observe these universal human rights in fulfilling their treaty
obligations; and (4) the duty of this Court to balance the rights of the individual under our fundamental law, on
one hand, and the law on extradition, on the other. The modern trend in public international law is the primacy
placed on the worth of the individual person and the sanctity of human rights. Slowly, the recognition that the
individual person may properly be a subject of international law is now taking root. The vulnerable doctrine that
the subjects of international law are limited only to states was dramatically eroded towards the second half of the
past century. For one, the Nuremberg and Tokyo trials after World War II resulted in the unprecedented
spectacle of individual defendants for acts characterized as violations of the laws of war, crimes against peace,
and crimes against humanity. Recently, under the Nuremberg principle, Serbian leaders have been persecuted
for war crimes and crimes against humanity committed in the former Yugoslavia. These significant events show
that the individual person is now a valid subject of international law.

Same; Same; Same; Same; Same; While this Court in Government of the United States of America v. Purganan,
389 SCRA 623 (2002), limited the exercise of the right to bail to criminal proceedings, however, in light of the
various international treaties giving recognition and protection to human rights, particularly the right to life and
liberty, a reexamination of this Courts ruling in Purganan is in order.The Philippines, along with the other
members of the family of nations, committed to uphold the fundamental human rights as well as value the worth
and dignity of every person. This commitment is enshrined in Section II, Article II of our Constitution which
provides: The State values the dignity of every human person and guarantees full respect for human rights.
The Philippines, therefore, has the responsibility of protecting and promoting the right of every person to liberty
and due process, ensuring that those detained or arrested can participate in the proceedings before a court, to
enable it to decide without delay on the legality of the detention and order their release if justified. In other words,
the Philippine authorities are under obligation to make available to every person under detention such remedies
which safeguard their fundamental right to liberty. These remedies include the right to be admitted to bail. While
this Court in Purganan limited the exercise of the right to bail to criminal proceedings, however, in light of the
various international treaties giving recognition and protection to human rights, particularly the right to life and
liberty, a reexamination of this Courts ruling in Purganan is in order.

Same; Same; Same; Same; Same; If bail can be granted in deportation cases, the Court sees no justification
why it should not also be allowed in extradition casesclearly, the right of a prospective extraditee to apply for
bail must be viewed in the light of the various treaty obligations of the Philippines concerning respect for the
promotion and protection of human rights.In Mejoff v. Director of Prisons, 90 Phil. 70 (1951) and Chirskoff v.
Commission of Immigration, 90 Phil. 256 A(1951), this Court ruled that foreign nationals against whom no formal
criminal charges have been filed may be released on bail pending the finality of an order of deportation. As
previously stated, the Court in Mejoff relied upon the Universal declaration of Human Rights in sustaining the
detainees right to bail. If bail can be granted in deportation cases, we see no justification why it should not also
be allowed in extradition cases. Likewise, considering that the Universal Declaration of Human Rights applies to
deportation cases, there is no reason why it cannot be invoked in extradition cases. After all, both are
administrative proceedings where the innocence or guilt of the person detained is not in issue. Clearly, the right
of a prospective extraditee to apply for bail in this jurisdiction must be viewed in the light of the various treaty
obligations of the Philippines concerning respect for the promotion and protection of human rights. Under these
treaties, the presumption lies in favor of human liberty. Thus, the Philippines should see to it that the right to
liberty of every individual is not impaired.

Page 165 of 236


Constitutional Law 2 Bill of Rights Armando Santiago Jr

GOVERNMENT OF HONG KONG VS. OLALIA (OTHER


VERSION)42

PONENCIA:
TOPIC:
TRIGGER OF THE FACTS:
TRIGGER OF THE ISSUE:
TRIGGER OF THE RULING:
VERDICT:
END POINT:

This case discusses whether the right to bail guaranteed under the Bill of Rights extends to a prospective
extradite in an extradition proceeding.

On January 30, 1995, the Republic of the Philippines and the then British Crown Colony of Hong Kong signed an
"Agreement for the Surrender of Accused and Convicted Persons." It took effect on June 20, 1997.

The Petitioner is the Government of Hong Kong Special Administrative Region, represented by the Philippine
Department of Justice

The Respondents are Judge Felix Olalia and Juan Antonio Muoz

FACTS
Private respondent Muoz was charged before the Hong Kong Court with three (3) counts of the offense of
"accepting an advantage as agent," in violation of Section 9 (1) (a) of the Prevention of Bribery Ordinance,
Cap. 201 of Hong Kong. He also faces seven (7) counts of the offense of conspiracy to defraud, penalized by
the common law of Hong Kong. Warrants of arrest were issued against him. If convicted, he faces a jail term of
seven (7) to fourteen (14) years for each charge.

On September 13, 1999, the DOJ received from the Hong Kong Department of Justice a request for the
provisional arrest of private respondent. The RTC, Branch 19, Manila issued an Order of Arrest against
private respondent. That same day, the NBI agents arrested and detained him.

Private respondent filed a petition for bailwhich was opposed by petitioner. After hearing, Judge Bernardo, Jr.
issued an Order denying the petition for bail, holding that there is no Philippine law granting bail in extradition
cases and that private respondent is a high "flight risk." Judge Bernardo, Jr. inhibited himself from further hearing
the case, it was then raffled off to Branch 8 presided by respondent judge. Private respondent filed a motion for
reconsideration of the Order denying his application for bail and this was granted by respondent judge.

Petitioner filed an urgent motion to vacate the above Order, but it was denied by respondent judge. Hence, the
instant petition.

ISSUE
Whether or not respondent judge acted with grave abuse of discretion amounting to lack or excess of
jurisdiction as there is no provision in the Constitution granting bail to a potential extraditee.

HELD
No. Bearing in mind the purpose of extradition proceedings, the premise behind the issuance of the arrest
warrant and the "temporary detention" is the possibility of flight of the potential extraditee. This is based on the

42 G.R. No. 153675, April 19, 2007


Page 166 of 236
Constitutional Law 2 Bill of Rights Armando Santiago Jr

assumption that such extraditee is a fugitive from justice. Given the foregoing, the prospective extraditee thus
bears theonus probandiof showing that he or she is not a flight risk and should be granted bail.

RATIO
The Philippines, along with the other members of the family of nations, committed to uphold the fundamental
human rights as well as value the worth and dignity of every person. Clearly, the right of a prospective
extraditee to apply for bail in this jurisdiction must be viewed in the light of the various treaty obligations of
the Philippines concerning respect for the promotion and protection of human rights. Under these
treaties, the presumption lies in favor of human liberty. Thus, the Philippines should see to it that the right to
liberty of every individual is not impaired.

Extradition is not a trial to determine the guilt or innocence of the potential extraditee.Nor is it a full-blown
civil action, but one that is merely administrative in character. Its object is to prevent the escape of a
person accused or convicted of a crime and to secure his return to the state from which he fled, for the
purpose of trial or punishment. It does not necessarily mean that in keeping with its treaty obligations, the
Philippines should diminish a potential extraditees rights to life, liberty, and due process. More so, where
these rights are guaranteed, not only by our Constitution, but also by international conventions, to which the
Philippines is a party. We should not, therefore, deprive an extraditee of his right to apply for bail, provided that a
certain standard for the grant is satisfactorily met.

In his Separate Opinion in Purganan, then Associate Justice Puno, proposed that a new standard which he
termed "clear and convincing evidence"should be used in granting bail in extradition cases.According to
him, this standard should be lower than proof beyond reasonable doubt but higher than preponderance of
evidence. The potential extraditee must prove by "clear and convincing evidence" that he is not a flight risk
and will abide with all the orders and processes of the extradition court.

In this case, there is no showing that private respondent presented evidence to show that he is not
a flight risk. Consequently, this case should be remanded to the trial court to determine whether private
respondent may be granted bail on the basis of "clear and convincing evidence.

WHEREFORE, we DISMISS the petition. This case is REMANDED to the trial court to determine whether
private respondent is entitled to bail on the basis of "clear and convincing evidence." If not, the trial court should
order the cancellation of his bail bond and his immediate detention; and thereafter, conduct the extradition
proceedings with dispatch.

GOVERNMENT OF THE UNITED STATES OF AMERICA HON.


GUILLERMO G. PURGANAN, MORALES, AND PRESIDING
JUDGE, REGIONAL TRIAL COURT OF MANILA AND MARK B.
JIMENEZ A.K.A. MARIO BATACAN CRESPO43
PONENCIA:
TOPIC:
TRIGGER OF THE FACTS:
TRIGGER OF THE ISSUE:
TRIGGER OF THE RULING:
VERDICT:
END POINT:

FACTS

43 G.R. No. 148571. September 24, 2002


Page 167 of 236
Constitutional Law 2 Bill of Rights Armando Santiago Jr

The United States Government sent to the Philippine Government Note Verbale No. 0522 dated June 16, 1999,
supplemented by Note Nos. 0597, 0720 and 0809 requesting the extradition of Mark B. Jimenez, also known as
Mario Batacan Crespo.

Upon learning of the request for his extradition, Jimenez sought and was granted a TRO by the RTC of Manila,
Branch 25. The TRO prohibited the DOJ from filing with the RTC a petition for his extradition.

Before the RTC could act on the Petition, Respondent Jimenez filed before it an Urgent Manifestation/Ex-Parte
Motion, which prayed that petitioners application for an arrest warrant be set for hearing.
The RTC granted the Motion of Jimenez and set the case for hearing on June 5, 2001. After the hearing, the
court a quo required the parties to submit their respective memoranda. In his Memorandum, Jimenez sought an
alternative prayer: that in case a warrant should issue, he be allowed to post bail in the amount of P100, 000.

The alternative prayer of Jimenez was also set for hearing on June 15, 2001. Thereafter, the court below issued
its questioned July 3, 2001 Order, directing the issuance of a warrant for his arrest and fixing bail for his
temporary liberty at one million pesos in cash. After he had surrendered his passport and posted the required
cash bond, Jimenez was granted provisional liberty via the challenged Order dated July 4, 2001.

ISSUES
1. Whether or not Jimenez is entitled to notice and hearing before a warrant for his arrest can be issued,
and

2. Whether or not he is entitled to bail and to provisional liberty while the extradition proceedings are
pending.

HELD
1. No. There is no requirement to notify and hear the accused before the issuance of warrants of arrest.
The case under consideration is an extradition and not a criminal action; therefore it is not sufficient to justify the
adoption of a set of procedures more protective of the accused.

2. No. The constitutional provision on bail applies only when a person has been arrested and detained for
violation of Philippine criminal laws. It does not apply to extradition proceedings, because extradition courts do
not render judgments of conviction or acquittal.

SYLLABI
Same; Same; Same; Persons to be extradited are presumed to be flight risks.Persons to be extradited are
presumed to be flight risks. This prima facie presumption finds reinforcement in the experience of the executive
branch: nothing short of confinement can ensure that the accused will not flee the jurisdiction of the requested
state in order to thwart their extradition to the requesting state.

Same; Same; Statutory Construction; Section 6 of PD 1069, our Extradition Treaty, uses the word immediate to
qualify the arrest of the accused, a qualification would be rendered nugatory by setting for hearing the issuance
of the arrest warrantarrest subsequent to a hearing can no longer be considered immediate.It is significant
to note that Section 6 of PD 1069, our Extradition Law, uses the word immediate to qualify the arrest of the
accused. This qualification would be rendered nugatory by setting for hearing the issuance of the arrest warrant.
Hearing entails

sending notices to the opposing parties, receiving facts and arguments from them, and giving them time to
prepare and present such facts and arguments. Arrest subsequent to a hearing can no longer be considered
immediate. The law could not have intended the word as a mere superfluity but, on the whole, as a means of
imparting a sense of urgency and swiftness in the determination of whether a warrant of arrest should be issued.
Same; Same; Same; By using the phrase if it appears, the law fur ther conveys that accuracy is not as
important as speed at such early stage.By using the phrase if it appears, the law further conveys that
Page 168 of 236
Constitutional Law 2 Bill of Rights Armando Santiago Jr

accuracy is not as important as speed at such early stage. The trial court is not expected to make an exhaustive
determination to ferret out the true and actual situation, immediately upon the filing of the petition. From the
knowledge and the material then available to it, the court is expected merely to get a good first impressiona
prima facie findingsufficient to make a speedy initial determination as regards the arrest and detention of the
accused.

Same; Same; Warrants of Arrest; Grave Abuse of Discretion; A judge gravely abuses his discretion when he sets
for hearing the application for the issuance of an arrest warrant in an extradition proceeding after having already
determined from the petition itself and its supporting documents that a prima facie finding exists.We stress that
the prima facie existence of probable cause for hearing the petition and, a priori, for issuing an arrest warrant
was already evident from the Petition itself and its supporting documents. Hence, after having already
determined therefrom that a prima facie finding did exist, respondent judge gravely abused his discretion when
he set the matter for hearing upon motion of Jimenez.

Same; Same; Same; Statutory Construction; The silence of the Extradition Law and the Treaty leans to the more
reasonable interpretation that there is no intention to punctuate with a hearing every little step in the entire
proceedings.Moreover, the law specifies that the court sets a hearing upon receipt of the answer or upon
failure of the accused to answer after receiving the summons. In connection with the matter of immediate arrest,
however, the word hearing is notably absent from the provision. Evidently, had the holding of a hearing at that
stage been intended, the law could have easily so provided. It also bears emphasizing at this point that
extradition proceedings are summary in nature. Hence, the silence of the Law and the Treaty leans to the more
reasonable interpretation that there is no intention to punctuate with a hearing every little step in the entire
proceedings.

Same; Same; Bail; Statutory Construction; As suggested by the use of the word conviction in Art. III, Section 13
of the Constitution, the constitutional provision on bail, as well as Section 4 of Rule 114 of the Rules of Court,
applies only when a person has been arrested and detained for violation of Philippine criminal lawsit does not
apply to extradition proceedings where the presumption of innocence is not at issue.We agree with petitioner.
As suggested by the use of the word conviction, the constitutional provision on bail quoted above, as well as
Section 4 of Rule 114 of the Rules of Court, applies only when a person has been arrested and detained for
violation of Philippine criminal laws. It does not apply to extradition proceedings, because extradition courts do
not render judgments of conviction or acquittal. Moreover, the constitutional right to bail flows from the
presumption of innocence in favor of every accused who should not be subjected to the loss of freedom as
thereafter he would be entitled to acquittal, unless his guilt be proved beyond reasonable doubt. It follows that
the constitutional provision on bail will not apply to a case like extradition, where the presumption of innocence is
not at issue.

Same; Same; Same; Same; The provision in the Constitution stating that the right to bail shall not be impaired
even when the privilege of the writ of habeas corpus is suspended does not detract from the rule that the
constitutional right to bail is available only in criminal proceedings.The provision in the Constitution stating that
the right to bail shall not be impaired even when the privilege of the writ of habeas corpus is suspended does
not detract from the rule that the constitutional right to bail is available only in criminal proceedings. It must be
noted that the suspension of the privilege of the writ of habeas corpus finds application only to persons judicially
charged for rebellion or offenses inherent in or directly connected with invasion. Hence, the second sentence in
the constitutional provision on bail merely emphasizes the right to bail in criminal proceedings for the
aforementioned offenses. It cannot be taken to mean that the right is available even in extradition proceedings
that are not criminal in nature.

Same; Same; Same; In the absence of any provisionin the Constitution, the law or the treatyexpressly
guaranteeing the right to bail in extradition proceedings, adopting the practice of not granting them bail, as a
general rule, would be a step towards deterring fugitives from coming to the Philippines to hide from or evade
their prosecutors.Too, we cannot allow our country to be a haven for fugitives, cowards and weaklings who,
instead of facing the consequences of their actions, choose to run and hide. Hence, it would not be good policy
to increase the risk of violating our treaty obligations if, through overprotection or excessively liberal treatment,
persons sought to be extradited are able to evade arrest or escape from our custody. In the absence of any
provisionin the Constitution, the law or the treatyexpressly guaranteeing the right to bail in extradition

Page 169 of 236


Constitutional Law 2 Bill of Rights Armando Santiago Jr

proceedings, adopting the practice of not granting them bail, as a general rule, would be a step towards
deterring fugitives from coming to the Philippines to hide from or evade their prosecutors.

Same; Same; Same; To best serve the ends of justice, the Court holds that, after a potential extraditee has been
arrested or placed under the custody of the law, bail may be applied for and granted as an exception, only upon
a clear and convincing showing (1) that, once granted bail, the applicant will not be a flight risk or a danger to the
community, and (2) that there exist special, humanitarian and compelling circumstances including, as a matter of
reciprocity, those cited by the highest court in the requesting state when it grants provisional liberty in extradition
cases therein.The rule, we repeat, is that bail is not a matter of right in extradition cases. However, the
judiciary has the constitutional duty to curb grave abuse of discretion and tyranny, as well as the power to
promulgate rules to protect and enforce constitutional rights. Furthermore, we believe that the right to due
process is broad enough to induce the grant of basic fairness to extraditees. Indeed, the right to due process
extends to the life, liberty or property of every person. It is dynamic and resilient, adaptable to every situation
calling for its application. Accordingly and to best serve the ends of justice, we believe and so hold that, after a
potential extraditee has been arrested or placed under the custody of the law, bail may be applied for and
granted as an exception, only upon a clear and convincing showing (1) that, once granted bail, the applicant will
not be a flight risk or a danger to the community; and (2) that there exist special, humanitarian and compelling
circumstances including, as a matter of reciprocity, those cited by the highest court in the requesting state when
it grants provisional liberty in extradition cases therein.

Same; Same; Same; Since the exception to the grant of bail in extradition proceedings has no express or
specific statutory basis, and since it is derived essentially from general principles of justice and fairness, the
applicant bears the burden of proving the two-tiered requirement with clarity, precision and emphatic
forcefulness.Since this exception has no express or specific statutory basis, and since it is derived essentially
from general principles of justice and fairness, the applicant bears the burden of proving the above two-tiered
requirement with clarity, precision and emphatic forcefulness. The Court realizes that extradition is basically an
executive, not a judicial, responsibility arising from the presidential power to conduct foreign relations. In its
barest concept, it partakes of the nature of police assistance amongst states, which is not normally a judicial
prerogative. Hence, any intrusion by the courts into the exercise of this power should be characterized by
caution, so that the vital international and bilateral interests of our country will not be unreasonably impeded or
compromised. In short, while this Court is ever protective of the sporting idea of fair play, it also recognizes the
limits of its own prerogatives and the need to fulfill international obligations.

Page 170 of 236


Constitutional Law 2 Bill of Rights Armando Santiago Jr

RIGHT OF THE ACCUSED


Bill of rights deals with the protection of the person facing criminal investigation or prosecution. The person
suspected or accused of a crime is entitled to the specific safeguards embodied in Section 12, 13, 14, 15, 17, 19
and 21 of the Art. 3 against his arbitrary prosecution or punishment.

It comprises of, to wit:


1. Criminal Due Process
2. Bail
3. Presumption of Innocence
4. Right to be Heard
5. Assistance of Counsel
6. Right to be Informed
7. Right to Speedy, Impartial and Public Trial
8. Right to Confrontation
9. Compulsory Process
10. Trials in Absentia

Preliminary investigation - is not among the rights granted to the accused in the Bill of rights. it is purely
statutory. it is component part of due process in criminal justice. mere deprivation of such investigation will
constitute a deprivation of his right to due process.

in order to satisfy the due process clause, it is not enough that the preliminary investigation is conducted in the
sense of making sure that the trasgressor shall not escape with impunity. A preliminary investigation serves not
only the purpose of the state. it is part of the guarantee of freedom and fair play which are birthrights of all who
live in our country.

basic ingredients of criminal due process is a trial conducted in accordance with the principles of fair play.

The requirement of proof beyond reasonable doubt is a necessary corollary of the constitutional right to be
presumed innocent. [People vs. Dramavo (1971)]

The accused cannot present evidence before the prosecution does so, even if the accused pleads guilty. It
violates the presumption of innocence. [Alejandro vs. Pepito (1980)]

The presumption of regularity (in official duties) cannot by itself prevail over the presumption of innocence of the
accused. But where it is not the sole basis for conviction, the presumption of regularity of performance of official
functions may prevail over the constitutional presumption of innocence. [People vs. Acuram (2000)]

EQUIPOISE RULE: Where the evidence adduced by the parties is evenly balanced, the constitutional
presumption of innocence should tilt the balance in favor of the accused. [Corpuz vs. People (1991)]
In order that circumstantial evidence may warrant conviction, the following requisites must concur:
(1) There is more than one circumstance
(2) The facts from which the inferences are derived are proven
(3) The combination of all the circumstances is such as to produce conviction beyond reasonable doubt.
[People v. Bato, G.R. No. 113804, January 16, 1998]

Right to be Heard

Page 171 of 236


Constitutional Law 2 Bill of Rights Armando Santiago Jr

SEC. 12, ART. III. 1987 CONSTITUTION.


Any person under investigation for the commission of an offense
shall have the right to be informed of his right to remain silent and to
have competent and independent counsel preferably of his own
choice. If the person cannot afford the services of counsel, he must
be provided with one. These rights cannot be waived except in
writing and in the presence of counsel.

It means the accused is amply accorded legal assistance extended by a counsel who commits himself to the
cause of the defense and acts accordingly. It is an efficient and truly decisive legal assistance, and not simply a
perfunctory representation. [People v. Bermas, G.R. No. 120420, April 21, 1999]

Right to be Informed

Procedural due process requires that the accused must be informed why he is being prosecuted and what
charge he must meet. [Vera vs. People, supra]

RIGHT TO SPEEDY, IMPARTIAL AND PUBLIC TRIAL


RA 7438. Rights of Persons under Custodial Investigation. SEC. 2. Rights of Persons Arrested, Detained or
Under Custodial Investigation; Duties of Public Officers. (a) Any person arrested detained or under custodial
investigation shall at all times be assisted by counsel; ART. III. SEC. 16. All persons shall have the right to a
speedy disposition of their cases before all judicial, quasi-judicial, or administrative bodies.

The accused has the right to be informed of the accusation against him including clear statement of the crime of
the which was violated. According to the supreme court every information must alleged one crime. hindi
pwedeng dagdagan that would be called duplicity (Charging more than one offense in a single information or
complaint)

A law and statue must not be vague or ambiguous.

WAIVER OF PRESENCE
the accused has the right to be present at each and every stage of their trial once he has spotified that there is a
trial he himself may choose not to come which is considered waived.

Exception: Accused has to be present for identification (sa court itatanong if you are to see him today can you
point on him?) for assurance that the person being accused is the one who is referring to in the testimony. He
cannot waived his appearance for it is necessary for the hearing.

IDENTIFICATION (TIME WHEN THEY CANNOT WAIVED THEIR RIGHT TO BE


PRESENT)
1. Arraignment
2. when the information or a complaint where to the accused is ask whether he is guilty or not guilty of the
accusation. he has to be there
3. at the promulgation of the judgment (general rule: He has to be there kasi baka guilty)
Page 172 of 236
Constitutional Law 2 Bill of Rights Armando Santiago Jr

TRIAL MUST BE PUBLIC


Rule 119 section 21
Exclusion of the public. The judge may, motu proprio, exclude the
public from the courtroom if the evidence to be produced during the
trial is offensive to decency or public morals. He may also, on motion
of the accused, exclude the public from the trial except court
personnel and the counsel of the parties.

ART. III. SEC. 3.


Civilian authority is, at all times, supreme over the military. xxx

Sec. 10. Law on speedy trial not a bar to provision on speedy trial in
the Constitution No provision of law on speedy trial and no rule
implementing the same shall be interpreted as a bar to any charge of
denial of the right to speedy trial guaranteed by Section 14(2), Article
III, of the 1987 Constitution.
IMPARTIAL TRIAL:
A civilian cannot be tried by a military court so long as the civil courts are open and operating, even during
Martial Law. [Olaguer vs. Military Commission (1987)]

Dismissal based on the denial of the right to speedy trial amounts to an acquittal. [Acevedo vs. Sarmiento
(1970)]

Note: RA 8493 provides: a 30-day arraignment within the filing of the information or from the date the accused
appeared before the court; trial shall commence 30 days from the arraignment, as fixed by the court. The entire
trial period shall not exceed 180 days, except as otherwise authorized by the SC Chief Justice.

The right to a speedy trial is violated only when the proceeding is attended by vexatious, capricious and
oppressive delays, or when unjustified postponements of the trial are asked for and secured, or when without
cause or justifiable motive, a long period of time is allowed to elapse without the party having his case tried.
[dela Rosa v. Court of Appeals, 253 SCRA 499; Tai Lim v. Court of Appeals, G.R. No. 131483, October 26,
1999]

The different interests of the defendant which the right to speedy trail are designed to protect are:
(1) To prevent oppressive pre-trail incarceration,
(2) To minimize anxiety and concern of the accused,
(3) To limit the possibility that the defense will be impaired.

But the right to speedy trail cannot be invoked where to sustain the same would result in a clear denial of due
process to the prosecution. In essence, the right to a speedy trial does not preclude the peoples equally
important right to public justice. [Uy v. Hon. Adriano, G.r. No. 159098, October 27, 2006]

RA 8493 is a means of enforcing the right of the accused to a speedy trial. The spirit of the law is that the
accused must go on record in the attitude of demanding a trial or resisting delay. If he does not do this, he must
be held, in law, to have waived the privilege. [Uy v. Hon. Adriano, G.R. No. 159098, October 27, 2006]

Page 173 of 236


Constitutional Law 2 Bill of Rights Armando Santiago Jr

Right of Confrontation
THIS IS THE BASIS OF THE RIGHT TO CROSS-EXAMINATION.
Testimony of a witness who has not submitted himself to cross examination is not admissible in evidence. The
affidavits of witnesses who are not presented during the trial, hence not subjected to cross examination, are
inadmissible because they are hearsay. [People v. Quidate, G.R. No. 117401, October 1, 1998; Cariago v.
Court of Appeals, G.R. No. 143561, June 6, 2001]

Trials In Absentia
WHEN CAN TRIAL IN ABSENTIA BE DONE
Accused failed to appear for trial despite postponement and notice to his bondsmen. The Court then allowed
prosecution to present evidence despite the fact that accused had not been arraigned. Petitioner was found
guilty. The issue is WON the court has jurisdiction. The Court held that because accused was not arraigned, he
was not informed of the nature and cause of accusation against him, Therefore, the Court has no jurisdiction.
The indispensable requisite for trial in absentia is that it should come after arraignment. [Borja vs. Mendoza
(1977)]

After arraignment, during which accused pleaded not guilty, case was set for hearing but the accused escaped.
He was tried in absentia. Lower court held the proceedings against him in abeyance to give him the opportunity
to cross examine witnesses against him and present his evidence.

The Court held that abeyance of proceedings was invalid. Such right to cross examine and present evidence on
his behalf is waived by failure to appear during the trial of which he had notice. [Gimenez vs. Nazareno (1988)]

WHEN PRESENCE OF THE ACCUSED IS A DUTY


(1) Arraignment and Plea
(2) During Trial, for identification
(3) Promulgation of Sentence
(Exception: Light offense -> can be via counsel)

Petitioner challenges the jurisdiction of military commissions to try him (for murder, illegal possession of firearms
and for violation of the Anti- Subversion Act) arguing that he being a civilian, such trial during martial law
deprives him of his right to due process.

An issue has been raised as to WON petitioner could waive his right to be present during trial.
On a 7-5 voting: SEVEN justices voted that petitioner may waive his right to be present at ALL stages of the
proceedings while FIVE voted that this waiver is qualified, he cannot waive when he is to be identified.
Trial in Absentia: As a general rule, subject to certain exceptions, any constitutional or statutory right may be
waived if such waiver is not against public policy.

Considering Art IV, Sec 19, 1973 Constitution (trial of a capital offense may proceed even in the absence of the
accused) and the absence of any law specifically requiring his presence at all stages of his trial, there appears,
no logical reason why petitioner, although he is charged with a capital offense, should be precluded from waiving
his right to be present in the proceedings for the perpetuation of testimony, since this right was conferred upon
him for his protection and benefit. [Aquino vs. Military Commission (1975)]

Page 174 of 236


Constitutional Law 2 Bill of Rights Armando Santiago Jr

Writ of Habeas Corpus


To produce the Body

A Writ directed to a person detaining another and commanding him to produce the body of the prison of a certain
time and place (Meaning of the court, at the hearing schedule) with cause of his capture to explain why the
person is detained and if the nature was not satisfactory the court will order his release.

HABEAS CORPUS
The Supreme Court may:

1) review,
2) in an appropriate proceeding;
3) filed by any citizen,
4) the sufficiency of the factual basis of the proclamation of martial law or the suspension of the privilege of the
writ or the extension thereof, and
5) must promulgate its decision thereon within thirty days from its filing.

A state of martial law does not suspend the operation of the Constitution, nor supplant the functioning of the civil
courts or legislative assemblies, nor authorize the conferment of jurisdiction on military courts and agencies over
civilians where civil courts are able to function, nor automatically suspend the privilege of the writ.

The suspension of the privilege of the writ shall apply only to persons judicially charged for rebellion or offenses
inherent in or directly connected with invasion.

A PRIME SPECIFICATION OF AN APPLICATION FOR A WRIT OF HABEAS


CORPUS IS RESTRAINT OF LIBERTY.
The essential object and purpose of the writ of habeas corpus is to inquire into all manner of involuntary restraint
as distinguished from voluntary, and to relieve a person therefrom if such restraint is illegal. Any restraint which
will preclude freedom of action is sufficient.

The forcible taking of these women from Manila by officials of that city, who handed them over to other parties,
who deposited them in a distant region, deprived these women of freedom of locomotion just as effectively as if
they had been imprisoned. Placed in Davao without either money or personal belongings, they were prevented
from exercising the liberty of going when and where they pleased.

The restraint of liberty which began in Manila continued until the aggrieved parties were returned to Manila and
released or until they freely and truly waived his right.

The true principle should be that, if the respondent is within the jurisdiction of the court and has it in his power to
obey the order of the court and thus to undo the wrong that he has inflicted, he should be compelled to do so.
Even if the party to whom the writ is addressed has illegally parted with the custody of a person before the
application for the writ is no reason why the writ should not issue. [Villavicencio vs. Lukban (1919)]

Petitioners were arrested without warrants and detained, upon the authority of Proclamation 889 (Which
suspended the privilege of the Writ of Habeas Corpus) and subsequently filed a petition for writ of habeas
corpus, assailing the validity of the said Proclamation and their detention.

The Court upheld the violation of the Proclamation and dismissed the petitions. The Supreme Court held that the
authority to suspend the privilege of the writ is circumscribed, confined and restricted, not only by the prescribed
setting or the conditions essential to its existence, but, also, as regards the time when and the place where it
may be exercised.

Page 175 of 236


Constitutional Law 2 Bill of Rights Armando Santiago Jr

Thus, the Court has the authority to inquire into the existence of the factual bases for the proclamation in
order to determine its constitutional sufficiency. The test for such judicial inquiry is whether or not the
Executive acted arbitrarily in issuing the Proclamation. The test is not correctness, but arbitrariness. For the
suspension of the privilege of the writ to be valid,

(a) there must be "invasion, insurrection or rebellion" or, pursuant to paragraph (2), section 10 of Art. VII of the
Constitution, "imminent danger thereof"; and

(b) public safety must require the aforementioned suspension. The President declared in Proclamation No.
889, as amended, that both conditions are present, and the Supreme Court agreed. The President did not act
arbitrarily; the Court acknowledged the existence of a sizeable group of men (Communists and the NPA) who
have publicly risen in arms to overthrow the government and have thus been and still are engaged in
rebellion against the Government of the Philippines. [Lansang vs. Garcia (1971)]

WRIT SUSPENDED VS. PRIVILEGE OF THE WRIT WAS SUSPENDED


Suspension of the writ, the constitution did not allow the suspension of the writ, because if its suspended it may
never be raised the validity of his detention, whereas, if the privilege of writ was suspended, a person may file an
a writ assailing the validity of persons detention, but the court cannot resolve the writ filed during the suspension,
it may only resolved after suspension. (habang suspended and writ di pwedeng iresolve ng court pero
pagkatapos ng suspension tsaka pala lang pwedeng i-resolve o i-review ng court)

The constitution allow the suspension of the privilege of the writ. (writ was never suspended only the privilege)

It is not physical restraint alone which can be inquired into by means of the writ of habeas corpus. In this case,
the petition is valid as petitioners temporary release from detention is accompanied with restrictions w/ the ff
effects:

1) curtailed freedom of movement by the condition that he must get approval of respondents for any travel
outside Metro Manila,
2) abridged liberty of abode because prior approval of respondent is required in case petitioner wants to change
place of residence,
3) abridged freedom of speech due to prohibition from taking any interviews inimical to national security, and
4) petitioner is required to report regularly to respondents or their reps. [Moncupa vs. Enrile (1986)]

It being undeniable that if the Hernandez ruling were to be given retroactive effect, petitioners had served the full
term for which they could have been legally committed, is habeas corpus the appropriate remedy?

YES. Cruz vs. Director of Prisons (1910): "The courts uniformly hold that where a sentence imposes punishment
in excess of the power of the court to impose, such sentence is void as to the excess. The rule is that the
petitioner is not entitled to his discharge on a writ of habeas corpus unless he has served out so much of the
sentence as was valid."

While the above decision speaks of a trial judge losing jurisdiction over the case, insofar as the remedy of
habeas corpus is concerned, the emphatic affirmation that it is the only means of benefiting the accused by the
retroactive character of a favorable decision holds true. Petitioners clearly have thus successfully sustained the
burden of justifying their release. [Gumabon vs. Director of Prisons (1971)]

Sombong claims that she is the mother of the child Christina, who is under the custody of Neri, and filed a
petition for the issuance of the writ of habeas corpus. The Supreme Court denied the petition.

In order to justify the grant of the writ of habeas corpus, the restraint of liberty must be in the nature of an illegal
and involuntary deprivation of freedom of action. However, habeas corpus may still be resorted to even if the
restraint is voluntary in cases where the rightful custody of any person is withheld from the person entitled
thereto. The said writ is the proper legal remedy to enable parents to regain the custody of a minor child even if
the child is in the custody of a third person of her own free will.
Page 176 of 236
Constitutional Law 2 Bill of Rights Armando Santiago Jr

Sombong does not have the right of custody over the child, because the evidence adduced does not warrant the
conclusion that Christina is the same person as her child Arabella. [Sombong vs. CA (1990)]
Larkins was arrested after a certain Alinea filed a complaint-affidavit for rape against him before the NBI. There
was no warrant. A complaint for rape was subsequently filed before the RTC. His common-law wife filed a
petition for habeas corpus.

The Supreme Court held that even if the arrest of a person is illegal, supervening events may bar his release
or discharge from custody. The court must thus look into the legality of his detention as of, at the earliest,
the filing of the application for a writ of habeas corpus, for even if the detention is at its inception illegal, it may,
by reason of some supervening events, such as the instances mentioned in Section 4 of Rule 102, be no longer
illegal at the time of the filing of the application.

Among such supervening events are:

(1) The issuance of a judicial process preventing the discharge of the detained person.
(2) Another is the filing of a complaint or information for the offense for which the accused is detained.

By then, the restraint of liberty is already by virtue of the complaint or information and, therefore, the writ of
habeas corpus is no longer available.

Section 4 of Rule 102 reads in part as follows: "Nor shall anything in this rule be held to authorize the discharge
of a person charged with or convicted an offense in the Philippines." It may also be said that by filing his motion
for bail, Larkins admitted that he was under the custody of the court and voluntarily submitted his person to its
jurisdiction. [Velasco vs. CA (1995)]

RULE 102 of the rules of court


Habeas Corpus Section 1.To what habeas corpus extends. Except as otherwise expressly provided by law,
the writ of habeas corpus shall extend to all cases of illegal confinement or detention by which any person is
deprived of his liberty, or by which the rightful custody of any person is withheld from the person entitled thereto.

1. Illegal detained
2. Custody battle - Ex. yung asawa mo lalake kinuha yung anak nyo below 7 years old. As the law provides, a
child 7 below dapat hindi mahihiwalay sa mother nya tapos kinuha nung tatay. the mother may files a writ
of habeas corpus para ilabas nya yung bata at ibalik sa mother. (suspension made by the president is not
considered in this scenario)

Suspension of the Writ of the habeas corpus


Only the president has the power to suspend the writ of habeas corpus.

WHAT IS A WRIT OF HABEAS CORPUS?


Habeas corpus is a Latin phrase which literally means you have the body. Basically, it is a writ directed to the
person detaining another, commanding him to produce the body of the prisoner at a designated time and place,
with the day and cause of his capture and detention, to do, submit to, and receive whatsoever the court or judge
awarding the writ shall consider in that behalf.

WHAT IS THE SCOPE OF THE WRIT OF HABEAS CORPUS?


The writ of habeas corpus generally extends to all cases of illegal confinement or detention by which a person is
deprived of liberty, or the rightful custody of a person is withheld from the person entitled thereto.

WHO MAY ISSUE THE WRIT OF HABEAS CORPUS?

Page 177 of 236


Constitutional Law 2 Bill of Rights Armando Santiago Jr

The writ may be issued by the Supreme Court or by the Court of Appeals or any member thereof, enforceable
anywhere in the Philippines.

It may also be issued by the Regional Trial Court (RTC) or any judge thereof, enforceable only within the judicial
district.

TO WHOM IS THE WRIT DIRECTED?


If the detention is by an officer, the writ shall be directed to him, commanding him to bring the body of the person
restrained of liberty before the court at the time and place specified. If the detention is by a person other than an
officer, then the writ shall be directed to an officer commanding him to the same effect and to summon the
person restraining. The respondent will be asked to explain the cause of the detention.

WHEN SHALL THE SUBJECT PERSON BE RELEASED?


When the prisoner is unlawfully restrained, the court or judge shall order his discharge which shall not be
effective until a copy of the order is served on the officer or person detaining the prisoner. If such officer or
person does not desire to appeal, the prisoner shall be forthwith released.

CASES

SUMMARY
MALIWAT vs. CA and REPUBLIC OF 107041, 5/15/'96 Accused was charged of falsification of
THE PHILIPPINES public and official documents.

GIMENEZ and MERCADO vs. HON. L-37933 4/15/'88 Application of trial in absentia to a person
NAZARENO and DE LA VEGA, JR who escaped from his detention

People vs. BUENVIAJE l-22945 3 3 '25 Violation of Medical Act. they are merely
different ways or means of committing the
same offense and both of these means are
closely related to each other and usually
employed together

CORPUZ vs. PEOPLE 74259 2 14 '91 Presumption of innocence invoked by corpuz


was not considered which cause the
affirmation of the SC convicting him of crime
of malversation of public funds.

People vs Monteron 130709 3 6 '02 Accused charge of rape wherein the


determination of SC was lighter attempted
rape / Purpose of Arraignment /
constitutional guarantee

Estrada vs Sandiganbayan 148560 11 19 '01

People vs. Purisima l-42050-66; l-46229-32;


l-46313-16; l-46997 11
20 '78

Page 178 of 236


Constitutional Law 2 Bill of Rights Armando Santiago Jr

FELIX BARCELON, vs. G.R. No. 2808


DAVID J. BAKER, JR., AND JOHN
DOE THOMPSON,

IN THE ISSUANCE OF THE WRIT OF G.R. No. L-61388


HABEAS CORPUS FOR DR.
AURORA PARONG ET. AL vs.
JUAN PONCE ENRILE ET. AL

Lim vs. CA G.R. No. 131483

IN THE MATTER OF THE PETITION GR L-33964


FOR HABEAS CORPUS OF DEL
ROSARIO, and ALCALA, vs.GARCIA

MALIWAT VS. COURT OF APPEALS AND REPUBLIC OF THE


PHILIPPINES44
PONENTE: PADILLA, J.

TOPIC: Fair Trial / Rights of the accused

TRIGGER OF THE FACTS: The petitioner was charged separately of falsification of public and official
documents.

TRIGGER OF THE ISSUE: WON petitioner was given a fair trial in the trial court.

TRIGGER OF THE RULING: kahit hindi sya yung mismong gumawa nung crime o nagfalsify still sya ang
considered na master mind ng krimen dahil sya ang makikinificiary nung
resulta nun if magprosper. unless he may prove na hindi sya so sya na ang
magpapakita ng evidence. burdened of proof was transferred to the accused

VERDICT: WHEREFORE, the petition is hereby DENIED and the decision of the Court of
Appeals in CA G.R. Nos. 0942829 dated 29 November 1991, which upholds
the amended decision of the Court of First Instance of Cavite dated 28 June
1988 in Criminal Cases Nos. 158-77 and 159-77 is hereby AFFIRMED en
toto. Costs against petitioner.

END POINT:

CITED DOCTRINE:

SEQUEL OF THE CASE:

IMPORTANT POINT/s

FACTS
The petitioner charged of falsification of public and official documents
Accused having somehow obtained possession of a blank form of a transfer certificate title which is public and
official documents

44 GR. 107041, 5/15/'96


Page 179 of 236
Constitutional Law 2 Bill of Rights Armando Santiago Jr

and filing, typing and inserting on the blank spaces the technical description of a parcel of land ang making it
appear that the same is the owners reconstituted copy of transfer of certificate. wherein, accused was aware
that the said parcel of land is already registered in the name of Green Valley Realty Corporation.
the said public and official document was reconstituted by virtue of the order of the trial court.
Accused pleaded not guilty of crime charged.
certain Atty. Santiago, examined the owners duplicate copies presented and scrutinized by him
a suspicion of having a familiar customary signature of Escolastico Cuevas and appearing in 2 titles.
Atty. Santiago, wrote a letter to the NBI director to report the alleged existence of dubious certificate of title.
Upon investigation, accused maliwat was subpoenas to testify before the NBI

ISSUE
Whether or not petitioner was given a fair trial in the lower court

RULING

SYLLABI
Constitutional Law; Due Process; An accused cannot claim denial of due process where he was able to
testify on his own behalf, and though he was unable to adduce additional documentary evidence that he
claims would establish his innocence, he had sought the postponements and cancellations of the
hearings for no less than forty (40) times, from the date of his arraignment to the promulgation of
judgment, a fact that spanned almost a decade.Under the foregoing facts and circumstances, Maliwat
certainly cannot claim that he was denied due process. The records show that he did testify on his own behalf
and was cross-examined by the prosecution. Admittedly, he was unable to adduce additional documentary
evidence that he claims would establish his innocence and which he now attaches as annexes in his petition for
review and memorandum of law before the Court. But as noted earlier, it was Maliwat who had sought the
postponements and cancellations of the hearings for no less than forty (40) times, from the date of his
arraignment to the promulgation of judgment, a fact that spanned almost a decade (1978 to 1988).

Same; Same; Judges; The fact that the judge, when still a clerk of court, testified for the prosecution in
regard to certain facts directly connected with or arising from the performance of his official duties as
clerk of court, without any reference to or pronouncement as to the innocence or guilt of the accused,
does not render him legally disqualified from sitting and deciding the case.Although admittedly a belated
plea, petitioner argues that there was a mistrial since a vital prosecution witness, then Clerk of Court Rolando
Diaz, became the judge of the case and had no choice but to render a judgment of conviction against him. The
records show that Rolando Diaz, then Clerk of Court of the CFI of Cavite City, indeed testified for the
prosecution. But as explained by the Solicitor General, his testimony was limited to certain facts directly
connected with or arising from the performance of his official duties as Clerk of Court, without any reference to or
pronouncement as to the innocence of guilt of the accused. Under Rule 137, Sec. 1 of the Rules of Court, Judge
Diaz previous actuations did not render him legally disqualified from sitting and deciding the case. The
suggestion that he is not wholly free, disinterested and independent could have been buttressed by the exercise
of his sound discretion in voluntarily disqualifying himself. Yet, the manner in which he exhibited himself during
the trial negates any suspicion of prejudgment in the case.

Same; Same; Same; A judge must not only render a just, correct and impartial decision but should do so
in such a manner as to be free from any suspicion as to his fairness, impartiality and integrity.The
guiding rule is that a judge must not only render a just, correct and impartial decision but should do so in such a
manner as to be free from any suspicion as to his fairness, impartiality and integrity. As applied to the case at
bar, the attitude exhibited by Judge Diaz speaks more of extraordinary leniency to the accused in granting all his
requests for postponements, even to the extent of reconsidering his orders declaring the accused as having
waived his right to present further evidence.

Page 180 of 236


Constitutional Law 2 Bill of Rights Armando Santiago Jr

GIMENEZ AND MERCADO VS. HON. NAZARENO AND DE LA


VEGA, JR45
PONENTE: Gancayco J.

TOPIC: Trial in Absentia / Cross-examination / Right of the accused

TRIGGER OF THE FACTS: Petitioners and private respondent were charged with the crime of rape and
four of them were dismissed and the excluded person DELA VEGA was
remained for he was at large. Petitioners raised the constitutional right of
DELA VEGA as they elevate the case to the SC

TRIGGER OF THE ISSUE: 1. jurisdiction over escapee


2. trial in absentia have the constitutional right
TRIGGER OF THE RULING: Requisites of trial in absentia was complied by the accused therefrom. Thus,
SC ruled granting the petition of the private respondent dismissing the case.

VERDICT: Judgement for the respondent Teodoro dela vega Jr. is hereby reversed and
set aside. the respondent judge is directed to render judgement upon the
innocence or guilt of the herein private respondent. Petition raised was
granted for Dela Vega

END POINT: Trial in absentia has the constitutional right even if the person accused
escape from his detention

CITED DOCTRINE None

SEQUEL OF THE CASE In order to apply trial in absentia the person must complied with the
requisites provided which require that the accused must showed up in
arraignment and escape or fail to appear in trial (as an application of the trial
in absentia) for it is a constitutional right.

IMPORTANT POINT/s The person who escape from his detention retain his rights to cross-examine,
right to confrontation and to present evidence on his behalf. these rights are
can be waived by such failure provided by the court (on this case, failure to
appear during the trial when he was notice may virtually waived)

FACTS
Petitioner were charged with the crime of murder
Accused were arraigned and each of them pleaded not guilty to the crime charged
first hearing, prior thereto, private respondent escaped from his detention center and on the said date failed to
appear in court
petitioner herein, filed a motion with the trial court to proceed with the hearing of the case against all the
accused
praying that respondent Dela Vega Jr. be tried in absentia invoking the provision46 provided in 1973
constitution
Pursuant to the constitutional provision thereof, the trial court proceeded with the trial but nevertheless gave
the private respondent the opportunity to take the witness stand the moment he shows up in court
the lower court rendered a decision dismissing the case against the five accused

45 GR L-37933 4/15/'88

46 Section 19, Article IV of the 1973 Constitution


Page 181 of 236
Constitutional Law 2 Bill of Rights Armando Santiago Jr

except the respondent who had escape from his detention shall remain pending without prejudice on the part
of the said accused to cross examine the witnesses for the prosecution and to present his defense whenever
the court acquire back the jurisdiction over his person.
the petitioners filed a motion for reconsideration questioning the dispositive portion on the ground that it will
render Nugatory the constitutional provision on trial in absentia
but the trial court denied its motion

ISSUE
1. WON a court loses jurisdiction over an accused who after being arraigned, escapes from the custody of the
law
2. WON under Section 199, Article IV of the 1973 Constitution, an accused who has been duly tried in absentia
retains his right to present evidence on his own behalf and to confront and cross-examine witness who
testified against him.

RULING
1. Dela vega acquired jurisdiction over the person of the accused when he appeared during the arraignment
and he pleaded not guilty to the crime charged. In Criminal cases, jurisdiction over the person of the accused
is acquired either by his arrest for voluntary appearance in court.

Such voluntary appearance is accomplished by appearing for arraignment as what respondent did in the
case. he appeared in the arraignment and pleads not guilty to the crime charged, jurisdiction is acquired by
the court over his person and this continues until the termination of the case, NOTWITHSTANDING his
escape from the custody of the law (even if the respondent was at large, the case shall continue)

2. Trial in absentia may be had when the following requisite are present:
(1) that there has been an arraignment;
(2) that the accused has been notified; and
(3) that he fails to appear and his failure to do so is unjustified.

In the application of the requisites thereof:


1. the respondent was arraigned and in the said arraignment he pleaded not guilty
2. he was informed of the scheduled hearing and evidenced by his signature on the issued by the trial court
3. that the respondent escaped from his detention. No explantion for his failure to appear in court in any of the
scheduled hearing was given. even the trial court considered his absence unjustified

Upon termination, the court duty to rule upon the evidence presented in court. the court need not to wait for the
time until the accused who escape from the custody finally decides to appear in court to present his evidence
and the witnesses against him.

Judge contention

A judgment of conviction must still be based upon the evidence presented in court. evidence must prove him
guilty beyond reasonable doubt. there can be no violation of due process since the accused was given the
opportunity to be heard. (innocent unless prove beyond reasonable doubt)

The person who escape from his detention retains his rights to cross-examine and to present evidence on his
behalf. failure to appear on the schedule trial which he had notice, he virtually waived these rights. (Right to
confrontation and cross-examine are personal right and it can be waived)

The person who escape from his detention who has tried in absentia waives his right to present evidence on his
own behalf and to confront an cross examine witnessed who testified against him.

Thus, Respondent Dela Vega was also dismissed from being excluded from four whom dismissed by the court

Page 182 of 236


Constitutional Law 2 Bill of Rights Armando Santiago Jr

SYLLABI
Remedial Law; Criminal Procedure; Jurisdiction; Jurisdiction once acquired is not lost upon the
instance of the parties but continues until the case is terminated.But the question is thiswas that
jurisdiction lost when the accused escaped from the custody of the law and failed to appear during the trial? We
answer this question in the negative. As We have consistently ruled in several earlier cases, jurisdiction once
acquired is not lost upon the instance of parties but continues until the case is terminated.

Same; Same; Same; Arraignment; Where the accused appears at the arraignment and pleads not guilty
to the crime charged, jurisdiction is acquired by the courts over his person and continues until
termination of the case, despite his escape from custody of the law.To capsulize the foregoing
discussion, suffice it to say that where the accused appears at the arraignment and pleads not guilty to the crime
charged, jurisdiction is acquired by the court over his person and this continues until the termination of the case,
notwithstanding his escape from the custody of the law.

Same; Same; Same; Same; Trial in absentia: Conditions for a trial in absentia to be present.Going to the
second part of Section 19, Article IV of the 1973 Constitution aforecited a trial in absentia may be had when the
following requisites are present: (1) that there has been an arraignment; (2) that the accused has been notified;
and (3) that he fails to appear and his failure to do so is unjustified.

Same: Same: Same: Same; Same; Upon termination of a trial in absentia, the court has the duty to rule
upon the evidence presented in court; Reason.Upon the termination of a trial in absentia, the court has the
duty to rule upon the evidence presented in court. The court need not wait for the time until the accused who
escaped from custody finally decides to appear in court to present his evidence and cross-examine the
witnesses against him. To allow the delay of proceedings for this purpose is to render ineffective the
constitutional provision on trial in absentia.

Same; Same; Same; Same; Same; Constitutional Law; Right of the accused to be presumed innocent,
not violated if the judgment is rendered as to the accused tried in absentia; Reason.The contention of
the respondent judge that the right of the accused to be presumed innocent will be violated if a judgment is
rendered as to him is untenable. He is still presumed innocent. A judgment of conviction must still be based upon
the evidence presented in court. Such evidence must prove him guilty beyond reasonable doubt. Also, there can
be no violation of due process since the accused was given the opportunity to be heard.

Same; Same; Same; Same; Same; Waiver of the escapees right who has been tried in absentia to
confrontation and cross-examination of witnesses and to present evidence by his failure to appear
during the trial of which he had notice: Right is a personal right and may be waived.Nor can it be said
that an escapee who has been tried in absentia retains his rights to cross-examine and to present evidence on
his behalf. By his failure to appear during the trial of which he had notice, he virtually waived these rights. This
Court has consistently held that the right of the accused to confrontation and cross-examination of witnesses is a
personal right and may be waived. In the same vein, his right to present evidence on his behalf, a right given to
him for his own benefit and protection, may be waived by him.

PEOPLE VS. BUENVIAJE47


PONENTE: Ostrand J.

TOPIC: Duplicity / Right of the Accused

TRIGGER OF THE FACTS: The accused was charged of the crime which has two complaint stated in the
information

47 GR L-22945 3/3/25
Page 183 of 236
Constitutional Law 2 Bill of Rights Armando Santiago Jr

TRIGGER OF THE ISSUE: WON information charged more than one offense

TRIGGER OF THE RULING: There is no duplicity. Both are in violation of the Medical Law and carry the
same penalty. they are merely different ways or means of committing the
same offense and both of these means are closely related to each other and
usually employed together.

VERDICT: CA decision affirmed convict the appellant of crime charged.

END POINT: we do not think it was the intention of the legislator that each single act should
be regarded as a separate offense and separate information presented for
each.

CITED DOCTRINE None

SEQUEL OF THE CASE a crime was charged to the accused, there was 2 or more charges were
charged to the accused and these charges constitute duplicity. they are not
duplicitous for they are not separate crimes but different means of committing
it.

IMPORTANT POINT/s they are merely different ways or means of committing the same offense and
both of these means are closely related to each other and usually employed
together.

Duplicity is illegal/invalid/unconstitutional because it violated the right of the


accused to be informed of the charge against him (hindi mo kasi alam kung
ano yung dapat mong depensahan)

DUPLICITY - one or more

FACTS
The defendant is accused of the violation of the Medical Act
accused obtained without from the board of Medical Examiners the corresponding certificate of registration for
the practice of medicine in the Philippines voluntarily, illegally and criminally compensation for assisting,
treating and manipulating the head and body of Regino Noble for the purpose of curing him of the ailments,
diseases, pains and physical defects from which she distributed and by letterhead and signs which she
exposed on the door of her office.
she added and prefixed to her name the letters DRA which means Doctor for the purpose of causing the
public to believe that she had received the corresponding title of doctor
On the records, she had no certificate from the board of medical examiners authorizing her to practice
medicine in the Philippines
on that day, she treated and manipulate the head and body of Noble to cure him and the treatment
consisting a thrust by means of application of the hand to the spinal column
she also advertised herself as a doctor of chiropractic wherein she claimed that she was graduate of
doctor in chiropractic
Trial court ruled convicting the respondent guilty of violation of section 2678 of the Administrative code

ISSUE
1. WON demurrer to the information should have been sustained on the ground that said information charged
more than one offense

Page 184 of 236


Constitutional Law 2 Bill of Rights Armando Santiago Jr

2. WON defendant is in reality accused of two separate and distinct offenses, namely, illegal practice of
medicine and illegally representing oneself as a doctor.

RULING
There is no duplicity. there is only one crime charged in the information and that is violation of the Medical
practice Act. there are different acts alleging information they are not separate crimes but different ways of
violating the medical practice act, both of which the accused committed here.

Petition were denied and decision of the appellate court commits no error.

Both are in violation of the Medical Law and carry the same penalty. they are merely different ways or means of
committing the same offense and both of these means are closely related to each other and usually employed
together. we do not think it was the intention of the legislator that each single act should be regarded as a
separate offense and separate information presented for each.

Intention of the legislature was not like this. A person who was found to be smoking opium could be charged in
three different complaints.

1. with the illegal possession of the pipe


2. illegal possession of the opium
3. for smoking the opium

The action of one is an element to commit the whole crime it will constitute duplicity and it is unconstitutional.

SYLLABI
CRIMINAL PROCEDURE; INFORMATION STATING SEVERAL DIFFERENT MEANS OF COMMITTING THE
SAME OFFENSE.The defendant was accused of the violation of the Medical Law. The information charged
both illegal practice of medicine and illegally advertising oneself as a doctor. Held: That the information was not
bad for duplicity inasmuch as the acts charged were merely different means of committing the same offense,
notwithstanding the fact that they are prohibited by separate sections of the statute.

STATUTORY DEFINITIONS.Within the territory over which the legislature has jurisdiction a statutory definition
of a term prevails over the ordinary definition of the same term.

ID.; PRACTICE OF MEDICINE; CHIROPRACTIC.Where chiropractic is by statute made a form of the practice
of medicine, a person holding himself out as a doctor of chiropractic in legal effect represents himself to be a
doctor of medicine.

CONSTITUTIONAL LAW; TlTLE OF AMENDATORY STATUTE.Under constitutional provisions similar to


those of the Organic Act of the Government of the Philippine Islands, the general rule is that a title which
declares a mandatory statute to be an act to amend a' designated section, or the like, of a specified code, is
sufficient and the precise nature of the amendatory act need not be further stated.

ID.; POLICE POWER; CHIROPRACTIC, KNOWLEDGE REQUIRED FOR ENGAGING IN.The subjects in
which an examination is required by section 778 of the Administrative Code, as amended by Act No. 3111, relate
to matters of which a thorough knowledge may be necessary for the proper diagnosis of diseases of the human
body and it is within the police power of the State to require that persons engaging in chiropractic or other
methods of curing human ills should possess such knowledge.

CORPUZ VS. PEOPLE48


48 GR 74259 2/14/91
Page 185 of 236
Constitutional Law 2 Bill of Rights Armando Santiago Jr

PONENTE: CRUZ J.

TOPIC: Presumption of Innocence / Right of the Accused

TRIGGER OF THE FACTS: Corpuz was convicted of the crime of malversation of public funds for being
negligent in the shortage costs of 50K. Presumption of innocence as his
defense was not considered by the SC.

TRIGGER OF THE RULING: His innocence as a defense was not meritorious because he fail to file a case
against those people who malversed the public funds. it is negligent on his
part, he failed to enter on his cash book which may cost shortage.

VERDICT: The petition of Corpuz was hereby denied and convicted of malversation of
public funds

END POINT: Defense of Innocence was not considered for there was no equipoise in this
case.

CITED DOCTRINE: equipoise doctrine is the rule which states that when the evidence of the
prosecution and the defense are so evenly balanced the appreciation of such
evidence calls for tilting of the scales in favor of the accused.

Thus, the evidence for the prosecution must be heavier to overcome the
presumption of innocence of the accused. The constitutional basis of the rule
is Bill of Rights which finds expressions in Sec. 1, par. (a), Rule 115 of the
1985 Rules on Criminal Procedure as amended

(see People v. Argawamon, 215 SCRA 652; People v. Ramilla, G.R. No.
101435, 8 November 1993; People v. De la Iglesia, G.R. No. 110991-92, 24
Feb. 1995).

he invoked equipoise doctrine. In such cases, when the evidence are evenly
weighted, the decision shall be in favor of the accused since his guilt was not
proven beyond reasonable doubt.

SEQUEL OF THE CASE:

IMPORTANT POINT/s: (1) when he entered the said amount in his cash book in March 1981,
he did not make any notation that said amount, though entered, was
not actually received;
(2) At the time he signed the certificate of turn-over, he did not make
any certification that the amount of P50,000.00 should not be charged
against him;
(3) Despite his insistence that Pineda and Martinez misappropriated
the money, he did not file any case, whether civil, criminal or
otherwise, against either or both.

FACTS
As Supervising Accounting Clerk in the Office of the Provincial Treasurer of Nueva Vizcaya
Generozo Corpuz was designated Acting Supervising Cashier in the said Office.
In this capacity, he received collections, disbursed funds and made bank deposits and withdrawals pertaining
to government accounts.
in his designation as Acting Supervising Cashier was terminated,
a Transfer of Accountabilities was effected between the petitioner and his successor.

Page 186 of 236


Constitutional Law 2 Bill of Rights Armando Santiago Jr

The Certificate of Turnover revealed a shortage in the amount of P72,823.08.


A letter of demand requiring Corpuz to produce the missing amount but he was able to pay only P10,159.50.
The balance was demanded in another letter
This was subsequently reduced by P12,067.51
through the payment to Corpuz of temporarily disallowed cash items and deductions from his salary before
his dismissal from the service.
a final letter of demand for the total deficiency of P50,596.07 was sent to Corpuz.
The demand not having been met, an information for malversation of the said amount was filed against him
with the Sandiganbayan
Corpuz insists, that he is not guilty of the charge because the shortage imputed to him was malversed by other
persons.
the court found Corpuz guilty beyond reasonable doubt as principal of the crime of Malversation of Public
Funds,
He was punished with reclusion temporal and to restitute to the provincial government of Nueva Vizcaya the
sum of P50,596.07 which is the amount misappropriated, and to pay the costs of the suit.
Further, the court ordered Corpuz to suffer the penalty of perpetual special disqualification, and to pay a fine
equal to the amount embezzled. Hence, the petition.

ISSUE
Whether the equipoise rule applies in the present case.

HELD
Except for Check 958525, which was only entered in Corpuz's Cash Book on 31 March 1981, or 3 months after
its issuance and encashment, all the other 3 were duly entered. Then Check 956639 which was issued and
encashed on the same day as Check 958525, was duly entered in his Cash Book. Non-entry of the latter check
on time was a subtle way of camouflaging the embezzlement of its money equivalent.

There seems to be no logical reason why Checks 956639 and 958525 could not have been liquidated together
by Diosdado Pineda who used the proceeds to pay salary differentials of government officials and employees of
the province of Nueva Vizcaya, since these have been issued and encashed on the same day.

Corpuz could not have been absent since his Employee's Leave Card, wherein his earned leaves are indicated,
shows that during the month of December 1980, he earned 1.25 days vacation leave and 1.25 days sick leave,
which is the same number of days vacation and sick leaves that he earned monthly from 7 July 1976 to October
1981.

Moreover, even if it were true that he was absent on 23 December 1980, the day when Check 958525 was
issued and encashed, yet, the other check which was issued and encashed on the same day was duly
liquidated.

These findings are mainly factual and are based on substantial evidence. There is no reason to disturb them,
absent any of the exceptional circumstances that will justify their review and reversal.

On the contrary,

the Court is convinced that the facts as established point unmistakably to Corpuz's guilt of the offense
charged.
This conclusion is bolstered by the Solicitor General's observation that Corpuz's denial of responsibility for the
missing P50,000.00 is negated by the following factors:

(1) when he entered the said amount in his cash book in March 1981, he did not make any notation that
said amount, though entered, was not actually received;
(2) At the time he signed the certificate of turn-over, he did not make any certification that the amount of
P50,000.00 should not be charged against him;

Page 187 of 236


Constitutional Law 2 Bill of Rights Armando Santiago Jr

(3) Despite his insistence that Pineda and Martinez misappropriated the money, he did not file any case,
whether civil, criminal or otherwise, against either or both.

Corpuz's claim that he is the victim of a "sinister design" to hold him responsible for a crime he has not
committed is less than convincing.
His attempt to throw the blame on others for his failure to account for the missing money only shows it is he
who is looking for a scapegoat.
The plaintive protest that he is "a small fry" victimized by the "untouchables" during the Marcos regime is a
mere emotional appeal that does not impress at all.
The suggestion that the supposed injustice on Corpuz would be abetted by the Supreme Court unless his
conviction is reversed must be rejected as an arrant presumptuousness.

The equipoise rule invoked by Corpuz is applicable only where the evidence of the parties is evenly
balanced, in which case the constitutional presumption of innocence should tilt the scales in favor of the
accused. There is no such equipoise here.

The evidence of the prosecution is overwhelming and has not been overcome by Corpuz with his nebulous
claims of persecution and conspiracy. The presumed innocence of the accused must yield to the positive
finding that he malversed the sum of P50,310.87 to the prejudice of the public whose confidence he has
breached.

His conviction must be affirmed.

SYLLABI
Same; Same; Same; The equipoise rule invoked by petitioner is applicable only where the evidence of
the parties is evenly balanced, in which case the constitutional presumption of innocence should tilt the
scales in favor of the accused.The equipoise rule invoked by the petitioner is applicable only where the
evidence of the parties is evenly balanced, in which case the constitutional presumption of innocence should tilt
the scales in favor of the accused. There is no such equipoise here. The evidence of the prosecution is
overwhelming and has not been overcome by the petitioner with his nebulous claims of persecution and
conspiracy. The presumed innocence of the accused must yield to the positive finding that he malversed the sum
of P50,310.87 to the prejudice of the public whose confidence he has breached. His conviction must be affirmed.

PEOPLE VS MONTERON49
PONENTE: Ynares-santiago J.

TOPIC: Right of the Accused / Arraignment / Constitutional Guarantee

TRIGGER OF THE FACTS: Mary was walking when she was slingshot by the Monteron to commit his
plans which is to rape mary but he failed to produce the same, for the
resistance of Mary. RTC had erred in convicting Monteron with the crime of
rape

TRIGGER OF THE RULING: RTC erred in ascertaining the facts of the case which cause excessive
judgement through arraignment and constitutional guarantee and in the end
lighter penalty was imposed to the accused which is attempted rape and not
consummated rape

SC ruled in favor of mary and affirmed the decision of the trial court with
modification from consummated rape to attempted rape.

49 GR 130709 3/6/02
Page 188 of 236
Constitutional Law 2 Bill of Rights Armando Santiago Jr

VERDICT: Accused Appellant is found guilty beyond reasonable doubt of the crime of
attempted rape.

END POINT: Arraignment and constitutional guarantee is a condition sine qua non in the
pendency of the case. to ascertain the action and placed it on the right
conviction.

CITED DOCTRINE: N/A

SEQUEL OF THE CASE:

IMPORTANTE POINT/s: Purpose of Arraignment - to apprise the accused of the possible loss of
freedom, even of his life, depending on the nature of the crime imputed to
him, or at the very least to inform him of why the prosecuting arm of the state
mobilized agaisnt him.

Notice of Appeal - Indicative of his innocent

Constitutional due process demands that the accused in a criminal case


should be informed of the nature and cause of the accusation against him.
The rationale behind this constitutional guarantee are:

First, to furnish the accused with the description of the charge against him as
will enable him to make his defense;

second, to avail himself of his conviction or acquittal, for protection against a


further prosecution for the same cause; and third, to inform the court of the
facts alleged, so that it may decide whether they are sufficient in law to
support a conviction, if one should be had.

FACTS
Certain Mary Ann Martenez was walking home from her school in Davao City
while she was waking she was hit on the head by a slingshot
she turned to see where the stone came from
she was hit again on the mouth where she fell down unconscious
when mary get conscious, she found herself lying on the grass and naked
Monteron was lying on top and also naked
she struggled but Monteron was stronger
Monteron placed his penis on top of Mary vagina, where Mary grabbed his erect penis and pushed it away
from her.
thereafter, Mary ran in pain and escape from his possession
Examination result was negative from laceration
on the pendency of the case, Monteron was charge with rape and in the arraignment he pleaded not guilty
RTC ruled in favor of Mary and convicted Monteron of the crime of rape
case was elevated to CA

ISSUE
Whether or not the trial court has committed an error in not acquitting Monteron of the crime charged in the
information

RULING
SC ruled in favor of Mary but modified the decision rendered by the trial court for it was not an consummated
rape but rather attempted rape. it was clearly provides that Monteron only placed his penis on top of her vagina.

Page 189 of 236


Constitutional Law 2 Bill of Rights Armando Santiago Jr

the pain she felt may have been caused by Monteron attempts to insert his organ into the vagina of Mary which
means Monteron penis was merely on top of her vagina and has not actually entered the same. It was Mary who
violently resist to prevent the insertion of Monterons penis in Marys vagina.

SEE SYLLABI to Connect the necessity of constitutional due process and the purpose of arraignment

SYLLABI
Criminal Law; Right to be Informed; Rationale.Constitutional due process demands that the accused in a
criminal case should be informed of the nature and cause of the accusation against him. The rationale behind
this constitutional guarantee are:

First, to furnish the accused with the description of the charge against him as will enable him to make his
defense;

second, to avail himself of his conviction or acquittal, for protection against a further prosecution for the same
cause; and third, to inform the court of the facts alleged, so that it may decide whether they are sufficient in law
to support a conviction, if one should be had.

Same; Same; Criminal Procedure; Arraignment; The purpose of arraignment is to apprise the accused of
the possible loss of freedom, even of his life, depending on the nature of the crime imputed to him, or at
the very least to inform him of why the prosecuting arm of the State is mobilized against him; When an
accused enters a negative plea during his arraignment, the same is not binding on the court as an
indication of his innocence.In fulfillment of the aforesaid constitutional guarantee, Rule 116, Section 1 (a) of
the Rules of Court mandates that an accused be arraigned in open court and asked to enter a plea of guilty or
not guilty of the crime charged. The purpose of arraignment is, thus, to apprise the accused of the possible loss
of freedom, even of his life, depending on the nature of the crime imputed to him, or at the very least to inform
him of why the prosecuting arm of the State is mobilized against him. Consequently, when accused-appellant
entered a negative plea during his arraignment, the same was not binding on the court as an indication of his
innocence. Rather, it is a general denial of the charges impugned against him and an exercise of his right to be
heard of his plea.

Witnesses; Time-tested is the rule that between the positive assertions of prosecution witness and the
negative averments of the accused, the former undisputedly deserves more credence and entitled to
greater evidentiary weight.On the whole, accused-appellant denies having abused and raped Mary Ann.
Time-tested is the rule that between the positive assertions of prosecution witnesses and the negative
averments of the accused, the former undisputedly deserves more credence and entitled to greater evidentiary
weight. Moreover, positive identification of the accused by prosecution witnesses as to his participation in the
crime cannot be overcome by his denial of participation.

Same; It is axiomatic that negative assertions cannot prevail over the positive testimonies of credible
witnesses.It is axiomatic that negative assertions cannot prevail over the positive testimonies of credible
witnesses. Thus, the accused-appellants denial, not being substantiated by clear and convincing evidence, is
negative and self-serving evidence bearing no weight in law. Moreover, the defense of denial is inherently weak
and has always been viewed with disfavor by the courts due to the facility with which they can be concocted.

Same; It is a basic precept that relationship per se of a witness with the victim does not necessarily
mean that he is biased.Accused-appellant challenges the testimony of Arnel Arat, saying that he was a
biased witness because he is Mary Anns cousin. It is a basic precept that relationship per se of a witness with
the victim does not necessarily mean that he is biased. On the contrary, relatives have more interest in telling the
truth for they want the real culprits to be meted their punishment. To be sure, there is no law disqualifying a
person from testifying in a criminal case in which his relative is involved if the former was at the scene of the
crime and witnessed the execution of the crime. Thus, the relationship of Arnel Arat to Mary Ann does not impair
the credibility of his testimony, especially so when the same was given in a clear, convincing and straightforward
manner.

Page 190 of 236


Constitutional Law 2 Bill of Rights Armando Santiago Jr

ESTRADA VS SANDIGANBAYAN50
PONENTE: BELLOSILLO, J.

TOPIC: vagueness of the plunder law

TRIGGER OF THE FACTS: Estrada was charged of violation of plunder law but he alleged that the
plunder law was vague or ambigous

TRIGGER OF THE ISSUE: WON the plunder law is vague or ambigous

TRIGGER OF THE RULING: The law is not vague and it violates the right of the accused to be informed of
the charges against him.

VERDICT: PREMISES CONSIDERED, this Court holds that RA 7080 otherwise known
as the Plunder Law, as amended by RA 7659, is CONSTITUTIONAL.
Consequently, the petition to declare the law unconstitutional is DISMISSED
for lack of merit.

END POINT: If the law is vague, it violates the right of the accused to be informed of the
charges against him.

CITED DOCTRINE None

SEQUEL OF THE CASE there was a law charged to the accused thereof, the law is vague or has a
defect the accused right was violated which is to inform of the charges against
him because he doesnt now how the law was violated because the law is not
clear.

IMPORTANT POINT/s A law is vague

1. if fails to give fair warning as to what conduct to avoid (hindi mo alam kung
ano yung gagawin mo, kung ano yung iiwasan mo at kung ano ang bawal)
2. it gives law enforces unbridled (limitless) discretion in its enforcement
(hindi mo alam kung naviolate mo ba yung batas ohindi sila ang
magsasabing vinaolate mo.)

If the law is vague, it violates the right of the accused to be informed of the
charges against him.

FACTS
Petitioner Joseph Estrada prosecuted An Act Defining and Penalizing the Crime of Plunder,
wishes to impress upon the Court that the assailed law is so defectively fashioned that it crosses that thin but
distinct line which divides the valid from the constitutionally infirm.
His contentions are mainly based on the effects of the said law that it suffers from the vice of vagueness;
it dispenses with the "reasonable doubt" standard in criminal prosecutions;
and it abolishes the element of mens rea in crimes already punishable under The Revised Penal Code saying
that it violates the fundamental rights of the accused.
The focal point of the case is the alleged vagueness of the law in the terms it uses.
Particularly, this terms are: combination, series and unwarranted.
Because of this, the petitioner uses the facial challenge on the validity of the mentioned law.

50 GR. 148560, 11/19/01


Page 191 of 236
Constitutional Law 2 Bill of Rights Armando Santiago Jr

ISSUE
1. Whether or not the petitioner possesses the locus standi to attack the validity of the law using the facial
challenge.
2. Whether or not Plunder law was vague or ambigous

RULING
On how the law uses the terms combination and series does not constitute vagueness. The petitioners
contention that it would not give a fair warning and sufficient notice of what the law seeks to penalize cannot be
plausibly argued. Void-for-vagueness doctrine is manifestly misplaced under the petitioners reliance since
ordinary intelligence can understand what conduct is prohibited by the statute. It can only be invoked against
that specie of legislation that is utterly vague on its face, wherein clarification by a saving clause or construction
cannot be invoked. Said doctrine may not invoked in this case since the statute is clear and free from ambiguity.
Vagueness doctrine merely requires a reasonable degree of certainty for the statute to be upheld, not absolute
precision or mathematical exactitude.

On the other hand, overbreadth doctrine decrees that governmental purpose may not be achieved by means
which sweep unnecessarily broadly and thereby invade the area of protected freedoms.

Doctrine of strict scrutiny holds that a facial challenge is allowed to be made to vague statute and to one which is
overbroad because of possible chilling effect upon protected speech. Furthermore, in the area of criminal law,
the law cannot take chances as in the area of free speech. A facial challenge to legislative acts is the most
difficult challenge to mount successfully since the challenger must establish that no set of circumstances exists.

Doctrines mentioned are analytical tools developed for facial challenge of a statute in free speech cases. With
respect to such statue, the established rule is that one to who application of a statute is constitutional will not be
heard to attack the statute on the ground that impliedly it might also be taken as applying to other persons or
other situations in which its application might be unconstitutional. On its face invalidation of statues results in
striking them down entirely on the ground that they might be applied to parties not before the Court whose
activities are constitutionally protected. It is evident that the purported ambiguity of the Plunder Law is more
imagined than real.

The crime of plunder as a malum in se is deemed to have been resolve in the Congress decision to include it
among the heinous crime punishable by reclusion perpetua to death.

Supreme Court holds the plunder law constitutional and petition is dismissed for lacking merit.

SYLLABI
Constitutional Law; Anti-Plunder Law (R.A. 7080); Statutes; Statutory Construction; The whole gamut of
legal concepts pertaining to the validity of legislation is predicated on the basic principle that a
legislative measure is presumed to be in harmony with the Constitution.Preliminarily, the whole gamut of
legal concepts pertaining to the validity of legislation is predicated on the basic principle that a legislative
measure is presumed to be in harmony with the Constitution. Courts invariably train their sights on this
fundamental rule whenever a legislative act is under a constitutional attack, for it is the postulate of constitutional
adjudication. This strong predilection for constitutionality takes its bearings on the idea that it is forbidden for one
branch of the government to encroach upon the duties and powers of another. Thus it has been said that the
presumption is based on the deference the judicial branch accords to its coordinate branchthe legislature. If
there is any reasonable basis upon which the legislation may firmly rest, the courts must assume that the
legislature is ever conscious of the borders and edges of its plenary powers, and has passed the law with full
knowledge of the facts and for the purpose of promoting what is right and advancing the welfare of the majority.
Hence, in determining whether the acts of the legislature are in tune with the fundamental law, courts should
proceed with judicial restraint and act with caution and forbearance. Every intendment of the law must be
adjudged by the courts in favor of its constitutionality, invalidity being a measure of last resort. In construing
therefore the provisions of a statute, courts must first ascertain whether an interpretation is fairly possible to
sidestep the question of constitutionality.

Page 192 of 236


Constitutional Law 2 Bill of Rights Armando Santiago Jr

Same; Same; Same; Same; Criminal Law; As it is written, the Plunder Law contains ascertainable
standards and well-defined parameters which would enable the accused to determine the nature of his
violation; As long as the law affords some comprehensible guide or rule that would inform those who
are subject to it what conduct would render them liable to its penalties, its validity will be sustained.As
it is written, the Plunder Law contains ascertainable standards and well-defined parameters which would enable
the accused to determine the nature of his violation. Section 2 is sufficiently explicit in its description of the acts,
conduct and conditions required or forbidden, and prescribes the elements of the crime with reasonable certainty
and particularity. x x x As long as the law affords some comprehensible guide or rule that would inform those
who are subject to it what conduct would render them liable to its penalties, its validity will be sustained. It must
sufficiently guide the judge in its application; the counsel, in defending one charged with its violation; and more
importantly, the accused, in identifying the realm of the proscribed conduct. Indeed, it can be understood with
little difficulty that what the assailed statute punishes is the act of a public officer in amassing or accumulating ill-
gotten wealth of at least P50,000,000.00 through a series or combination of acts enumerated in Sec. 1, par. (d),
of the Plunder Law.

Same; Same; Same; Same; Void for Vagueness Doctrine; A statute is not rendered uncertain and void
merely because general terms are used therein, or because of the employment of terms without defining
them; much less do we have to define every word we use.Petitioner, however, bewails the failure of the
law to provide for the statutory definition of the terms combination and series in the key phrase a combination
or series of overt or criminal acts found in Sec.1,par.(d),andSec.2,and the word pattern in Sec. 4. These
omissions, according to petitioner, render the Plunder Law unconstitutional for being impermissibly vague and
overbroad and deny him the right to be informed of the nature and cause of the accusation against him, hence,
violative of his fundamental right to due process. The rationalization seems to us to be pure sophistry. A statute
is not rendered uncertain and void merely because general terms are used therein, or because of the
employment of terms without defining them; much less do we have to define every word we use. Besides, there
is no positive constitutional or statutory command requiring the legislature to define each and every word in an
enactment. Congress is not restricted in the form of expression of its will, and its inability to so define the words
employed in a statute will not necessarily result in the vagueness or ambiguity of the law so long as the
legislative will is clear, or at least, can be gathered from the whole act, which is distinctly expressed in the
Plunder Law.

Same; Same; Same; Same; It is a well-settled principle of legal hermeneutics that words of a statute will
be interpreted in their natural, plain and ordinary acceptation and signification, unless it is evident that
the legislature intended a technical or special legal meaning to those words.It is a well-settled principle
of legal hermeneutics that words of a statute will be interpreted in their natural, plain and ordinary acceptation
and signification, unless it is evident that the legislature intended a technical or special legal meaning to those
words. The intention of the lawmakerswho are, ordinarily, untrained philologists and lexicographersto use
statutory phraseology in such a manner is always presumed. Thus, Websters New Collegiate Dictionary
contains the following commonly accepted definition of the words combination and series: Combinationthe
result or product of combining; the act or process of combining. To combine is to bring into such close
relationship as to obscure individual characters. Seriesa number of things or events of the same class coming
one after another in spatial and temporal succession.

Same; Same; Same; Same; Words and Phrases; Combination, Explained.Thus when the Plunder Law
speaks of combination, it is referring to at least two (2) acts falling under different categories of enumeration
provided in Sec. 1, par. (d), e.g., raids on the public treasury in Sec. 1, par. (d), subpar. (1), and fraudulent
conveyance of assets belonging to the National Government under Sec. 1, par. (d), subpar. (3).

Same; Same; Same; Same; Same; Series, Explained.On the other hand, to constitute a series there
must be two (2) or more overt or criminal acts falling under the same category of enumeration found in Sec. 1,
par. (d), say, misappropriation, malversation and raids on the public treasury, all of which fall under Sec. 1, par.
(d), subpar. (1). Verily, had the legislature intended a technical or distinctive meaning for combination and
series, it would have taken greater pains in specifically providing for it in the law.

Same; Same; Same; Same; Same; Pattern, Explained.As for pat-tern, we agree with the observations of
the Sandiganbayan that this term is sufficiently defined in Sec.4, in relation to Sec.1,par.(d),andSec.2.As for

Page 193 of 236


Constitutional Law 2 Bill of Rights Armando Santiago Jr

pattern, we agree with the observations of the Sandiganbayan that this term is sufficiently defined in Sec. 4, in
relation to Sec. 1, par. (d), and Sec. 2xxxx under Sec. 1 (d) of the law, a pattern consists of at least a
combination or series of overt or criminal acts enumerated in subsections (1) to (6) of Sec. 1 (d). Secondly,
pursuant to Sec. 2 of the law, the pattern of overt or criminal acts is directed towards a common purpose or goal
which is to enable the public officer to amass, accumulate or acquire ill-gotten wealth. And thirdly, there must
either be an overall unlawful scheme or conspiracy to achieve said common goal. As commonly understood,
the term overall unlawful scheme indicates a general plan of action or method which the principal accused and
public officer and others conniving with him follow to achieve the aforesaid common goal. In the alternative, if
there is no such overall scheme or where the schemes or methods used by multiple accused vary, the overt or
criminal acts must form part of a conspiracy to attain a common goal.

Same; Same; Criminal Law; Void for Vagueness Doctrine; Words and Phrases; The void-for-
vagueness doctrine has been formulated in various ways, but is most commonly stated to the effect
that a statute establishing a criminal offense must define the offense with sufficient definiteness that
persons of ordinary intelligence can understand what conduct is prohibited by the statuteit can only be
invoked against that specie of legislation that is utterly vague on its face, i.e., that which cannot be clarified either
by a saving clause or by construction.It cannot plausibly be contended that the law does not give a fair
warning and sufficient notice of what it seeks to penalize. Under the circumstances, petitioner s reliance on the
void-for-vagueness doctrine is manifestly misplaced. The doctrine has been formulated in various ways, but is
most commonly stated to the effect that a statute establishing a criminal offense must define the offense with
sufficient definiteness that persons of ordinary intelligence can understand what conduct is prohibited by the
statute. It can only be invoked against that specie of legislation that is utterly vague on its face, i.e., that which
cannot be clarified either by a saving clause or by construction.

Same; Same; Same; Same; Due Process; When a statute lacks comprehensible standards that men of
common intelligence must necessarily guess at its meaning and differ in its application, the statute is
repugnant to the Constitution in two (2) respectsit violates due process for failure to accord persons,
especially the parties targeted by it, fair notice of what conduct to avoid, and, it leaves law enforcers unbridled
discretion in carrying out its provisions and becomes an arbitrary flexing of the Government muscle.A statute
or act may be said to be vague when it lacks comprehensible standards that men of common intelligence must
necessarily guess at its meaning and differ in its application. In such instance, the statute is repugnant to the
Constitution in two (2) respectsit violates due process for failure to accord persons, especially the parties
targeted by it, fair notice of what conduct to avoid; and, it leaves law enforcers unbridled discretion in carrying
out its provisions and becomes an arbitrary flexing of the Government muscle. But the doctrine does not apply
as against legislations that are merely couched in imprecise language but which nonetheless specify a standard
though defectively phrased; or to those that are apparently ambiguous yet fairly applicable to certain types of
activities. The first may be saved by proper construction, while no challenge may be mounted as against the
second whenever directed against such activities. With more reason, the doctrine cannot be invoked where the
assailed statute is clear and free from ambiguity, as in this case.

Same; Same; Same; Same; The test in determining whether a criminal statute is void for uncertainty is
whether the language conveys a sufficiently definite warning as to the proscribed conduct when
measured by common understanding and practice; The vagueness doctrine merely requires a
reasonable degree of certainty for the statute to be upheldnot absolute precision or mathematical
exactitude.The test in determining whether a criminal statute is void for uncertainty is whether the language
conveys a sufficiently definite warning as to the proscribed conduct when measured by common understanding
and practice. It must be stressed, however, that the vagueness doctrine merely requires a reasonable degree
of certainty for the statute to be upheldnot absolute precision or mathematical exactitude, as petitioner seems
to suggest. Flexibility, rather than meticulous specificity, is permissible as long as the metes and bounds of the
statute are clearly delineated. An act will not be held invalid merely because it might have been more explicit in
its wordings or detailed in its provisions, especially where, because of the nature of the act, it would be
impossible to provide all the details in advance as in all other statutes.

Same; Same; Same; Same; Overbreadth Doctrine; Facial Challenges; The allegations that the Plunder
Law is vague and overbroad do not justify a facial review of its validity.Moreover, we agree with, hence
we adopt, the observations of Mr. Justice Vicente V. Mendoza during the deliberations of the Court that the

Page 194 of 236


Constitutional Law 2 Bill of Rights Armando Santiago Jr

allegations that the Plunder Law is vague and overbroad do not justify a facial review of its validityThe void-
forvagueness doctrine states that a statute which either forbids or requires the doing of an act in terms so vague
that men of common intelligence must necessarily guess at its meaning and differ as to its application, violates
the first essential of due process of law. The overbreadth doctrine, on the other hand, decrees that a
governmental purpose may not be achieved by means which sweep unnecessarily broadly and thereby invade
the area of protected freedoms. A facial challenge is allowed to be made to a vague statute and to one which is
overbroad because of possible chilling effect upon protected speech. The theory is that [w]hen statutes
regulate or proscribe speech and no readily apparent construction suggests itself as a vehicle for rehabilitating
the statutes in a single prosecution, the transcendent value to all society of constitutionally protected expression
is deemed to justify allowing attacks on overly broad statutes with no requirement that the person making the
attack demonstrate that his own conduct could not be regulated by a statute drawn with narrow specificity. The
possible harm to society in permitting some unprotected speech to go unpunished is outweighed by the
possibility that the protected speech of others may be deterred and perceived grievances left to fester because
of possible inhibitory effects of overly broad statutes. This rationale does not apply to penal statutes. Criminal
statutes have general in terrorem effect resulting from their very existence, and, if facial challenge is allowed for
this reason alone, the State may well be prevented from enacting laws against socially harmful conduct. In the
area of criminal law, the law cannot take chances as in the area of free speech. The overbreadth and vagueness
doctrines then have special application only to free speech cases. They are inapt for testing the validity of penal
statutes.

same; Same; Same; Same; Same; Statutory Construction; Ambiguity, where none exists, cannot be
created by dissecting parts and words in the statute to furnish support to critics who cavil at the want of
scientific precision in the law; It will take more than nitpicking to overturn the wellentrenched
presumption of constitutionality and validity of the Plunder Law.In light of the foregoing disquisition, it is
evident that the purported ambiguity of the Plunder Law, so tenaciously claimed and argued at length by
petitioner, is more imagined than real. Ambiguity, where none exists, cannot be created by dissecting parts and
words in the statute to furnish support to critics who cavil at the want of scientific precision in the law. Every
provision of the law should be construed in relation and with reference to every other part. To be sure, it will take
more than nitpicking to overturn the well-entrenched presumption of constitutionality and validity of the Plunder
Law. A fortiori, petitioner cannot feign ignorance of what the Plunder Law is all about. Being one of the Senators
who voted for its passage, petitioner must be aware that the law was extensively deliberated upon by the Senate
and its appropriate committees by reason of which he even registered his affirmative vote with full knowledge of
its legal implications and sound constitutional anchorage.

Criminal Law; Anti-Plunder Law; Presumption of Innocence; Reasonable Doubt Standard; In a criminal
prosecution for plunder, as in all other crimes, the accused always has in his favor the presumption of
innocence which is guaranteed by the Bill of Rights, and unless the State succeeds in demonstrating by
proof beyond reasonable doubt that culpability lies, the accused is entitled to an acquittalthe use of the
reasonable doubt standard is indispensable to command the respect and confidence of the community in the
application of criminal law.The running fault in this reasoning is obvious even to the simplistic mind. In a
criminal prosecution for plunder, as in all other crimes, the accused always has in his favor the presumption of
innocence which is guaranteed by the Bill of Rights, and unless the State succeeds in demonstrating by proof
beyond reasonable doubt that culpability lies, the accused is entitled to an acquittal. The use of the reasonable
doubt standard is indispensable to command the respect and confidence of the community in the application of
criminal law. It is critical that the moral force of criminal law be not diluted by a standard of proof that leaves
people in doubt whether innocent men are being condemned. It is also important in our free society that every
individual going about his ordinary affairs has confidence that his government cannot adjudge him guilty of a
criminal offense without convincing a proper factfinder of his guilt with utmost certainty. This reasonable doubt
standard has acquired such exalted stature in the realm of constitutional law as it gives life to the Due Process
Clause which protects the accused against conviction except upon proof beyond reasonable doubt of every fact
necessary to constitute the crime with which he is charged.

Same; Same; Constitutional Law; Death Penalty Law (R.A. 7659); It is now too late in the day to resurrect
the issue of the constitutionality of R.A. 7659, the same having been eternally consigned by People v.
Echegaray, 267 SCRA 682 (1997), to the archives of jurisprudential history.To clinch, petitioner likewise
assails the validity of RA 7659, the amendatory law of RA 7080, on constitutional grounds. Suffice it to say,

Page 195 of 236


Constitutional Law 2 Bill of Rights Armando Santiago Jr

however, that it is now too late in the day for him to resurrect this long dead issue, the same having been
eternally consigned by People vs. Echegaray to the archives of jurisprudential history. The declaration of this
Court therein that RA 7659 is constitutionally valid stands as a declaration of the State, and becomes, by
necessary effect, assimilated in the Constitution now as an integral part of it.

CONCURRING
Same; Same; Strict scrutiny is used today to test the validity of laws dealing with the regulation of
speech, gender, or race and facial challenges are allowed for this purpose.Hence, strict scrutiny is used
today to test the validity of laws dealing with the regulation of speech, gender, or race and facial challenges are
allowed for this purpose. But criminal statutes, like the Anti-Plunder Law, while subject to strict construction, are
not subject to strict scrutiny. The two (i.e., strict construction and strict scrutiny) are not the same. The rule of
strict construction is a rule of legal hermeneutics which deals with the parsing of statutes to determine the intent
of the legislature. On the other hand, strict scrutiny is a standard of judicial review for determining the quality and
the amount of governmental interest brought to justify the regulation of fundamental freedoms. It is set opposite
such terms as deferential review and intermediate review.

Same; Same; Under deferential review, laws are upheld if they rationally further a legitimate
governmental interest, without courts seriously inquiring into the substantiality of such interest and
examining the alternative means by which the objectives could be achieved.Thus, under deferential
review, laws are upheld if they rationally further a legitimate governmental interest, without courts seriously
inquiring into the substantiality of such interest and examining the alternative means by which the objectives
could be achieved. Under intermediate review, the substantiality of the governmental interest is seriously looked
into and the availability of less restrictive alternatives are considered. Under strict scrutiny, the focus is on the
presence of compelling, rather than substantial, governmental interest and on the absence of less restrictive
means for achieving that interest.

Same; Same; Void for Vagueness Doctrine; Overbreadth Doctrine; Facial Challenges; Words and
Phrases; The void-for-vagueness doctrine states that a statute which either forbids or requires the
doing of an act in terms so vague that men of common intelligence must necessarily guess at its
meaning and differ as to its application, violates the first essential of due process of law; The
overbreadth doctrine decrees that a governmental purpose may not be achieved by means which
sweep unnecessarily broadly and thereby invade the area of protected freedoms.Nor do allegations that
the Anti-Plunder Law is vague and overbroad justify a facial review of its validity. The void-for-vagueness
doctrine states that a statute which either forbids or requires the doing of an act in terms so vague that men of
common intelligence must necessarily guess at its meaning and differ as to its application, violates the first
essential of due process of law. The over-breadth doctrine, on the other hand, decrees that a governmental
purpose may not be achieved by means which sweep unnecessarily broadly and thereby invade the area of
protected freedoms.

Same; Same; Same; Same; Same; Same; A facial challenge is allowed to be made to a vague statute and
to one which is overbroad because of possible chilling effect upon protected speech, the theory being
that [w]hen statutes regulate or proscribe speech and no readily apparent construction suggests itself
as a vehicle for rehabilitating the statutes in a single prosecution, the transcendent value to all society of
constitutionally protected expression is deemed to justify allowing attacks on overly broad statutes with
no requirement that the person making the attack demonstrate that his own conduct could not be
regulated by a statute drawn with narrow specificity.A facial challenge is allowed to be made to a vague
statute and to one which is overbroad because of possible chilling effect upon protected speech. The theory is
that [w]hen statutes regulate or proscribe speech and no readily apparent construction suggests itself as a
vehicle for rehabilitating the statutes in a single prosecution, the transcendent value to all society of
constitutionally protected expression is deemed to justify allowing attacks on overly broad statutes with no
requirement that the person making the attack demonstrate that his own conduct could not be regulated by a
statute drawn with narrow specificity. The possible harm to society in permitting some unprotected speech to go
unpunished is outweighed by the possibility that the protected speech of others may be deterred and perceived
grievances left to fester because of possible inhibitory effects of overly broad statutes.

Page 196 of 236


Constitutional Law 2 Bill of Rights Armando Santiago Jr

Same; Same; Same; Same; Same; Same; The overbreadth and vagueness doctrines have special
application only to free speech casesthey are inapt for testing the validity of penal statutes.This rationale
does not apply to penal statutes. Criminal statutes have general in terrorem effect resulting from their very
existence, and, if facial challenge is allowed for this reason alone, the State may well be prevented from
enacting laws against socially harmful conduct. In the area of criminal law, the law cannot take chances as in the
area of free speech. The overbreadth and vagueness doctrines then have special application only to free speech
cases. They are inapt for testing the validity of penal statutes. As the U.S. Supreme Court put it, in an opinion by
Chief Justice Rehnquist, we have not recognized an overbreadth doctrine outside the limited context of the
First Amendment. In Broadrick v. Oklahoma, the Court ruled that claims of facial overbreadth have been
entertained in cases involving statutes which, by their terms, seek to regulate only spoken words and, again,
that overbreadth claims, if entertained at all, have been curtailed when invoked against ordinary criminal laws
that are sought to be applied to protected conduct. For this reason, it has been held that a facial challenge to a
legislative act is the most difficult challenge to mount successfully, since the challenger must establish that no
set of circumstances exists under which the Act would be valid. As for the vagueness doctrine, it is said that a
litigant may challenge a statute on its face only if it is vague in all its possible applications. A plaintiff who
engages in some conduct that is clearly proscribed cannot complain of the vagueness of the law as applied to
the conduct of others.

Same; Same; Same; Same; Same; Same; The doctrines of strict scrutiny, overbreadth, and vagueness
are analytical tools developed for testing on their faces statutes in free speech cases or, as they are
called in American law, First Amendment Cases.In sum, the doctrines of strict scrutiny, overbreadth, and
vagueness are analytical tools developed for testing on their faces statutes in free speech cases or, as they are
called in American law, First Amendment cases. They cannot be made to do service when what is involved is a
criminal statute. With respect to such statute, the established rule is that one to whom application of a statute is
constitutional will not be heard to attack the statute on the ground that impliedly it might also be taken as
applying to other persons or other situations in which its application might be unconstitutional. As has been
pointed out, vagueness challenges in the First Amendment context, like overbreadth challenges typically
produce facial invalidation, while statutes found vague as a matter of due process typically are invalidated [only]
as applied to a particular defendant. Consequently, there is no basis for petitioners claim that this Court review
the Anti-Plunder Law on its face and in its entirety.

PEOPLE VS. PURISIMA51


PONENTE: MUOZ PALMA, J.

TOPIC:

TRIGGER OF THE FACTS: Violation of PD. 9 found in possession of a knife outside his residence.
according to the accused it does not applied to them because PD 9 must be
on the occasion of insurrection, rebellion and lawless violence.

TRIGGER OF THE ISSUE:

TRIGGER OF THE RULING: the preamble of PD 9

51 Consolidated cases in one decision GR. L-42050-66; l-46229-32; l-46313-16; l-46997 11/20/78
Page 197 of 236
Constitutional Law 2 Bill of Rights Armando Santiago Jr

VERDICT: WHEREFORE, We DENY these 26 Petitions for Review and We AFFIRM the
Orders of respondent Judges dismissing or quashing the Information
concerned, subject however to Our observations made in the preceding of
this Decision regarding the right of the State or Petitioner herein to file either
an amended Information under Presidential Decree No. 9, paragraph 3, or a
new one under other existing statute or city ordinance as the facts may
warrant.

END POINT:

CITED DOCTRINE:

SEQUEL OF THE CASE:

IMPORTANT POINT/s: The preamble of PD No. 9 also clearly concurs to that, though the preamble is
not a part of the statute, it is the key to determine what is the intent and spirit
of the decree and determine what acts fall within the purview of a penal
statute.

FACTS
Informations were filed to 26 individuals from Manila and Samar, individually and separately, before the Courts
of First Instance of Manila and Samar for illegal possession of deadly weapon
or violation of Presidential Decree No. 9 pursuant to Proclamation No. 1081
On the motion to quash by the accused, the three respondent judges:
Judge Purisima and Judge Macaren, both of CFI of Manila; and Judge Polo of CFI of Samar, issued in the
respective cases filed before them an order to quash or dismiss the informations on a common ground
Lack of essential elements to constitute an offense penalized by PD No. 9. The respondent judges stated
that to constitute the said offense, two elements must be present;
(1) possession of any bladed, blunt or pointed weapon outside of residence as stated in par 3;
(2) and intended to use it to commit or abet subversion, rebellion, etc as stated in the preamble of the said
PD.
The People, as petitioners, thru the Solicitor General, contended that the prohibited acts need not be related
to subversive activities and the intent of the accused are irrelevant since its is a statutory offense and
punishing the possession of such deadly weapon is not only to eradicate subversive acts but also criminality in
general. The petitioners also argued that the preamble is not an essential part of an act and cannot prevail
over the text of the law itself.

ISSUE
Whether or not the petitioners arguments as to the intention and scope of PD No. 9 (3) correct?

RULING
NO. The Supreme Court says that the intention of PD No. 9 (3) is to penalize the acts which are those related to
the desired result of Proc. No. 1081 and Gen. Orders Nos. 6 and 7 which are to suppress those who commit or
abet lawlessness, rebellion, subversive acts and the like. The preamble of PD No. 9 also clearly concurs to that,
though the preamble is not a part of the statute, it is the key to determine what is the intent and spirit of the
decree and determine what acts fall within the purview of a penal statute.

SYLLABI
Constitutional Law; Criminal Procedure; It is imperative under the Constitution and Rules of Court, that
an information should designate or mention the specific statute violated.It is a constitutional right of any
person who stands charged in a criminal prosecution to be informed of the nature and cause of the accusation
against him. Pursuant to the above, Section 5, Rule 110 of the Rules of Court, expressly requires that for a
complaint or information to be sufficient it most, inter alia, state the designation of the offense by the statute, and
Page 198 of 236
Constitutional Law 2 Bill of Rights Armando Santiago Jr

the acts of omissions complained of as constituting the offense. This is essential to avoid surprise on the
accused and to afford him the opportunity to prepare his defense accordingly. To comply with these fundamental
requirements of the Constitution and the Rules on Criminal Procedure, it is imperative for the specific statute
violated to be designated or mentioned in the charge. In fact, another compelling reason exists why a
specification of the statute violated is essential in these cases. As stated in the order of respondent Judge
Maceren the carrying of so-called deadly weapons is the subject of another penal statute and a Manila City
Ordinance.

BARCELON VS.BAKER, JR., AND THOMPSON52


PONENTE: JOHNSON, J.

TOPIC: can the supreme court investigate on the suspension of writ issued by the
executive branch / right of the accused

TRIGGER OF THE FACTS: Barcelon was detained and restrained of his liberty and person who applied was
wholly without legal authority and not by virtue of any court. Governor-General
suspending the privilege of the writ of habeas corpus because of the fact that
certain organized bands of ladrones in said provinces were in open insurrection
against the constituted authorities; and the said bands, or parts of them, and
some of their leaders, were still in open resistance to the constituted authorities.

TRIGGER OF THE ISSUE: WON the supreme court has the power to investigate on the issued suspension
of the writ of habeas corpus.

TRIGGER OF THE RULING: No. The SC may not for it is out of their jurisdiction and they are not equipped
with the facilities to conduct such investigation.

VERDICT: For all of the foregoing reasons, the application for the writ of habeas corpus
should be denied, and it is so ordered

END POINT: This case ruled that the supreme court has no power to investigate against the
suspension issued by the executive. for they are not equipped with facilities.

CITED DOCTRINE: NONE

SEQUEL OF THE CASE: Writ of habeas corpus was suspended by the executive under the capacity of
governor-general on the grounds that there is insurrection which cause of
suspension to a certain province. Supreme court cannot investigate for the
validity of the suspension issued by the executive for they are not equipped in
the determination of the truth and untruth prevailing. (investigation is out of
power of the supreme court as this case posit)

IMPORTANT POINT/s: The authority to suspend the privilege of writ of habeas corpus is exclusively
vested in the legislative and executive branches of the government and their
decision is final and conclusive upon the Judicial Department and upon all
persons.
Suspension of writ of habeas corpuz may be valid in this cases of rebellion,
insurrection, or invasion, when the public safety might require it.

FACTS
This case was an application for a writ of habeas corpus which it alleged that Barcelon is detained and
restrained of his liberty at the town of Batangas, in the Province of Batangas,
and that the detention and restraint of the said applicant is wholly without legal authority and not under or by
virtue of any process issued by any court.

52 G.R. No. 2808


Page 199 of 236
Constitutional Law 2 Bill of Rights Armando Santiago Jr

Respondents admit that they are detaining the body of the said Felix Barcelon, but deny the right of the court
to inquire into the reasons therefor by virtue of the resolution issued by the Philippine Commission and the
executive order of the Governor-General suspending the privilege of the writ of habeas corpus in the
Provinces of Cavite and Batangas.
The Philippine Bill section 5 provides that the Governor-General is hereby authorized to suspend writ of
habeas corpus in the said provinces because of the fact that certain organized bands of ladrones in said
provinces were in open insurrection against the constituted authorities; and the said bands, or parts of them,
and some of their leaders, were still in open resistance to the constituted authorities.

ISSUE
Whether or not the judicial department of the Government may investigate the facts upon which the legislative
and executive branches of the Government acted in providing for the suspension of the privilege of the writ of
habes corpus in the province of Cavite and Batangas

RULING
The suspension of the writ of habeas corpus was valid. Supreme court states that they cannot look into the
validity. Since the ground for suspension was insurrection, only the executive has the best access to information
when there is actually rebellion or insurrection and etc. the supreme court is not equipped with the proper
facilities to determine a truth or untruth prevailing in the Philippines. (mas alam ng executive ang issue, unlike
SC they are only within the court) they decide issues but not investigation.

NO. It is the duty of the legislative branch of the Government to make such laws and regulations as will
effectually conserve peace and good order and protect the lives and property of the citizens of the State. It is the
duty of the Governor-General to take such steps as he deems wise and necessary for the purpose of enforcing
such laws. If the judicial department of the Government, or any officer in the Government, has a right to contest
the orders of the President or of the Governor-General under the conditions above supposed, before complying
with such orders, then the hands of the President or the Governor General may be tied until the very object of
the rebels or insurrectors or invaders has been accomplished.

IN THIS CASE, Congress had authority to provide that the President, or the Governor-General, with the approval
of the Philippine Commission, might suspend the privilege of the writ of habeas corpus in cases of rebellion,
insurrection, or invasion, when the public safety might require it.

the conclusion set forth in the said resolution and the said executive order, as to the fact that there existed in the
Provinces of Cavite and Batangas open insurrection against the constituted authorities, was a conclusion
entirely within the discretion of the legislative and executive branches of the Government, after an investigation
of the facts. That one branch of the United States Government in the Philippine Islands has no right to
interfere or inquire into, for the purpose of nullifying the same, the discretionary acts of another
independent department of the Government.

The doctrine that whenever the Constitution or a statute gives a discretionary power to any person, to be
exercised by him upon his own opinion of certain facts, such person is to be considered the sole and exclusive
judge of the existence of those facts has been recognized in this case.

The authority to suspend the privilege of writ of habeas corpus is exclusively vested in the legislative and
executive branches of the government and their decision is final and conclusive upon the Judicial Department
and upon all persons.

Therefore, the application for the writ of habeas corpus is denied

SYLLABI
Page 200 of 236
Constitutional Law 2 Bill of Rights Armando Santiago Jr

WRIT OF HABEAS CORPUS, SUSPENSION OF THE PRIVILEGE OF THE SAME BY THE EXECUTIVE AM)
LEGlSLATIVE DEPARTMENTS OF THE GOVERNMENT.The privilege of the writ of habeas corpus may be
suspended in the Philippine Islands in the case of rebellion, insurrection, and invasion, when the public safety
requires it, by the President of the United States, or by the Governor-General of the Philippine Islands with the
approval of the Philippine Commission.

ID.; ID.A paragraph of section 5 of the act of Congress of July 1, 1002, confers upon the President, or the
Governor-General and the Philippine Commission discretionary power to determine (a) whether or not there
exists the status of rebellion, insurrection, or invasion; (b) whether the public safety requires the suspension of
the privilege of the writ of habeas corpus.

ID.; ID.When the President, or the Governor-General with the approval of the Philippine Commission, declares
that a state of rebellion, insurrection, or invasion exists, this declaration or conclusion is conclusive against the
judicial department of the Government.

ID.; ID.Whenever a statute gives discretionary power to any person, to be exercised by him upon his own
opinion of certain facts, such statute constitutes him the sole and exclusive judge of the existence of those facts.

ID.; ID.After the President, or the Governor-General with the approval of the Philippine Commission, declares
that a state of rebellion, insurrection, or invasion exists, that condition will be considered by the judicial
department of the Government as continuing until the President or the Governor-General shall declare it to be at
an end.

IN THE ISSUANCE OF THE WRIT OF HABEAS CORPUS FOR


DR. PARONG ET. AL. VS. ENRILE ET. AL.53
PONENTE: DE CASTRO, J.

TOPIC: Right of the accused / Reversal of the Lansang Doctrine & Reinstatement of the
Montenegro Doctrine / suspension of habeas corpuz and right to bail at the
same time

TRIGGER OF THE FACTS: padilla ang his companion was raided by PC for there was a valid search
warrant but padilla et al was arrested without just cause and without arrest
warrant. Arrested people were transferred to the place where only arresting
officer knows of the place. Mother of sabino file a petition for writ of habeas
corpuz

TRIGGER OF THE ISSUE: WON was the arrest valid?

TRIGGER OF THE RULING: SC decision in the Lansang Case was reversed and the ruling in the Barcelon
Case & the Montenegro Case was again reinstated.

VERDICT: WHEREFORE, pursuant to Section 8 of Presidential Decree No. 1877 and


Section 8 of the Rules and Regulations Implementing Presidential Decree No.
1877-A, the motion for reconsideration should have been granted,

and the writ of habeas corpus ordering the release of the detainees covered by
such Section 8 issued, but in the light of the foregoing manifestation as to
Sabino Padilla et al having been released, the petition as to them has been
declared moot and academic.

As to Dr. Aurora Parong, since a warrant of arrest against her was issued by the
municipal court for illegal possession of firearm and ammunitions, the petition
is likewise declared moot and academic. No costs.

53 G.R. No. L-61388


Page 201 of 236
Constitutional Law 2 Bill of Rights Armando Santiago Jr

END POINT: Rationale behind of suspension of right to bail while the writ of Habeas corpuz
was suspended at the same time If the right to bail may be demanded during
the continuance of the rebellion, and those arrested, captured and detained in
the course thereof will be released, they would, without the least doubt, rejoin
their comrades(colleague) in the field thereby jeopardizing the success of
government efforts to bring to an end the invasion, rebellion or insurrection

CITED DOCTRINE: LANSANG DOCTRINE (Abandoned)

RULING: The doctrine established in Barcelon and Montenegro was


subsequently abandoned in this case where the SC declared that it had the
power to inquire into the factual basis of the suspension of the privilege of the
writ of habeas corpus by Marcos in Aug 1971 and to annul the same if no legal
ground could be established. Accordingly, hearings were conducted to receive
evidence on this matter, including two closed-door sessions in which relevant
classified information was divulged by the government to the members of the
SC and 3 selected lawyers of the petitioners. In the end, after satisfying itself
that there was actually a massive and systematic Communist-oriented campaign
to overthrow the government by force, as claimed by Marcos, the SC
unanimously decided to uphold the suspension of the privilege of the Writ of
Habeas Corpus.

SEQUEL OF THE CASE: Raid was authorized by a search warrant issued by the judge, but certain people
were arrested without just cause and without arrest warrant. these arrested
people was transferred to unknown place where only the arresting officer know
the place.

IMPORTANT POINT/s: This ruling was rescind by Sec 18, Art 7 of the 1987 Constitution which
expressly constitutionalized the Lansang Doctrine. Note as well that under Art 3
(Sec 13) of the Constitution it is stated that the right to bail shall not be
impaired even if the privilege of the writ of habeas corpus is suspended.

Lansang Doctrine - it was decided in the time of 1937 constitution which state
the absolute suspension of the habeas corpuz and right to bail thereafter
pronouncement of the thereof suspension.

Montenegro Doctrine - it was decided in the time of 1987 constitution which may
invoke the right to bail even if writ of habeas corpuz was suspended.

FACTS
Reversal of the Lansang Doctrine & Reinstatement of the Montenegro Doctrine

Sabino Padilla, together w/ 8 others who were having a conference in a house in Bayombong, NV, were
arrested by members of the Philippine Constabulary (Police).
The raid of the house was authorized by a search warrant issued by Judge Sayo.
Josefina, mother of Sabino, opposed the arrest averring that no warrant of arrest was issued but rather it was
just a warrant of arrest hence the arrest of her son and the others was w/o just cause.
Sabino and companions together with 4 others were later transferred to a facility only the Philippine
Constabularys know.
Josefina petitioned the court for the issuance of the writ of habeas corpus.

ISSUE
Whether or not the arrests done against Sabino et al is valid.

RULING
Page 202 of 236
Constitutional Law 2 Bill of Rights Armando Santiago Jr

In a complete about face, the SC decision in the Lansang Case was reversed and the ruling in the Barcelon
Case & the Montenegro Case was again reinstated.

The questioned power of the president to suspend the privilege of the writ of habeas corpuz was once again
held as discretionary in the president.

The SC again reiterated that the suspension of the writ was a political question to be resolved solely by the
president. It was also noted that the suspension of the privilege of the writ of habeas corpus must, indeed, carry
with it the suspension of the right to bail, if the governments campaign to suppress the rebellion is to be
enhanced and rendered effective. If the right to bail may be demanded during the continuance of the rebellion,
and those arrested, captured and detained in the course thereof will be released, they would, without the least
doubt, rejoin their comrades in the field thereby jeopardizing the success of government efforts to bring to an end
the invasion, rebellion or insurrection.

SYLLABI
Habeas Corpus; Moot and Academic; The release of persons detained by Presidential Order or the
issuance of warrants of arrests by the court against them in connection with criminal cases filed in
court, renders habeas corpus petition academic and moot.WHEREFORE, pursuant to Section 8 of
Presidential Decree No. 1877 and Section 8 of the Rules and Regulations Implementing Presidential Decree No.
1877-A, the motion for reconsideration should have been granted, and the writ of habeas corpus ordering the
release of the detainees covered by such Section 8 issued, but in the light of the foregoing manifestation as to
Norberto Portuguese, Sabino Padilla, Francis Divinagracia, Imelda de los Santos, Benjamin Pineda, Zenaida
Mallari, Mariano Soriano, Tito Tanguilig. Letty Ballogan, Bienvenido Garcia, Eufronio Ortiz, Jr., Juanito Granada,
and Tom Vasquez, having been released, the petition as to them has been declared moot and academic. As to
Dr. Aurora Parong, since a warrant of arrest against her was issued by the municipal court of Bayombong on
August 4, 1982, for illegal possession of firearm and ammunitions, the petition is likewise declared moot and
academic.

CONCURRING
ABAD SANTOS, J.,

Habeas Corpus; Judgment; The tone of the original decision is slavish.I concur in the result, i.e. to
dismiss the case for having become moot and academic. And if I had my way I would set the original decision
aside because of its slavish tone.

SEPARATE OPINION
FERNANDO, C.J.,

Habeas Corpus; Judgments; The original decision does not state the controlling principles on the questions at
bar.Let me state that while I signed the above per curiam opinion as it clarifies the duration of preventive
detention, I am not persuaded that the original decision expresses what to my mind should be the controlling
principles as to the questions dealt with in my separate opinion.

TEEHANKEE, J.,

Constitutional Law; Habeas Corpus; The Supreme Court has the authority to inquire into the factual
bases of the Chief Executive in suspension of the writ of habeas corpus.I maintain my original dissent
from the decision of April 20, 1983, thus: I am constrained to dissent from the all-encompassing scope of the
main opinion of Mr. Justice de Castro which would overturn the landmark doctrine erf Lansang vs. Garcia which
upheld the Supreme Courts authority to inquire into the existence of factual bases for the Presidents
suspension of the privilege of writ of habeas corpus in order to determine the constitutional sufficiency thereof
and would revert to the retrogressive and colonial era ruling of Barcelon vs. Baker and Montenegro vs.
Castaeda that the Presidents decision to so suspend the privilege of the writ is final and conclusive upon the
courts and all other persons, and would further deny the right to bail even after the filing of charges in court to
Page 203 of 236
Constitutional Law 2 Bill of Rights Armando Santiago Jr

persons detained under Presidential Commitment Orders, and The continuous flow of petitions for habeas
corpus filed with this Court should not be decried nor discouraged. The Court stands as the guarantor of the
constitutional and human rights of all persons within its jurisdiction and must see to it that the rights are
respected and enforced.

Same; Same; The Courts resolution at bar reaffirms the restrictive interpretation of preventive detention
orders.It should be noted that the Courts Resolution at bar reaffirms the restrictive interpretation of preventive
detention under section 3 of P.D. 1877 dated July 21, 1983 adopted by it in the Jimenez and Villaber, cases, that
it will set at liberty persons preventively detained without charges for over one year. Specifically cited is section 8
of the Defense Ministers implementing rules and regulations duly approved by the President that The period of
detention of all persons presently detained by virtue of a Presidential Commitment Order or its derivatives shall
not extend beyond one (1) year from and after the date of effectivity of Presidential Decree No. 1877, as
amended. Upon the effectivity of these rules and regulations, all cases of persons presently detained under a
presidential commitment order or its derivatives shall be governed by Presidential Decree No. 1877, as
amended, and its implementing rules and regulations. The Courts Resolution further authoritatively states with
the unqualified concurrence of at least ten (10) members that T)here is no question, therefore, that the force
and effectivity of a presidential commitment order issued as far back as July 12, 1982 had ceased to have any
force and effect, since the detention exceeded the prescribed one-year limitation and that pursuant to Section 8
of Presidential Decree No. 1877 and Section 8 of the Rules and Regulations Implementing Presidential Decree
No. 1877-A, the motion for reconsideration should have been granted, and the writ of habeas corpus ordering
the release of the detainees covered by such Section 8 issued, but for the release of the detainees effected
earlier, mostly in January this year.

LIM VS. CA54


PONENTE: BUENA, J.

TOPIC: Right to a speedy trial

TRIGGER OF THE FACTS: The petitioner filed a motion to dismiss and invoke his right to a speedy trial for
his scheduled trial was postponed for 11 times. he alleged that the trial court
commit grave abuse of discretion in denying motion to dismiss.

TRIGGER OF THE ISSUE: WON Petitioners right to a speedy trial was violated

TRIGGER OF THE RULING: SC ruled denying the petition for it was lack of merit. postponent was
reasonable, it was delay for there was no subpoena served, and postponent due
to beyond control of prosecution

VERDICT: WHEREFORE, the petition is DENIED for lack of showing that the Court of
Appeals committed reversible error.

END POINT: Unjustified postponements which prolong the trial for an unreasonable length of
time are what offend the right of the accused to speedy trial. The right to speedy
trial allows reasonable continuance so as not to deprive the prosecution its day
in court.

The right of an accused to a speedy trial is guaranteed to him by the


constitution but the same shall not be utilized to deprive the State of a
reasonable opportunity of fairly prosecuting criminals. It secures rights to an
accused but it does not preclude the rights of the State to seek justice. Both the
State and the accused are entitled to due process.

CITED DOCTRINE: None

54 G.R. No. 131483


Page 204 of 236
Constitutional Law 2 Bill of Rights Armando Santiago Jr

SEQUEL OF THE CASE: Postponement is reasonable in the pendency of a trial it will not constitute
vexatious, capricious and oppressive delays and also it may not render violation
on his constitutional right to a speedy trial

IMPORTANT POINT/s: The right of an accused to a speedy trial is guaranteed to him by the
constitution but the same shall not be utilized to deprive the State of a
reasonable opportunity of fairly prosecuting criminals. It secures rights to an
accused but it does not preclude the rights of the State to seek justice. Both the
State and the accused are entitled to due process.

FACTS
Petition for review to set aside the decision rendered by the Court of Appeals
in denying petitioners motion to dismiss invoking his right to a speedy trial
The petitioner was arraigned and pleaded not guilty to the charger
initial trial of the case was ensued, however, schedule of trial was reset for 11 times
the petitioner filed a motion to dismiss invoking his right to a speedy trial
The prosecution failed to file its Comment within the period granted it. the building housing the court was
burned and the court records had to be reconstituted.
Trial court ordered to dismiss the case for failure of the prosecution to prove its case despite eleven (11)
postponements spread over an unreasonably long period time in violation of the right of the petitioner to a
speedy trial.
CA dismissed the petition.
petitioner elevate the case to the supreme court.

ISSUE
Whether or not the petitioners right was violated as an accused to a speedy trial

RULING
The petition is without merit.

1. not all the eleven postponements were made at the instance of the prosecution. The hearing was reset
because the petitioner was without counsel, and the hearing was reset because petitioners newly engaged
counsel was not available.
2. Contrary to the allegation,
1. the reasons for the prosecutions postponements were reasonable and were not intended merely to delay
the proceedings of the case.
2. The hearing of the case had to be postponed several times because there was no proof that the
prosecution witnesses were duly served with subpoena.
3. It would be unjust to pounce on the absence of the witnesses as a basis for dismissing the case when
there was a valid excuse for their absence, that is, there was no proof that they were duly served with
subpoena.
4. The other reasons for the postponement of the hearing, such as the re-raffling of the case to another
branch and the fire which razed the building housing the court and its records, are circumstances beyond
the control of the prosecution.

It bears stressing that the Petitioner is a detention prisoner and inordinate delays in the prosecution of the case
will be grossly prejudicial not only to the petitioner but to the State as well and the administration of justice.

The right of an accused to a speedy trial is guaranteed to him by the constitution but the same shall not be
utilized to deprive the State of a reasonable opportunity of fairly prosecuting criminals. It secures rights to an
accused but it does not preclude the rights of the State to seek justice. Both the State and the accused are
entitled to due process.
Page 205 of 236
Constitutional Law 2 Bill of Rights Armando Santiago Jr

Unjustified postponements which prolong the trial for an unreasonable length of time are what offend the right of
the accused to speedy trial. The right to speedy trial allows reasonable continuance so as not to deprive the
prosecution its day in court.

SC defined right

1. a trial conducted according to law of criminal procedure and the rules and regulations, free from vexatious,
capricious and oppressive delays.
2. what the constitution prohibits is vexatious, capricious and oppressive delays, manufactured by them
ministers of justice. Not every delay in the trial is vexatious, capricious or oppressive. In the legal firmament.
The terms have distinct connotations. Vexatious suggests an act which is willful and without reasonable
cause, for the purpose of annoying and embarrasing another or one lacking justification and intended to
harass
3. Oppressive connotes an unjust or cruel exercise of power or authority. Capricious action, on the other hand,
means willful and unreasoning action xxx

The Court finds that there is no basis for petitioners allegation that his constitutional right to speedy trial has
been violated. One must take into account that a trial is always subject to reasonable delays and
postponements, and in the absence of any showing that the same were capricious, the State should not be
deprived of a reasonable opportunity of prosecuting petitioner.

Petition were Denied.

RATIO
SYLLABI
Constitutional Law; Criminal Law; Right to Speedy Trial; The right of an accused to a speedy trial is
guaranteed to him by the constitution but the same shall not be utilized to deprive the State of a
reasonable opportunity of fairly prosecuting criminals.The right of an accused to a speedy trial is
guaranteed to him by the constitution but the same shall not be utilized to deprive the State of a reasonable
opportunity of fairly prosecuting criminals. It secures rights to an accused but it does not preclude the rights of
the State to seek justice. Both the State and the accused are entitled to due process.

Same; Same; Same; The right to speedy trial allows reasonable continuance so as not to deprive the
prosecution its day in court.Unjustified postponements which prolong the trial for an unreasonable length of
time are what offend the right of the accused to speedy trial. The right to speedy trial allows reasonable
continuance so as not to deprive the prosecution its day in court.

Same; Same; Same; One must take into account that a trial is always subject to reasonable delays and
postponements, and in the absence of any showing that the same were capricious, the State should not
be deprived of a reasonable opportunity of prosecuting petitioner.The Court finds that there is no basis
for petitioners allegation that his constitutional right to speedy trial has been violated. One must take into
account that a trial is always subject to reasonable delays and postponements, and in the absence of any
showing that the same were capricious, the State should not be deprived of a reasonable opportunity of
prosecuting petitioner.

Same; Same; Same; Mere mathematical reckoning of the time involved would not be sufficient.In the
recent case of Binay vs. Sandiganbayan, et al. we held that the right to a speedy disposition of a case, like the
right to speedy trial, is deemed violated only when the proceedings is attended by vexatious, capricious, and
oppressive delays; or when unjustified postponements of the trial are asked for and secured, or when without
cause or justifiable motive a long period of time is allowed to elapse without the party having his case . . . A mere
mathematical reckoning of the time involved, therefore, would not be sufficient.

Page 206 of 236


Constitutional Law 2 Bill of Rights Armando Santiago Jr

IN THE MATTER OF THE PETITION FOR HABEAS CORPUS


OF LANSANG VS.GARCIA
PONENTE: CONCEPCION, C.J.

TOPIC: Application of suspended the privilege of the writ of habeas corpus

TRIGGER OF THE FACTS: Granade was thrown in a liberal party caucus which causing death of 8 people
because of this event, Marcos issued BP889 which suspending the privilege writ
of habeas corpuz, thereafter, lansangan et al were invited to interrogate and
further investigation. Lansangan et. al questioned the validity of BP889 for not
of constitutional requisite

TRIGGER OF THE ISSUE: WON suspension of Habeas Corpuz is unconstitutional


WON the supreme court can review the suspension of the writ of habeas corpus

TRIGGER OF THE RULING: doctrine established in Barcelon and Montenegro was subsequently abandoned

SC declared that it had the power to inquire into the factual basis of the
suspension of the privilege of the writ of habeas corpus by Marcos and to annul
the same if no legal ground could be established.

Issuance of BP889 was issued with proper investigation and found out there is a
valid reason to uphold BP889

VERDICT: 1. Declaring that the President did not act arbitrarily in issuing Proclamation
No. 889
2. Dismissing the petitions in L-33964, L-33965, L-33982, L-34004, L-34013,
L-34039 and L-34265, insofar as petitioners Teodosio Lansang et al are
concerned
3. The Court of First Instance of Rizal is hereby directed to act with utmost
dispatch in conducting the preliminary examination and/or investigation of
the charges for violation of the Anti-Subversion Act filed against herein
petitioners and to issue the corresponding warrants of arrest, if probable
cause is found to exist against them, or, otherwise, to order their release

END POINT: the constitution mandated the supreme court may look into the truth of the
suspension of the writ.

CITED DOCTRINE: In this case, the supreme court REVISITED the Doctrine Held in the Barcelon
Case & the Montenegro Case

SEQUEL OF THE CASE: Suspension of the writ of habeas corpus were issued on the ground that there
was a rebellion, insurrection and etc.

IMPORTANT POINT/s: IT REVERSE THE STATEMENT ABOUT THE POWER OF THE SUPREME COURT:
had the power to inquire into the factual basis of the suspension of the privilege
of the writ of habeas corpus by Marcos and to annul the same if no legal ground
could be established

The Supreme Court may review, in an appropriate proceeding filed by any


citizen, the sufficiency of the factual basis of the proclamation of martial law or
the suspension of the privilege of the writ or the extension thereof, and must
promulgate its decision thereon within thirty days from its filing. (1987 changed
this thats why these cases was happend)

Abandonment of the Doctrine Held in the Barcelon vs. Baker Case & the Montenegro Case

Page 207 of 236


Constitutional Law 2 Bill of Rights Armando Santiago Jr


FACTS
Due to the throwing of two hand grenades in a Liberal Party caucus in 1971 causing the death of 8 people,
Marcos issued BP 889 which suspended the privilege of the writ of habeas corpus.
Marcos urged that there is a need to reduce the growth of Maoist (Communist) groups.
Subsequently, Lansang et al were invited by the Philippine Constabularya headed by Garcia for interrogation
and investigation.
Lansang et al questioned the validity of the suspension of the writ averring that the suspension does not meet
the constitutional requisites.

ISSUE
1. Whether or not the suspension is constitutional.
2. Wether or not the Supreme court can review the suspension of the writ of habeas corpus

RULING
The supreme court may check the arbitrariness/baseless of the issuance of suspension of the writ of habeas
corpus. but if the executive state that there was a susbstantial basis then the supreme court cannot decide for
that is political question. they cannot look into the truth but they can look into the manner whether such
suspension was issued whimsically or has baseless ground. since there was investigation made by the
executive there is susbstantial basis of issuing the suspension.

The doctrine established in Barcelon and Montenegro was subsequently abandoned in this case where the SC
declared that it had the power to inquire into the factual basis of the suspension of the privilege of the
writ of habeas corpus by Marcos and to annul the same if no legal ground could be established.
Accordingly, hearings were conducted to receive evidence on this matter, including two closed-door sessions in
which relevant classified information was disclose by the government to the members of the SC and 3 selected
lawyers of the petitioners. In the end, after satisfying itself that there was actually a massive and systematic
Communist-oriented campaign to overthrow the government by force, as claimed by Marcos, the SC
unanimously decided to uphold the suspension of the privilege of the Writ of Habeas Corpus.

This provision changed the 1973, now allowing an individual to file a question to the
supreme court assailing the validity of the suspension. congress has the power to approve
the writ issued by the executive.

Art. 7, SECTION 18. The President shall be the Commander-in-Chief of all armed forces
of the Philippines and whenever it becomes necessary, he may call out such armed forces to
prevent or suppress lawless violence, invasion or rebellion. In case of invasion or rebellion,
when the public safety requires it, he may, for a period not exceeding sixty days, suspend
the privilege of the writ of habeas corpus or place the Philippines or any part thereof under
martial law. Within forty-eight hours from the proclamation of martial law or the suspension
of the privilege of the writ of habeas corpus, the President shall submit a report in person or
in writing to the Congress. The Congress, voting jointly, by a vote of at least a majority of all
its Members in regular or special session, may revoke such proclamation or suspension,
which revocation shall not be set aside by the President. Upon the initiative of the President,
the Congress may, in the same manner, extend such proclamation or suspension for a
period to be determined by the Congress, if the invasion or rebellion shall persist and public
safety requires it.

The Congress, if not in session, shall, within twenty-four hours following such
proclamation or suspension, convene in accordance with its rules without any need of a call.

Page 208 of 236


Constitutional Law 2 Bill of Rights Armando Santiago Jr

The Supreme Court may review, in an appropriate proceeding filed by any citizen, the
sufficiency of the factual basis of the proclamation of martial law or the suspension of the
privilege of the writ or the extension thereof, and must promulgate its decision thereon
within thirty days from its filing.

A state of martial law does not suspend the operation of the Constitution, nor supplant
the functioning of the civil courts or legislative assemblies, nor authorize the conferment of
jurisdiction on military courts and agencies over civilians where civil courts are able to
function, nor automatically suspend the privilege of the writ.

The suspension of the privilege of the writ shall apply only to persons judicially
charged for rebellion or offenses inherent in or directly connected with the invasion.

During the suspension of the privilege of the writ, any person thus arrested or
detained shall be judicially charged within three days, otherwise he shall be released.

SYLLABI
Constitutional law; Judicial review; Habeas Corpus.The Supreme Court has the authority under the
Constitution to inquire into the existence of a factual basis for the issuance of a presidential proclamation
suspending the privilege of the writ of habeas corpus for the purpose of determining the constitutional sufficiency
thereof.

Same; Grant of power to suspend writ privilege neither absolute nor unqualified.Far from being full and
plenary, the authority to suspend the privilege of the writ is circumscribed, confined and restricted, not only by
the prescribed setting or the conditions essential to its existence, but, also, as regards the time when and the
place where it may be exercised. Like the limitations imposed by the Fundamental Law upon the legislative
department, adherence thereto and compliance therewith may, within proper bounds, be inquired into by the
courts of justice. Otherwise, the explicit constitutional provisions thereon would be meaningless.

Same; Requisites for valid suspension of writ of habeas corpus.For a valid suspension of the privilege
of the writ: (a) there must be invasion, insurrection or rebellion or pursuant to paragraph (2), section 10 of
Art. VII of the Constitution imminent danger thereof; and (b) public safety must require the aforementioned
suspension. The President declared in Proclamation No. 889, as amended, that both conditions are present.

Same; Separation of Powers.Pursuant to the principle of separation of powers underlying our system of
government, the Executive is supreme within his own sphere. However, the separation of powers, under the
Constitution, is not absolute. What is more, it goes hand in hand with the system of checks and balances, under
which the Executive is supreme, as regards the suspension of the privilege, but only if and when he acts within
the sphere allotted to him by the Basic Law, and the authority to determine whether or not he has so acted is
vested in the Judicial Department, which, in this respect, is, in turn, constitutionally supreme.

Same;Same.In the exercise of such authority, the function of the Court is merely to check not to
supplant the Executive, or to ascertain merely whether he hasgone beyond the constitutional limits of his
jurisdiction, not to exercise the power vested in him or to determine the wisdom of his act.

Same; Same; Test for determining validity of presidential suspension of writ.The Solicitor General
urged that judicial inquiry into the basis of the questioned proclamation can go no further than to satisfy the
Court not that the Presidents decision is correct and that public safety was endangered by the rebellion and
justified the suspension of the writ, but that in suspending the writ, the President did not act arbitrarily. No
cogent reason has been submitted to warrant the rejection of such test.

Same; Same; Same.Considering the data in the possession of the President, the Court is not prepared to
hold that the Executive had acted arbitrarily or gravely abused his discretion when he then concluded that public
safety and national security required the suspension of the privilege of the writ, particularly if the NPA were to
strike simultaneously with violent demonstrations staged by the two hundred forty-five (245) KM chapters, all

Page 209 of 236


Constitutional Law 2 Bill of Rights Armando Santiago Jr

over the Philippines, with the assistance and cooperation of the dozens of GPP organizations, and the bombing
of water mains and conduits, as well as electric power plants and installations a possibility which, no matter
how remote, he was bound to forestall, and a danger he was under obligation to anticipate and arrest.

CONCURRING
CASTROand BARREDO, JJ.,concurring:

Constitutional law; Release of detained persons when writ is suspended and they are charged in court.The
dissenting opinion is based on the fallacy that when a formal charge is filed against a person he is thereby
surrendered to the court and the arresting officer is thereby divested of custody over him. Except in a
metaphorical sense, the detainee is not delivered or surrendered at all to the judicial authorities. What the
phrase delivered to the court simply means is that from the time a person is indicted in court, the latter acquires
jurisdiction over the subject-matter. The detainee remains in the custody of the detaining officer, under the same
authority invoked for the detention, until the court decides whether there is probable cause to order his arrest.
Criminal procedure; Duty of arresting officer.The arresting officer should hold the person detained until the
court can act, with the only difference that where the privilege of the writ is available, the arresting officer must
release the detainee upon the expiration of the maximum detention time allowed by law, if he has not delivered
the detainee to the court within that period.

DISSENTING
FERNANDO, J.,concurring and dissenting:

Constitutional law; Separation of Powers; Persons detained due to writs suspension should be released when
charged in court until warrant is issued.In the case of Nava vs. Gatmaitan (90 Phil. 172) the majority of the
Court reached the conclusion (although lacking enough votes to make it binding) that the suspension of the
privilege of the writ does not suspend the right to bail. Thus, By the same token, if and when formal complaint is
presented, the court steps in and the executive steps out. The detention ceases to be an executive and becomes
a judicial concern. Thereupon the corresponding court assumes its role and the judicial process takes its course
to the exclusion of the executive or the legislative departments. Henceforward, the accused is entitled to demand
all the constitutional safeguards and privileges essential to due process. It could follow then that the petitioners
still detained ought not to be further deprived of their liberty in the absence of a warrant of arrest for whatever
offense they may be held to answer, to be issued by a judge after a finding of probable cause. That is to comply
with the constitutional requirement against unreasonable search and seizure. Same; Violation of due process of
law.Moreover, to keep them inconfinement after the ordinary processes of the law are to be availed of, as
thereafter decreed by the Executive itself, is to ignore the safeguard in the Bill of Rights that no person shall be
held to answer for a criminal offense without due process of law.

BAR QUESTION (1987-2006)

Rights of the Accused; Counsel of his Choice (Q8-2005)


(1) Mariano was arrested by the NBI as a suspect in the shopping mall bombings. Advised of his rights, Mariano
asked for the assistance of his relative, Atty. Santos. The NBI noticed that Atty. Santos was inexperienced,
incompetent and inattentive. Deeming him unsuited to protect the rights of Mariano, the NBI dismissed Atty.
Santos. Appointed in his place was Atty. Barroso, a bar topnotcher who was in the premises visiting a relative.
Atty. Barroso ably assisted Mariano when the latter gave a statement. However, Mariano assailed the
investigation claiming that he was deprived of counsel of his choice.

Was the NBI correct in dismissing Atty. Santos and appointing Atty. Barroso in his stead? Is Mariano's statement,
made with the assistance of Atty. Barroso, admissible in evidence? (5%)

Page 210 of 236


Constitutional Law 2 Bill of Rights Armando Santiago Jr

ALTERNATIVE ANSWER:
The NBI was not correct in dismissing Atty. Santos and appointing Atty. Barroso in his stead. Article III, Section
12(1) of the 1987 Constitution requires that a person under investigation for the commission of an offense shall
have no less than "competent and independent counsel preferably of his own choice " This is meant to stress the
primacy accorded to the voluntariness of the choice under the uniquely stressful conditions of a custodial
investigation' Thus, the lawyer called to be present during such investigation should be as far as reasonably
possible, the choice of the individual undergoing questioning. The appointment of Atty. Barroso is questionable
because he was visiting a relative working in the NBI and thus his independence is doubtful. Lawyers engaged
by the police, whatever testimonials are given as proof of their probity and supposed independence, are
generally suspect, as in many areas, the relationship between lawyers and law enforcement authorities can be
symbiotic. Considering that Mariano was deprived of counsel of his own choice, the statement is inadmissible in
evidence. (People v. Januario, G.R. No. 98252, February 7, 1997)

ALTERNATIVE ANSWER:
The NBI was correct in dismissing Atty. Santos as he was incompetent. The 1987 Constitution requires counsel
to be competent and independent. Atty. Barroso, being a bar topnotcher ably assisted Mariano and there is no
showing that his having a relative in the NBI affected his independence. Moreover, the accused has the final
choice of counsel as he may reject the one chosen for him and ask for another. A lawyer provided by the
investigators is deemed engaged by the accused where he raises no objection against the lawyer during the
course of the investigation, and the accused thereafter subscribes to the truth of his statement before the
swearing officer. Thus, once the prosecution shows there was compliance with the constitutional requirement on
pre-interrogation advisories, a confession is presumed to be voluntary and the declarant bears the burden of
proving that his confession is involuntary and untrue. A confession is admissible until the accused successfully
proves that it was given as a result of violence, intimidation, threat or promise of reward or leniency which are
not present in this case. Accordingly, the statement is admissible. (People v. Jerez, G.R. No. 114385,
January 29, 1998)

Rights of the Accused; Presumption of Innocence vs. Presumption of


Theft (2004)
(5-b) OZ lost five head of cattle which he reported to the police as stolen from his barn. He requested several
neighbors, including RR, for help in looking for the missing animals. After an extensive search, the police found
two head in RR's farm. RR could not explain to the police how they got hidden in a remote area of his farm.
Insisting on his innocence, RR consulted a lawyer who told him he has a right to be presumed innocent under
the Bill of Rights. But there is another presumption of theft arising from his unexplained possession of stolen
cattle under the penal law.

Are the two presumptions capable of reconciliation In this case? If so, how can they be reconciled? If not, which
should prevail? (5%)

SUGGESTED ANSWER:
The two presumptions can be reconciled. The presumption of innocence stands until the contrary is proved. It
may be overcome by a contrary presumption founded upon human experience. The presumption that RR is the
one who stole the cattle of OZ is logical, since he was found in possession of the stolen cattle. RR can prove his
innocence by presenting evidence to rebut the presumption. The burden of evidence is shifted to RR, because
Page 211 of 236
Constitutional Law 2 Bill of Rights Armando Santiago Jr

how he came into possession of the cattle is peculiarly within his knowledge. (Dizon-Pamintuan v. People, 234
SCRA 63 (1994)).

Rights of the Accused; Right to Bail (1993)


No. 9: Johann learned that the police were looking for him in connection with the rape of an 18-year old girl, a
neighbor. He went to the police station a week later and presented himself to the desk sergeant. Coincidentally.
the rape victim was in the premises executing an extrajudicial statement. Johann, along with six (6) other
suspects, were placed in a police line- up and the girl pointed to him as the rapist. Johann was arrested and
locked up in a cell. Johann was charged with rape in court but prior to arraignment invoked his right to
preliminary investigation. This was denied by the judge, and thus, trial proceeded. After the prosecution
presented several witnesses, Johann through counsel, invoked the right to ball and filed a motion therefor, which
was denied outright by the Judge. Johann now files a petition for certiorari before the Court of Appeals arguing
that:

3) He is entitled to bail as a matter of right, thus the Judge should not have denied his motion to fix ball outright.
Decide.

SUGGESTED ANSWER:
3) In accordance with Art. III. sec. 13 of the Constitution, Johann may be denied bail if the evidence of his guilt is
strong considering that the crime with which he is charged is punishable by reclusion perpetua. It is thus not a
matter of right for him to be released on bail in such case. The court must first make a determination of the
strength of the evidence on the basis of evidence already presented by the prosecution, unless it desires to
present some more, and give the accused the opportunity to present countervailing evidence. If having done this
the court finds the evidence not to be strong, then it becomes the right of Johann to be admitted to bail. The error
of the trial court lies in outrightly denying the motion for bail of Johann.

Rights of the Accused; Right to Bail; Capital Offense (Q4-2006)

State whether or not the law is constitutional. Explain briefly.


2. A law denying persons charged with crimes punishable by reclusion perpetua or death the right to bail. (2%)

SUGGESTED ANSWER:
The law is invalid as it contravenes Section 13, Article III of the 1987 Constitution which provides that "all
persons, except those charged with offenses punishable by reclusion perpetua when evidence of guilt is strong,
shall, before conviction, be bailable by sufficient sureties, or be released on recognizance as may be provided by
law." The accused may not be deprived of his constitutional right to bail even if charged with a capital offense
where the evidence of guilt is not strong.

Rights of the Accused; Right to Bail; Deportation Case (1989)


No. 15: May an alien invoke the constitutional right to bail during the pendency of deportation proceedings?

SUGGESTED ANSWER:

Page 212 of 236


Constitutional Law 2 Bill of Rights Armando Santiago Jr

No. an alien may not invoke the constitutional right to bail during the pendency of deportation proceedings. In
Harvey vs Santiago, 162 SCRA 840, it was held that the constitutional guarantee to bail may not be invoked in
deportation proceedings, because they do not partake of the nature of a criminal action.

Rights of the Accused; Right to Bail; Matter of Right or a Matter of


Discretion (Q7-2005)
a) State with reason(s) whether bail is a matter of right or a matter of discretion in the following cases: (4%) The
imposable penalty for the crime charged is reclusion perpetua and the accused is a minor;

SUGGESTED ANSWER:
If the accused is a minor where the imposable penalty for the crime charged is reclusion perpetua, bail would be
a matter of right. Under Article 68 of the Revised Penal Code, when the offender is a minor under eighteen years
of age, he is entitled to a penalty, depending on his age, lower by one or two degrees than that prescribed by law
for the crime committed. The Constitution withholds the guaranty of bail from one who is accused of a capital
offense where the evidence of guilt is strong. The obvious reason is that one who faces a probable death
sentence has a particularly strong temptation to flee. This reason does not hold where the accused has been
established without objection to be a minor who by law cannot be sentenced to death. (Bravo v. Borja, G.R. No.
L-65228, February 18, 1985)

b) The imposable penalty for the crime charged is life imprisonment and the accused is a minor;

ALTERNATIVE ANSWER:
If the accused is a minor and the imposable penalty for the crime charged is life imprisonment, bail would not be
a matter of right. In the instant case, assuming that evidence of guilt strong, bail shall be denied as the privileged
mitigating circumstance of minority is not available for violation of special laws penalized by life imprisonment.

ALTERNATIVE ANSWER:
Although the Constitution mentions only reclusion perpetua, Rule 114 of the Rules of Court adds life
imprisonment, and therefore, applying the PRO REO DOCTRINE, bail would still be a matter of right, since it is
favorable to the accused.

c) The accused has been convicted of homicide on a charge of murder and sentenced to suffer an indeterminate
penalty of from eight (8) years and one (1) day of prision mayor, as minimum, to twelve (12) years and four (4)
months of reclusion temporal, as maximum.

SUGGESTED ANSWER:
If the accused has been convicted of homicide on a charge of murder and sentenced to suffer imprisonment of
from 8 to 12 years, bail is a matter of discretion. Under Rule 114, Sec. 5, par. 1 of the Rules of Court, if the
decision of the trial court convicting the accused changed the nature of the offense from non-bailable to bailable,
the application for bail may be filed and acted upon by the appellate court. Admission to bail is discretionary.

Rights of the Accused; Right to Speedy Trial (2000)

Page 213 of 236


Constitutional Law 2 Bill of Rights Armando Santiago Jr

No XV. Charged by Francisco with libel, Pablo was arraigned on January 3, 2000, Pre-trial was dispensed with
and continuous trial was set for March 7, 8 and 9, 2000. On the first setting, the prosecution moved for its
postponement and cancellation of the other settings because its principal and probably only witness, the private
complainant Francisco, suddenly had to go abroad to fulfill a professional commitment. The judge instead
dismissed the case for failure to prosecute.

a) Would the grant of the motion for postponement have violated the accused's right to speedy trial? (2%)

SUGGESTED ANSWER
The grant of the motion for postponement would not have violated the right of the accused to speedy trial. As
held In People v. Leviste, 255 SCRA 238 (1996). since the motion for postponement was the first one requested,
the need for the offended party to attend to a professional commitment is a valid reason, no substantial right of
the accused would be prejudiced, and the prosecution should be afforded a fair opportunity to prosecute its
case, the motion should be granted.

ALTERNATIVE ANSWER:
Since continuous trial of cases is required and since the date of the initial hearing was set upon agreement of all
parties, including the private complainant, the judge properly dismissed the case for failure to prosecute.

BAR QUESTION (2007-2013)

Rights of the Accused; Right to Bail (2009)


No.XII. William, a private American Citizen, a university graduate and frequent visitor to the Philippines, was
inside the US embassy when he got into a heated argument with a private Filipino citizen. Then, in front of many
shocked witnesses, he killed the person he was arguing with. The police came, and brought him to the nearest
police station. Upon reaching the station, the police investigator, in halting English, informed William of his
Miranda rights, and assigned him an independent local counsel. William refused the services of the lawyer, and
insisted that he be assisted by a Filipino lawyer currently based in the US. The request was denied, and the
counsel assigned by the police stayed for the duration of the investigation.

William protested his arrest.

(b) He also claimed that his Miranda Rights were violated because he was not given the lawyer of his choice;
that being an American, he should have been informed of his rights in proper English; and that he should have
been informed of his rights as soon as he was taken into custody, not when he was already at the police station.
Was William denied his Miranda rights? Why or why not?

SUGGESTED ANSWER:
William was not denied with his Miranda rights. True that he has the right to counsel preferably of his choice. But
if he cannot afford the services of a counsel, he should be provided with one.

Page 214 of 236


Constitutional Law 2 Bill of Rights Armando Santiago Jr

Moreover, the Miranda rights are available only during custodial investigation that is, from the moment the
investigating officer begins to ask questions for the purpose of eliciting admissions, confessions or any
information from the accused. therefore, it is proper that he was only informed of his right at the police station.

ALTENATIVE ANSWER:
The fact that the police officer gave him the Miranda warning in halting English does not detract from its validity.
Under Section 2(b) of Republic Act No. 7438, it is sufficient that the language used was known to and
understood by him. William need not be given the Miranda warning before the investigation started. William was
not denied his Miranda rights. It is not practical to require the police officer to provide a lawyer of his own choice
from the United States (Gamboa vs. Cruz, 162 SCAR 642, [1998]).

(c) If William applies for bail, claiming that he is entitled thereto under the international standard of justice and
that he comes from a US State that has outlawed capital punishment, should William be granted bail as a matter
of right? Reasons.

SUGGESTED ANSWER:
William is not entitled to bail as a matter of right. His contention is not tenable. Observing the territorial
jurisdiction of commission of the offense, the applicable law in the case is Philippine laws not the law of the
country to where he is a national (Section 13, Art. III of the Constitution). Under our law, bail is not a matter of
right if the felony or offense committed has an imposable penalty of reclusion perpetua or higher and the
evidence of guilt is strong.

Rights of the Accused; Right to Bail; Double Jeopardy (2008)


No. VII. JC, a major in the Armed Forces of the Philippine, is facing prosecution before the Regional Trial Court
of Quezon City for the murder of his neighbor whom he suspected to have molested his (JCs) 15 year-old daughter.

(a) Is JC entitled to bail? Why or why not? (3%)

SUGGESTED ANSWER:
As a general rule, bail is not a matter of right when the offense charged carries with an imposable penalty of
reclusion perpetua or higher.

In the present case, JC is charged with murder which has a penalty of reclusion perpetua, hence he cannot be
allowed bail. However, should the evidence of guilt be found weak after hearing, the court may in its discretion,
fix bail for temporary liberty.

(b) Assume that upon being arraigned, JC entered a plea of guilty and was allowed to present evidence to prove
mitigating circumstances. JC then testified to the effect that he stabled the deceased in self- defense because
the latter was strangling him and that he voluntarily surrendered to the authorities. Subsequently, the trial court
rendered a decision acquittal violate JCs right against double jeopardy? Why or why not? (3%)

SUGGESTED ANSWER:
By presenting evidence of self-defense, JC effectively withdrew his plea of guilty (Peo vs. Balisacan, G.R. No.
L-26376, Aug. 31, 1966). In the absence of a valid plea, an essential element for jurisdiction of the Court and first
jeopardy was absent. Consequently, the court had no jurisdiction to acquit JC. Thus, an appeal by the
prosecution would not violate the rule against second jeopardy.

Page 215 of 236


Constitutional Law 2 Bill of Rights Armando Santiago Jr

ALTERNATIVE ANSWER:
Double jeopardy sets in when the first jeopardy has attached. There is already first jeopardy when the accused
has validly entered his plea before the appropriate court having jurisdiction over the subject matter and his
person and that he has been convicted or acquitted or that the case against him has been terminated without his
express consent.

In the present case, JC validly entered his plea of guilty but during the presentation of evidence he submits
evidence of self-defense. the consequence thereof is for the court to withdraw the plea of guilty and enter a plea
of not guilty. The validity of entering his plea is not affected.

Therefore, his acquittal shall bar any similar indictment that may be filed against him because of double
jeopardy.

Rights of the Accused; Right to Counsel (2012)


No. III. Mr. Brown, a cigarette vendor, was invited by PO1 White to a nearby police station. Upon arriving at the
police station, Brown was asked to stand side-by-side with five (5) other cigarette vendors in a police line-up.
PO1 White informed them that they were looking for a certain cigarette vendor who snatched the purse of a
passer-by and the line-up was to allow the victim to point at the vendor who snatched her purse. No questions
were to be asked from the vendors.

(a) Brown, afraid of a "set up" against him, demanded that he be allowed to secure his lawyer and for him to be
present during the police line-up. Is Brown entitled to counsel? Explain (5%)

SUGGESTED ANSWER
Brown is not entitled to counsel during the police line-up. He was not yet being asked to answer for a criminal
offense. (Gamboa vs. Cruz, 162 SCRA 642.)

(b) Would the answer in (a.) be the same if Brown was specifically invited by White because an eyewitness to
the crime identified him as the perpetrator? Explain. (3%)

SUGGESTED ANSWER
Brown would be entitled to the assistance of a lawyer. He was already considered as a suspect and was
therefore entitled to the rights under custodial investigation. (People vs. Legaspi, 331 SCRA 95.)

(c) Briefly enumerate the so-called "Miranda Rights". (2%)

SUGGESTED ANSWER
The Miranda warning means that a person in custody who will be interrogated must be informed of the following:

(a) He has the right to remain silent;


(b) Anything said can be used as evidenced against him;
(c) He has the right to have counsel during the investigation; and
(d) He must be informed that if he is indigent, a lawyer will be appointed to represent him. (Miranda vs. Arizona,
384 U.S. 436.)

Page 216 of 236


Constitutional Law 2 Bill of Rights Armando Santiago Jr

Arnold did not contest the guard's statement; he steadfastly remained silent and refused to give any written
statement. Later in court, the guard testified and narrated the statements he gave the police over Arnold's
counsel's objections. While Arnold presented his own witnesses to prove that his possession and apprehension
had been set-up, he himself did not testify. The court convicted Arnold, relying largely on his admission of the
charge by silence at the police investigation and during trial.

From the constitutional law perspective, was the court correct in its ruling? (6%)

SUGGESTED ANSWER:
The court was wrong in relying on the silence of Arnold during the police investigation and during the trial. Under
Article III, Section 12 of the 1987 Constitution, he had the right to remain silent. His silence cannot be taken as a
tacit admission, otherwise, his right to remain silent would be rendered nugatory. Considering that his right
against self-incrimination protects his right to remain silent, he cannot be penalized for exercising it (People vs.
Galvez, 519 SCRA 521).

Rights of the Accused; Right to Remain Silent (2013)


No.VII. As he was entering a bar, Arnold - who was holding an unlit cigarette in his right hand -was handed a
match box by someone standing near the doorway. Arnold unthinkingly opened the matchbox to light his
cigarette and as he did so, a sprinkle of dried leaves fell out, which the guard noticed. The guard immediately
frisked Arnold, grabbed the matchbox, and sniffed its contents. After confirming that the matchbox contained
marijuana, he immediately arrested Arnold and called in the police. At the police station, the guard narrated to
the police that he personally caught Arnold in possession of dried marijuana leaves.

ALTERNATIVE ANSWER:
No, the court has erred in its ruling of convicting Arnold relying solely on his admission of the charge by silence
at the police investigation and during trial.

allegedly seen being held for three days and tortured before he finally disappeared. The CHR lawyers requested
one Lt. Valdez for a photocopy of the master plan of Camp Aquino and to confirm in writing that he had custody
of the master plan. Lt. Valdez objected on the ground that it may violate his right against self-incrimination.
Decide with reasons. (4%).

SUGGESTED ANSWER:
The objection of Lt. Valdez is not valid. The right against self-incrimination refers to testimonial evidence and
does not apply to the production of a photocopy of the master plan of Camp Aquino, because it is a public
record. He cannot object to the request for him to confirm his custody of the master plan, because he is the
public officer who had custody of it. (Almonte vs. Vasquez, 244 SCRA 286 [1995]).

ALTERNATIVE ANSWER:
The objection is without merit. Right against self-incrimination is not violated because the right is simply against
testimonial compulsion. But the prohibition also extends to the compulsion for the production of documents,
papers and chattels that may be used as evidence against the witness, except where the State has a right to
inspect the same such as in this case.

The duty of the lawyer includes ensuring that the suspect under custodial investigation is aware that the right of
an accused to remain silent may be invoked at any time (People v. Sayaboc, G.R. No. 147201, January 15,
2004).

Page 217 of 236


Constitutional Law 2 Bill of Rights Armando Santiago Jr

ALTERNATIVE ANSWER:
The court correctly convicted Arnold. There is no showing that the evidence for the prosecution was insufficient.
When Arnold remained silent, he run the risk of an inference of guilt from non- production of evidence in his
behalf (People vs. Solis, 128 SCRA 217).

Page 218 of 236


Constitutional Law 2 Bill of Rights Armando Santiago Jr

Right against self-incrimination


Section 17 of Art. 3
No person shall be compelled to be a witness against himself
Self-incrimination - if the disclosure made is capable of being used against the defendant as a confession of a
crime then such disclosure would be an accusation against himself [US vs. Navarro]

JUSTIFICATION OF GUARANTEE: [US VS. NAVARRO]


1. Public Policy
if the party were required to testify, it would place the witnes under the strongest temptation to commit
perjury55
2. Humanity
it would prevent the extorting of confession by duress56

PURPOSE
Prohibit compulsory57 oral examination of prisoner before trial, for the purpose of extorting unwilling
confessions or declarations implicating them in the commission of the crime [US vs. Tan Teng]
Also covers the fact that an accused may not be forced to create evidence that will implicate himself [Beltran
vs. Samson]

WHO MAY DETERMINE: INCRIMINATING


The Court

SCOPE
1. as long as the question tend to incriminate58 privilege may apply.
2. in all other cases, a person may not refuse to answer the question is RELEVANT and otherwise allowed even
if the answer may tend to embarrass the person or subject him to civil liability
3. Rights may not be invokes where the question ask related to a past criminality for which the person may no
longer be prosecuted.
4. A person may not refuse to answer where he has been previously granted immunity under a validly enacted
statute

COMPULSION TO TAKE THE WITNESS STAND OR TO ANSWER AN INCRIMINATING QUESTION [Chavez


vs. CA]

Taking the witness Taking the witness Answer an incriminating


stand stand in a criminal case question

Accused Can be compelled Cannot be compelled Cannot be compelled

Witness Can be compelled Can be compelled Cannot be compelled


with an exception substantiated in the next section

55 the act or crime of knowingly making a false statement (as about a material matter) while under oath or bound by an affirmation or other
officially prescribed declaration that what one says, writes, or claims is true compare false swearing

56 compulsion by threat

57 obligatory

58 incriminate - to cause (someone) to appear guilty of or responsible for something (such as a crime)
Page 219 of 236
Constitutional Law 2 Bill of Rights Armando Santiago Jr

Can also be applied in administrative proceedings partaking the nature of a criminal proceeding or analogious
to a criminal proceeding
[Cabal vs. Kapunan Jr.] - Forfeiture of property
[Pascual Jr. Vs. Board of medical Examiners] - Revocation of license

NOT AN ABSOLUTE RIGHT


It does not prohibit every form of preliminary investigation
Preliminary investigation is often the only means of discovering the persons who may be reasonably
charged with a crime so as to enable to fiscal to prepare his complaint or information.
The guarantee does not prohibit a party litigant from using his adversary as a
witness when the case is not a criminal case where the intended witness is the
accused himself.
If a subpoena was issued by the Senate to issue queries in aid of legislation, the witness (even if accused in a
criminal case) may be compelled to take the witness stand even if what is to be asked is related to the criminal
case.
However, they may still invoke the right if they find that the question issued to them is incriminating.

WHEN TO RAISE THE PRIVILEGE


Before taking the witness Before answering an
stand (Accused) incriminating question
(Ordinary Witness)
Accused in a criminal X
case

Accused not in a criminal x


case

Witness x

WHEN AVAILABLE TO INVOKE THE PRIVILEGE AGAINST SELF-INCRIMINATION


When and as the incriminating question is asked, since the witness has no way of knowing in advance the
nature or effect of the question to be put to him.

Ordinary Witness - when the question is asked

Accused - when asked to take a stand as witness

PROHIBITION DOES NOT PRECLUDE A BODY CHECKDUP; MECHANICAL ACTS


Substance emitting from the body of the defendant was received as evidence in a prosecution for a rape case.
(2) US v. Tan Teng
Morphine forced out of the mouth of the accused was received as evidence. (3) US v. Ong Sui Hong
Pregnancy test is not a violation of the right against self incrimination as long as it is not done with torture or
force. (3) Villaflor v. Summers

HANDWRITING
Writing is not a purely mechanical act, because it requires the application of intelligence and attention. (5)
Beltran v. Samson
Forcing someone to furnish a specimen of his handwriting is equal to compelling a person to produce a
document which is a violation of this right the witness is required to furnish evidence against himself.

Page 220 of 236


Constitutional Law 2 Bill of Rights Armando Santiago Jr

WAIVER OF RIGHT
When a person voluntarily answers an incriminating question, he is deemed to have waived his right. Such
right may be waived, either directly or by a failure to invoke it. waiver is certain and unequivocal59 and
intelligently, understandingly and willingly made.
Denial on the part of the accused when it was said that it was she who wrote on the documents does not
constitute a waiver and therefore cannot be required to produce a sample of her handwriting. (6) Bermudez v.
Castillo

DOCUMENTS AND RECORDS


The right against self incrimination covers documentary evidence.
This right is a personal one thus applying only to a natural person.
Hence, a corporation may be compelled to submit to the visitorial powers of the state even if this
results in disclosure of criminal acts of the corporation.

CASES

CASE CONTENT FORMAT


CASE TITLE GR NO. CASE HINT

People vs. GR 133025, 17 February 2000 The taking of pictures of an accused even
Gallarde without the assistance of counsel, being a
purely mechanical act, is not a violation of
his constitutional right against self-
incrimination

REGALA G.R. No. 105938. September 20, 1996 The lawyer may not disclose his
vs.SANDIGAN
BAYAN

Beltran vs. GR 32025, 23 September 1929 Writing is not a purely mechanical act,
Samson because it requires the application of
intelligence and attention.

ROSETE vs G.R. No. 136051 June 8, 2006 The respondent filed a notice of oral
JULIANO LIM deposition against the petitioner and the
petitioner contends that it is violative to their
constitutional rights for there was an existing
criminal case.

PEOPLE VS. GALLARDE60


PONENTE: Davide, Jr., CJ

TOPIC: Right against self-incrimination / Essential part of this right

TRIGGER OF THE FACTS: Gallarde was convicted of a crime of murder wherein allegation of such
violation of his right against self-incrimination

59 very strong and clear: not showing or allowing any doubt

60 GR 133025, 17 February 2000


Page 221 of 236
Constitutional Law 2 Bill of Rights Armando Santiago Jr

TRIGGER OF THE ISSUE: Whether The taking of pictures of an accused violates of his constitutional
right against self incrimination.

TRIGGER OF THE RULING: No. It did not violate the right of the accused. Mechanical Act will not
constitute incrimination.

VERDICT: WHEREFORE, the assailed decision of the Regional Trial Court in Criminal
Case finding accused-appellant RADEL GALLARDE guilty of the crime of
murder is hereby modified. As modified, RADEL GALLARDE is hereby found
guilty beyond reasonable doubt, as principal, of the crime of Homicide costs
against accused-appellant RADEL GALLARDE in both instances.

END POINT: Writing is not a purely mechanical act, because it requires the application of
intelligence and attention

CITED DOCTRINE: None

SEQUEL OF THE CASE:

IMPORTANT POINT/s: It was not mentioned in the facts regarding of taking pictures of the accused.
But the supreme court ruled that taking of picture is a mechanical act and will
not constitute incrimination.

FACTS
at the house of spouses Eduardo and Elena Talan
their neighbors converged.
Among them were Radel Gallarde, and all surnamed Fernandez Idling by was Editha, 10 year old daughter of
spouses Talan.
After a while, Roger stood up and invited Jaime and Gallarde to dine in the kitchen.
As they partook of the meal, Gallarde suddenly left.
Jaime, too, stepped out of the kitchen to urinate.
Outside the house, he chanced upon Gallarde and Editha talking to each other. Jaime whistled at Gallarde but
instead of minding him, the latter run towards the road leading to his house.
Thereafter, Editha entered the kitchen and took hold of a kerosene lamp.
Jaime followed her and asked where she was going. Editha answered that she would look for Gallarde. Soon
Editha left enroute to where Gallarde fled. By 10:00 p.m., the drinking buddies had dispersed but Jaime,
Francisco, Edwin and Rose regrouped at Renato's place where they talked and relaxed.
Moments later, Roger arrived and informed them that Editha was missing.
Roger asked the group to help look for her. Elena Talan informed his uncle, Barangay Ex-kagawad Mario
Fernandez, about her daughter's disappearance.
The latter, together with his son Edwin, wife Virginia and nephew Freddie Cortez wasted no time in joining
their neighbours search the houses, dikes and fields to look for the missing child.
When Jaime mentioned that Gallarde was the last person he saw talking to Editha, the searchers went back to
the house of Gallarde.
The searchers found Gallarde squatting with his short pants at the toilet about 6 meters away from Gallarde's
house; his hands and knees covered with soil. Asked where Editha was, Gallarde replied: "I do not know, I did
not do anything to her." To the question, "where did you come from since a while ago you were not yet in this
toilet?" Gallarde answered "I was with Kiko, I was asleep in their house. One of the searchers Mario Bado, got
angry and countered that Gallardes statement was impossible because Kiko was with him drinking. After the
confrontation at the toilet, Ex-kagawad Fernandez brought Gallarde to Brgy. Captain Felicisimo Mendoza,
informing the latter that Gallarde was the last person seen talking with the missing child.
Fernandez then rejoined the searchers. Back in the field, Virginia Fernandez tripped on a wet ground. The
searchers, thereafter, noticed disheveled grasses, and a wide hole among the disheveled grass. When Ex-
kagawad Fernandez forthwith scratched some earth aside and then Editha's hand pitted out. Fernandez
screamed in terror.

Page 222 of 236


Constitutional Law 2 Bill of Rights Armando Santiago Jr

Meantime, Barangay Captain Mendoza heard shouts saying: "She is here, she is now here already dead!"
Mindful of Gallarde's safety, Brgy. Captain Mendoza decided to bring Gallarde to the municipal building. On
their way though, they met policemen on board a vehicle. He flagged them down and turned over the person
of Gallarde, saying: "Here is the suspect in the disappearance of the little girl.
Since you are already here, I am giving him to you." The policemen together with Gallarde proceeded to
where the people found Editha. One of the policemen shoved more soil aside. The lifeless Editha was
completely naked when she was recovered. A picture of Gallarde was taken without any counsel present.
Gallarde was charged with the special complex crime of rape with homicide.
During the arraignment Gallarde, with the assistance of counsel, entered a plea of not guilty. Trial of the case
immediately ensued as the defense waived the holding of the pre-trial conference.
the Regional Trial Court rendered a decision convicting Gallarde of the crime of murder only, not of the
complex crime of rape with homicide because of the lack of proof of carnal knowledge
His motion for reconsideration, having been denied by the trial court in its Resolution
Gallarde appealed to the Supreme Court.

ISSUE
Whether or not the taking of pictures of an accused violates of his constitutional right against self incrimination.

RULING
The taking of pictures of an accused even without the assistance of counsel, being a purely mechanical act, is
not a violation of his constitutional right against self-incrimination.

The constitutional right of an accused against self-incrimination prohibit the use of physical or moral
compulsion(obligation) to extort(force) communications from the accused and not the inclusion of his body in
evidence when it may be material.

Purely mechanical acts are not included in the prohibition as the accused does not thereby speak his guilt,
hence the assistance and guiding hand of counsel is not required.

The essence of the right against self-incrimination is testimonial compulsion, that is, the giving of evidence
against himself through a testimonial act. Hence, it has been held that a woman charged with adultery may be
compelled to submit to physical examination to determine her pregnancy; and an accused may be compelled to
submit to physical examination and to have a substance taken from his body for medical determination as to
whether he was suffering from gonorrhoea which was contracted by his victim; to expel morphine from his
mouth; to have the outline of his foot traced to determine its identity with bloody footprints; and to be
photographed or measured, or his garments or shoes removed or replaced, or to move his body to enable the
foregoing things to be done.

CLASS ANNOTATION
SYLLABI
Constitutional Law; Right of Accused; The taking of pictures of an accused even without the assistance
of counsel, being a purely mechanical act, is not a violation of his constitutional right against self-
incrimination.We cannot agree with the trial courts rejection of the photographs (Exhibits I, J and K)
taken of GALLARDE immediately after the incident on the ground that the same were taken while [GALLARDE]
was already under the mercy of the police. The taking of pictures of an accused even without the assistance of
counsel, being a purely mechanical act, is not a violation of his constitutional right against self-incrimination. The
constitutional right of an accused against self-incrimination proscribes(prohibits) the use of physical or moral
compulsion to extort communications from the accused and not the inclusion of his body in evidence when it
may be material. Purely mechanical acts are not included in the prohibition as the accused does not thereby
speak his guilt, hence the assistance and guiding hand of counsel is not required. The essence of the right

Page 223 of 236


Constitutional Law 2 Bill of Rights Armando Santiago Jr

against self-incrimination is testimonial compulsion(obligation), that is, the giving of evidence against
himself through a testimonial act.

REGALA VS.SANDIGANBAYAN61
PONENTE: KAPUNAN, J.

TOPIC: Right of the Accused

TRIGGER OF THE FACTS: PCGG Mandated the petitoner to reveal the identity of his client in order to
affirm their allegation that regala was co-principal in the commission of the
crime against ill gotten wealth.

TRIGGER OF THE ISSUE: WON fiduciary duty may be asserted by petitioner on refusal to disclose
names of their clients (privilege information)

TRIGGER OF THE RULING: SC upheld the right of petitioners to refuse disclosure of names of their clients
under the pain of breach of fiduciary relationship with their client.

VERDICT: the Resolutions of respondent Sandiganbayan promulgated are hereby


ANNULLED and SET ASIDE. Respondent Sandiganbayan is further ordered
to exclude petitioners Teodoro D. Regala, Edgardo J. Angara, Avelino V. Cruz,
Jose C. Concepcion, *Rogelio A. Vinluan, Victor P. Lazatin, Eduardo U.
Escueta and Paraja G. Hayuduni as parties-defendants in SB Civil Case No.
0033 entitled "Republic of the Philippines v. Eduardo Cojuangco, Jr., et al.".

END POINT:

CITED DOCTRINE: None

SEQUEL OF THE CASE: Money was received by the government, paid by persons who thereby
admitted they had not paid a sufficient amount in income taxes someone or
more years in the past. The names of the clients are useful to the government
for but one purpose - to ascertain which taxpayers think they were delinquent,
so that it may check the records for that one year or several years. The
voluntary nature of the payment indicates a belief by the taxpayers that more
taxes or interest or penalties are due than the sum previously paid, if any. It
indicates a feeling of guilt for nonpayment of taxes, though whether it is
criminal guilt is undisclosed. But it may well be the link that could form the
chain of testimony necessary to convict an individual of a federal crime.
Certainly the payment and the feeling of guilt are the reasons the attorney
here involved was employed to advise his clients what, under the
circumstances, should be done

IMPORTANT POINT/s: Right Against Self-Incrimination was violated relating documents to the
suspect transactions.

Atty-Client privilege was violated when touches the identity of the client.

FACTS
Corporation clients of petitioner consulted them regarding corporate structure and financial matters upon
which legal advice were given by petitioners.
Said corporation is subject to investigation by the PCGG involving ill gotten wealth.

61 G.R. No. 105938. September 20, 1996


Page 224 of 236
Constitutional Law 2 Bill of Rights Armando Santiago Jr

Petitioner refuses to provide information on fear that it may implicate them in the very activity from which legal
advice was sought from them and it may breach the fiduciary relationship of the petitioner with their client.

ISSUE
WON fiduciary duty may be asserted by petitioner on refusal to disclose names of their clients (privilege
information)

RULING
SC upheld the right of petitioners to refuse disclosure of names of their clients under the pain of breach of
fiduciary relationship with their client.

As a general rule, a lawyer MAY NOT INVOKE THE PRIVILEGE BECAUSE:

1. The court has the right to know that the client whose privilege is sought to be protected is flesh and blood.
2. Privilege begins to exist only after the Atty - client relationship has been established.
3. Privilege generally pertains to be the subject matter of the relationship.
4. With due process consideration, the opposing party should know his adversary.[a party suing or sued is
entitled to know his opponent is. He cannot be obliged to grope in the dark against unknown forces.]

EXCEPTION: LAWYERS MAY INVOKE THE PRIVILEGE WHEN:


1. Strong probability exists that revealing the clients name would implicate the client in the very activity for
which he sought the lawyers advice. Revealing the identity of the client might be detrimental and may
implicate his client.
2. Disclosure would open to civil liability of client. (present in this case) PCGG mandates the petitioner to
produce or disclose the identity of his client to trace the merit of the case against the client.
3. Government lawyers have no case against the lawyers client unless by revealing the clients name it would
provide them the only link that would form the chain of testimony necessary to convict an individual of a
crime. (present in this case) this case was not filed against the lawyers client but on the lawyer itself. So
meaning disclosure of such identity may harm the lawyers client.
4. Relevant to the subject matter of the legal problem on which client seeks legal assistance. (present in this
case) the important distinction must be made between a case where a client takes on the services of an
attorney for illicit purposes, seeking advice about how to go around the law for the purpose of committing
illegal activities and a case where a client thinks he might have previously committed something illegal and
consult his attorney about it. THE PRIVILEGE CANNOT BE INVOKED OR USED AS A SHIELD FOR AN
ILLEGAL ACT. BECAUSE IT IS NOT WITHIN THE PROFESSIONAL CHARACTER OF A LAWYER TO GIVE
AN ADVICE ON THE COMMISSION OF A CRIME.
5. Nature of atty-client relationship has been previously disclosed and it is the identity which is intended to be
confidential.

Old Code of Civil Procedure enacted by the Philippine Commission on August 7, 1901,:Section 383 of the
Code specifically "forbids counsel, without authority of his client to reveal any communication made by
the client to him or his advice given thereon in the course of professional employment." 28 Passed on
into various provisions of the Rules of Court, the attorney-client privilege, as currently worded
provides:Sec. 24. Disqualification by reason of privileged communication. The following persons cannot
testify as to matters learned in confidence in the following cases:An attorney cannot, without the consent of his
client, be examined as to any communication made by the client to him, or his advice given thereon in the course
of, or with a view to, professional employment, can an attorney's secretary, stenographer, or clerk be examined,
without the consent of the client and his employer, concerning any fact the knowledge of which has been
acquired in such capacity. 29Further, Rule 138 of the Rules of Court states:Sec. 20. It is the duty of an attorney:
(e) to maintain inviolate the confidence, and at every peril to himself, to preserve the secrets of his client, and to
accept no compensation in connection with his client's business except from him or with his knowledge and
approval.This duty is explicitly mandated in Canon 17 of the Code of Professional Responsibility which provides
that:Canon 17. A lawyer owes fidelity to the cause of his client and he shall be mindful of the trust and
confidence reposed in him.Canon 15 of the Canons of Professional Ethics also demands a lawyer's fidelity to

Page 225 of 236


Constitutional Law 2 Bill of Rights Armando Santiago Jr

client:The lawyers owes "entire devotion to the interest of the client, warm zeal in the maintenance and defense
of his rights and the exertion of his utmost learning and ability," to the end that nothing be taken or be withheld
from him, save by the rules of law, legally applied. No fear of judicial disfavor or public popularity should restrain
him from the full discharge of his duty. In the judicial forum the client is entitled to the benefit of any and every
remedy and defense that is authorized by the law of the land, and he may expect his lawyer to assert every such
remedy or defense. But it is steadfastly to be borne in mind that the great trust of the lawyer is to be performed
within and not without the bounds of the law. The office of attorney does not permit, much less does it demand of
him for any client, violation of law or any manner of fraud or chicanery. He must obey his own conscience and
not that of his client.

CLASS ANNOTATION
SYLLABI
Attorneys; Lawyer-Client Relationship; Petitioners are being prosecuted solely on the basis of activities
and services performed in the course of their duties as lawyers.It would seem that petitioners are merely
standing in for their clients as defendants in the complaint. Petitioners are being prosecuted solely on the basis
of activities and services performed in the course of their duties as lawyers. Quite obviously, petitioners inclusion
as co-defendants in the complaint is merely being used as leverage to compel them to name their clients and
consequently to enable the PCGG to nail these clients. Such being the case, respondent PCGG has no valid
cause of action as against petitioners and should exclude them from the Third Amended Complaint.

Same; Same; An attorney is more than a mere agent or servant because he possesses special powers of
trust and confidence reposed on him by his client.In modern day perception of the lawyer-client
relationship, an attorney is more than a mere agent or servant, because he possesses special powers of trust
and confidence reposed on him by his client. A lawyer is also as independent as the judge of the court, thus his
powers are entirely different from and superior to those of an ordinary agent. Moreover, an attorney also
occupies what may be considered as a quasi-judicial office since he is in fact an officer of the Court and
exercises his judgment in the choice of courses of action to be taken favorable to his client.

Same; Same; In the creation of lawyer-client relationship there are rules, ethical conduct and duties that
breathe life into it.Thus, in the creation of lawyer-client relationship, there are rules, ethical conduct and
duties that breathe life into it, among those, the fiduciary duty to his client which is of a very delicate, exacting
and confidential character, requiring a very high degree of fidelity and good faith, that is required by reason of
necessity and public interest based on the hypothesis that abstinence from seeking legal advice in a good cause
is an evil which is fatal to the administration of justice.

Same; Same; Generally, a lawyer may not invoke the privilege and refuse to divulge the name or identity
of his client.As a matter of public policy, a clients identity should not be shrouded in mystery. Under this
premise, the general rule in our jurisdiction as well as in the United States is that a lawyer may not invoke the
privilege and refuse to divulge the name or identity of his client.

Same; Same; Client identity is privileged where a strong probability exists that revealing the clients
name would implicate that client in the very activity for which he sought the lawyers advice.Client
identity is privileged where a strong probability exists that revealing the clients name would implicate that client
in the very activity for which he sought the lawyers advice.

Same; Same; Where disclosure would open the client to civil liability his identity is privileged.Where
disclosure would open the client to civil liability, his identity is privileged. For instance, the peculiar facts and
circumstances of Neugass v. Terminal Cab Corporation, prompted the New York Supreme Court to allow a
lawyers claim to the effect that he could not reveal the name of his client because this would expose the latter to
civil litigation.

Same; Same; The content of any client communication to a lawyer lies within the privilege if it is relevant
to the subject matter of the legal problem on which the client seeks legal assistance.Apart from these

Page 226 of 236


Constitutional Law 2 Bill of Rights Armando Santiago Jr

principal exceptions, there exist other situations which could qualify as exceptions to the general rule. For
example, the content of any client communication to a lawyer lies within the privilege if it is relevant to the
subject matter of the legal problem on which the client seeks legal assistance. Moreover, where the nature of the
attorney-client relationship has been previously disclosed and it is the identity which is intended to be
confidential, the identity of the client has been held to be privileged, since such revelation would otherwise result
in disclosure of the entire transaction.

Same; Same; The lawyer-client confidentiality privilege and lawyers loyalty to his client extends even
after the termination of the relationship.The utmost zeal given by Courts to the protection of the lawyer-
client confidentiality privilege and lawyers loyalty to his client is evident in the duration of the protection, which
exists not only during the relationship, but extends even after the termination of the relationship.

SEPARATE OPINION
VITUG, J., Separate Opinion:

Attorneys; Lawyer-Client Relationship; It is unreasonable for the Sandiganbayan to compel petitioners


to breach the trust reposed on them and succumb to a thinly disguised threat of incrimination.I see in
the case before us, given the attendant circumstances already detailed in the ponencia, a situation of the
Republic attempting to establish a case not on what it perceives to be the strength of its own evidence but on
what it could elicit from a counsel against his client. I find it unreasonable for the Sandiganbayan to compel
petitioners to breach the trust reposed on them and succumb to a thinly disguised threat of incrimination.

DISSENTING OPINION
DAVIDE, JR., J., Dissenting Opinion:

Attorneys; Lawyer-Client Relationship; The prerogative to determine who shall be made defendant in a
civil case is initially vested in the plaintiff.The prerogative to determine who shall be made defendants in a
civil case is initially vested in the plaintiff, or the PCGG in this case. The control of the Court comes in only when
the issue of interest (2, Rule 3, Rules of Court) as, e.g., whether an indispensable party has not been joined,
or whether there is a misjoinder of parties (7, 8, and 9, Id.), is raised.

Same; Same; The rule of confidentiality under the lawyer-client relationship is not a cause to exclude a
party.In view of their adamantine position, the petitioners did not, therefore, allow themselves to be like Roco.
They cannot claim the same treatment, much less compel the PCGG to drop them as defendants, for nothing
whatsoever. They have no right to make such a demand for until they shall have complied with the conditions
imposed for their exclusion, they cannot be excluded except by way of a motion to dismiss based on the grounds
allowed by law (e.g., those enumerated in 1, Rule 16, Rules of Court). The rule of confidentiality under the
lawyer-client relationship is not a cause to exclude a party. It is merely a ground for disqualification of a witness
(24, Rule 130, Rules of Court) and may only be invoked at the appropriate time, i.e., when a lawyer is under
compulsion to answer as witness, as when, having taken the witness stand, he is questioned as to such
confidential communication or advice, or is being otherwise judicially coerced to produce, through subpoenae
duces tecum or otherwise, letters or other documents containing the same privileged matter. But none of the
lawyers in this case is being required to testify about or otherwise reveal any [confidential] communication made
by the client to him, or his advice given thereon in the course of, or with a view to, professional employment.
PUNO, J., Dissenting Opinion:

Attorneys; Lawyer-Client Relationship; The relation of attorney and client cannot exist for the purpose of
counsel in concocting crimes.Communications to an attorney having for their object the commission of a
crime x x x partake the nature of a conspiracy, and it is not only lawful to divulge such communications, but
under certain circumstances it might become the duty of the attorney to do so. The interests of public justice
require that no such shield from merited exposure shall be interposed to protect a person who takes counsel
Page 227 of 236
Constitutional Law 2 Bill of Rights Armando Santiago Jr

how he can safely commit a crime. The relation of attorney and client cannot exist for the purpose of counsel in
concocting crimes. In the well chosen words of retired Justice Quiason, a lawyer is not a gun for hire.

Same; Same; As a general rule, the attorney-client privilege does not include the right of non-disclosure
of client identity.Assuming then that petitioners can invoke the attorney-client privilege since the PCGG is no
longer proceeding against them as co-conspirators in crimes, we should focus on the more specific issue of
whether the attorney-client privilege includes the right not to divulge the identity of a client as contended by the
petitioners. As a general rule, the attorney-client privilege does not include the right of non-disclosure of client
identity. The general rule, however, admits of well-etched exceptions which the Sandiganbayan failed to
recognize.

Same; Same; The person claiming the privilege or its exceptions has the obligation to present the
underlying facts demonstrating the existence of the privilege.Be that as it may, I part ways with the
majority when it ruled that petitioners need not prove they fall within the exceptions to the general rule. I
respectfully submit that the attorney-client privilege is not a magic mantra whose invocation will ipso facto and
ipso jure drape he who invokes it with its protection. Plainly put, it is not enough to assert the privilege. The
person claiming the privilege or its exceptions has the obligation to present the underlying facts demonstrating
the existence of the privilege. When these facts can be presented only by revealing the very information sought
to be protected by the privilege, the procedure is for the lawyer to move for an inspection of the evidence in an in
camera hearing. The hearing can even be in camera and ex-parte.

BELTRAN VS. SAMSON62


PONENTE: First Division, Romualdez (J): 6 concur

TOPIC: Right against self incrimination

TRIGGER OF THE FACTS: The judge ordered the petitioner to appear before the provincial prosecutor
take the dictations and write to compare the handwriting of beltran if he is the
falsifier. He invoke his constitutional right

TRIGGER OF THE ISSUE: WON The right of the petitioner were violated

TRIGGER OF THE RULING: Yes, He may invoke his right and refuse to do so. Because of this action it
may incriminate him and render his conviction.

VERDICT: We find the present action well taken, and it is ordered that the respondents
and those under their orders desist and abstain absolutely and forever from
compelling the petitioner to take down dictation in his handwriting for the
purpose of submitting the latter for comparison.

END POINT: Writing is not a purely mechanical act, because it requires the application of
intelligence and attention.

CITED DOCTRINE: First National Bank vs. Robert: He was then cross-examined the question
in regard to his having signed papers not in the case, and was asked in
particular whether he would not produce signatures made prior to the note in
suit, and whether he would not write his name there in the court. The judge
excluded all these inquiries, on objection, and it is of these rulings that
complaint is made. The object of the questions was to bring into the case
extrinsic signatures, for the purpose of comparison by the jury, and we think
that the judge was correct in ruling against it.

62 GR 32025, 23 September 1929


Page 228 of 236
Constitutional Law 2 Bill of Rights Armando Santiago Jr

SEQUEL OF THE CASE: Forcing someone to furnish a specimen of his handwriting is equal to
compelling a person to produce a document which is a violation of this right
the witness is required to furnish evidence against himself.

IMPORTANT POINT/s:

FACTS
Felix Samson, Judge of the Second Judicial District ordered Francisco Beltran to appear before the Provincial
Fiscal of Isabela, Francisco Jose, to take dictations in his own handwriting from the latter. The purpose for such
was for the fiscal to compare Beltran's handwriting and to determine if it is he who wrote certain documents
supposed to be falsified. Beltran filed a petition for a writ of prohibition.

ISSUE
Whether the writing from the fiscal's dictation by Beltran for the purpose of comparing the latters handwriting
and determining whether he wrote certain documents supposed to be falsified, constitutes evidence against
himself within the scope and meaning of the constitutional provision (i.e. "Nor shall he be compelled in any
criminal case to be a witness against himself.").

RULING
The fiscal under section 1687 of the Administrative Code, and the proper judge, upon motion of the fiscal, may
compel witnesses to be present at the investigation of any crime of misdemeanor. But this power must be
exercised without prejudice to the constitutional rights of persons cited to appear. The privilege is found in the
Jones Law, which provides that "Nor shall he be compelled in any criminal case to be a witness against himself."
This text is not limited to declaracion but says "to be a witness."

As to its scope, this privilege is not limited precisely to testimony, but extends to all giving or furnishing of
evidence. Writing is something more than moving the body, or the hand, or the fingers. Writing is not a purely
mechanical and attention. Herein, writing means that Beltran is to furnish a means to determine or not he is the
falsifier, as the petition of the provincial fiscal clearly states. Except that it is more serious, the present case is
similar to that of producing documents of chattels in one's possession. And as to such production of documents
or chattels, which is not so serious as present, the same eminent Professor Wigmore, in his work cited, says
(volume 4, page 864): "2264, Production or Inspection of Documents and Chattels.

1. It follows that the production of documents or chattels by a person (whether ordinary witness or party-witness)
in response to a subpoena, or to a motion to order production, or to other form of process treating him as a
witness (i. e. as a person appearing before the tribunal to furnish testimony on his moral responsibility for truth-
telling), may be refused under the protection of the privilege; and this is universally conceded." Thus, for the
purposes of the constitutional privilege, there is a similarity between one who is compelled to produce a
document, and one who is compelled to furnish a specimen of his handwriting, for in both cases, the witness is
required to furnish evidence against himself. The present case is more serious than that of compelling the
production of documents or chattels, because here the witness is compelled to write and create, by means of the
act of writing, evidence which does not exist, and which may identify him as the falsifier. It cannot be contended
that if permission to obtain a specimen of Beltran's handwriting is not granted, the crime would go unpunished.
Considering the circumstance that Beltran is a municipal treasurer, it should not be a difficult matter for the fiscal
to obtain genuine specimens of his handwriting. But even supposing it is impossible to obtain a specimen or
specimens without resorting to the means complained of, that is not reason for trampling upon a personal right
guaranteed by the constitution. It might be true that in some cases criminals may succeed in evading the hand of
justice, but such cases are accidental and do not constitute the raison d'etre of the privilege. This constitutional
privilege exists for the protection of innocent persons. Hence, the Court ordered the judge and the fiscal and
those under their orders desist and abstain absolutely and forever from compelling Beltran to take down dictation
in his handwriting for the purpose of submitting the latter for comparison.

Page 229 of 236


Constitutional Law 2 Bill of Rights Armando Santiago Jr

CLASS ANNOTATION
SYLLABI
CRIMINAL PROCEDURE; COMPULSORY APPEARANCE OF WITNESSES AT FlSCAL'S INVESTIGATIONS;
REFUSAL OF WlTNESS TO WRITE FROM DICTATION.The fiscal under section 1687 of the Administrative
Code, and the competent judge, at the request of the fiscal, may compel witnesses to be present at the
investigation of any crime or misdemeanor. But this power must be exercised without prejudice to the
constitutional rights of persons cited to appear. The petitioner, in refusing to write down what the fiscal had to
dictate to him for the purpose of verifying his handwriting and determining whether he had written certain
documents alleged to have been falsified, seeks protectionhis constitutional privilege.

2.ID.; RIGHTS OF DEFENDANT; TEXT OF CONSTITUTIONAL PROVISION.This right was promulgated,


both in the Organic Law of the Philippines of July 1, 1902 and in paragraph 3, section 3 of the Jones Law, which
provides (in Spanish); "Ni se le obligar (defendant) a declarar en contra suya, en ningn proceso criminal," and
recognized in our Criminal Procedure (General Orders, No. 58) in section 15 (No. 4) and section 56. The English
text of the Jones Law reads as follows: "Nor shall he be compelled in any criminal case to be a witness against
himself," thus, the prohibition is not restricted to not compelling him to testify, but extends to not compelling him
to be a witness.

3.ID.; ID.; SCOPE OF CONSTITUTIONAL PRIVILEGE."The rights intended to be protected by the


constitutional provision that no man accused of crime shall be compelled to be a witness against himself is so
sacred, and the pressure toward their relaxation so great when the suspicion of guilt is strong and the evidence
obscure, that it is the duty of courts liberally to construe the prohibition in favor of personal rights, and to refuse
to permit any steps tending toward their invasion. Hence, there is the well-established doctrine that the
constitutional inhibition is directed not merely to giving of oral testimony, but embraces as well the furnishing of
evidence by other means than by word of mouth, the divulging, in short, of any fact which the accused has a
right to hold secret." (28 R. C. L., par. 20, page 434, and notes.)

4.ID. ; ID. ; CASES INAPPLICABLE.There have been cases where it was lawful to compel the accused to
write in open court while he was under cross-examination (Bradford vs. People, 43 Pacific Reporter, 1013), and
to make him write his name with his consent during the trial of his case (Sprouse vs. Com., 81 Va., 374, 378);
but in the first case, the defendant, in testifying as witness in his own behalf waived his constitutional privilege
not to be compelled to act as witness; and in the second, he also waived said privilege because he acted
voluntarily.

5.ID. ; ID. ; PREPARATION AND CREATION OF EVIDENCE BY TESTIMONIAL ACT.This constitutional


prohibition embraces the compulsory preparation and creation by a witness of self-incriminatory evidence by
means of a testimonial act. "For though the disclosure thus sought" (the production of documents and chattels)
"be not oral in form, and though the documents or chattels be already in existence and not desired to be first
written and created by a testimonial act or utterance of the person in response to the process, still no line can be
drawn short of any process which treats him as a witness; because in virtue of it he would be at any time liable to
make oath to the identity or authenticity or origin of the articles produced." (4 Wigmore on Evidence 864', 865,
latest edition.) In the case before us, writing is something more than moving the body, or hand, or fingers; writing
is not a purely mechanical act; it requires the application of intelligence and attention; writing means for the
petitioner here to furnish, through a testimonial act, evidence against himself.

6.ID.; ID.; PROSECUTION OF CRIMES; PRIVILEGE, REASON FOR EXISTENCE OF.It cannot be
contended in the present case that if permission to obtain a specimen of the petitioner's handwriting is not
granted, the crime would go unpunished. The petitioner is a municipal treasurer, and it should not be difficult for
the fiscal to obtain a genuine specimen of his handwriting by some other means. But even supposing that it is
impossible to secure such specimen without resorting to the means herein complained of by the petitioner, that is
no reason for trampling upon a personal right guaranteed by the constitution. It might be true that in some cases
criminals may succeed in evading the hand of justice, but such cases are accidental and do not constitute the
raison d'etre of the privilege. This constitutional privilege exists for the protection of innocent persons.

Page 230 of 236


Constitutional Law 2 Bill of Rights Armando Santiago Jr

7.ID.; ID.; DISTINCTION BETWEEN VILLAFLOR-SUMMERS CASE AND CASE AT BAR.The difference
between this case and that of Villaflor vs. Summers (41 Phil., 62), is that in the latter the object was to have the
petitioner's body examined by physicians, without being compelled to perform a positive act, but only an
omission, that is, not to prevent the examination, which could be, and was, interpreted by this court as being no
compulsion of the petitioner to furnish evidence by means of a testimonial act; all of which is entirely different
from the case at bar, where it is sought to make the petitioner perform a positive testimonial act, silent, indeed,
but effective, namely, to write and give a sample of his handwriting for comparison.

ROSETE VS JULIANO LIM63


PONENTE: CHICO-NAZARIO, J.

TOPIC: Right Against Self-Incrimination

TRIGGER OF THE FACTS: The Petitioner acquired the land of the respondent. The respondent filed an
oral deposition against the petitioner. The trial court grant the petition and the
petitioner filed contending that there is an existing criminal case and such
deposition may incriminate them in their criminal case.

TRIGGER OF THE ISSUE: WON the trial court comitted grave abuse of discretion in granting the petition
for oral deposition raised by the respondent

TRIGGER OF THE RULING: SC ruled in the negative. such deposition is violative to their constitutional
right.

VERDICT: All the foregoing considered, the instant petition is dismissed for lack of merit.

END POINT:

CITED DOCTRINE: In the Ayson case, it is evident that the Court treats a party in a civil case as
an ordinary witness, who can invoke the right against self-incrimination only
when the incriminating question is propounded. Thus, for a party in a civil
case to possess the right to refuse to take the witness stand, the civil case
must also partake of the nature of a criminal proceeding.

SEQUEL OF THE CASE:

IMPORTANT POINT/s: Court said exception applies to parties in civil actions which are criminal in
nature.

FACTS
Petitioner filed a complaint for Annulment of specific performance with damages against AFP retirement and
separation benefits system
espreme realty and development corporation it tasked that the deed of sale executed by AFP-RSBS covering
certain parcels of lands in favor of espreme realty and the titles under the name of the latter be annulled
the AFP-RSBS and espreme realty be ordered to execute the necessary documents to restore ownership and
title of said lands to respondents
the register of deeds be ordered to cancel the titles of said land under the name of espreme realty and to
transfer the same in the names of respondents
respondents filed a notice to take deposition upon oral examination giving notice that they will cause the
deposition of petitioner mapalo and rosete

63 G.R. No. 136051 June 8, 2006


Page 231 of 236
Constitutional Law 2 Bill of Rights Armando Santiago Jr

contents that since there are two criminal cases pending before the city prosecutors involving the same
set of facts as in the present case wherein respondent Lim is the private complainant and petitioners are
the respondents to permit the taking of the deposition would be violative of their constitutional right
against self-incrimination because by means if the oral deposition, respondents would seek to establish
the allegations of the fact in the complaint which are also the allegations of fact in the complaint-affidavit
in the said criminal cases.
due to this, the case was appealed

ISSUE
Whether or not RTC acted in grave abuse of discretion in declaring in its order that the constitutional right of
oscar mapalo and chito rosete would not be violated by the taking of their deposition in the civil case files in the
lower court although they are also repondents or defendants in the aformentioned criminal cases filed by herein
private respondent lim involving the same or identical set of facts

RULING
The court did not abuse its discretion when if refused to recognize petitioners Mapalo and Rosetes constitutional
right against self incrimination. It allowed and scheduled the taking of their deposition by way of oral
examination. They refuse to give their depositions due to the pendency of two criminal cases (estafa and BP22)
because their answer would expose them to criminal action or liability since they would be furnishing evidence
against themselves in the said criminal cases, thus, there is a tendency to incriminate both mapalo and chito
rosete.

The right against self-incrimination is accorded to every person who gives evidence whether VOLUNTARY or
UNDER COMPULSION OF SUBPOENA in any civil, criminal or administrative proceeding.

IN CIVIL CASE (ORDINARY WITNESS) - Under compulsion of subpoena, the witness receiving it must obey the
subpoena take the stand and answer question, but when there is an incriminatory question the right against self
incrimination may be invoked.

CRIMINAL CASE - The constitutional right may be invoke.

EXCEPTION IN REFUSAL OF GIVING TESTIMONY

1. to be exempted from being a witness against himself


2. To testify as witness in his own behalf; but if he offers himself as a witness he may be cross-examined as any
other witness; however, his neglect or refusal to be a witness shall not in any manner prejudice or be used
against him.

Right of the defendant in a criminal case to be exempt from being a witness against himself signifies that he
cannot be compelled to testify or produce evidence in the criminal case in which he is the accused or one of the
accused.

He cannot be compelled to do so even by subpoena or other process of the court. He cannot be required to be a
witness either for the prosecution or for a co-accused or even himself.

The right tot take the stand does not generally apply to parties in administrative cases or proceedings.

In order for petitioners to exercise the right to refuse to take the witness stand and to give their depositions, the
case must partake of the nature of a criminal proceeding. The case on hand certainly cannot be categorized as
such. The fact that there are two criminal cases pending which are allegedly based on the same set of facts as
that of the civil case will not give them the right to refuse to take the witness stand and to give their depositions.
They are not facing criminal charges in the civil case. Like an ordinary witness, they can invoke the right against
self-incrimination only when the incriminating question is actually asked of them. Only if and when incriminating
questions are thrown their way can they refuse to answer on the ground of their right against self-incrimination.
Page 232 of 236
Constitutional Law 2 Bill of Rights Armando Santiago Jr


CLASS ANNOTATION
SYLLABI
Criminal Procedure; Rights of the Accused; Self-Incrimination; The right against self-incrimination is
accorded to every person who gives evidence, whether voluntary or under compulsion of subpoena, in
any civil, criminal or administrative proceeding.The right against self-incrimination is accorded to every
person who gives evidence, whether voluntary or under compulsion of subpoena, in any civil, criminal or
administrative proceeding. The right is not to be compelled to be a witness against himself. It secures to a
witness, whether he be a party or not, the right to refuse to answer any particular incriminatory question, i.e., one
the answer to which has a tendency to incriminate him for some crime. However, the right can be claimed only
when the specific question, incriminatory in character, is actually put to the witness. It cannot be claimed at any
other time. It does not give a witness the right to disregard a subpoena, decline to appear before the court at the
time appointed, or to refuse to testify altogether. The witness receiving a subpoena must obey it, appear as
required, take the stand, be sworn and answer questions. It is only when a particular question is addressed to
which may incriminate himself for some offense that he may refuse to answer on the strength of the
constitutional guaranty.

Same; Same; Same; Under the Rules of Court, in all criminal prosecutions the defendant is entitled
among others, to the following.An accused occupies a different tier of protection from an ordinary witness.
Under the Rules of Court, in all criminal prosecutions the defendant is entitled among others1) to be exempt
from being a witness against himself, and 2) to testify as witness in his own behalf; but if he offers himself as a
witness he may be cross-examined as any other witness; however, his neglect or refusal to be a witness shall
not in any manner prejudice or be used against him.

Same; Same; Same; As long as the suit is criminal in nature, the party thereto can altogether decline to
take the witness standit is not the character of the suit involved but the nature of the proceedings that
controls.It is clear, therefore, that only an accused in a criminal case can refuse to take the witness stand. The
right to refuse to take the stand does not generally apply to parties in administrative cases or proceedings. The
parties thereto can only refuse to answer if incriminating questions are propounded. This Court applied the
exceptiona party who is not an accused in a criminal case is allowed not to take the witness standin
administrative cases/ proceedings that partook of the nature of a criminal proceeding or analogous to a criminal
proceeding. It is likewise the opinion of the Court that said exception applies to parties in civil actions which are
criminal in nature. As long as the suit is criminal in nature, the party thereto can altogether decline to take the
witness stand. It is not the character of the suit involved but the nature of the proceedings that controls.

BAR QUESTION
Rights of the Incrimination (2010) Accused; Self-incrimination
No. X. A, the wife of an alleged victim of enforced disappearance, applied for the issuance of a writ of amparo
before a Regional Trial Court in Tarlac. Upon motion of A, the court issued inspection and production orders
addressed to the AFP chief of Staff to allow entry at Camp Aquino and permit the copying of relevant documents,
including the list of detainees, if any. Accompanied by court-designated Commission on Human Rights (CHR)
lawyers, A took photographs of a suspected isolation cell where her husband was allegedly seen being held for
three days and tortured before he finally disappeared. The CHR lawyers requested one Lt. Valdez for a
photocopy of the master plan of Camp Aquino and to confirm in writing that he had custody of the master plan.
Lt. Valdez objected on the ground that it may violate his right against self-incrimination. Decide with reasons.
(4%).

Page 233 of 236


Constitutional Law 2 Bill of Rights Armando Santiago Jr

SUGGESTED ANSWER:
The objection of Lt. Valdez is not valid. The right against self-incrimination refers to testimonial evidence and
does not apply to the production of a photocopy of the master plan of Camp Aquino, because it is a public
record. He cannot object to the request for him to confirm his custody of the master plan, because he is the
public officer who had custody of it. (Almonte vs. Vasquez, 244 SCRA 286 [1995]).

ALTERNATIVE ANSWER:
The objection is without merit. Right against self-incrimination is not violated because the right is simply against
testimonial compulsion. But the prohibition also extends to the compulsion for the production of documents,
papers and chattels that may be used as evidence against the witness, except where the State has a right to
inspect the same such as in this case.

Pursuant to the production order issued by the court, there can be compulsion for the production of documents
sought in the order.

Rights of the Accused; Self-Incrimination (1988)


No. 3: Dr. Juan Sto. Tomas is a practicing dentist in Marikina, Metro Manila. He was charged with immorality
before the Board of Dentistry by a lady patient, who claims that Dr. Sto. Tomas took liberties with her person and
kissed her while she was under the treatment at the latter's clinic.

At the initial hearing of the administrative complaint, the complainant's counsel called the respondent as his first
witness. The respondent through counsel, objected vigorously, claiming his constitutional right to be exempt from
being a witness against himself. The Board noted the objection, but ruled that in the next scheduled hearing, a
month and a half later, the respondent would be called to testify as a witness, as the right he claims is not
available in administrative investigations, but only in criminal prosecutions.

Dr. Sto. Tomas is decided not to testify. As his lawyer, what would you do? Why?

SUGGESTED ANSWER:
I will file a petition for prohibition with prayer for preliminary injunction with the Regional Trial Court. The privilege
against self incrimination is available not only in judicial proceedings but also in administrative investigations. In
Pascual v. Board of Medical Examiners, 28 SCRA 344 (1969), it was held that the revocation of a license as a
medical practitioner can be an even greater deprivation than mere forfeiture of property. In some aspects it is
similar to criminal proceedings and, therefore, the respondent can not be made to testify as a witness for the
complainant.

Rights of the Accused; Self-Incrimination (1990)

No. 4: The privilege of self-incrimination must be timely invoked, otherwise it is deemed waived.
1. In a CIVIL CASE, the plaintiff called the defendant a hostile witness and announced that the defendant would
be asked incriminating questions in the direct examination. When should the defendant invoke the privilege
against self- incrimination?

Page 234 of 236


Constitutional Law 2 Bill of Rights Armando Santiago Jr

2. In a CRIMINAL CASE, the prosecution called the accused to the witness stand as the first witness in view of
certain facts admitted by the accused at the pre-trial. When should the accused invoke the privilege against
self-incrimination?

3. In an administrative case for malpractice and the cancellation of license to practice medicine filed against C,
the complainant called C to the witness stand. When should C invoke the privilege against self-
incrimination?

Explain your answers to the three questions.

SUGGESTED ANSWER:
(1) As held in Bagadiong v, De Guzman, 94 SCRA 906, the defendant should take the witness stand and object
when a question calling for an incriminating question is propounded. Unlike in proceedings which are criminal
in character in which the accused can refuse to testify, the defendant must wait until a question calling for an
incriminatory answer is actually asked. (Suarez v. Tongco, 2 SCRA 71)

(2) As held in Chavez v. Court of Appeals, 24 SCRA 663, in a criminal case the accused may altogether refuse to
take the witness and refuse to answer any question, because the purpose of calling him as a witness for the
prosecution has no other purpose but to incriminate him.

(3) As in a criminal case, C can refuse to take the witness stand and refuse to answer any question. In Pascual
v. Board of Medical Examiners, 28 SCRA 344, it was held that an administrative case for malpractice and
cancellation of the license to practice medicine is penal in character, because an unfavorable decision would
result in the revocation of the license of the respondent to practice medicine. Consequently, he can refuse to
take the witness stand.

Rights of the Accused; Self-Incrimination (1992)


No, 3; Congress is considering a law against drunken driving. Under the legislation, police authorities may ask
any driver to take a "breathalyzer test", wherein the driver exhales several times into a device which can
determine whether he has been driving under the influence of alcohol. The results of the test can be used, in any
legal proceeding against him. Furthermore, declaring that the issuance of a driver's license gives rise only to a
privilege to drive motor vehicles on public roads, the law provides that a driver who refuses to take the test shall
be automatically subject to a 90-day suspension of his driver's license,
Cite two [2] possible constitutional objections to this law. Resolve the objections and explain whether any such
infirmities can be cured.

SUGGESTED ANSWER:
Possible objections to the law are that requiring a driver to take the breathalyzer test will violate his right against
self-incrimination, that providing for the suspension of his driver's license without any hearing violates due
process, and that the proposed law will violate the right against unreasonable searches and seizures, because it
allows police authorities to require a drive to take the breathalyzer test even if there is no probable cause.

Requiring a driver to take a BREATHALYZER TEST does not violate his right against self- incrimination, because
he is not being compelled to give testimonial evidence. He is merely being asked to submit to a physical test.
This is not covered by the constitutional guarantee against self-incrimination. Thus, in South Dakota vs. Neville,
459 U.S. 553, it was held for this reason that requiring a driver to take a blood-alcohol test is valid.

As held in Mackey vs. Afontrya 443 U.S. 1, because of compelling government interest in safety along the
streets, the license of a driver who refuses to take the breathalyzer test may be suspended immediately pending
a post- suspension hearing, but there must be a provision for a post-suspension hearing. Thus, to save the

Page 235 of 236


Constitutional Law 2 Bill of Rights Armando Santiago Jr

proposed law from unconstitutionally on the ground of denial of due process, it should provide for an immediate
hearing upon suspension of the driver's license. The proposed law violates the right against unreasonable
searches and seizures. It will authorize police authorities to stop any driver and ask him to take the breathalyzer
test even in the absence of a probable cause.

Rights of the Accused; Self-Incrimination (2000)


No XI. b) A man was shot and killed and his killer fled. Moments after the shooting, an eyewitness described to
the police that the slayer wore white pants, a shirt with floral design, had boots and was about 70 kilos and 1.65
meters. Borja, who fit the description given, was seen nearby. He was taken into custody and brought to the
police precinct where his pants, shirt and boots were forcibly taken and he was weighed, measured,
photographed, fingerprinted and subjected to paraffin testing. At his trial, Borja objected to the admission in
evidence of the apparel, his height and weight, his photographs, fingerprints comparison and the results of the
paraffin test, asserting that these were taken in violation of his right against self-incrimination. Rule on the
objection. (2%)

SUGGESTED ANSWER:
b) The objection of Borja is not tenable. As held in People v. Paynor, 261 SCRA 615 (1996), the rights
guaranteed by Section 12, Article in of the Constitution applies only against testimonial evidence. An accused
may be compelled to be photographed or measured, his garments may be removed, and his body may be
examined.

Rights of the Accused; Self-Incrimination (Q7-2006)


Select the best answer and explain.
1. An accused's right against self-incrimination is violated in the following cases: (5%)
A. When he is ordered by the trial court to undergo a paraffin test to prove he is guilty of murder;
B. When he is compelled to produce his bankbooks to be used as evidence against his father charged with
plunder;
C. When he is ordered to produce a sample of his handwriting to be used as evidence that he is the author of a
letter wherein he agreed to kill the victim;
D. When the president of a corporation is sub- poenaed to produce certain documents as proofs he is guilty of
illegal recruitment.

SUGGESTED ANSWER:
The best answer is c) when he is ordered to produce a sample of his handwriting to be used as evidence that he
is the author of a letter wherein he agreed to kill the victim. Under Article HI, Section 17 of the 1987 Constitution,
"no person shall be compelled to be a witness against himself." Since the provision prohibits compulsory
testimonial incrimination, it does not matter whether the testimony is taken by oral or written means as either way
it involves the USE OF INTELLECTUAL FACULTIES. The purpose of the privilege is to avoid and prohibit
thereby the repetition and recurrence of compelling a person, in a criminal or any other case, to furnish the
missing evidence necessary for his conviction (Bermudez v. Castillo, Per Rec. No. 714-A, July 26, 1937; Beltran
v. Samson, G.R. No. 32025, September 23,1929).

Page 236 of 236

Você também pode gostar